The Argument from Reason

Wayfarer June 21, 2023 at 03:00 11125 views 498 comments
The argument from reason challenges the proposition that everything that exists, and in particular thought and reason, can be explained solely in terms of natural or physical processes. It is, therefore, an argument against materialist philosophy of mind. According to the argument, if such theories were true, our thoughts, and so also our reasoning, would be determined on the molecular level by neurochemistry, leaving no role for the free exercise of reason.

The basis of the argument is, then, that if materialism were correct, our thoughts would be the product of physical processes which are in themselves devoid of any purpose or intentionality (in line with the axioms of materialism, which holds that everything in the Universe is the product of physical laws and product of non-intentional and non-purposive processes). However our ability to engage in rational inference presupposes the existence of intentionality and purpose. Reasoning involves forming beliefs based on evidence, making logical inferences, and seeking to arrive at a true understanding. If our thoughts were merely the result of physical causation, they would lack the ability to either genuinely apprehend truth, or to be rationally justified in making truth claims.

The Target of Criticism
In order to clearly frame the argument from reason, it is necessary to understand what it is opposed to. This is usually said to be ‘naturalism’, but I will instead propose that its target is better named physicalism or materialism.

Thomas Nagel uses the term "the materialist neo-Darwinian concept of mind" [sup]1[/sup] to refer to a view that combines materialist philosophy of mind with the principles of neo-Darwinism, which is the modern synthesis of Darwinian evolution and genetics. Advocates include Daniel Dennett, Alex Rosenberg, D.M. Armstrong, J C C Smart, and others. These are strict reductionists, claiming that states of mind are brain-states, and therefore ultimately reducible to physical laws. For example, Daniel Dennett, speaking of the organic molecule, says that ‘An impersonal, unreflective, robotic, mindless little scrap of molecular machinery is the ultimate basis of all the agency, and hence meaning, and hence consciousness, in the universe’[sup]2[/sup]

C S Lewis
The current form of the argument from reason was popularised by C S Lewis in 1947, subsequently revised and reformulated after criticism from G.E.M. Anscombe. The argument is as follows:

1. No belief is rationally inferred if it can be fully explained in terms of nonrational causes.

This is because rational inference requires insight into logical relations. A process of reasoning (P therefore Q) is rational only if the reasoner sees that Q follows from P, and accepts Q on that basis. Thus, reasoning is veridical only if it involves a specific kind of causality, namely, rational insight. If a belief can be fully explained by nonrational causes, such as a physical influence then it cannot be said to be the product of reason. Lewis gives the example:

[quote=C S Lewis, Miracles, Chap 3]We can say, 'Grandfather is ill today because he ate lobster yesterday.' We can also say, 'Grandfather must be ill today because he hasn't got up yet (and we know he is an invariably early riser when he is well).' In the first sentence 'because' indicates the relation of Cause and Effect: The eating made him ill. In the second, it indicates the relation of what logicians call Ground and Consequent. The old man's late rising is not the cause of his disorder but the reason why we believe him to be disordered. There is a similar difference between 'He cried out because it hurt him' (Cause and Effect) and 'It must have hurt him because he cried out' (Ground and Consequent). The first indicates a dynamic connection between events or 'states of affairs' i.e. 'eating lobster caused him to be ill'; the first, a logical relation between beliefs.

Now a train of reasoning has no value as a means of finding truth unless each step in it is connected with what went before in the Ground-Consequent relation. If our B does not follow logically from our A, we think in vain. If what we think at the end of our reasoning is to be true, the correct answer to the question, 'Why do you think this?' must begin with the Ground-Consequent 'because'[/quote]

2. If naturalism is true, then all beliefs can be fully explained in terms of non-rational causes.

Support: Naturalism holds that nature is all that exists, and that all events in nature can in principle be explained without invoking mental causation. As a matter of definition physicalists claim that all events must have physical causes, and that therefore human thoughts can ultimately be explained in terms of material causes or physical events (such as neurochemical events in the brain) that are nonrational. In Lewis' terms, this would entail that our beliefs are a result of a physical chain of causes, not held as a result of insight into a ground-consequence relationship.

3. Therefore, if naturalism is true, then no belief is rationally inferred (from 1 and 2).

4. We have good reason to accept naturalism only if it can be rationally inferred from good evidence.

5. Therefore, there is not, and cannot be, good reason to accept naturalism.

(As noted, I feel the argument is better expressed by the term 'physicalism' rather than 'naturalism', as today's naturalism is not so clearly physicalist as it was at the time of Lewis' argument, although that in itself is an interesting discussion point.)

My view of the argument

I myself am not a particular admirer of C S Lewis (although I enjoyed a collective biography of him and his confrères, The Inklings, P & C Zalenski - the other Inklings being Owen Barfield, C.S Williams, and Tolkein). Nor do I present the argument as any kind of 'proof of God' (God forbid). However, I'm convinced that the basic drift of the argument is sound, because, as it says, reasoned inference comprises wholly and solely the relationship of ideas - not of neurochemicals across synapses.

'Hang on', you might say, 'if we but had sensitive enough instrumentation, would it not be possible to detect the neural configurations of such ideas in the operations of the brain?' To which I would point out that there's an inevitable circularity in such an attempt. This is because to establish any kind of logical relationship between the data, and the exercise of reason, would itself be an exercise of reason. I would argue that reason is internal to the act of thought - that it can't be detected in external data, such as neurological data, without drawing on the very faculty which you're seeking to explain. So I would argue that there's a problem of recursion - you can't see reason 'from the outside', as it were (another of Thomas Nagel's arguments.)

That's all for now - there are many more points to discuss, but it's already a long OP so will leave it there pending any comments.

--------
1. Thomas Nagel, Mind and Cosmos: Why the Materialist Neo-Darwinian Conception of Nature Is Almost Certainly False. In this book, Nagel makes the case that this is, as it were, the default philosophy of the secular intelligentsia. A detailed abstract is available here (NY Times).
2. Daniel Dennett, Darwin’s Dangerous Idea: Evolution and the Meanings of Life (New York: Simon and Schuster, 1995), 202-3.

Comments (498)

Tom Storm June 21, 2023 at 05:30 #816608
Reply to Wayfarer That's a really nicely presented OP.

I suspect Richard Rorty would argue that what looks like reason and rationality to humans is pretty much just a trick of language and contingency.

Isn't it the case that in nature animals survive and thrive if they make certain choices and not others? Couldn't it be argued that reason is just the choices that allow us to have more efficacious outcomes? In more vulgar Darwinian terms, natural selection privileges rational behavior as it enhances our chances of survival, and humans as pattern seeking creatures, adopt reason as the pattern which enhances the capacity to flourish. And of course reason has been painstakingly constructed over time and isn't all that popular in most areas of human life.
Srap Tasmaner June 21, 2023 at 05:46 #816610
Reply to Wayfarer
C S Lewis, Miracles, Chap 3:'Grandfather must be ill today because he hasn't got up yet (and we know he is an invariably early riser when he is well).'


C S Lewis, Miracles, Chap 3:'It must have hurt him because he cried out'


C S Lewis, Miracles, Chap 3:If our B does not follow logically from our A, we think in vain.


In neither of these examples does the B follow logically from the A, not the way we usually use "follow logically"; in each case the A's count as evidence for their respective B's, and it's the easiest thing in the world to construct a defeater. (Grandpa's not ill, he's dead, still angry about what you said about Trump last night, etc.)

I'm still not clear how the argument works. If I hold A and think B follows from A, infer B and then hold B on the basis of A, that's all you and Lewis need, right?
Wayfarer June 21, 2023 at 06:05 #816617
Reply to Srap Tasmaner I think the distinction is between different types of 'because' - 'because' in the sense of 'he became ill because he ate lobster' - the observation of a physical cause and effect. The second is based on the understanding of grandfather's behaviour - hence an understanding of 'ground and consequence'. I don't think the point is to prove that what the cause of the illness was, but simply to show that one could arrive at an understanding based on insight into the subject's behaviour.
Tom Storm June 21, 2023 at 06:19 #816620
Reply to Wayfarer If you bark twice you get a dog treat.
180 Proof June 21, 2023 at 06:34 #816621
Reply to Tom Storm :up:

Reply to Wayfarer At most (being charitable), the "argument from reason" only narrowly applies to reductive physicalism, otherwise a broader conclusion presupposes a false dichtomy (i.e. either "reductive physicalism" or "nonphysicalism") fallacy which invalidates the argument.
Srap Tasmaner June 21, 2023 at 06:43 #816622
Reply to Wayfarer

Sure, I get that. Two meaning of "because", two meanings of "reason". I get that, but I'm not clear how you make an argument out of this and if an argument has been made.

It's a matter of my psychological history that I have made the inferences I have, rational or not. But you and Lewis seem to think my good inferences are evidence of something that my bad inferences are not, and I don't understand why.

Take Grandpa. He hasn't come down, and I can reasonably infer that he's sick. Cool. I'm mister rationality and my behavior disproves naturalism.

But if there's a deer nibbling the grass in my yard when I step out on the porch, it will hear the slightest sound I make and freeze. If I make a considerable noise, it'll bolt. To me, Grandpa staying in bed is a sign that he's sick; to a deer the noises I make are a sign of danger. Is the deer's behavior also a refutation of naturalism? --- I mean, you can punt, because you don't need another refutation of naturalism if you've already got one, but what makes my behavior so special? (If you don't like the deer, substitute the dog that knows the particular sound of his owner's car.)
Wayfarer June 21, 2023 at 06:51 #816624
Quoting Srap Tasmaner
It's a matter of my psychological history that I have made the inferences I have, rational or not.


But then, that's reductionist, as well - of the type of reductionism that says that we hold the beliefs that we do, because of our psychological history, or because of some disposition. 'Psychologism', I believe it is called. Not because they're true, or can be true, regardless of those factors.
Wayfarer June 21, 2023 at 06:55 #816625
Quoting 180 Proof
At most (being charitable), the "argument from reason" only narrowly applies to reductive physicalism,


I was an undergrad at the University where D M Armstrong was head of school of philosophy. His best-known book is called A Materialist Theory of Mind. I would consider the argument from reason to be fatal to the premisses of that book.
Srap Tasmaner June 21, 2023 at 07:05 #816626
Reply to Wayfarer

It wasn't intended as the sort of reductionism you describe. You could claim if you want that the history of my mental states is not reducible to the history of my brain states.

My point was that we are talking about my mental behavior here, and if I have mental behavior -- rational or not -- that isn't reducible to biology, then you're good, naturalism is refuted. I don't understand the focus on my mental behavior you consider rational, and how its being rational makes it special evidence against naturalism.
Wayfarer June 21, 2023 at 07:24 #816627
Quoting Srap Tasmaner
I don't understand the focus on my mental behavior you consider rational, and how its being rational makes it special evidence against naturalism.


It is because of the physicalist assumptions of the kind of naturalism that the argument is aimed at. I mean, do you agree that 'An impersonal, unreflective, robotic, mindless little scrap of molecular machinery (i.e. the organic molecule) is the ultimate basis of all the agency, and hence meaning, and hence consciousness, in the universe?' If you don't agree, then you may not see the point of the argument, because it's not relevant to your own philosphical point of view.
180 Proof June 21, 2023 at 07:30 #816628
Reply to Wayfarer Back in the day, I couldn't find a copy of A Materialist Theory of Mind, but I'd eventually learned much from Armstrong's A Combinatorial Theory of Possibility and Sketch for a Systematic Metaphysics which, IME, are far more consistent with both formal and natural sciences than any 'immaterialist, antirealist or supernaturalist' speculations on the world, mind, etc I've come across.
Wayfarer June 21, 2023 at 07:34 #816629
Here's an example from D M Armstrong, A Materialist Theory of Mind - a paragraph chosen practically at random, expressing a kind of exasperation that anyone could dare think that the mind somehow is not an outcome of the law of physics or cannot be brought within the ambit of physics.

User image


I like sushi June 21, 2023 at 07:44 #816631
I think it is just a matter of not knowing. Any ‘theory’ that is given will necessarily be one that is ‘physical’/‘material’.

‘Love’ can be said to have ‘physical’ markers yet in and of itself there is more to experience than mere physical reduction. I am most convinced by Hussel’s approach when it comes to consciousness. There need be no answer just because we can ask a question. The problem is likely not understanding that some so-called ‘questions’ posed are not really questions at all.

Crisis was an attempt to highlight the problem of reducing psychology to materialism/physicalism.
Srap Tasmaner June 21, 2023 at 07:47 #816632
Quoting Wayfarer
It is because of the physicalist assumptions of the kind of naturalism that the argument is aimed at.


Still not seeing it.

This is true: the kind of reason I have for believing in UFOs is not the same kind of reason my treehouse fell down; those are two different senses of the word "reason", the former to do with inference and the latter with gravity.

This is unargued for: if I believe something for a reason, my belief is uncaused, or whatever you'd prefer to say there --- not describable without remainder in physical terms, blah blah blah.

Is there an argument from "because" having two senses to there being two realms, one ruled by Physics or Something and one ruled by Reason or Something? If that's even what we're going for.
Wayfarer June 21, 2023 at 08:07 #816634
Quoting Srap Tasmaner
s there an argument from "because" having two senses to there being two realms, one ruled by Physics or Something and one ruled by Reason or Something? If that's even what we're going for.


A story that comes to mind - I can't remember where I read it, or the details. But it was to do with a man who had a brain tumour (I think it was) who suddenly started to manifest extreme paedophilic behaviour. He had always been an upstanding citizen and this behaviour was completely out of character for him, but he was arrested and charged with innappropriate relations with minors. Whilst on remand, other symptoms began to manifest, and the tumour was discovered and excised. After this, his behaviour returned to normal. As I said, I can't remember all the details,but I think the court accepted that he had been influenced by this condition to perform acts against his own judgement.

You could generalise that to many cases where subjects are found not responsible for their actions because of some physical impediment or condition. I think that would be categorised as a physical cause. Whereas, it would be argued, if a subject were completely in possession of his faculties, and still decided to pursue such activities, then they would be held responsible.

I suppose that is a rhetorical example, but I think it draws out one of the implications of the argument, does it not?

Quoting I like sushi
Crisis was an attempt to highlight the problem of reducing psychology to materialism/physicalism.


Right. That book is sitting here on my desk as I write this. I'm perfectly confident that Edmund Husserl was not the target of this kind of argument.
Wayfarer June 21, 2023 at 09:28 #816642
Quoting I like sushi
Any ‘theory’ that is given will necessarily be one that is ‘physical’/‘material’.


On further thought, I’m rather intrigued by why you would say that, and why it appears obvious to you.
I like sushi June 21, 2023 at 12:42 #816673
Reply to Wayfarer Because a theory only has meaning if it can be tested. It is not a theory that god exists it is a belief. They are quite different. If proof of gods existence was provided it would necessarily constitute something that refers to a ‘physical’/‘materialist’ framework rather than based on some pure logic.

Miasma theory did not hold up when explaining malaria BUT there was a material/physical connection. If we are searching for a COMPLETE understanding I think that is a faulty approach to begin with.
Srap Tasmaner June 21, 2023 at 13:03 #816677
Quoting Wayfarer
the implications of the argument


The implications of what argument?

I thought in this thread you were presenting a specific argument, credited to Lewis with an assist from Anscombe, not just the usual clash of beliefs, and not just the bare claim that "we are not our brains" or something.

I don't yet see what the argument is.

Is it equivalent to an argument about free will and responsibility?

Doesn't seem to be, unless you wanted to say that abusers of children rationally infer that they should do what they do. (And I can't believe you'd reach for such an example after pooh-poohing psychological history, when it is widely known that abusers were often themselves abused.)
wonderer1 June 21, 2023 at 14:02 #816691
Quoting Wayfarer
The argument from reason challenges the proposition that everything that exists, and in particular thought and reason, can be explained solely in terms of natural or physical processes. It is, therefore, an argument against materialist philosophy of mind. According to the argument, if such theories were true, our thoughts, and so also our reasoning, would be determined on the molecular level by neurochemistry, leaving no role for the free exercise of reason.


This appears to be begging the question, by presuming that the exercise of reason is something different than information processing occurring in our brains.

Smuggling in a dualism which isn't part of the materialist view doesn't do anything to contradict a materialist view.

Then there is the issue of "free exercise of reason", which suggests to me a desire to maintain a belief in libertarian free will. However, giving up a naive notion of libertarian free will is a small price to pay for the more accurate understanding that comes with a well informed materialist perspective. (Assuming we want to compare appeals to consequences.)
wonderer1 June 21, 2023 at 14:57 #816709
Quoting Wayfarer
The basis of the argument is, then, that if materialism were correct, our thoughts would be the product of physical processes which are in themselves devoid of any purpose or intentionality (in line with the axioms of materialism, which holds that everything in the Universe is the product of physical laws and product of non-intentional and non-purposive processes).


Assuming you are using "intentionality" as discussed in the SEP, this 3blue1brown video provides a good sketch of how the outputs of a neural network can be *about* numerical digits in the visual field provided as inputs to the neural network. So intentionality can be recognized as emerging at relatively low levels in our neurological information processing.
wonderer1 June 21, 2023 at 15:37 #816720
Quoting Wayfarer
2. If naturalism is true, then all beliefs can be fully explained in terms of non-rational causes.


Sorry about the piecemeal response. I'm at work right now, and addressing things as I have small windows of time.

I disagree with your premise above. Naturalism being true does not entail being able to fully explain all physical systems and in particular there are insurmountable issues to fully explaining complex systems, and brains are the most complicated physical systems we know of. Naturalism being true only requires beliefs being *caused*, by what at the lowest level are non-rational causes.
introbert June 21, 2023 at 15:46 #816721
From my interpretation of what you wrote the argument from reason assumes natural physics are deterministic without full knowledge of workings. Cliffords argument for determinism that action is based on sequential stimuli, could also cause near unlimited possibilities in a physical structure like the brain X 'ideas'. Obviously ideas to a determinist are direct observstions not flights of fancy, so anyone not proceeding in a reasonable sequence is predestined by design to fail and then?
I like sushi June 21, 2023 at 16:05 #816725
The general problem in the argument is framing things as True or Not True in relation to phenomenon instead of understanding it as an abstract game that helps guide us through ‘reality’ rather than something that is directly applicable to ‘reality’.
180 Proof June 21, 2023 at 18:04 #816739
wonderer1 June 21, 2023 at 21:19 #816794
Quoting I like sushi
The general problem in the argument is framing things as True or Not True in relation to phenomenon instead of understanding it as an abstract game that helps guide us through ‘reality’ rather than something that is directly applicable to ‘reality’.


What are your thoughts on replacing "true" and "false" with "more accurate" and "less accurate"?

Throwing away the notions of true or false altogether seems a bit extreme to me. Wouldn't we, in effect, be throwing out logic as well?
wonderer1 June 21, 2023 at 21:24 #816801
Another thought regarding [url=https://plato.stanford.edu/entries/intentionality/] which I'll preface with an exverpt from the SEP entry:

In philosophy, intentionality is the power of minds and mental states to be about, to represent, or to stand for, things, properties and states of affairs. To say of an individual’s mental states that they have intentionality is to say that they are mental representations or that they have contents. Furthermore, to the extent that a speaker utters words from some natural language or draws pictures or symbols from a formal language for the purpose of conveying to others the contents of her mental states, these artifacts used by a speaker too have contents or intentionality.


Do the outputs of ChatGPT have intentionality? Why or why not?
180 Proof June 21, 2023 at 21:37 #816805
Quoting wonderer1
Do the outputs of ChatGPT have intentionality?

Well, it's not altogether clear even that human thoughts "have intentionality" ... :chin:
wonderer1 June 21, 2023 at 21:45 #816807
Quoting 180 Proof
Well, it's not altogether clear even that human thoughts "have intentionality" ... :chin:


The SEP goes on to say:
‘Intentionality’ is a philosopher’s word: ever since the idea, if not the word itself, was introduced into philosophy by Franz Brentano in the last quarter of the nineteenth century, it has been used to refer to the puzzles of representation, all of which lie at the interface between the philosophy of mind and the philosophy of language.


So do we blame old Franz for creating all of this confusion? :gasp:
Tom Storm June 21, 2023 at 21:50 #816809
Quoting wonderer1
What are your thoughts on replacing "true" and "false" with "more accurate" and "less accurate"?

Throwing away the notions of true or false altogether seems a bit extreme to me. Wouldn't we, in effect, be throwing out logic as well?


Generally I hold to a view that some ideas are useful for certain purposes and some ideas are not. We never get to ultimate truth as such. Just things which work or don't. Does logic work everywhere?

Quoting 180 Proof
Well, it's not altogether clear even that human thoughts "have intentionality" ... :chin:


:fire: Next you'll be telling us qualia is nonsense...

You clearly believe that natural processes were able to lead human animals to the use of reason.

The argument here is essentially that naturalism isn't workable. But why wouldn't our ability to reason be advantageous for survival? I'm not arguing that evolution selects for truth, but that minds which can realistically understand the world around it (food that is safe to eat, predators to avoid, etc) are likely to survive better. A reasoning functionality in the brain would be advantageous - minds that survive, that act in accordance with truth, are more likely to survive the material world around them.

So the underlying issue here from @Wayfarer perspective is that naturalism presupposes intentionality; our capacity for thoughts to be about stuff. How can physical things give rise to such thought? But isn't intentionality essentially about memory - our ability to observe things and recall them?

We're back to the discussion about consciousness - how do we get to the mental?

The question we're faced with: is it impossible that conscious processes could evolve from natural causes? Surely we can't say no.

Wayfarer June 21, 2023 at 21:52 #816813
Quoting 180 Proof
Well, it's not altogether clear even that human thoughts "have intentionality”


So how did this entry become written? By mistake?
Wayfarer June 21, 2023 at 21:55 #816815
Quoting wonderer1
This appears to be begging the question, by presuming that the exercise of reason is something different than information processing occurring in our brains.


Which question, exactly? It starts with the presumption that we can arrive at true beliefs through reasoned inference, and then asks what must be the case in order for this to be so.

Quoting I like sushi
The general problem in the argument is framing things as True or Not True in relation to phenomenon instead of understanding it as an abstract game that helps guide us through ‘reality’ rather than something that is directly applicable to ‘reality’.


This really makes no sense. Again the argument is about the means by which reasoned inference may result in true beliefs. And any argument which has to place reality in scare quotes ought to be looked at askance.

Philosophim June 21, 2023 at 22:08 #816817
Good post Wayfarer, the time and detail that went into this is appreciated.

I'm ok with point 1 at the moment, so lets go into the proposed contradiction.

Quoting Wayfarer
As a matter of definition physicalists claim that all events must have physical causes, and that therefore human thoughts can ultimately be explained in terms of material causes or physical events (such as neurochemical events in the brain) that are nonrational. In Lewis' terms, this would entail that our beliefs are a result of a physical chain of causes, not held as a result of insight into a ground-consequence relationship.


Quoting Wayfarer
A process of reasoning (P therefore Q) is rational only if the reasoner sees that Q follows from P, and accepts Q on that basis. Thus, reasoning is veridical only if it involves a specific kind of causality, namely, rational insight.


If this is the case, how is it not rational to conclude that the physical brain causes the mind? Its not an irrational argument. In simple terms, if brain state = X, then mind state = Y is the claim right? If this can be confirmed through testing, then I would say this is a completely rational argument. If you lacked rational insight, then yes, you would not see it as rational. But you have rational insight. How is this not rational then?

Are you saying that underlying physical process don't process the term rationality like we do in our mind? Because that's not what naturalism is stating. Its perfectly rational to observe that gravity pulls something down at a steady acceleration. Are we to say that gravity is irrational because it doesn't realize or think that it should accelerate at a steady pace? Of course not. That's not a counter of naturalism, that's just a misapplication of the term "rational".

wonderer1 June 21, 2023 at 22:45 #816825
Quoting Wayfarer
Which question, exactly? It starts with the presumption that we can arrive at true beliefs through reasoned inference, and then asks what must be the case in order for this to be so.


I meant beg the question in the sense discussed here, assuming that would be the usage most common on the forum. In any case, what I meant by beg the question was assume the conclusion. I interpreted you saying, "...if such theories were true, our thoughts, and so also our reasoning, would be determined on the molecular level by neurochemistry, leaving no role for the free exercise of reason.", as suggesting that exercise of reason is assumed to be incompatible with the determinism of physics, when that is what your argument seeks to show.

Having looked at that sentence from your opening paragraph again, I'll also point out that my theory is that neurons supervene on the molecular level, but it is at the level of networks of neurons that our reasoning is determined in the most interesting regards. That's simplistic of course, since among other things, various organs dumping hormones into our bloodstreams and intake of a variety of substances also plays a deterministic role in how we reason. (Speaking of which... [reaches for a gummie]) However, I think consideration of the role of networks of neurons, and disregarding the molecular details on which the neurons supervene, is an appropriate level of looking at things for the purpose of this discussion. That's the level at which intentionality seems to emerge.
Wayfarer June 21, 2023 at 23:15 #816833
Quoting Tom Storm
why wouldn't our ability to reason be advantageous for survival?...So the underlying issue here from Wayfarer perspective is that naturalism presupposes intentionality; our capacity for thoughts to be about stuff. How can physical things give rise to such thought? But isn't intentionality essentially about memory - our ability to observe things and recall them?


I think viewing reason through that criterion of whether it 'helps us survive' is reductionist. What is advantageous to survival is an essential consideration in evolutionary theory. But classifying reason along with other traits - tentacles, claws, physical speed or strength - undermines the sovereignty, thus the credibility, of reason. Surely if reason is to have meaning, it has to be able to stand on it's own feet, so to speak. If it appeals to the court of 'what helps survival', then reason becomes subordinated to other purposes. (Both Donald Hoffman and Alvin Plantinga make use of this line of argument, for different purposes. It's also discussed in Nagel's Evolutionary Naturalism essay which I've previously referred to.)

My view: evolutionary biology certainly provides the account of how h. sapiens physically evolved. But thanks to the rapid evolution of the massive sapient fore-brain, h. sapiens has developed powers of perception which are almost entirely absent in other creatures, chief amongst them reason (hence 'the rational animal'). That faculty, along with language, tool-making, story-telling, and the capacity for self-transcendence, enables us to 'transcend our biology' so to speak. An intuition of that, I contend, is what is behind the various forms of philosophical dualism, such as the rational soul in the physical body. These need not be literally true in order to be metaphorically accurate, to tell us something vital about human nature.

I think the nature of reason is tied up with the ability to abstract and to generalise, which is the basis of both language and logic. And I think the Greek philosophers realised this - you can see the origins of it in Parmenides and Plato and the discussions of forms and universals. That's a digression, but it's also part of the background of this argument.

Quoting Tom Storm
The question we're faced with: is it impossible that conscious processes could evolve from natural causes?


Everyone assumes that 'mind is organised matter' and that the processes understood by the natural sciences will one day illustrate the fine detail of how this happened through what is called a-biogenesis (life from non-life). But, I refer again to the first four paragraphs of The Core of MInd and Cosmos, which discusses the sense in which 'the subject' was deliberately omitted or excluded at the formation of the modern natural sciences of Galileo, Descartes and Newton, with many concommitant philosophical implications. One could argue that the perspective of the subject (subject-hood, as distinct from subjectivity) is being re-introduced through phenomenology and embodied cognition (although It's still not considered in the kind of physicalism which this argument is addressing.)

I am contemplating the idea that right from the very first life-forms, life *is* the earliest manifestation of intentionality. As the complexity of organisms evolves over the aeons, so too their intelligence, apparently arriving at h. sapiens, through which the whole process has become critically self-aware.

//we arrive at the ability to understand abstract truths and the like. They're not simply 'a product' of the human mind, although having such a mind, we can produce, e.g. imaginary number systems and the like. But I maintain the furniture of reason such as logical laws, are discovered not invented, and certainly are not the products of a biological process.//

Quoting wonderer1
Do the outputs of ChatGPT have intentionality? Why or why not?


Well, taking it from the horses' mouth:

Question: In philosophy, 'intentionality' is the power of minds and mental states to be about, to represent, or to stand for, things, properties and states of affairs. Do AI systems such as ChatGPT possess this power?


ChatGPT: AI systems, including ChatGPT, do not possess intentionality in the same way that humans do. Intentionality is typically associated with consciousness and subjective experience, which are currently not attributes of AI systems.

While AI systems like ChatGPT can process and generate information about things, properties, and states of affairs, they do not possess a genuine understanding or subjective experience of the concepts they deal with. Their ability to represent and respond to information is based on statistical patterns learned from vast amounts of training data.

AI systems operate through algorithms and statistical models that enable them to process and generate text based on patterns in the data they were trained on. They don't possess personal goals, beliefs, desires, or subjective experiences. Their responses are determined by the patterns in the input they receive and the way they were trained, rather than by any internal mental states or subjective understanding.

While AI systems can be sophisticated in generating responses that appear intentional or meaningful, these responses are ultimately the result of complex computational processes rather than true intentionality.

Verbatim!


Quoting Philosophim
In simple terms, if brain state = X, then mind state = Y is the claim right? If this can be confirmed through testing, then I would say this is a completely rational argument.


So I'm saying, you can't get to such simple terms in neuroscience. What you're arguing here is 'brain-mind identity theory'. To respond in terms of the argument from reason, I would say that the brain-mind identity theory collapses or blurs the distinction between logical necessity and physical causation. If mental states are said to be identical to specific brain states or processes, this suggest that there is a direct correspondence between logical propositions and specific physical configurations.

In the brain-mind identity theory, the identity statement between a mental state and a brain state is typically understood as a necessary identity. This means that if a specific mental state is instantiated, it is necessarily identical to a specific brain state. The logical necessity is derived from the supposed one-to-one correspondence between mental and brain states. By collapsing the distinction between logical necessity and physical causation, the brain-mind identity theory implies that the truth of a logical proposition is causally determined by the physical state of the brain. In this view, the brain state corresponding to a particular mental state is thought to be both the cause and the logical ground for the associated mental experience or thought.

But while there may be correlations between mental states and brain states, this doesn't necessarily imply a strict identity between them. Logical propositions and their truth values are abstract entities that exist independently of any specific physical realization, such as brain states. 'If X >Y and A>X, then it must be the case that A>Y'. This is a logical proposition, but note that its validity is not dependent on any configuration of physical symbols. I could choose to represent it (and any number of different propositions) in different symbolic form and in different media, all whilst still preserving the meaning of the proposition. Hence the distinction between logical necessity and physical causation is preserved, and you can't show that 'brain-states' are causal, in respect of propositional content.

Quoting wonderer1
I interpreted this as suggesting that exercise of reason is assumed to be incompatible with the determinism of physics, when that is what your argument seeks to show.


And I think the argument does show that. It distinguishes between insight based on reasoned inference (knowing that X must be so on account of Y) and observation of a cause-and-effect relationship. No question is being begged, a case is being made.

Quoting wonderer1
I think consideration of the role of networks of neurons, and disregarding the molecular details on which the neurons supervene, is an appropriate level of looking at things for the purpose of this discussion


It might be, were this a computer science or neuroscience forum.
Tom Storm June 21, 2023 at 23:47 #816837
Quoting Wayfarer
But classifying reason along with other traits - tentacles, claws, physical speed or strength - undermines the sovereignty, thus the credibility, of reason. Surely if reason is to have meaning, it has to be able to stand on it's own feet, so to speak.


I don't think I share this view but it interests me. I don't see how reason needs to have transcendent meaning. But I'm open to considering this further.

Quoting Wayfarer
I think the nature of reason is tied up with the ability to abstract and to generalise, which is the basis of both language and logic. And I think the Greek philosophers realised this - you can see the origins of it in Parmenides and Plato and the discussions of forms and universals. That's a digression, but it's also part of the background of this argument.


I get this but I am not sure where this leads us.

Quoting Wayfarer
One could argue that the perspective of the subject (subject-hood, as distinct from subjectivity) is being re-introduced through phenomenology and embodied cognition (although It's still not considered in the kind of physicalism which this argument is addressing.)


Yes, and this is a rich, fascinating (and largely incomprehensible area to me).

Quoting Wayfarer
I am contemplating the idea that right from the very first life-forms, life *is* the earliest manifestation of intentionality. As the complexity of organisms evolves over the aeons, so too their intelligence, apparently arriving at h. sapiens, through which the whole process has become critically self-aware.

//we arrive at the ability to understand abstract truths and the like. They're not simply 'a product' of the human mind, although having such a mind, we can produce, e.g. imaginary number systems and the like. But I maintain the furniture of reason such as logical laws, are discovered not invented, and certainly are not the products of a biological process.//


I guess your project is a form of Platonism, a story about reality which you are more or less convinced of. I just don't think we (and certainly not I) know enough to go there. But it's interesting material. I don't think we can rule out naturalism at this point.

Thanks



Srap Tasmaner June 21, 2023 at 23:55 #816840
Quoting Wayfarer
To respond in terms of the argument from reason, I would say that the brain-mind identity theory collapses or blurs the distinction between logical necessity and physical causation.


Logical necessity never holds between one belief state and another; it only holds between the contents of one belief state and the contents of another.

That's why you need an actual argument showing that if brain state A, with contents P, causes brain state B, with contents Q, that a causal relation between A and B is incompatible with a logical relation between P and Q.
Gnomon June 22, 2023 at 00:07 #816843
Quoting Wayfarer
The argument from reason challenges the proposition that everything that exists, and in particular thought and reason, can be explained solely in terms of natural or physical processes. It is, therefore, an argument against materialist philosophy of mind. According to the argument, if such theories were true, our thoughts, and so also our reasoning, would be determined on the molecular level by neurochemistry, leaving no role for the free exercise of reason.

On the TPF forum, this a no-win argument. Both Physicalists and Metaphysicalists typically agree on the details of physics, neuro-chemistry, and cosmology all the way back to the rationally-inferred Big Bang, but disagree on the metaphysical question of direction vs randomness.

So, the argument eventually boils down to A> a rational intentional Creation ( temporal Cosmos) vs B> accidental random Causation (timeless Chaos), dating back to the beginning of our little pocket of space-time. Each party, exercising Reason & Inference, can find evidence to support his conclusion, based on that original Axiomatic assumption. But they arrive at different rational conclusions : a world that makes sense to the rational mind vs a world that makes sense for the sensory body*1.

Ontological question : Is the universe a self-organizing self-learning Program*2, or a random sequence of accidents that over eons has stumbled upon a formula to cause a few constellations of atoms to imagine that they exist, simply because they can think. What do you think? :smile:


*1. Is the World Rational? :
Our preliminary hypothesis asserts that the world has a certain property owing to which it can be successfully investigated by us. We call it the hypothesis of the rationality of the world (or simply the rationality of the world).
https://link.springer.com/chapter/10.1007/978-3-540-77626-0_5

*2. The Conservation of Information :
I'd be surprised if materialist/physicalist/deterministic scientists would think in terms of "learning" in a law-limited "deterministic" system*1. However computer scientists, and Information theorists, do sometimes use such anthro-morphic terminology metaphorically*2. So, if the "laws of nature" are imagined as a computer program, the universe could conceivably learn, in the same sense that Artificial Intelligence does*3, by means of "non-deterministic algorithms"*4.

But AI is not natural, and currently requires a natural Programmer to establish the parameters of the system. Would a self-organizing, self-learning world also require the services of a preter-natural Programmer to bootstrap the system?
https://thephilosophyforum.com/discussion/comment/816834
Tom Storm June 22, 2023 at 00:16 #816848
Quoting Gnomon
Is the universe a self-organizing self-learning Program*1, or a random sequence of accidents that over eons has stumbled upon a formula to cause a few constellations of atoms to imagine that they exist, simply because they can think. What do you think?


I'm not convinced we know what is random versus that which is not random. We detect patterns, as far as human cognition allows and we ascribe characteristics to those patterns - again in human terms. But words like 'random' or 'accidental' seem to have emotional connotations and function as tips of icebergs.
Wayfarer June 22, 2023 at 00:37 #816857
Quoting Srap Tasmaner
Logical necessity never holds between one belief state and another; it only holds between the contents of one belief state and the contents
of another.


That's what I'm arguing, it's exactly what I said - 'Logical propositions and their truth values (meaning their content) are abstract entities that exist independently of any specific physical realization'

Quoting Srap Tasmaner
That's why you need an actual argument showing that....


Sorry, but I don't follow this part.

Quoting Tom Storm
I don't see how reason needs to have transcendent meaning.


Useful to bear in mind the tricky Kantian distinction between 'transcendent' and 'transcendental'. The former refers to what is beyond experience; the latter to what must be presumed to be the case, in order for such and such a statement to be true. The argument from reason is very much a transcendental argument.

Quoting Gnomon
On the TPF forum, this a no-win argument.


I am dissappointed, but never surprised, to observe the routine deprecation of the faculty of reason. I think the classical notion of reason is rather non-PC, for various reasons, chief among them that it distinguishes humans from other species.

Quoting Tom Storm
I guess your project is a form of Platonism,


Lloyd Gleeson, who is one of the leading academics in this area, says in his most recent book Platonism vs Naturalism, that Platonism is philosophy, in that it delineates the specific questions and subject matter unique to philosophy as distinct from natural science. I don't expect that will win anyone over, though ;-) (See Edward Feser, Join the Ur-Platonist Alliance!)

Quoting Gnomon
Ontological question: Is the universe a self-organizing self-learning Program*2, or a random sequence of accidents that over eons has stumbled upon a formula to cause a few constellations of atoms to imagine that they exist, simply because they can think. What do you think?


Again, take a look at the chapter headings and abstracts (all available online) of Mind and the Cosmic Order, Charles Pinter. He has a compelling answer to at least part of this question.
Tom Storm June 22, 2023 at 00:55 #816862
Quoting Wayfarer
The argument from reason is very much a transcendental argument.


Good to know.

Quoting Wayfarer
Lloyd Gleeson


Do you mean Lloyd Gerson? I've read some of his papers.

Quoting Wayfarer
it delineates the specific questions and subject matter unique to philosophy as distinct from natural science.


Yes, I am aware of this position. I am simply unable to determine whether any of this scholarship is meaningful or not. My job in philosophy to be aware of the key questions and positions. With no expertise in these areas of enquiry, my own commitments are intuitions and of no real importance.

Joshs June 22, 2023 at 01:06 #816863
Reply to Wayfarer

Quoting Wayfarer
Lloyd Gleeson, who is one of the leading academics in this area, says in his most recent book Platonism vs Naturalism, that Platonism is philosophy, in that it delineates the specific questions and subject matter unique to philosophy as distinct from natural science. I don't expect that will win anyone over, though ;-) (See Edward Feser, Join the Ur-Platonist Alliance!)



I’m with Deleuze here:

“The whole of Platon­ism is dominated by the idea of drawing a distinction between 'the thing itself' and the simulacra. Difference is not thought in itself but re­lated to a ground, subordinated to the same and subject to mediation in mythic form. Overturning Platonism, then, means denying the primacy of original over copy, of model over image; glorifying the reign of simulacra and reflections.” (Difference and Repetition)
Wayfarer June 22, 2023 at 01:19 #816864
Reply to Joshs Thanks, although I suspect I'd have to read a lot more of that milieu to understand the drift (and must admit, feel little compulsion to do so.)

Quoting Tom Storm
Do you mean Lloyd Gerson?


Yeah sorry :yikes: I find him a very difficult read, because so much of his work is addressing other scholars and historical questions of interpretation. So I've only read snippets - come to think of it, that applies to many of my sources - but really got a lot from a lecture of which I also have the hard copy. That lecture conveys the gist of what was to become his latest book. I have this quotation from it in my scrapbook:

[quote=Lloyd Gerson]Aristotle, in De Anima, argued that thinking in general (which includes knowledge as one kind of thinking) cannot be a property of a body; it cannot, as he put it, 'be blended with a body'. This is because in thinking, the intelligible object or form is present in the intellect, and thinking itself is the identification of the intellect with this intelligible. Among other things, this means that you could not think if materialism is true… . Thinking is not something that is, in principle, like sensing or perceiving; this is because thinking is a universalising activity. This is what this means: when you think, you see - mentally see - a form which could not, in principle, be identical with a particular - including a particular neurological element, a circuit, or a state of a circuit, or a synapse, and so on. This is so because the object of thinking is universal, or the mind is operating universally.

….the fact that in thinking, your mind is identical with the form that it thinks, means (for Aristotle and for all Platonists) that since the form 'thought' is detached from matter, 'mind' is immaterial too. [/quote]

which, as it happens, beautifully supports 'the argument from reason'.

Tom Storm June 22, 2023 at 01:26 #816865
Quoting Wayfarer
but really got a lot from a lecture of which I also have the hard copy. I have this quotation in my scrapbook:


Yes, that's one I found pretty interesting too. Gerson is the go to guy on this subject as I understand it.

Quoting Joshs
Overturning Platonism, then, means denying the primacy of original over copy, of model over image; glorifying the reign of simulacra and reflections.” (Difference and Repetition)


That's an interesting call to arms but I guess it's hard for most of us to apprehend how we can do this? Is it an act of will? Pardon my literalism but in glorifying the reign of simulacra, does my Picasso print become equal to the one hanging in the museum?
wonderer1 June 22, 2023 at 01:34 #816870
Question: In philosophy, 'intentionality' is the power of minds and mental states to be about, to represent, or to stand for, things, properties and states of affairs. Do AI systems such as ChatGPT possess this power?


ChatGPT: AI systems, including ChatGPT, do not possess intentionality in the same way that humans do. Intentionality is typically associated with consciousness and subjective experience, which are currently not attributes of AI systems.
Reply to Wayfarer

That was funny, but note ChatGPT simply says ChatGPT does not have intentionality "in the same way that humans do." Which of course any well informed materialist would agree with. Then ChatGPT goes on producing output which doesn't directly respond to the question.

Whether ChatGPT has consciousness is not relevant. The point was to consider intentionality at low (subconscious) levels, on which intentionality at conscious levels could supervene.

Also, you didn't ask ChatGPT the question I proposed which was, "Do the outputs of ChatGPT have intentionality?" Since you posted an output of ChatGPT, do you think that output is about something?

Tom Storm June 22, 2023 at 01:36 #816871
Reply to wonderer1 Do animals have intentionality? They seem to from my perspective. What does this add to the discussion?
Joshs June 22, 2023 at 01:39 #816873
Reply to Tom Storm

Quoting Tom Storm
Overturning Platonism, then, means denying the primacy of original over copy, of model over image; glorifying the reign of simulacra and reflections.” (Difference and Repetition)
— Joshs

That's an interesting call to arms but I guess it's hard for most of us to apprehend how we can do this? Is it an act of will? Pardon my literalism but in glorifying the reign of simulacra, does my Picasso print become equal to the one hanging in the museum


It’s not as act of will , it’s an act of insight, understanding that there is no such thing as an original. It’s only as a contrivance, a sleight of hand, an illusion that we refer contingent particulars back to propositional or axiomatic forms, universals, foundations. Ever see Orson Wells’ film ‘F for Fake’?
Wayfarer June 22, 2023 at 01:48 #816875
Quoting wonderer1
Which of course any well informed materialist would agree with


OK then, give us a well-informed materialists' account of the significance of intentionality.

As for whether output - written text - has any kind of ‘intentionality’, I would say, clearly not. Written text means nothing without being interpreted.

Quoting Joshs
there is no such thing as an original


Now we're on solid po-mo territory. Oh, wait..... :yikes:
Tom Storm June 22, 2023 at 01:49 #816876
Quoting Joshs
Ever see Orson Wells’ film ‘F for Fake’?


One of my favorite films. An extraordinary cinematic essay. I understand his argument to be slightly different - a talented forger can fake things and they may pass as real, especially amongst the rich who want them to be real and the 'experts' who pass them off and take a cut. Wells also argues (elsewhere) that Shakespeare is objectively great and that Welles films are original even if flawed. I'm not sure he's our guy for this but I get your point, he loves to evoke and explore the notion of fakery

But forgery and fakery are only possible if there is an original - so how does this all work?
Wayfarer June 22, 2023 at 01:50 #816877
Quoting wonderer1
Also, you didn't ask ChatGPT the question I proposed which was, "Do the outputs of ChatGPT have intentionality?"


Let's do that.

Do the outputs of ChatGPT have intentionality?

ChatGPT: No, the outputs of ChatGPT do not have intentionality. ChatGPT is a language model that generates responses based on patterns it has learned from the training data. It does not possess consciousness or subjective experiences, and its responses are not driven by internal goals or intentions. ChatGPT generates text based on statistical patterns and associations in the training data and attempts to provide coherent and relevant responses to the input it receives.
wonderer1 June 22, 2023 at 01:51 #816878
Quoting Tom Storm
Do animals have intentionality? They seem to from my perspective. What does this add to the discussion?


In my experience, people have widely varying concepts in mind when using the word "intentionality" so many may disagree with what I am talking about being intentionality, but yes. I'd say animal with enough of a neural network to call a brain, likely have the low level intentionality I've been discussing. That's a different matter though, than having 'a big enough pile' of low level intentionality for consciousness to emerge.

Edit: I forgot to answer your last question. I don't have a clear idea of what you are asking with your question, but what I see it as adding to the discussion, is further consideration and clarification of the paradigm I'm presenting.
Joshs June 22, 2023 at 01:56 #816880
Reply to Tom Storm Quoting Tom Storm
But forgery and fakery are only possible if there is an original - so how does this all work?


Why not forgery and fakery all the way down? As Nietzsche asks:

“Why shouldn't the world that is relevant to us – be a fiction? And if someone asks: “But doesn't fiction belong with an author?” – couldn't we shoot back: “Why? Doesn't this ‘belonging' belong, perhaps, to fiction as well? Aren't we allowed to be a bit ironic with the subject, as we are with the predicate and object? Shouldn't philosophers rise above the belief in grammar? With all due respect to governesses, isn't it about time philosophy renounced governess-beliefs?” – The world with which we are concerned is false, i.e., is not fact but fable and approximation on the basis of a meager sum of observations; it is "in flux," as something in a state of becoming, as a falsehood always changing but never getting near the truth: for--there is no "truth" (1901/1967 Will to Power.)
Tom Storm June 22, 2023 at 01:58 #816881
Reply to Joshs I love the idea but I can’t find a way to fit it in.
Wayfarer June 22, 2023 at 02:02 #816882
Let's not dissolve the entire dialogue in the acid bath of post-modern relativism.

Quoting wonderer1
people have widely varying concepts in mind when using the word "intentionality"

The SEP entry would be a good starting point https://plato.stanford.edu/entries/intentionality
wonderer1 June 22, 2023 at 02:16 #816883
Quoting Wayfarer
OK then, give us a well-informed materialists' account of the significance of intentionality.


The sort of low level intentionality I have been discussing provides the subconscious infrastructure for consciousness.

As for whether output - written text - has any kind of ‘intentionality’, I would say, clearly not. Written text means nothing without being interpreted.


And have you interpreted ChatGPT's output as being [i]about[/I] something? (Like your conscious mind interpret's the outputs of your subconscious as being [i]about[/I] something.)

wonderer1 June 22, 2023 at 02:20 #816885
Quoting Wayfarer
The SEP entry would be a good starting point https://plato.stanford.edu/entries/intentionality


You know I've already provided that link, right? Is your ego so bruised already that you need to try to put me down in the estimations of the rest of the forum?

We could talk about the evolutionary psychology of that.
Tom Storm June 22, 2023 at 02:25 #816887
Reply to Joshs Sorry, I rarely get what Nietzsche says. I like him best when he sounds like a truculent Oscar Wilde. The question I always have when I read this kind of hyperbolic provocation is, why?

Sorry @Wayfarer I might start a thread on postmodernism and reason.
Wayfarer June 22, 2023 at 02:31 #816888
Quoting wonderer1
You know I've already provided that link, right?


Sorry, I forgot you had mentioned it.

Philosophim June 22, 2023 at 02:33 #816889
Quoting Wayfarer
But while there may be correlations between mental states and brain states, this doesn't necessarily imply a strict identity between them.


You know this is a completely false statement. You can't just claim they are correlations, you have to prove it. To prove a correlation, you need to remove the brain and still have a mind. Does anyone with brain death have a mind? Are the countless neuroscience discoveries, medicine, psychiatrics, etc. all just correlations? Of course not. You're too well versed to make a claim like that.

This is a rationalization. Despite knowing this isn't true, you believe this regardless. Why? What do you gain out of it Wayfarer? That's the only reason why people hold things they know are false to be true. Do you do it because you fear you'll lose something? Maybe I can help you hold onto what you want without you having to hold to this false notion. We're in philosophy. The point is to be razor sharp with are arguments and suppositions as we cut down our rationalizations and false beliefs.

Quoting Wayfarer
Logical propositions and their truth values are abstract entities that exist independently of any specific physical realization, such as brain states.


No, they aren't. There always has to be something to process those logical proposition and truth values. It doesn't exist in a vacuum. If there is nothing, there is no logic Wayfarer. We are the brains abstracting these identities. No brains, no abstract identity of logic. Apart from brains, does such logic just float out there? Where is it if it is not in the brains of logically capable thinking beings?

Quoting Wayfarer
I could choose to represent it and any number of different propositions in different symbolic systems and different media, whilst still preserving the logic.


And what is doing this thinking? Your brain.

You seem to take the argument like this: "My brain's physical capabilities let me think of abstracts and logic and rationality. Therefore such things exist apart from the physical capabilities that my brain produces. Its a contradiction Wayfarer. Go get drunk and watch logic disappear. Look at a brain damaged individual and see how they process.

Quoting Wayfarer
I think consideration of the role of networks of neurons, and disregarding the molecular details on which the neurons supervene, is an appropriate level of looking at things for the purpose of this discussion
— wonderer1

It might be, were this a computer science or neuroscience forum.


Here you are also mistaken. The best philosophers of history were often times mathematicians and scientists as well. Philosophy has to discuss the material that we know of today, or it is an exercise in futility. You cannot discuss the philosophy of mind without neuroscience. That is a person who is in the dark ages and will be left behind. Why isn't neuroscience looking to arguments such as your Wayfarer? Because they offer nothing. They're wrong. Its not that neuroscience is full of itself and can't comprehend what you're saying. They do. And its so off base as to be brushed aside without a second thought.

I've said this before, and I'll say it again. Poor philosophy wonders at what could be. Great philosophy wonders at what is, and attempts to solve it. But we have to address what we know, not ignore it for our ideology.
wonderer1 June 22, 2023 at 02:37 #816890
Wayfarer June 22, 2023 at 02:42 #816892
Quoting Philosophim
Are the countless neuroscience discoveries, medicine, psychiatrics, etc. all just correlations? Of course not.


But they don't entail what you say they entail. Have you ever encountered the book The Philosophical Foundations of Neuroscience, by Hacker and Bennett? Can you say anything about that? Up until I mentioned the term 'brain-mind identity theory' had you ever heard it? Do you know who those philosophers are that I mentioned, and what they say?

From my perspective, everything you write on the forum comprises wholly and solely what Philosophim thinks is obvious, accompanied by a strong sense of indignation that someone else can question what, to you, are obvious facts. This is your response to everything I address to you.

Quoting Philosophim
We're in philosophy.


Have you ever written a term paper in philosophy? Ever actually studied it? Because I can see no indication of that. You never mention any philosophers, or philosophical arguments, apart from your belief that everything is matter-energy and the mind is the product of the brain, which, to you, is obvious.
Wayfarer June 22, 2023 at 03:09 #816898
Quoting I like sushi
Any ‘theory’ that is given will necessarily be one that is ‘physical’/‘material’.

Wayfarer: On further thought, I’m rather intrigued by why you would say that, and why it appears obvious to you.

I like sushi - Because a theory only has meaning if it can be tested.


Right - but that only applies to empirical theories, which the argument from reason is not. It's closer in nature to Kant's transcendental theories (i.e. given that we know X, what must be the case in order that we know it.)

Quoting Srap Tasmaner
Is there an argument from "because" having two senses to there being two realms, one ruled by Physics or Something and one ruled by Reason or Something? If that's even what we're going for.


That is exactly what is at issue. There's a detailed discussion of this issue in this .pdf file, The Argument from Reason, Victor Reppert, Pp 356- . I won't try and re-state it again, but it is distinguishing between physical cause-and-effect and logical ground-and-consequence, and saying that the latter can't be reduced to, or explained in terms of, the former.
wonderer1 June 22, 2023 at 04:28 #816905
Quoting Wayfarer
I think consideration of the role of networks of neurons, and disregarding the molecular details on which the neurons supervene, is an appropriate level of looking at things for the purpose of this discussion
— wonderer1

It might be, were this a computer science or neuroscience forum.


It's disappointing to see such anti-intellectualism here. Especially in light of Fooloso4 having so recently posted this.

Quoting Fooloso4
Philosophy has become in large part insular and self-referential. Written by philosophers for philosophers. With a specialized language designed only for the initiated, a cramped style of writing intended to ward off attack, overburdened by its own theory laden stranglehold on thinking and seeing, enamored by its linguistic prowess and the production of problems that only arise within this hermetically sealed sterile environment. It either laments the fact that it is regarded as irrelevant or takes this to be the sign of its superiority.


Wayfarer June 22, 2023 at 04:33 #816906
Quoting wonderer1
It's disappointing to see such anti-intellectualism here.


It's not 'anti-intellectualism'. You haven't actually addressed the topic of the OP - what you've tried to do, is shift the discussion to discussion about neural networks and evolutionary psychology. Perhaps you could demonstrate how viewing the question in those terms, casts light on the basic contention, which is about the relationship of physical causation and logical necessity, and the sense in which rational inference can or can't be reduced to, or understood in terms of, the physical sciences.

Wayfarer June 22, 2023 at 04:36 #816910
Reply to wonderer1 And incidentally, see my response to the very post that you reference. You think that response is also 'anti-intellectual'?
I like sushi June 22, 2023 at 04:55 #816915
Quoting wonderer1
What are your thoughts on replacing "true" and "false" with "more accurate" and "less accurate"?


No problem whatsoever. You would then need evidence though and come to the conclusion that the claim is nonsense because there is no possible non-physical or non-material evidence.
I like sushi June 22, 2023 at 05:05 #816919
Quoting Wayfarer
This really makes no sense. Again the argument is about the means by which reasoned inference may result in true beliefs. And any argument which has to place reality in scare quotes ought to be looked at askance.


Logic is abstract. Reality is not. Any abstract argument should be applied to reality with care. Non-natural and non-material are nothing/other. If there is some other means we have yet to bring it into the light.
Wayfarer June 22, 2023 at 05:15 #816920
Quoting I like sushi
Logic is abstract. Reality is not. Any abstract argument should be applied to reality with care.


Modern science has gotten many spectacular results from the interpretation of data in the light of mathematical abstractions. That is the subject of Eugene Wigner's famous paper, The Unreasonable Effectiveness of Mathematics in the Natural Sciences. An excellent illustration is Dirac's discovery of anti-matter, which was formulated purely on the basis of mathematical projection, and not empirically validated until years later.
I like sushi June 22, 2023 at 06:24 #816926
Reply to Wayfarer Validation is kind of important. If it was not validated then what. If something happens it happens and if it does not it does not. The logic and rationality behind it are determiners but they are useful guides.

If your position that Rationality exists in some supernatural realm that is fine. Logic is flawless in an abstract realm becuase it is discrete. Reality may or may not map 1 to 1 onto some logical principles or it may not.

Any such arguments always fall sort as we are limited. I will state though that we know things due to experience (including rationalism).
Tom Storm June 22, 2023 at 06:30 #816927
Quoting wonderer1
Edit: I forgot to answer your last question. I don't have a clear idea of what you are asking with your question, but what I see it as adding to the discussion, is further consideration and clarification of the paradigm I'm presenting.


No worries. I guess where I was heading is that if animals have rudimentary intentionality, what does this say about a more evolved human version? Is intentionality just a hallmark of complexity (an idea mocked by many). @Wayfarer argues that human rationality and intentionality is special. He's not the only one. Can we infer anything additional about this matter from understanding animal behaviour?
waarala June 22, 2023 at 08:19 #816947
Reply to Joshs

Nietzsche had his own theories how the world functions. I think his extremely cynical views represent biologism. Or that the world becomes "fatally" ordered or disordered through the battle of strong and weak ones.
wonderer1 June 22, 2023 at 10:38 #816956
Delete
introbert June 22, 2023 at 10:39 #816957
Reply to waarala thats classical canon aswell strong is alazon and weak is eiron. Alazon makes a bunch of deceptive truth claims that people believe that give him power, the eiron is underdog thay must transend using a body that stresses from mental exertion.
wonderer1 June 22, 2023 at 13:15 #816973
Quoting Tom Storm
No worries. I guess where I was heading is that if animals have rudimentary intentionality, what does this say about a more evolved human version? Is intentionality just a hallmark of complexity (an idea mocked by many).


I would mock that idea as well. It's not simply a matter of degree of complexity. I think that only physical systems with outputs, that are about some aspect of their inputs have intentionality.

I strongly encourage watching this video (or otherwise look into the subject) in order to develop an intuitive recognition of how networks of neurons can be systems with outputs that are about some aspect of the system's input. For me, understanding this stuff has been of immense benefit in understanding intuition, when to trust and distrust my intuitions, and how to improve my intuitions. Understanding the fine details of what is discussed in the video isn't so important for the context of this thread, as developing a recognition of how intentionality can emerge in what is a relatively simple system.

Quoting Tom Storm
Wayfarer argues that human rationality and intentionality is special. He's not the only one. Can we infer anything additional about this matter from understanding animal behaviour?


Well, a substantial amount has been learned about human brains from studying animal brains, but if "this matter" is the human capacity for philosophical thought, then I'd think understanding of animal behavior would be of limited usefulness. (Now if "this matter" was the behavior of humans on a philosophy forum, much might be learned by watching a documentary on chimpanzees.)

Anyway, in response to your question I'm at an I don't know where to begin state, so I'll defer to Kurt Vonnegut:

Tiger got to hunt, bird got to fly;
Man got to sit and wonder 'why, why, why? '
Tiger got to sleep, bird got to land;
Man got to tell himself he understand.

NOS4A2 June 22, 2023 at 16:00 #817000
The fallacy of division reminds us that that which true for a whole needn’t be true of its parts. So a vast combination of non-intentional and non-purposeful processes could form an intentional and purposeful being.

Reasoning, forming beliefs, making logical inferences, seeking to arrive at a true understanding—these are the activities of a human being.

Human beings are physical and chock-full of neurochemicals and synapses. Human beings are often described as intentional and purposeful. They are rational.

Since human beings are physical, and their actions are often intentional and purposeful and rational, beliefs are inferred by both physical and rational causes.

So beliefs can still be fully explained in terms of physical causes, and naturalism remains true.

Gnomon June 22, 2023 at 16:11 #817004
Quoting Tom Storm
I'm not convinced we know what is random versus that which is not random. We detect patterns, as far as human cognition allows and we ascribe characteristics to those patterns - again in human terms. But words like 'random' or 'accidental' seem to have emotional connotations and function as tips of icebergs.

I suppose you are referring to the problem of determining if a string of numbers is random. In judgments of randomness, there is always a degree of doubt. Statistical analysis is inherently limited to probabilities instead of certainties*1. But I was talking about Philosophy, not Mathematics. For philosophical purposes, we routinely make judgements about Necessity vs Chance. I don't know about animals, but human nature seems to have an innate sense of Order vs Disorder. And, of course, there may be emotional reactions in those faced with Orderly/Predictable vs Disorderly/Unpredictable situations.

But this is a calm reasonable intellectual philosophy forum --- no heretics in dungeons --- so what I'm talking about is the Logical Connotations of an Ontological question : " Is the universe a self-organizing self-learning Program, or a random sequence of accidents". If the universe is a series of accidents, going nowhere, then the project of Science is impossible*2. But, if there is at least some perceptible order within background randomness, the project of Philosophy --- to make sense of the world --- is reasonable*3. For now, you can ignore the "self-learning" interpretation of some observers. We can get into that later.

For this post, my question to you is this : do you think the universe is -- on the whole -- A> organized (lawful, predictable) or B> disorganized (lawless, unpredictable)? Are you able --- can you convince yourself --- to make such a philosophical generalization? Caution, your answer may have emotional implications. I'm not asking you to go on record though ; it's just you and me here. Are you afraid to make such a summary judgment of the historical patterns of evolutionary development over 14 billion years? :smile:


*1. "The Unreasonable Effectiveness of Mathematics in the Natural Sciences" is a 1960 article by the physicist Eugene Wigner. In the paper, Wigner observes that a physical theory's mathematical structure often points the way to further advances in that theory and even to empirical predictions. ___Wikipedia

*2. Nature of Science :
Scientific Knowledge Assumes an Order and Consistency in Natural Systems. Science assumes that objects and events in natural systems occur in consistent patterns that are understandable through measurement and observation.
https://www.shapeoflife.org/nature-science-scientific-knowledge-assumes-order-and-consistency-natural-systems

*3. Laws of Nature :
Within metaphysics, there are two competing theories of Laws of Nature. On one account, the Regularity Theory, Laws of Nature are statements of the uniformities or regularities in the world; they are mere descriptions of the way the world is. On the other account, the Necessitarian Theory, Laws of Nature are the “principles” which govern the natural phenomena of the world. That is, the natural world “obeys” the Laws of Nature. This seemingly innocuous difference marks one of the most profound gulfs within contemporary philosophy, and has quite unexpected, and wide-ranging, implications.
https://iep.utm.edu/lawofnat/
Philosophim June 22, 2023 at 16:28 #817011
Quoting Wayfarer
Are the countless neuroscience discoveries, medicine, psychiatrics, etc. all just correlations? Of course not.
— Philosophim

But they don't entail what you say they entail. Have you ever encountered the book The Philosophical Foundations of Neuroscience, by Hacker and Bennett?


Of course they entail what they entail. All you have to do is show that brain death and a lack of mind are not a correlate. All you have to do is demonstrate how when neuroscientists analyze the brain, they can predict accurately what a person will think or say next up to 10 seconds before they say it. If my points are so easy to counter, then you should be able to easily give a counter to them. Citing a book vaguely does nothing. That's an appeal to authority, not an argument.

Quoting Wayfarer
From my perspective, everything you write on the forum comprises wholly and solely what Philosophim thinks is obvious, accompanied by a strong sense of indignation that someone else can question what, to you, are obvious facts. This is your response to everything I address to you.


And yet if they were not obvious facts, you would be able to counter them easily wouldn't you? Instead you retreat and answer with things like: Quoting Wayfarer
Have you ever written a term paper in philosophy? Ever actually studied it? Because I can see no indication of that.


This is someone who is insecure about their own intelligence. Don't be Wayfarer. You're a smart person. But a smart person should not be so easily caught up in their own ego. Its a poison trap of smart people to think that "If I just read a bunch of papers and cite them, people will think I'm smart." You have knowledge, but you seem unable to critically think about that knowledge when its challenged from a new perspective. Thus you retreat. I call this out so that you'll attempt to improve Wayfarer instead of getting haughty and making poor appeals to authority.

Here's the truth. It doesn't matter what the background of someone is in philosophy. It matters if you can think logically, critically, and honestly. You attempting to put up barriers when you're countered is unbecoming. If you must know, am I formally educated? Yes. Am I intelligent? Objectively yes. I do not post my background as a "flex" because I don't want people to just agree with me for the wrong reason. The arguments I give should stand on their own, as should yours. Eliminating such inconsequential considerations such as "status" lets us get right to the arguments instead of our egos.
Gnomon June 22, 2023 at 17:15 #817016
Quoting Wayfarer
I am dissappointed, but never surprised, to observe the routine deprecation of the faculty of reason. I think the classical notion of reason is rather non-PC, for various reasons, chief among them that it distinguishes humans from other species.

I suppose, in order to avoid the historical slavery of political/religious Spiritualism (soul more important/essential than body, and ideals worth dying for), Materialism has gone to the opposite extreme : a mundane real body without a spooky ideal mind ; hence, free-range animals with guns & computers instead of teeth & claws.

However, my interest in philosophy/science is that it allows us to do what animals can't : to know thyself. A bit of introspection can make us both proud of human culture, and ashamed of its imperfections. We may be almost indistinguishable from animals in our biology, but human psychology allows us to use tools for leverage to move the world.

Yet, when ungoverned by Reason, those tools can turn us into blood-thirsty savages. "Guns don't kill people; People with guns, knives, tanks, missiles kill people". Often for irrational reasons : e.g. Putin's political dreams of a glorious ideal empire justify Ukrainocide. Aren't humans distinguished!

Does Materialism/Physicalism inherently turn us into secular humanists & pacifists? Is there a philosophical middle ground, where physical bodies & metaphysical minds can coexist? :worry:

Quoting Wayfarer
Again, take a look at the chapter headings and abstracts (all available online) of Mind and the Cosmic Order, Charles Pinter. He has a compelling answer to at least part of this question.

I just ordered a copy of the book from Amazon. It seems to address some of the common sticking points on this forum. I'm guessing that he leans toward a Platonic worldview, but I'll try to remain open-minded. :smile:

RogueAI June 22, 2023 at 17:42 #817021
Quoting Philosophim
That's an appeal to authority, not an argument.


That's a copout. We cite books and philosophers in discussions here constantly. It's not a fallacy in informal discussions if the authority is a valid one.
RogueAI June 22, 2023 at 17:44 #817022
Quoting Philosophim
Of course they entail what they entail. All you have to do is show that brain death and a lack of mind are not a correlate. All you have to do is demonstrate how when neuroscientists analyze the brain, they can predict accurately what a person will think or say next up to 10 seconds before they say it. If my points are so easy to counter, then you should be able to easily give a counter to them.


Would any of that be different if this were all a dream?
Philosophim June 22, 2023 at 18:06 #817025
Quoting RogueAI
That's an appeal to authority, not an argument.
— Philosophim

That's a copout. We cite books and philosophers in discussions here constantly. It's not a fallacy in informal discussions if the authority is a valid one.


No, citing a book without any specific arguments from the book is an appeal to authority. If an argument from the book had been presented to counter my point, that would have been fine.

Quoting RogueAI
Of course they entail what they entail. All you have to do is show that brain death and a lack of mind are not a correlate. All you have to do is demonstrate how when neuroscientists analyze the brain, they can predict accurately what a person will think or say next up to 10 seconds before they say it. If my points are so easy to counter, then you should be able to easily give a counter to them.
— Philosophim

Would any of that be different if this were all a dream?


Can you prove that this is all a dream? That's like saying "Would it all be different if we were all made out of cotton candy?" Its a fun thing to explore, but without providing an argument that we are in fact, made out of cotton candy, its not an argument worth considering in a discussion of facts. It is not a correlation or supposition that the mind comes from the brain. It is a scientific fact. Not that a fact cannot be overturned, or we can't suppose there's more out there than we currently know. If you're going to say a fact is wrong, you need to prove it.
RogueAI June 22, 2023 at 18:11 #817026
Quoting Philosophim
Can you prove that this is all a dream? That's like saying "Would it all be different if we were all made out of cotton candy?" Its a fun thing to explore, but without providing an argument that we are in fact, made out of cotton candy, its not an argument worth considering in a discussion of facts.


I can't prove it's all a dream. I'm simply asking you if all the science that's been done would necessarily be any different if all this was a dream. Would it?

Also: we dream and create worlds every night. Your cotton candy example is absurd. Idealism is taken seriously by people and philosophers. It can't be dismissed with a hand-wave.
Philosophim June 22, 2023 at 18:23 #817029
Quoting RogueAI
I can't prove it's all a dream. I'm simply asking you if all the science that's been done would necessarily be any different if all this was a dream. Would it?


I don't know. You're asking about a fictional reality. We can't make judgements about fictional realities, because they're fictional. Can we create a fictional reality where we decide science is different? Sure. Can we create a fictional reality where we decide science is the same? Sure. Its fiction, so there are no limits on what we can do.
RogueAI June 22, 2023 at 18:27 #817030
Quoting Philosophim
I don't know. You're asking about a fictional reality. We can't make judgements about fictional realities, because they're fictional. Can we create a fictional reality where we decide science is different? Sure. Can we create a fictional reality where we decide science is the same? Sure. Its fiction, so there are no limits on what we can do.


OK, so all the neuroscience that's been done is consistent with an idealistic reality. Why should I then believe that the prima facie neural causation model that you champion is actual causation? I would if the model you describe could explain how things are conscious and why consciousness is present at all, but materialism/physicalism/naturalism has utterly failed to solve the mind-body problem. How long are we going to put up with that failure before we start to explore new theories? What if the mind-body problem is still around 1,000 years from now? At what point do you start to question your metaphysical assumptions?
Tom Storm June 22, 2023 at 19:25 #817041
Quoting Gnomon
I suppose you are referring to the problem of determining if a string of numbers is random.


No, I was talking about how things seem to us as opposed to how they might really be. When we talk about order, it is based on our models of what order appears to be to us.

Quoting Gnomon
For this post, my question to you is this : do you think the universe is -- on the whole -- A> organized (lawful, predictable) or B> disorganized (lawless, unpredictable)?


My point is simple. How would we know? We seem to have discovered some regularities in our little patch. We can claim no such knowledge about the whole universe. I'm not even certain physics works the same across the universe - what's to say it isn't largely a function/invention of human cognition?

Philosophim June 22, 2023 at 19:45 #817044
Quoting RogueAI
OK, so all the neuroscience that's been done is consistent with an idealistic reality. Why should I then believe that the prima facie neural causation model that you champion is actual causation?


Some of neuroscience is almost certainly idealistic, in which case idealistic philosophy has free reign. But the fact that the mind comes from the brain is not idealistic, it is decades of research and experiments that continue to confirm this as a fact. From brain surgery, anesthesia, brain damage research, psychadelics, and psychiatric medicine there are a host of things to choose from. If the brain did not cause the mind, then all of these fields which rely on this fact, would have catastrophic failure rates and be no more than charlatans.

You can even test it on yourself. Go get drunk tonight and see how it affects your mind. That is due to the alcohol impacting your brain. Its an extremely simple test to confirm for yourself while having a little fun.

Quoting RogueAI
I would if the model you describe could actually explain how things are conscious and why consciousness is present at all, but materialism/physicalism/naturalism has utterly failed to solve the mind-body problem.


If you are talking about certain details of consciousness, of course we don't understand everything yet. For example, we'll never know what its like to exist as a brain from the subjective viewpoint of the brain. That's outside of our measurement. But we can most certainly impact consciousness by manipulating the brain. Anesthesia knocks you unconscious for surgerys. You think that's all just a happy accident? That's all based on the fact that brain affects the mind, and anesthesia affects the brain in a particular manner.

Don't confuse not fully mapping out the brain with being unable to make certain conclusions about the brain. We're trying to reverse engineer the brain's specifics, but we have overall conclusions about how it works that have continually held up to tests and critiques. If you reversed engineered a car, you might not understand how magnetism works, but you could understand the parts of the car and how they interact. The car does not run if the engine is not active, despite not knowing all the details on how gas combustion causes the engine to run.

Quoting RogueAI
How long are going to put up with that failure before we start to explore new theories? What if the mind-body problem is still around 1,000 years from now? At what point do you start to question your metaphysical assumptions?


You misunderstand. You can always question and wonder at alternatives. I can sit and ponder that all of physics is somehow a big misunderstanding. I can have a lot of fun coming up with other theories. But those are all suppositions, untested, and non-factual. None of those override facts themselves. If I said my crazy physics alternative fixed everything with physics, but I could not adequately demonstrate this, I would be a fraud.

So have fun with idealism. Say, "What if...?" Explore and think on alternatives. But until there is something factually substantial behind those musings, it is entirely inappropriate to say they counter known facts.
RogueAI June 22, 2023 at 19:51 #817046
Quoting Philosophim
You can always question and wonder at alternatives.


The alternatives gain more credibility as the mainstream theories fail to explain observations. We see this already with Modified Newtonian dynamics gaining more ground as the mystery of dark matter continues to be unresolved. If the Hard Problem is still around 1,000 years from now, it will be devastating for materialism/physicalism. You're already seeing a resurgence in idealism and panpsychism because of it.
Philosophim June 22, 2023 at 20:50 #817051
Quoting RogueAI
If the Hard Problem is still around 1,000 years from now, it will be devastating for materialism/physicalism.


Not at all. The only viable version of the hard problem is it stands today is that we cannot know what another subject is experiencing from that subjects viewpoint. We could take two subjects and stimulate identical brain states to where they both said, "I see a green tree." We could never independently verify what that green tree looked like specifically to subject 1 or 2. No one can. To my mind, there's no theory that ever could either. This in no way invalidates the fact that brain causes the mind.
RogueAI June 22, 2023 at 20:53 #817052
Quoting Philosophim
The only viable version of the hard problem is it stands today is that we cannot know what another subject is experiencing from that subjects viewpoint. We could take two subjects and stimulate identical brain states to where they both said, "I see a green tree." We could never independently verify what that green tree looked like specifically to subject 1 or 2. No one can. To my mind, there's no theory that ever could either.


That's not the only viable problem. How does consciousness arise from matter? Why is consciousness present at all? Why are only certain arrangements of matter conscious?

If these questions are still unanswered after 1,000 years, no will believe in materialism. Why would they? It will have failed to answer some of the most basic questions.
Philosophim June 22, 2023 at 20:56 #817054
Quoting RogueAI
That's not the only viable problem. How does consciousness arise from matter? Why is consciousness present at all? Why are only certain arrangements of matter conscious?

If these questions are still unanswered after 1,000 years, no will believe in materialism. Why would they? It will have failed to answer some of the most basic questions.


Those are easy problems, not hard problems. Easy and hard do not denote their difficulty in finding a solution, but their difficulty in finding a path to a solution at all. The hard problem I noted has no pathway to a solution. Your questions have clear pathways of investigation and testability. Considering the amount of progress we've made in just the last 30 years, there seems to be no reason to alter course for the next 30, let alone 1000.

Something else to think about, but your questions can equally be applied to almost any other state of matter. How does water arise from H20? Why is water a possible existence at all? Why are only certain arrangements of atoms water while others are not? We know that water is made out of molecules, and consciousness comes from the brain, but there are still deeper questions that we continue to look into.

Still a lot to discover!
RogueAI June 22, 2023 at 21:25 #817056
Quoting Philosophim
Those are easy problems, not hard problems.


I don't think so. I think we'll see science continue to flounder, and it will reach a crisis when the Ai's start operating at human levels, which won't be too long from now, and science will have no answer to the question everyone will be asking: are these things conscious?

But maybe you're right and there will be a breakthrough soon. Then you can resurrect this and laugh at me, but I don't think that's going to happen.
Joshs June 22, 2023 at 21:26 #817057
Reply to waarala Quoting waarala
Nietzsche had his own theories how the world functions. I think his extremely cynical views represent biologism. Or that the world becomes "fatally" ordered or disordered through the battle of strong and weak ones.


Deleuze, Foucault and Heidegger were profoundly sly influenced by Nietzsche’s ideas. None of them would label his views biologism. Instead, in their readinga he offers an overturning of Platonic metaphysics by placing difference as prior to identity.
Gnomon June 22, 2023 at 21:29 #817058
Quoting Tom Storm
No, I was talking about how things seem to us as opposed to how they might really be. When we talk about order, it is based on our models of what order appears to be to us.

Sounds like you are being evasive. Barring divine revelation, how else would we know anything about the world, except as they "seem to us" : via our senses & inferences? And how they seem is what our mental models tell us. Is your seemly model/map of the world orderly enough for us to understand it and discuss it, or disorderly enough to keep us forever in the dark about ultimate philosophical questions? As the OP inquired : do we humans possess " the ability to either genuinely apprehend truth, or to be rationally justified in making truth claims". It's not a trick question : do you find the world orderly enough for you to find your way around the local terrain, and to draw inferences about its wider patterns of Geology*1? :smile:

*1. Geology : "the science that deals with the earth's physical structure and substance, its history, and the processes that act on it".


Quoting Tom Storm
My point is simple. How would we know? We seem to have discovered some regularities in our little patch. We can claim no such knowledge about the whole universe. I'm not even certain physics works the same across the universe - what's to say it isn't largely a function/invention of human cognition?

Are you claiming complete ignorance about the world, or just "profound skepticism"? Is mathematics simply a child's game of counting fish? Or a science that allows us to guess about what happens next, and what happened before. Kant was skeptical about our ability to know what's what, but despite that handicap, he wrote thousands of words to instruct us about the positive & negative aspects of Epistemology.

On this forum, few of us claim to speak from absolute authority. We just share personal opinions/models, and that's how we expand & refine our "little patch" of reliable knowledge. By comparing our worldviews, we may learn what ideas are imaginary "inventions", and which are realistic enough to be reliable "knowledge". :nerd:

Epistemological rationalism :
Humans will always find things arranged in certain patterns because it is they who have unwittingly so arranged them. Kant held, however, that these certainties were bought at a heavy price. Just because a priori insights are a reflection of the mind, they cannot be trusted as a reflection of the world outside the mind. Whether the rational order in which sensation is arranged—the order, for example, of time, space, and causality—represents an order holding among things-in-themselves (German Dinge-an-sich) cannot be known. Kant’s rationalism was thus the counterpart of a profound skepticism.
https://www.britannica.com/topic/rationalism/Epistemological-rationalism-in-modern-philosophies#ref561225
Philosophim June 22, 2023 at 21:38 #817060
Quoting RogueAI
But maybe you're right and there will be a breakthrough soon. Then you can resurrect this and laugh at me, but I don't think that's going to happen.


No, I wouldn't laugh at you RogueAI. Just want to clarify this isn't a ego thing or jeering in any way. Please continue to have a fascination for alternatives than the status quo!
Tom Storm June 22, 2023 at 21:49 #817067
Quoting Gnomon
On this forum, few of us claim to speak from absolute authority. We just share personal opinions/models, and that's how we expand & refine our "little patch" of reliable knowledge.


Quoting Gnomon
Kant was skeptical about our ability to know what's what, but despite that handicap, he wrote thousands of words to instruct us about the positive & negative aspects of Epistemology.


I connect these two quotes because no one here is Kant or seems to have his prodigious capacities.

I am not being evasive, simply mildly incredulous at the claims we sometimes make about 'reality'. This is a legitimate view some philosophers arrive at. And yes, I am a skeptic.

There is no God’s Eye point of view that we can know or usefully imagine; there are
only the various points of view of actual persons reflecting various interests and
purposes that their descriptions and theories subserve.


- Hilary Putnam, Reason, Truth and History,
Gnomon June 22, 2023 at 22:20 #817074
Quoting Tom Storm
?wonderer1
Do animals have intentionality? They seem to from my perspective. What does this add to the discussion?

What did you mean (intend) by that question? :joke:

Courts of Law often spend thousands of attorney hours in trying to prove or deny Intention --- after the fact. But during an action, the intent is fairly obvious to the human mind. We seem to have a talent for interpreting intentions, such as stalking behavior. For example, if we see a cheetah approaching an antelope, crouching slowly, hairs raised, ears forward & eyes fixed, it could be just playing, or it could be intent on murder. Likewise, Nature --- as a whole system --- seems to display intentional patterns of behavior, that can be rationalized into a purposive, meaningful, goal oriented, worldview. But proving it, after the fact, is arduous.

What does that ability to interpret behavior add to a discussion about Rationality? For humans the innate ability to recognize patterns can be enhanced by the addition of Rational analysis of the situation, as in the courtroom example. Reason allows humans to make fine distinctions that may not be apparent to an animal. If you point a gun at an antelope, it may not interpret your intentions as murderous. Artificial/cultural elements of the modern world require reason to enhance instinct. That may be why some exhausted thinkers idealize a return to a "state of nature" where arduous & fallible reasoning & argumentation is not required for survival.

The intention of the OP, seems to argue that rational humans are not mere instinctive animals. Hence more than just aggregations of atoms & tangles of neurons. It's that little extra immaterial essence --- je nais se quoi --- that distinguishes human nature from animal nature. :smile:

WHAT ARE THE ANIMAL'S INTENTIONS?
User image

Reply to Wayfarer
Wayfarer June 22, 2023 at 23:32 #817086
Quoting wonderer1
Assuming you are using "intentionality" as discussed in the SEP...


Which states that:

In philosophy, intentionality is the power of minds and mental states to be about, to represent, or to stand for, things, properties and states of affairs. To say of an individual’s mental states that they have intentionality is to say that they are mental representations or that they have contents.


So I can't see how your proposed definition:

Quoting wonderer1
I think that only physical systems with outputs, that are about some aspect of their inputs have intentionality.


squares with what is given in the SEP article. You've already suggested a couple of times that ChatGPT might possess intentionality, which in both cases, ChatGPT itself has rejected. Besides, when you mention neural networks or artificial intelligence, you do so precisely because of what they represent: you are saying that they represent the way in which physical systems are able to embody intentionality. So again your argument is recursive - you are imputing intentionality to those systems on the basis of your rational ability to draw reasoned conclusions, which is the very faculty that is in question.

Quoting Gnomon
The intention of the OP, seems to argue that rational humans are not mere instinctive animals. Hence more than just aggregations of atoms & tangles of neurons. It's that little extra immaterial essence --- je nais se quoi --- that distinguishes human nature from animal nature.


As noted, C S Lewis was arguing as a believer. My motivation is different - whilst I'm not atheistic, I also have no intention of evangalising for belief in God. My view is that the commonly-held materialism of secular culture is something like a popular mythology, an aggregation of views that 'everyone knows' must be true (science says so!) But that many of the elements of that worldview can be called into question by philosophical analysis.

In regards the question of the nature of 'intention', the following is glossed from the essay by Victor Reppert, where he defines what exactly is being called into question by the argument.

Basic assumptions of materialism

First the materialist worldview presumes a mechanistic base level. This doesn't mean necessarily deterministic - there can be chance at the basic level of reality in a mechanistic worldview (e.g. Heisenberg's uncertainty principle and the 'quantum leap'.) However at the level of basic physics, nature is free of purpose, free of meaning or intentionality, free of normativity, and absent of any and all forms of subjectivity. If one is operating within a materialistic framework, then one cannot attribute purpose to what happens at the basic level. Talk of purpose may be appropriate for macrosystems, such as animals and humans, but that is a purpose that is ultimately the product of a purposeless basic physics. This is the level at which the role of intentionality in nature is to be debated.

Furthermore, 'what something means' cannot be an element of reality at the basic level. Meaning is only ever imputed by subjects, and subjects don't exist on that level - so the Universe, absent subjects, is also devoid of meaning.

There is nothing normative in basic physics. We can never say that some particle of matter is doing what it is doing because it ought to be doing that. Rocks in a landslide do not go where they go because it would be a good idea to go there. To quote Wittgenstein, 'in the world everything is as it is and happens as it does happen. In it there is no value—and if there were, it would be of no value. If there is a value which is of value, it must lie outside all happening and being-so. For all happening and being-so is accidental.' (TLP 6.41)

Basic physics is lacking in subjectivity. The basic elements of the universe have no 'points of view,' and no subjective experience (this is the point which is disputed by panpsychism). Consciousness, if it exists, must be a 'macro' feature of basic elements massed together. This is presumed to be where evolutionary biology enters the picture - by providing the mechanism through which simple elements are combined so as to give rise to the emergence of consciousness.

The level of basic physics must be causally closed. That is, if a physical event has a cause at time t, then it has a physical cause at time t. Even that cause is not a determining cause; there cannot be something nonphysical that plays a role in producing a physical event. If you knew everything about the physical level (the laws and the facts) before an event occurred, you could add nothing to your ability to predict where the particles will be in the future by knowing anything about anything outside of basic physics.

Finally whatever is not physical, at least if it is in space and time, must supervene on the physical. Given the physical, everything else is a necessary consequence, including minds and purposive behaviour.

In summary, the world is at bottom a mindless system of events at the level of fundamental particles and fields, behaving in the manner described by physical laws, and everything else that exists must exist consequentially to what is going on at that basic level. This understanding of a broadly materialist worldview is not a tendentiously defined form of reductionism; it is what most people who would regard themselves as being in the broadly materialist camp would agree with, a sort of “minimal materialism.” Reppert also maintains that any worldview that could reasonably be called “naturalistic” is going to have these features, and the difficulties that the 'argument from reason' presents against a “broadly materialist” worldview thus defined will be typical of naturalism insofar as it maintains these materialist tenets.

Incidentally, for anyone who is up for rather a long (i.e. >10,000 word) read, I've found an excellent essay by non-reductionist biologist Stephen Talbott, From Physical Causes to Organisms of Meaning, which discusses in depth and details how to get from the 'blind, non-purposive' causes of physics to the plainly purposive behaviours which animate the entire organic domain.





RogueAI June 22, 2023 at 23:42 #817091
RogueAI June 22, 2023 at 23:47 #817093
Quoting Wayfarer
In summary, the world is at bottom a mindless system of events at the level of fundamental particles and fields, behaving in the manner described by physical laws, and everything else that exists must exist consequentially to what is going on at that basic level.


This is pretty straightforward, but even this will get bogged down quickly when someone asks, "What, exactly, is a particle?" The materialist/physicalist ontology is very...fluid.
wonderer1 June 22, 2023 at 23:49 #817094
Quoting Tom Storm
I'm not convinced we know what is random versus that which is not random. We detect patterns, as far as human cognition allows and we ascribe characteristics to those patterns - again in human terms. But words like 'random' or 'accidental' seem to have emotional connotations and function as tips of icebergs.


I'd say it takes some effort to find something in the world that looks like it might be truly random.

https://en.m.wikipedia.org/wiki/Hardware_random_number_generator

You are correct that "random" has emotional connotations for many. In the Christian environment of my childhood, with the belief that there was a God that was going to make everything right, and the feeling of security that comes with that - it can be unnerving to consider the possibility that there is a random aspect of some sort, to the world. I, being very much a science nerd, realized fairly early on that it looks an awful lot like there is a random aspect to reality. It's a somewhat controversial topic in physics. The Multi Worlds Interpretation of QM is considered a deterministic interpretation. I'm kind of partial to the MWI, but not because I have anywhere near the expertise needed to judge between interpretations. I find it relatively easy to 'picture' an MWI world, as compared to the worlds of other interpretations of QM, and that undoubtedly biases my view.

Anyway, after that longer than intended digression, I was curious as to whether you found the following excerpt from that link to be emotional?

In computing, a hardware random number generator (HRNG) or true random number generator (TRNG) is a device that generates random numbers from a physical process, rather than by means of an algorithm. Such devices are often based on microscopic phenomena that generate low-level, statistically random "noise" signals, such as thermal noise, the photoelectric effect, involving a beam splitter, and other quantum phenomena. These stochastic processes are, in theory, completely unpredictable for as long as an equation governing such phenomena is unknown or uncomputable. This is in contrast to the paradigm of pseudo-random number generation commonly implemented in computer programs.

This TLS accelerator computer card uses a hardware random number generator to generate cryptographic keys to encrypt data sent over computer networks.
A hardware random number generator typically consists of a transducer to convert some aspect of the physical phenomena to an electrical signal, an amplifier and other electronic circuitry to increase the amplitude of the random fluctuations to a measurable level, and some type of analog-to-digital converter to convert the output into a digital number, often a simple binary digit 0 or 1. By repeatedly sampling the randomly varying signal, a series of random numbers is obtained.




Wayfarer June 22, 2023 at 23:52 #817095
Quoting RogueAI
This is pretty straightforward, but even this will get bogged down quickly when someone asks, "What, exactly, is a particle?" The materialist/physicalist ontology is very...fluid.


Sure! It is now. That is why a lot of people will say that quantum physics itself has undermined materialism (and I'm one of them). There is a strong idealist school of thought amongst physicists. But materialism in popular culture hasn't caught up with that.
RogueAI June 23, 2023 at 00:01 #817099
Reply to Wayfarer It was all so simple before QM.
RogueAI June 23, 2023 at 00:10 #817101
Quoting wonderer1
I'm kind of partial to the MWI, but not because I have anywhere near the expertise needed to judge between interpretations. I find it relatively easy to 'picture' an MWI world, as compared to the worlds of other interpretations of QM, and that undoubtedly biases my view.


Doesn't MWI violate the old axiom: Entities must not be multiplied beyond necessity? Isn't the following pretty awkward?

"In the Everett version of the cat puzzle, there is a single cat up to the point where the device is triggered. Then the entire Universe splits in two. Similarly, as DeWitt pointed out, an electron in a distant galaxy confronted with a choice of two (or more) quantum paths causes the entire Universe, including ourselves, to split. In the Deutsch–Schrödinger version, there is an infinite variety of universes (a Multiverse) corresponding to all possible solutions to the quantum wave function. As far as the cat experiment is concerned, there are many identical universes in which identical experimenters construct identical diabolical devices. These universes are identical up to the point where the device is triggered. Then, in some universes the cat dies, in some it lives, and the subsequent histories are correspondingly different. But the parallel worlds can never communicate with one another."
https://thereader.mitpress.mit.edu/the-many-worlds-theory/
Wayfarer June 23, 2023 at 00:17 #817106
Reply to RogueAI Discussion of interpretations of quantum physics is a sure death-knell for any thread. Let's not get dragged into that rabbit-hole.

The topic is 'the argument from reason' and the sense in which rational inference is or is not explicable in naturalistic terms.
Tom Storm June 23, 2023 at 00:24 #817109
Quoting wonderer1
Anyway, after that longer than intended digression, I was curious as to whether you found the following excerpt from that link to be emotional?


If boredom is an emotional reaction, then yes. :wink: Sorry - I find any kind of technical writing (or descriptions of methodologies, etc) almost unreadable. I don't have the attention span. It's on me, I know...
wonderer1 June 23, 2023 at 00:28 #817111
Reply to RogueAI

As I said, I am biased, because for me MWI is a useful tool for conceiving of aspects of the world. I know that I am not going to study QM to the point of being an expert, so a consequence of that is that I'm likely to remain biased. At least until some breakthrough comes along. OK?
wonderer1 June 23, 2023 at 00:34 #817112
180 Proof June 23, 2023 at 00:34 #817113
Quoting wonderer1
So do we blame old Franz for creating all of this confusion?

Blame millennia of folk psychology (e.g. Scholastic psychologism).

Quoting Tom Storm
Next you'll be telling us qualia is nonsense...

Not "nonsense", just (adaptive?) phenomenal noise in mammalian cognitive systems.

Quoting Wayfarer
So how did this entry become written? By mistake?

It was written – as our exchanges are written, Wayf – by deterministic nonlinear dynamic system-agents which reflexively confabulate ex post facto intentions-of-the-gaps. :sparkle: :eyes:
Wayfarer June 23, 2023 at 00:36 #817114
Quoting 180 Proof
It was written by deterministic nonlinear dynamic system-agents


waffle, 180. Pure and simple.
RogueAI June 23, 2023 at 00:38 #817117
wonderer1 June 23, 2023 at 00:41 #817118
Quoting 180 Proof
It was written – as our exchanges are written, Wayf – by deterministic nonlinear dynamic system-agents which reflexively confabulate ex post facto intentions-of-the-gaps. :sparkle: :eyes:


:up:
180 Proof June 23, 2023 at 00:54 #817124
Reply to Wayfarer And yet, besides a disapproving squeak, you cannot point out my error/s. :roll:
Wayfarer June 23, 2023 at 01:00 #817126
Reply to 180 Proof Life is short, 180.

wonderer1 June 23, 2023 at 01:04 #817129
Quoting Wayfarer
So I can't see how your proposed definition:

I think that only physical systems with outputs, that are about some aspect of their inputs have intentionality.
— wonderer1

squares with what is given in the SEP article.


Yes, I know it would take a paradigm shift for you to get it. You would need to spend some time, studying stuff, that I suspect you would find boring, to reach the point of grasping what I am trying to communicate, about what is involved in understanding oneself.

You are interested in Buddhism, right? Are you familiar with the Zen notion of beginners mind?

Your teacup is full.
180 Proof June 23, 2023 at 01:32 #817133
Quoting Wayfarer
Life is short, 180.

And denial is long.
Wayfarer June 23, 2023 at 02:17 #817140
Quoting wonderer1
Yes, I know it would take a paradigm shift for you to get it.


Much simpler than that - the source you provided doesn’t support your meaning of the term ‘intentionality’.
Wayfarer June 23, 2023 at 02:18 #817141
Quoting wonderer1
You are interested in Buddhism, right? Are you familiar with the Zen notion of beginners mind?


Owned the first edition. Nothing whatever to do with the point at issue.
wonderer1 June 23, 2023 at 02:57 #817146
Quoting Wayfarer
...You've already suggested a couple of times that ChatGPT might possess intentionality, which in both cases, ChatGPT itself has rejected.


Wayfarer, it seems unlikely that I'll be able to keep up with your rate of posting, but I do want to respond to this much for now.

As I've said, there are different ways of understanding intentionality. Furthermore, you haven't been paying close attention. What I asked was (going from memory) "Does the output of ChatGPT have intentionality?" and in response to the way you responded to that, I asked, "Do you interpret the output of ChatGPT as having intentionality?" (As being about something.) Which you haven't answered.

The reason I asked was to get you thinking about the question. I think that you do interpret the output of ChatGPT as being about something, after all, you've said that you have been making use of it a lot lately. Why would you do that, if you didn't think that the output is about something?

Quoting Wayfarer
Besides, when you mention neural networks or artificial intelligence, you do so precisely because of what they represent: you are saying that they represent the way in which physical systems are able to embody intentionality. So again your argument is recursive - you are imputing intentionality to those systems on the basis of your rational ability to draw reasoned conclusions, which is the very faculty that is in question.


You misunderstand, I'm not trying to make an argument in talking about neural nets. I am presenting a hypothesis for consideration. It would take effort on your part, that you haven't shown a willingness to make, for you to grasp the hypothesis, which is fine.


wonderer1 June 23, 2023 at 03:04 #817148
Quoting Wayfarer
Owned the first edition. Nothing whatever to do with the point at issue.


I didn't asked if you owned the book. I asked, "Are you familiar with the Zen notion of beginners mind?"

Srap Tasmaner June 23, 2023 at 03:36 #817150
Reply to Wayfarer

(( Unlikely I'll slog through Victor Reppert, but if I do, I'll let you know. Probably will look at the Stephen Talbott. ))

Back to business.

What does human reasoning look like? Let's go back to the Lewis example. He wants to contrast (1) Grandpa sleeping in because he's sick from (2) my inferring that he's sick because he's sleeping in.

(1) asserts a causal relation between two states of affairs: Grandpa is sick causes Grandpa sleeps in; (2) is considerably more complicated. I am said to know or believe a couple things: (a) Grandpa is sleeping in; (b) Grandpa sleeps in if and only if he is sick. From these, I deduce that (c) Grandpa is sick.

Unlike Grandpa, whose sleeping in is caused by his being sick, my state of believing (or knowing) that Grandpa is sick is not caused by my beliefs (a) and (b); it is a free choice (or act?) of mine to believe that (c) on account of (a) and (b). (a) and (b) together entail (c), and I choose to align my beliefs with what is logical, and so hold (c). Nothing forces me to believe (c), and I could (perversely) do otherwise if I choose. As a matter of logic, (c) flows automatically from (a) and (b), but my holding (c) does not flow automatically from my holding (a) and (b).

That's the Lewis account of the basic situation, but that account is actually intended to be embedded into another argument that will include the free act of inference, something like this: if I am caused to believe something, then I have not freely inferred it; if I have not freely inferred my belief, then I cannot consider it rational, for only beliefs arrived at by the use of reason are rational.

Is that a fair account of the argument from reason as you understand it?
wonderer1 June 23, 2023 at 05:00 #817155
Quoting Wayfarer
Which states that:

In philosophy, intentionality is the power of minds and mental states to be about, to represent, or to stand for, things, properties and states of affairs. To say of an individual’s mental states that they have intentionality is to say that they are mental representations or that they have contents.


By the way Wayfarer,

The sentence immediately following what you quoted there from the SEP says:

Furthermore, to the extent that a speaker utters words from some natural language or draws pictures or symbols from a formal language for the purpose of conveying to others the contents of her mental states, these artifacts used by a speaker too have contents or intentionality.


Do you agree with that statement?



Wayfarer June 23, 2023 at 05:04 #817156
Quoting wonderer1
Do you agree with that statement?


Yes. Note 'used by a speaker'. They are on that sense imbued with intentionality, namely, that of the speaker, to convey or represent something.

Quoting wonderer1
The reason I asked was to get you thinking about the question. I think that you do interpret the output of ChatGPT as being about something, after all, you've said that you have been making use of it a lot lately. Why would you do that, if you didn't think that the output is about something?


As you know, ChatGPT is a human invention, and trained in large language models to anticipate responses to questions. So of course the answer will be about something, and will represent something. But then, it has been designed to do that, it is a human artefact, after all, which we use for our purposes. Humans engineered the system and then interpret the output. In that respect, it emulates elements of intentionality, but as I already noted, ChatGPT itself says 'AI systems, including ChatGPT, do not possess intentionality in the same way that humans do. Intentionality is typically associated with consciousness and subjective experience, which are currently not attributes of AI systems.'

So all due respect, I think you're missing the philosophical point at issue, which has to do with the nature of reason. So then you try to re-frame the debate, not in the terms in which it was given, but in terms which suit your argument. It's very like the old anecdote of the fellow looking under the lampost for his keys, knowing they've actually been lost elsewhere, 'because the light's better here'. :wink:

I re-iterated in this thread the elements of the materialist/naturalist view that Victor Reppert claims that the argument from reason is opposing. If you think that depiction of materialist philosophy is wrong, or if you think that they're about right, but the argument from reason fails to address them, then that kind of analysis is what is appropriate (pretty much as Srap Tasmaner does.)







Wayfarer June 23, 2023 at 05:12 #817157
Quoting Srap Tasmaner
Is that a fair account of the argument from reason as you understand it?


I think so, overall. I'm not entirely persuaded by your meta-analysis here:

Quoting Srap Tasmaner
Grandpa is sick is not caused by my beliefs (a) and (b); it is a free choice (or act?) of mine to believe that (c) on account of (a) and (b). (a) and (b) together entail (c), and I choose to align my beliefs with what is logical, and so hold (c). Nothing forces me to believe (c), and I could (perversely) do otherwise if I choose. As a matter of logic, (c) flows automatically from (a) and (b), but my holding (c) does not flow automatically from my holding (a) and (b).


It is, of course, true that you're not compelled to believe 'c' by anything other than reason. You could act against your better judgement, you could draw different conclusions ('old bugger is probably hung over') or you could go upstairs and knock on the door (the empiricist approach). But that is not really the point, which is to differentiate the kind of causation involved in physical cause-and-effect, on the one side, and rational necessity - believing something due to reasons - on the other. As I think you already noted earlier the thread - a distinction is being drawn between two types of 'because'.

But overall, I think it's a fair description.
wonderer1 June 23, 2023 at 05:20 #817159
Quoting Wayfarer
Yes. Note 'used by a speaker'. They are on that sense imbued with intentionality, namely, that of the speaker, to convey or represent something.


What is this "imbuing"? Sounds kind of hand wavy.

What disqualifies ChatGPT from being a speaker imbuing it's output with intentionality, regardless of whether the information processing involved is different than what occurs in human brains?

Quoting Wayfarer
...that kind of analysis is what is appropriate...


Well, I'm autistic, and I have my own style that I find works for me.

Janus June 23, 2023 at 07:05 #817161
I see two possibilities regarding my belief about anything whatsoever: the first being that the information I have encountered regarding the thing my belief is about has convinced me that my belief is true, which means that my belief is determined by my understanding of the information.

The second possibility is that it is entirely up to me to believe whatever I like, regardless of whether of not the information I have encountered up to the time of the formation of the belief has convinced my reason. It may have convinced my reason, but failed to convince my desire that I should believe that which my reason indicates, because perhaps I don't like the implications. This second possibility sounds like it could only obtain if I decided to believe something contrary to what the evidence indicates. Now, why would I do that? Out of pure perversity, out of displeasure with what the evidence indicates? Either way it does not sound like my belief would be one of rational conviction.

Is it even possible to believe contrary to what the evidence indicates to me personally is the truth, constituted as I am, with all my built in presuppositions and emotionally driven biases? Remember, whether valid or invalid, reasoning is only as sound as its grounding premises, which are often based on unacknowledged prejudices, and not derived from reasoning at all.

So, it seems that if it is not possible to rationally believe something contrary to what I genuinely find, whether rationally or not, convincing (which itself is a function of how I am constituted emotionally and intellectually, which in turn does not seem to be something I can actually determine "causa sui"), and it follows then that my belief would not be rationally driven at all, but emotionally driven.

This line of thinking reminds me of Schopenhauer's perspective on free will (which refutes the libertarian conception): "A man can will he wants, but he cannot will what he wills". This in turn reminds me of Hume's “Reason is and ought only to be the slave of the passions and can never pretend to any other office than to serve and obey them.” .

It seems that both of those philosophers did not belief in free will as conceived by the libertarians, and if this right, which it seems it must be if my belief is not to be merely arbitrarily driven by desire contrary to reason, then the idea that beliefs could be purely driven by rationality refutes itself and it follows that we are determined to believe what we do by our desires, aversions, biases and prejudices, and this would mean that the best we can do is to try to become conscious of those desires, aversions, prejudices and biases and grow out of them...into better, more rational ones...which would seem to be a never-ending process, at least unto death.

So, in short it just doesn't look possible that reason could be sovereign, but that it must be content to work slowly and piecemeal to become aware of, and then, as needed in order to live with greater serenity, change my desires, aversions, prejudices and biases, without the remotest possibility of becoming completely free of them but, at best, being able to gradually obtain a more livable suite, a suite of convictions which brings more peace and yet does not contradict the most convincing evidence, for if it does I will have more work to do, and will not be as much at peace as I could be.

In some ways this seems like the (Ancient) skeptical solution; Ataraxia attendant on suspension of judgement, but only regarding absolutes, for pragmatism dictates that I should give provisional credence to what the evidence indicates seems to be the case, while at the same time not imputing that seeming to some imagined ultimate reality. The latter can only cause dissatisfaction, unless I abandon reason altogether and put my faith just in "what rings true".
Tom Storm June 23, 2023 at 08:25 #817163
Reply to Janus Nicely written.

Quoting Janus
Remember, whether valid or invalid, reasoning is only as sound as its grounding premises, which are often based on unacknowledged prejudices, and not derived from reasoning at all.


Yep. Presuppositions sink ships.

Quoting Janus
So, in short it just doesn't look possible that reason could be sovereign, but that it must be content to work slowly and piecemeal to become aware of, and then, as needed in order to live with greater serenity, change my desires, aversions, prejudices and biases, without the remotest possibility of becoming completely free of them but, at best, being able to gradually obtain a more livable suite, a suite of convictions which brings more peace and yet does not contradict the most convincing evidence, for if it does I will have more work to do, and will not be as much at peace as I could be.


Very interesting. This resonates with me.

Quoting Janus
pragmatism dictates that I should give provisional credence to what the evidence indicates seems to be the case, while at the same time not imputing that seeming to some imagined ultimate reality. The latter can only cause dissatisfaction, unless I abandon reason altogether and put my faith just in "what rings true".


I spend a lot of time in 'provisional credence' country. I hear alarm bells when people say they know something to be certain.
wonderer1 June 23, 2023 at 11:21 #817175
Reply to Janus

:up:

Very thoughtful post.


Srap Tasmaner June 23, 2023 at 13:52 #817199
Quoting Wayfarer
It is, of course, true that you're not compelled to believe 'c' by anything other than reason.


The version I presented really targets determinism rather than naturalism, but we'd also want not to say that a conclusion is rational if reached by a process at least one step of which was genuinely stochastic. (Big tent determinism.)

Still, how literally do you want to take "compelled by reason"? Does reason operate something like a natural law, compelling me to reach a particular conclusion as surely as night follows day? (The latter being, you know, physics.)

(That's certainly not the plan for Lewis, who's going to want the sort of libertarian free will I gestured at, and for whom this is a direct confrontation between theism and naturalism, not just between naturalism and 'something else (to be determined later)'.)

Quoting Wayfarer
But that is not really the point, which is to differentiate the kind of causation involved in physical cause-and-effect, on the one side, and rational necessity - believing something due to reasons - on the other.


But this is why I need clarification. This "rational necessity" you're talking about, I don't know what that is. We sometimes speak of "logical necessity" but most such talk is pretty loose; if you really need such a thing, it's just a necessity relation that doesn't include any facts or history or natural laws and so on. A "bare necessity", as it were. It just means logic and logic alone, and only applies to what logic applies to. (Propositions, even if those propositions are proxies for states of affairs.)

Your rational necessity sounds like something that applies to epistemic agents, compelling them to hold certain beliefs if they hold others. Logic alone, famously, can't pull this off, or the world would be a better place.
Srap Tasmaner June 23, 2023 at 14:51 #817213
Reply to Janus

While I think this is fundamentally the right sort of answer, it does require a wholesale rethinking of the idea of rationality, and that might be a bitter pill to swallow.

What happens when we are persuaded by an argument? When we are convinced to change our minds? Those idioms leave us a bit passive, as if an argument pushes and pulls our beliefs like so much gravity. It's more decorous to say that we find an argument persuasive or the evidence convincing; sounds like we've rendered a judgment, in keeping with our high station.

Neither of those is particularly attractive. I think it's easy to accept pragmatism in the abstract -- to think this must be the way we think -- but difficult to believe it in particular cases, where it seems to us we have closely examined the logic and the evidence and taken a position. When doing philosophy, in particular, this is what we tell ourselves, and each other.

We may claim to be comfortable distinguishing the logic of discovery and the logic of confirmation or justification, but I think mostly we aren't. Chess provides a clear example, as usual: there's a saying among masters that the move you want to play is the right move, even if it seems impossible. This is intuition, and the idea is that careful analysis will justify your inclination, so some part of your mind must have zipped through that analysis without bothering to keep you informed, which would only slow things down. That fits nicely with the two-systems model, because the fast system here is just the unconscious and efficient habits that used to be carried out laboriously and consciously. --- But that still suggests that the conscious analysis you do is properly modeled as reasoning of the most traditional sort. There's no difference in kind here, only a difference in implementation. (This algorithm is known to work, so we can run it on the fast but unconscious machine.)

What is difficult to accept is that reason is really and truly rationalization, that justification is always and only post-hoc, that we are simply incapable of relying on logic and evidence alone to form our beliefs even if we do so to justify them. We want to believe that the process by which we reach a conclusion is quite similar to the process we would use to justify reaching it, and we want to believe that includes a free act of judgment.

I don't believe we ever choose what to believe, but only find that we do or we don't. And that applies here as well. It's what I find I believe, and others find they believe differently. What's worse, I find I cannot help but believe I have considered evidence and argument to reach this conclusion, but even if that is so, at no point did I weigh it all up and freely judge that it is so. All I can say is that argument and evidence seem to assail me like so much sensory input and the result is that I believe what I believe. I have to hope that what reason I have has done a good job filtering and weighing its inputs to reach a sound conclusion. If I try to justify my belief, I will surely succeed. It's one of my best things, as it is for everyone; rationalizing is our super power. Now I have to hope, as well, that my post-hoc justifications are everything they seem to be.

So, yes, I broadly agree with what you posted, @Janus, but I reserve a bit of Humean horror that the foundations of my rationality are not themselves rational.
wonderer1 June 23, 2023 at 15:07 #817215
Quoting Srap Tasmaner
Chess provides a clear example, as usual: there's a saying among masters that the move you want to play is the right move, even if it seems impossible. This is intuition, and the idea is that careful analysis will justify your inclination, so some part of your mind must have zipped through that analysis without bothering to keep you informed...


Right. Going with intuition is relying on the deep learning which has occurred in neural nets between our ears.

wonderer1 June 23, 2023 at 15:47 #817220
Quoting Srap Tasmaner
I have to hope that what reason I have has done a good job filtering and weighing its inputs to reach a sound conclusion. If I try to justify my belief, I will surely succeed. It's one of my best things, as it is for everyone; rationalizing is our super power. Now I have to hope, as well, that my post-hoc justifications are everything they seem to be.


I'd suggest looking at logic/language as providing a quite valuable way of comparing different intuitons we have:

1. With each other. (In an internal dialogue.)
2. With the intuitions of other people.
3. With empirical evidence.

Quoting Srap Tasmaner
So, yes, I broadly agree with what you posted, Janus, but I reserve a bit of Humean horror that the foundations of my rationality are not themselves rational.


I wouldn't say that the foundations aren't rational, but that the foundations are intuitive, and intuition is a foundational aspect of human rationality. It's just that many philosophically minded people have tended to think simplistically of rationality as somewhat synonymous with logic.



Srap Tasmaner June 23, 2023 at 16:32 #817229
Quoting wonderer1
Going with intuition is relying on the deep learning which has occurred in neural nets between our ears.


Sure, but

Quoting wonderer1
I wouldn't say that the foundations aren't rational, but that the foundations are intuitive, and intuition is a foundational aspect of human rationality. It's just that many philosophically minded people have tended to think simplistically of rationality as somewhat synonymous with logic.


even though there's a story we can tell about sound Bayesian inference being adaptive, even if unconscious, cognitive biases tell another story, that, as Kahneman says, and I never tire of quoting him, system 1 is a machine for jumping to conclusions.

So I still think Hume's horror is hard to shrug off. Our thinking is not what we thought it was. We learn some things about it that are reassuring and some that aren't, but the real problem is there is no transparency here; we're in the land of "for all we know..."
Fooloso4 June 23, 2023 at 16:37 #817230
Quoting Tom Storm
Gerson is the go to guy on this subject as I understand it.


He may be the go to guy for Platonism, but for that reason not the go to guy for Plato or Aristotle. Of course he and other Platonists would not agree.

Lloyd Gerson:Aristotle, in De Anima, argued that thinking in general (which includes knowledge as one kind of thinking) cannot be a property of a body; it cannot, as he put it, 'be blended with a body'.


This is misleading. Thinking is a property of intelligent beings. The distinction between form and body is an abstraction.

Human beings are an embodied beings. For Plato, as Aristotle well knows, forms are hypothetical. See Phaedo on Socrates' Second Sailing where he explicitly calls the forms hypothesis and acknowledges their inadequacy, calling them "safe and ignorant"(105b). See also Plato's Timaeus where the static and ineffectual nature of Forms is criticized.
,
Gerson says:

Lloyd Gerson:….the fact that in thinking, your mind is identical with the form that it thinks, means (for Aristotle and for all Platonists) that since the form 'thought' is detached from matter, 'mind' is immaterial too.


The "form 'thought'" is a product of philosophical poiesis. I don't think it is a term that either Plato or Aristotle ever used.

The limits of reason drawn by both Plato and Aristotle allows for both greater play and greater work of imagination, that is, of poiesis, of making. Reason often imagines that it is the whole of the story.

It might be objected that Aristotle argues for the existence of Intellect or Mind Itself, a disembodied thinking.But he does not present an unambiguous argument. Some scholars argue that this is intentional and marks the limit of what we know. A theological account intended to stand against the theologians, giving the appearance of an answer while pointing to the aporia of theological claims.

Near the beginning of Metaphysics Aristotle says:

... it is through experience that men acquire science and art ... (981a)


For more

Wisdom for Aristotle is, as it is for Plato and Socrates, knowledge of ignorance. The Platonist belief in an immaterial realm of intelligible Forms accessible to thought is a creation of the human mind, philosophical poiesis. Contrary to this, both Plato and Aristotle are rooted in physis, nature.

Physis or nature is not an explanation for why things are as they are. It is as things emerge and come to be as they are, how they grow and develop according to the kind of being each is. Each kind of being has its own nature. It develops accordingly.

Aristotle regards living beings as self-sustaining functioning wholes. The four causes are inherent in a being being the kind of being it is, not something imposed on or interfering with it from the outside. Human beings are by their nature thinking beings. This is not an explanation, but a given. It has nothing to do with Gerson's "form 'thought'". Nothing to do with a transcendent realm accessible to the wise.

Rather than an argument from reason, @Wayfarer, Plato and Aristotle use reason to demonstrate the limits of reason.











wonderer1 June 23, 2023 at 16:42 #817231
Quoting Srap Tasmaner
So I still think Hume's horror is hard to shrug off. Our thinking is not what we thought it was. We learn some things about it that are reassuring and some that aren't, but the real problem is there is no transparency here; we're in the land of "for all we know..."


I can understand that. I started thinking along these lines 36 years ago, when I realized that "there is something weird about my brain" is the best explanation for a bunch of different aspects of my life experience. For me the horror faded away a long time ago.

You see, one thing is, I can live with doubt, and uncertainty, and not knowing. I think it's much more interesting to live not knowing than to have answers which might be wrong. I have approximate answers and possible beliefs and different degrees of certainty about different things. But I'm not absolutely sure of anything, and there are many things I don't know anything about, such as whether it means anything to ask why we're here, and what the question might mean. I might think about it a little bit; if I can't figure it out, then I go onto something else. But I don't have to know an answer. I don't feel frightened by not knowing things, by being lost in the mysterious universe without having any purpose, which is the way it really is, as far as I can tell -- possibly. It doesn't frighten me. [smiles]
Richard Feynman
Gnomon June 23, 2023 at 17:08 #817234
Quoting Tom Storm
I'm not convinced we know what is random versus that which is not random. We detect patterns, as far as human cognition allows and we ascribe characteristics to those patterns - again in human terms. But words like 'random' or 'accidental' seem to have emotional connotations and function as tips of icebergs.

Tom, your unwillingness to commit to at least a provisional position on the Random Chaos vs Rational Cosmos question is puzzling to me. Is it the "emotional connotations" that cause you to take a position of Profound Skepticism? If the world is all a "blooming buzzing confusion"*1, why bother to post on a philosophy forum? Doesn't a forum like this presuppose that we can eventually make sense of the complex patterns of Nature, and the even more confusing patterns of Culture? Do you think that Nature is "leaving no role for the free exercise of reason. — Wayfarer". :smile:

PS___Admittedly, sometimes forum threads, veering recklessly off-topic, seem to add to the original confusion that provokes the question. :joke:


In his book, The Principles of Psychology, William James defines the concept of 'blooming and buzzing confusion' to describe a baby's experience of the world as pure sensation that comes before any rationality. This experience becomes a reference to further interpretation of the coming sensations in life.
https://www.hamedkhosravi.com/A-Buzzing-Confusion-1

Reply to Wayfarer
Srap Tasmaner June 23, 2023 at 18:01 #817243
Reply to wonderer1

"Better a question that can't be answered than an answer that can't be questioned."

I think he said that. (Reading Surely You're Joking at I guess 16 or so was a formative experience for me.)

But I insist the phenomenology of this is hard.

I have sometimes said that many people on this forum don't seem to believe in disagreement: "if you seem to disagree with me, it can only be because you didn't understand what I said, so I'll say it again." We do recognize that even correct arguments don't always land with an audience, do not compel them with the force of reason, so we try different wordings, different analogies and examples, hoping that one of them will finally do the trick. --- My point here is only that we don't know what will work, why it will work, and what worked in our case. We hope to explain this lack of transparency by distinguishing form from content, as if it were the same as to say I've never been convinced by an argument presented in Polish, since I don't speak Polish. If you grasp the meaning at all, logic is supposed to carry the day, but experience tells us this is not so, though we believe it of ourselves. (This a little like @NOS4A2's suggestion that it is always other people we believe need to be kept in line by force, not us!)
Alkis Piskas June 23, 2023 at 18:12 #817246
Reply to Wayfarer
Excellent presentation, Wayferer! :up:

There are more arguments about the failure of materialist philosophy of the mind than what one can imagine. For me it is a dead case. But who am I to say. Leibniz has already set the point with his "Mill Argument Against Mechanical Materialism" 300 years ago:

"If we imagine a machine whose structure makes it think, sense, and have perceptions, we could conceive it enlarged, keeping the same proportions, so that we could enter into it, as one enters a mill. Assuming that, when inspecting its interior, we will find only parts that push one another, and we will never find anything to explain a perception. And so, one should seek perception in the simple substance and not in the composite or in the machine." (GP: VI, 609/AG: 215)

Long ago, before I came upon this, I had thought something quite similar: If thoughts were produced and stored in the brain, shouldn't neurologists or other specialized scientists,ehen they open or scan the brain, be able to trace them and identify them? Yet, not only there has been the least trace of such an identification but they have not even explained the process of thought, at least not in a provable and undisputed way. As Leibniz would say, they will "never find anything to explain a thought". And think that "perception" that Leibniz talks about is much more concrete and near to physicality that thinking.
Srap Tasmaner June 23, 2023 at 19:43 #817261
Reply to Alkis Piskas

That sounds a lot like "But where's the university?" It is a spectacularly awful argument.

The much-vaunted (around here) failure of neuroscience neglects two facts: (1) neuroscience is still in its infancy, maybe adolescence; (2) it has been having truly astonishing and accelerating success.

You evidently think we've been wandering down a blind alley since Phineas Gage's accident. I find that view incomprehensible, but you do you.
wonderer1 June 23, 2023 at 20:28 #817272
Quoting Alkis Piskas
...If thoughts were produced and stored in the brain, shouldn't neurologists or other specialized scientists,ehen they open or scan the brain, be able to trace them and identify them? Yet, not only there has been the least trace of such an identification but they have not even explained the process of thought, at least not in a provable and undisputed way. As Leibniz would say, they will "never find anything to explain a thought". And think that "perception" that Leibniz talks about is much more concrete and near to physicality that thinking.


Thoughts are more events than things. See the following link for information about scientists detecting thought events:

https://www.eedesignit.com/oh-no-ai-now-reads-minds/
Janus June 23, 2023 at 21:51 #817288
Quoting Tom Storm
I spend a lot of time in 'provisional credence' country. I hear alarm bells when people say they know something to be certain.


So do I. The demand for certainty seems to be the motivator for the kind of platonist thinking that wants to place reason high up on on a throne presiding over mere emotion. I don't understand this attitude, since the principles of reason of skillful thinking: coherence, validity and consistency are formal constraints, that by themselves tell us nothing, and must work with what we receive via the senses and our emotions to yield any knowledge.

Quoting wonderer1
Right. Going with intuition is relying on the deep learning which has occurred in neural nets between our ears.


I think that's right, but our intuitions can fool us, so we do need to examine the reasoning and its foundational presuppositions and our desires and aversions that underly our intuitions

Quoting Srap Tasmaner
So, yes, I broadly agree with what you posted, Janus, but I reserve a bit of Humean horror that the foundations of my rationality are not themselves rational.


I agree, it is kind of horrible, especially when you consider the dire situation humanity faces now, with all its competing narratives and interests. The flipside is the fear that if human life was governed by reason, and only reason; a civilization comprised of Vulcan-like or Spock-like beings, who either have no passions or keep their passions rigidly in check, that we would be a repressed, uptight bunch who live too much "in our heads". It seems a balance is hard to achieve it is either the individual at the expense of the collective or the collective at the expense of the individual; not a very attractive pair of options.

.
wonderer1 June 23, 2023 at 21:53 #817290
Quoting Srap Tasmaner
Reading Surely You're Joking at I guess 16 or so was a formative experience for me.


I think it was late 20s for me, but it still played a significant role in my subsequent thinking. I'd like to reread it, but I've been buying books at twice the rate I'm reading them for years. So probably not going to happen.

Quoting Srap Tasmaner
But I insist the phenomenology of this is hard.


I don't know much about the study of phenomenology in philosophy, so I can't really say anything about that. I suppose I've developed my own thoughts on phenomenology, but they would probably be "in Polish" from the perspective of most phenomenologists.

Quoting Srap Tasmaner
I have sometimes said that many people on this forum don't seem to believe in disagreement: "if you seem to disagree with me, it can only be because you didn't understand what I said, so I'll say it again." We do recognize that even correct arguments don't always land with an audience, do not compel them with the force of reason, so we try different wordings, different analogies and examples, hoping that one of them will finally do the trick.


Right. An aspect of my communication strategy, in venues like TPF, is an attempt to lay some subconscious groundwork in people's minds that might allow them to make a paradigm shift in their thinking somewhere down the line, but results are very scattershot.

Quoting Srap Tasmaner
My point here is only that we don't know what will work, why it will work, and what worked in our case.


I agree with this to a large extent. Although, in my experience, the more knowledge I have of how the person I am talking to thinks, the greater my ability to put thing in terms that connect for them. (Though that is something I can't really disentangle from my coping with being on the autism spectrum.)

Quoting Srap Tasmaner
If you grasp the meaning at all, logic is supposed to carry the day, but experience tells us this is not so, though we believe it of ourselves.


I know what you mean. Although with all the time I have put into thinking about this stuff, I think I am relatively conscious of the importance of catalyzing paradigm shifts (whether in myself or others) in the process of communicating.

To close, I have enjoyed this meeting of the minds immensely. I hope we will have occasion to discuss these sorts of things more in the future.

Wayfarer June 23, 2023 at 21:55 #817291
Quoting Janus
it seems that if it is not possible to rationally believe something contrary to what I genuinely find, whether rationally or not, convincing (which itself is a function of how I am constituted emotionally and intellectually, which in turn does not seem to be something I can actually determine "causa sui"), and it follows then that my belief would not be rationally driven at all, but emotionally driven.


All of which is quite irrelevant if you were doing an actual exercise in logic. But rationally-inferred propositions aren't a matter of belief at all - the hackneyed example I gave of 'if X>Y and A>X then X>Y' is not dependent on belief nor a matter of belief or sentiment.

Quoting Srap Tasmaner
This "rational necessity" you're talking about, I don't know what that is. We sometimes speak of "logical necessity" but most such talk is pretty loose; if you really need such a thing, it's just a necessity relation that doesn't include any facts or history or natural laws and so on. A "bare necessity", as it were. It just means logic and logic alone, and only applies to what logic applies to.


Again, how is a logical syllogism 'pretty loose'? Logic is quite precise. I don't think that the argument from reason is setting out to prove that reason is infallible or all-knowing - simply that it comprises the relationships of ideas, and so that can't be reduced to, or explained in terms of, the physical cause-and-effect relationships that are grist to the naturalist mill.

Quoting Srap Tasmaner
The much-vaunted (around here) failure of neuroscience neglects two facts: (1) neuroscience is still in its infancy, maybe adolescence; (2) it has been having truly astonishing and accelerating success.


I have relatives who owe their lives to neuroscience, and I would never disparage it. But I'm not sure how much of philosophy is indebted to it. It seems to me that many of the appeals to neuroscience are pretty close to scientism. There's a well-known book I learned about on this forum, The Philosophical Foundations of Neuroscience, Hacker and Bennett, which I'll probably never get around to reading in full, although this review gives a good précis. It is by a neuroscientist (Bennett) and philosopher (Hacker) and gives pretty short shrift to neuro-reductionism - which is not to disparage neuroscience in the least.

Considerably more smoke than light in most of the above.
Srap Tasmaner June 23, 2023 at 22:00 #817294
Quoting wonderer1
phenomenology


I didn't mean the school of thought, but the thing itself, the detailed account of a type of experience.

Quoting wonderer1
I have enjoyed this meeting of the minds immensely.


Likewise.
Wayfarer June 23, 2023 at 22:01 #817295
Quoting Alkis Piskas
As Leibniz would say, they will "never find anything to explain a thought". And think that "perception" that Leibniz talks about is much more concrete and near to physicality that thinking.


:up: Leibniz is frequently cited as one of the predecessors to David Chalmer's 'Facing up to the Problem of Consciousness'
Wayfarer June 23, 2023 at 22:01 #817296
Quoting wonderer1
An aspect of my communication strategy, in venues like TPF, is an attempt to lay some subconscious groundwork in people's minds that might allow them to make a paradigm shift


From what, to what?
Janus June 23, 2023 at 22:02 #817297
Quoting Wayfarer
All of which is quite irrelevant if you were doing an actual exercise in logic. But rationally-inferred propositions aren't a matter of belief at all - the hackneyed example I gave of 'if X>Y and A>X then X>Y' is not dependent on belief nor a matter of belief or sentiment.


But, so what? That kind of purely formal logical deduction has little to do with actual human life and reasoning. Your "Argument from Reason" itself is not that kind of deduction, but is an abductive speculation based on premises which are taken for granted.

It would help if you actually engaged in argumentation with your interlocutor's arguments rather than attempting to dismiss them with statements like "considerably more smoke than light in most of the above".
Srap Tasmaner June 23, 2023 at 22:03 #817298
Quoting Wayfarer
Considerably more smoke than light in most of the above.


Hey, you're doing the best that you can.
wonderer1 June 23, 2023 at 22:03 #817299
Quoting Wayfarer
From what, to what?


From what seems to me a less accurate view to what seems to me a more accurate view.
Wayfarer June 23, 2023 at 22:03 #817300
Quoting Janus
That kind of purely formal logical deduction has little to do with actual human life and reasoning.


It is the subject of the OP, and the basis for an argument.

Reply to wonderer1 Well, hopefully, that is what the overall aim of philosophy is for anyone engaged in it.
Janus June 23, 2023 at 22:08 #817303
Quoting Wayfarer
It is the subject of the OP, and the basis for an argument.


I wasn't referring to the subject: the argument from reason itself is not a purely deductive argument. You seem to be taking it for granted that a pure deduction cannot be at the same time a neural process.

Let me put it another way: iff it is accepted that a pure deduction cannot be at the same time a neural process then your argument form reason would stand, but that is the very point at issue. Where is the argument for the contention that a pure deduction cannot be at the same a neural process?
Wayfarer June 23, 2023 at 22:13 #817304
Reply to Janus Not 'taking for granted': presenting an argument for it, which I don't believe you've addressed - or at least, until this post.
Wayfarer June 23, 2023 at 22:16 #817305
Some notes on 'transcendental arguments': 'What is typical of transcendental arguments proper is that they purport to establish the conditions necessary for experience, or experience of a certain kind, as a whole; and, at their most controversial, to establish a conclusion about the nature and existence of the external world, or other minds, as these – and particularly the world’s existence – can be derived in consequence of paying attention to what has to be the case for there to be experience, or in order for experience to be as it is.” (A. C. Grayling, The Refutation of Skepticism, pg. 83). So the strategy of transcendental arguments is that they start with a given - some incontrovertible fact of experience - and seek to answer the question: “what must be true given this experience.”

Descartes' cogito ergo sum argument is one example. There are others in The Critique of Pure Reason.


Srap Tasmaner June 23, 2023 at 22:21 #817306
Quoting Wayfarer
I don't think that the argument from reason is setting out to prove that reason is infallible or all-knowing - simply that it comprises the relationships of ideas, and so that can't be reduced to, or explained in terms of, the physical cause-and-effect relationships that are grist to the naturalist mill.


This is the thing, though, you keep eliding the difference between propositions and epistemic agents holding those propositions true (or probable, whatever).

Logic is not the natural science of thought. That's psychology. Logic, taken narrowly, is a science of some of the relations among propositions, but includes no concept of an agent at all. You can take it somewhat more broadly, as it used to be, but then we're really looking to logic for normative guidance on what works. Decision theory kinda lands in here, and lives at the intersection of psychology, philosophy, and economics.

But to reiterate: taking "logic" as we usually do these days, my believing that P does not entail my believing that Q, even if P entails Q. If there's a relation between my believing that P and my believing that Q, logic may play some part in that, but it is unquestionably also a matter of psychology, as it's my belief states that are at issue.
Wayfarer June 23, 2023 at 22:26 #817307
Quoting Srap Tasmaner
Logic is not the natural science of thought. That's psychology.


Isn't that psychologism? The philosophical position that asserts that all meaningful statements or concepts can ultimately be reduced to psychological terms or explained in psychological language. It suggests that the study of psychology is foundational and can provide a comprehensive understanding of all aspects of human experience, including areas traditionally studied by other disciplines such as logic, mathematics, or philosophy.

Psychologism was particularly influential in the late 19th and early 20th centuries, primarily associated with figures like Wilhelm Wundt, Edward Titchener, and Oswald Külpe. They sought to establish psychology as a fundamental science that could explain all aspects of human cognition and behavior, including areas previously considered the domain of philosophy, such as logic and ethics. Frege criticized Husserl's philosophy of maths on the grounds of psychologism.

But critics argue that psychologism commits the "psychological fallacy" by conflating psychological descriptions or explanations with logical or conceptual analysis. They claim that not all meaningful concepts or statements can be reduced to psychological terms, and that the proper understanding of logic, mathematics, and philosophy couldn't be reduced to psychological terminology. Accordingly, sychologism was largely rejected as a philosophical position in the early 20th century.
Wayfarer June 23, 2023 at 22:28 #817309
The target of the argument is re-iterated in this post, the passage commencing 'The materialist worldview presumes a mechanistic base level. This doesn't mean necessarily deterministic - there can be chance at the basic level of reality in a mechanistic worldview (e.g. Heisenberg's uncertainty principle and the 'quantum leap'.) However at the level of basic physics, nature is free of purpose, free of meaning or intentionality, free of normativity, and absent of any and all forms of subjectivity. If one is operating within a materialistic framework, then one cannot attribute purpose to what happens at the basic level."
Srap Tasmaner June 23, 2023 at 22:34 #817311
Quoting Janus
You seem to be taking it for granted that a pure deduction cannot be at the same time a neural process.


Quoting Wayfarer
Not 'taking for granted': presenting an argument for it


I don't think so. That's what I asked for earlier:

Quoting Srap Tasmaner
That's why you need an actual argument showing that if brain state A, with contents P, causes brain state B, with contents Q, that a causal relation between A and B is incompatible with a logical relation between P and Q.


I'm not sure there's exactly an argument here at all, and even if there is, whether it works. Certainly if the whole thing turns on libertarian free will (that "obscure and panicky doctrine" as Peter Strawson called it), that's a kettle of fish of a different color.

On the other hand, I'm also inclined, as I've indicated in recent postings, just to let it rip. Maybe naturalism does show that the sort of reasoning we think we do is a myth.
Srap Tasmaner June 23, 2023 at 22:40 #817314
Quoting Wayfarer
Isn't that psychologism?


It isn't.

I don't have to reduce logic to psychology to point out that logic describes some relations between propositions and no relations among an epistemic agent's belief states. It's a known fact. You won't find a logician anywhere who would claim otherwise.
wonderer1 June 23, 2023 at 22:46 #817315
Quoting Janus
Right. Going with intuition is relying on the deep learning which has occurred in neural nets between our ears.
— wonderer1

I think that's right, but our intuitions can fool us, so we do need to examine the reasoning and its foundational presuppositions and our desires and aversions that underly our intuitions


Absolutely our intuitions can fool us. And logic is subject to GIGO, and can fool us as well. Since I have Feynman on the brain, another quote:

“The first principle is that you must not fool yourself, and you are the easiest person to fool.”


I would add to what you said above, that we can learn from the study of applications of Artificial Neural Nets (ANNs), to improve the effectiveness with which we use our brains. A key consideration with ANNs is the training set, or the set of inputs that were involved in an ANN learning whatever it learned. Analogously, we can consider the size and scope of the training set that went into the deep learning underlying our intuitions, and consider whether our intuitions are likely to be trustworthy or untrustworthy under whatever the present circumstances are. In doing so we might recognize a benefit to increasing the size and/or scope of our training sets, and improve the training of our neural nets, resulting in an improvement to the reliability to our intuitions in the future.

Wayfarer June 23, 2023 at 22:49 #817316
Quoting Srap Tasmaner
I don't have to reduce logic to psychology to point out that logic describes some relations between propositions and no relations among an epistemic agent's belief states.


However, if I adopt a view on account of logic, then that informs my 'belief states', I am willing to accept it, and act on it.

Quoting Srap Tasmaner
you need an actual argument showing that if brain state A, with contents P


How could you specify 'content' in this sense? How would you ascertain what the 'brain state' is for some ostensible content? Could 'believing that p'be described in terms of the state of billions of neurons at a given instant in time? I don't think it could, as the brain, being a dynamic neural network, is constantly changing. The element of constancy amongst that flux is syntactic and semantic - not physiological.

Janus June 23, 2023 at 22:51 #817317
Reply to Wayfarer But these "givens" that state what, for example, all experience must be like are not purely logically given (that is they are not tautologies in that their negations are not logical contradictions) they are synthetic generalizations from experience, and I think they represent what we find it impossible to imagine.
Wayfarer June 23, 2023 at 22:56 #817318
Reply to Janus That's a theory similar to J S Mill's. To which the Kantian rejoinder is, that it would be impossible to arrive at 'synthetic generalisations from experience' without the faculty of reason and the innate categories of the understanding which can contrast like and unlike within a context and against a background. But if you want to launch into explaining the foundations of logic, then go right ahead, but it would seem to me it's going to involve a lot of writing.
Janus June 23, 2023 at 23:08 #817320
Reply to Wayfarer There is no need to write a treatise on the foundations of logic; logical principles are actually fairly simple.

We arrive at synthetic generalizations from experience by recognizing what the general characteristics of different experiences are: so, it is a form of cognition: re-cognition. I don't see it as involving any logical deduction; it is more like pattern recognition.

Of course, we have to also think about it coherently as well, and that does involve thinking consistently or in logical terms in keeping with the principle of validity; that is we should avoid contradicting ourselves.

A simple example is that we can recognize that all our experiences of sensory objects are spatiotemporal, from which we generalize to saying that all experiences of sensory objects must be spatiotemporal because we cannot imagine how it could be otherwise.
Janus June 23, 2023 at 23:11 #817321
Quoting wonderer1
I would add to what you said above, that we can learn from the study of applications of Artificial Neural Nets (ANNs), to improve the effectiveness with which we use our brains. A key consideration with ANNs is the training set, or the set of inputs that were involved in an ANN learning whatever it learned. Analogously, we can consider the size and scope of the training set that went into the deep learning underlying our intuitions, and consider whether our intuitions are likely to be trustworthy or untrustworthy under whatever the present circumstances are. In doing so we might recognize a benefit to increasing the size and/or scope of our training sets, and improve the training of our neural nets, resulting in an improvement to the reliability to our intuitions in the future.


That sounds like an interesting avenue of investigation, that I am yet to take the first step upon. If only we had more time!

Can you recommend an introductory text?
Srap Tasmaner June 23, 2023 at 23:25 #817324
Quoting Wayfarer
However, if I adopt a view on account of logic, then that informs my 'belief states', I am willing to accept it, and act on it.


Adopt, willing, accept, act -- all of this is beyond the purview of logic. It's really straightforward. One proposition can entail another; one belief state cannot, in this same sense, entail another.

Quoting Wayfarer
How could you specify 'content' in this sense? How would you ascertain what the 'brain state' is for some ostensible content?


It was "for the sake of argument," but that branch of the discussion never materialized.
Srap Tasmaner June 23, 2023 at 23:28 #817325
Reply to wonderer1

Robert Kowalski (early developer of Prolog) has been suggesting that instead of trying to get machines to think like us, we ought to consider learning to think more like machines. Wrote a book about it.
Wayfarer June 23, 2023 at 23:36 #817327
Quoting Srap Tasmaner
One proposition can entail another; one belief state cannot, in this same sense, entail another.


Sure it can. If I believe that P, and this entails that I believe that Q, then that is entailment. You're splitting hairs.

Tom Storm June 23, 2023 at 23:38 #817328
Quoting Fooloso4
He may be the go to guy for Platonism, but for that reason not the go to guy for Plato or Aristotle. Of course he and other Platonists would not agree.


Yes, I meant Platonism.

Quoting Fooloso4
Aristotle regards living beings as self-sustaining functioning wholes. The four causes are inherent in a being being the kind of being it is, not something imposed on or interfering with it from the outside. Human beings are by their nature thinking beings. This is not an explanation, but a given. It has nothing to do with Gerson's "form 'thought'". Nothing to do with a transcendent realm accessible to the wise.

Rather than an argument from reason, Wayfarer, Plato and Aristotle use reason to demonstrate the limits of reason.


Thanks and very interesting.
Srap Tasmaner June 23, 2023 at 23:41 #817329
Quoting Wayfarer
You're splitting hairs.


Look at this way: we think of logic as normative, within limits; if P entails Q, and you believe P, then you ought to believe Q. Do people always do what they ought?
Tom Storm June 23, 2023 at 23:57 #817333
Quoting Gnomon
Tom, your unwillingness to commit to at least a provisional position on the Random Chaos vs Rational Cosmos question is puzzling to me.


I think that's mostly a problem for you and may explain things. Also 'unwillingness' is not a good word, it implies an ought - I 'ought' to be able to, right? I would say 'inability' would be more appropriate. I hold tentative positions on some matters, and was just writing elsewhere above -

Quoting Tom Storm
I spend a lot of time in 'provisional credence' country. I hear alarm bells when people say they know something to be certain.


Quoting Gnomon
If the world is all a "blooming buzzing confusion"*1, why bother to post on a philosophy forum?


What an odd question. It is precisely because things are far from clear that I am interested to see what other people make of things. One shouldn't come to a position and then say - 'That's it, I have arrived!' That's the thinking of fundamentalism or monomania.

Humans have cognitive limitations and individuals have intellectual/psychological limitations - to argue that we have equal access to an understanding of reality (whatever that might be) would be absurd. Many of the questions we ask are doubtless unanswerable or have incomprehensible answers for many of us. I am primarily interested in improving the questions.

Quoting Gnomon
Doesn't a forum like this presuppose that we can eventually make sense of the complex patterns of Nature, and the even more confusing patterns of Culture?


Christ no. A forum like this showcases opinions, values and beliefs (theorised and untheorised), which come from any number of sources and intellectual processes, some of which seem more credible than others.

For me the task here is mostly to ask what do you believe and why? And then pose the odd question to clarify or identify potential challenges to the belief. We are all here testing beliefs in the marketplace. Although it's clear some people hold dogmatic positions which sometimes seem rather fragile.

In this process skepticism for me isn't denialism or cynicism. It is simply the recognition of uncertainty in our experience and practice. Where possible things should be questioned and justified before they can pass for tentative knowledge. In this process there is also scope for us to change our views.

javra June 23, 2023 at 23:58 #817334
Reply to Wayfarer

“Reason” to me is a fairly ambiguous concept, and so then too is the notion of “rationality”. As one minor hint of this: The irrational argument is yet engaged in reasoning, albeit in false or wrong or inaccurate ways, and so it is not arational—for it yet makes use of the laws of thought to obtain its conclusions. On the other hand, to add to this ambiguity, and with a pointer to Janus’s comments, I don’t see rationality as distinct from emotions—finding emotions to hold their sub/unconscious reasoning and, conversely, finding all conscious reasoning to be hewed, if not governed, by emotive dispositions, ranging from overt forms like animosity or compassion to more subtle forms like sereneness, or wonder, or curiosity.

At any rate, due to the just mentioned, I would present what I take to be the OP’s underlying stance differently, this by addressing the issue of truth. (intending to come full circle at the end)

For lack of equivocation, by truth I’m referring to the generalized sum of all individual truths regardless of their type (e.g., abstract or concrete), such that an individual truth is understood to be an instance of conformity to that which is actual, aka that which is real. If this is too abstract in its given terse form, then I’ll here yield to the correspondence theory of truth. Given this:

Is all truth thus understood a fabricated creation or, otherwise, is it a brute aspect of the world that is thereby uncreated?

Of note, when we willfully fabricate what others then accept to be a truth, we as a society term this a deception, aka a lie. So, then, for the theistically minded, are all the mundane truths we accept and live by the fabrication—one can here technically say, the deception—of an omnipotent psyche (thus, an omnipotent deceiver)? On the other hand, for the atheistically minded, is truth an emergent condition fabricated from, i.e. created by, mindless subatomic particles and forces that themselves emerge from a quantum vacuum field as these engage in an emergent process of biological evolution—such that the referent to the term “truth” changes (evolves) with time, never holding any definitive existential property but, instead, only being (often enough, the changing) fabricated stories we tell ourselves about it?

Plenty from both camps will answer “yes, all truths are fabricated/created”, but then the relativism that unfolds either way becomes detrimental to human welfare. The deceptions of the despotically minded become the lived, incarnate truths of the populous—and opposition to these created truths become collectively condemned at the long-term expense of all. As one example, that climate change is a global hoax becomes truth because some autocrats so state (in large part to keep their wallets fat) and because others so choose to believe. And yet, the nonfabricated truth that it is not a hoax yet pulls us into the inevitable.

On the other hand, if one answers that truth is not a created fabrication, then there will be something more to existence than what physicalism proposes, as the OP suggests. And yes, this can be characterized as Platonism or Neoplatonism wherein the Good is the ultimate truth (and vice versa), and can be furthermore shunned on account of this presently unorthodox view.

But if truth where to be metaphysically uncreated, then this will resonate with the basic laws of thought: what is true can only be itself; it will thereby be impossible that two contradictory truths cooccur at the same time and in the same way; and in it will likewise be impossible that a proposition be both true and false at the same time and in the same way, entailing that the proposition can either be one or the other.

These basic, universal laws of thought will then form the foundations of all rationality. Here, then, rationality becomes a metaphysical derivative of the existential occurrence of (uncreated, rather than fabricated) truth. Nevertheless, truth and rationality can well be deemed in such an interpretation to be staple aspects of existence—this in line to a globally pervading logos—aspects toward whose greater comprehension the increased intelligence that occasionally gets selected for by evolutionary mechanisms develops toward.

-----

I’m still relatively short on time so I’ll likely not participate for long, and I know this post has some gaps in it. But I wanted to chime in a bit all the same, this in my support of the OP. Even if our perspectives might differ somewhat.
wonderer1 June 24, 2023 at 00:05 #817336
Quoting Janus
Can you recommend an introductory text?


I'm not really up to date on the books on the subject. One of the seminal texts, and the book I learned the basics from, is Parallel Distributed Processsing. It was written in 1987, but the first few Amazon reviews I read gave the impression that it is still considered an important introduction to the field. There may well be better introductory texts though, and Amazon's purchasers also read might be of some value.

However, I think that this 20 minute youtube video presents a quite good introduction to how trained neural nets work, and the same videographer has a part 2 video covering how learning in ANNs work. For the purposes of getting a sense of how the subject is relevant to our thinking, the first video might be sufficient.

If you are looking for a book on how research into ANNs can be applied to human thought, I don't know of any such books having been written yet. (Not to say I've looked for one.) I've learned by trial and error.
wonderer1 June 24, 2023 at 00:28 #817341
Quoting Srap Tasmaner
Robert Kowalski (early developer of Prolog) has been suggesting that instead of trying to get machines to think like us, we ought to consider learning to think more like machines. Wrote a book about it.


Thanks.
Janus June 24, 2023 at 00:40 #817343
Reply to wonderer1 :up: cheers
Wayfarer June 24, 2023 at 01:04 #817346
Quoting Srap Tasmaner
we think of logic as normative, within limits; if P entails Q, and you believe P, then you ought to believe Q. Do people always do what they ought?


Of course not, but you've already made that point, and I've said that it's not really relevant to the argument at hand.

I want to double back to this early comment to bring out this point.

Quoting wonderer1
This [i.e. the OP] appears to be begging the question, by presuming that the exercise of reason is something different than information processing occurring in our brains.

Smuggling in a dualism which isn't part of the materialist view doesn't do anything to contradict a materialist view.


The argument from reason doesn't rely on 'smuggling in' a dualism - it is an explicit appeal to dualism to highlight a fundamental weakness in the type of reducionist physicalism that is being criticized. It is not 'begging the question' but presenting an argument to that effect, which propenents of physicalism are then required to answer.

Quoting wonderer1
Naturalism being true only requires beliefs being *caused*, by what at the lowest level are non-rational causes.


Which is exactly what the argument from reason is criticising.

Diverting the thread to AI research and neural networks as a kind of 'general argument for physicalism' is just changing the subject.

Reply to javra Thanks, Javra, I'll mull that over, although I think it's rather more metaphysical than the argument itself warrants. But may come back with some more responses.

Quoting Fooloso4
Aristotle regards living beings as self-sustaining functioning wholes. The four causes are inherent in a being being the kind of being it is, not something imposed on or interfering with it from the outside. Human beings are by their nature thinking beings. This is not an explanation, but a given. It has nothing to do with Gerson's "form 'thought'". Nothing to do with a transcendent realm accessible to the wise.


Are the following not characteristic of Aristotle, then?

[quote=Nichomachean Ethics, Book X, 1177a11] if happiness [??????????/eudomonia] consists in activity in accordance with virtue, it is reasonable that it should be activity in accordance with the highest virtue; and this will be the virtue of the best part of us. Whether then this be the Intellect [????/nous], or whatever else it be that is thought to rule and lead us by nature, and to have cognizance of what is noble and divine, either as being itself also actually divine, or as being relatively the divinest part of us, it is the activity of this part of us in accordance with the virtue proper to it that will constitute perfect happiness; and it has been stated already* that this activity is the activity of contemplation [?????????/theoria].[/quote]

*In 6.7.2-3 Aristotle says that

Wisdom [?????] is the most perfect mode of knowledge. A wise person must have a true conception of unproven first principles and also know the conclusions that follow from them. 'Hence Wisdom must be a combination of Intelligence [Intellect; ????] and Scientific Knowledge [????????]: it must be a consummated knowledge of the most exalted objects.' Contemplation is that activity in which one's ???? intuits and delights in first principles."


Surely this does at least suggest 'a transcendent realm accessible to the wise'?
NOS4A2 June 24, 2023 at 01:39 #817353
Reply to Alkis Piskas

If you want to find the “process of thought” watch a person think. Human thought, like believing and reasoning, is an action performed by persons, and not by any other collection of things and processes. If you want to see a cartwheel or a backflip you do the same thing: watch a person perform these actions.

If thoughts are not persons thinking, beliefs are not persons believing, and reasons not persons reasoning, then they are nothing but words without a referent. There is no other way around it.
javra June 24, 2023 at 01:40 #817354
Reply to Wayfarer Yes, I suppose the issue is to me fully metaphysical regardless of the perspectives one may hold in relation to it. All the same, no worries. I was just passing through.
wonderer1 June 24, 2023 at 01:51 #817359
Quoting Wayfarer
Diverting the thread to AI research and neural networks as a kind of 'general argument for physicalism' is just changing the subject.


No, that's just the conclusion your insufficiently trained neural nets jumped to. What it is, is providing food for thought (training inputs) that are relevant to forming more accurate intuitions about how our minds/brains work. Admittedly, it is a bit of a, "He who hath ears to hear, let him hear." sort of thing, as I was discussing with Srap.

It's okay if you don't get it. Developing and strengthening new intuitions, to the point that an epiphany/paradigm shift can occur, takes time. Be patient.

By the way, I don't think you responded to my question about your familiarity with shoshin or beginners mind. Does this ring any bells?

Shoshin (Japanese: ??) is a concept from Zen Buddhism meaning beginner's mind. It refers to having an attitude of openness, eagerness, and lack of preconceptions when studying, even at an advanced level, just as a beginner would. The term is especially used in the study of Zen Buddhism and Japanese martial arts,[1] and was popularized outside of Japan by Shunry? Suzuki's 1970 book Zen Mind, Beginner's Mind.

The practice of shoshin acts as a counter to the hubris and closed-mindedness often associated with thinking of oneself as an expert.[2] This includes the Einstellung effect, where a person becomes so accustomed to a certain way of doing things that they do not consider or acknowledge new ideas or approaches.[3] The word shoshin is a combination of sho (Japanese: ?), meaning "beginner" or "initial", and shin (Japanese: ?), meaning "mind".[4]


Seems like something a few people here could use some practice at.
Janus June 24, 2023 at 01:52 #817360
Quoting Wayfarer
Surely this does at least suggest 'a transcendent realm accessible to the wise'?


What exactly leads you to think so?
Janus June 24, 2023 at 02:04 #817368
Reply to wonderer1 :up: It seems that there are, roughly speaking, two kinds of thinkers about the nature of the brain/mind: those who wish to discover new and better ways to think about it, and those who seek for support for what they want to be the case.

Many people dislike science because it is seen to be delivering a picture of humans as exhaustively material beings. Others value a scientific approach because the prime directive is to remain open to the idea that what might seem to "ring true" to us may be profoundly mistaken.
Wayfarer June 24, 2023 at 02:12 #817370
Quoting wonderer1
It's okay if you don't get it.


You haven't provided anything to 'get' save reference to a youtube video and various links.

If you're suggesting a superior paradigm, as distinct from an area of study, then go ahead and do that. So far I see nothing of the kind, beyond vague inferences.

It refers to having an attitude of openness, eagerness, and lack of preconceptions when studying


Something which you've in no way demonstrated. Start an OP on whatever it is you're trying to say, rather than constantly de-railing.


Quoting Janus
Many people dislike science because it is seen to be delivering a picture of humans as exhaustively material beings.


No, that's scientific materialism. It is not the same thing. It mainly comes from the attempt to apply scientific methodology to philosophical problems, as a few here are doing.


wonderer1 June 24, 2023 at 02:21 #817372
Quoting Wayfarer
rather than constantly de-railing.


I will bow out.

Thanks for the thread. I thoroughly enjoyed it.
Srap Tasmaner June 24, 2023 at 02:22 #817373
Quoting Wayfarer
it's not really relevant to the argument at hand.


That may be, but I assure you it was relevant to my understanding what you're talking about.

Quoting Wayfarer
1. No belief is rationally inferred if it can be fully explained in terms of nonrational causes.


Suppose I just deny this. As I understand it, Anscombe did, and argued that there was no reason you could not give both a causal and an inferential account of the same phenomenon.

I accept that the causal explanation is not the same as an inferential explanation, but I do not see why having the one excludes having the other.

How will you persuade me to accept (1)?
Wayfarer June 24, 2023 at 02:24 #817374
Quoting Srap Tasmaner
How will you persuade me?


By giving reasons, yes? As distinct from twisting your arm, or threatening you in some other way.
Janus June 24, 2023 at 02:28 #817376
Quoting Wayfarer
No, that's scientific materialism. It is not the same thing. It mainly comes from the attempt to apply scientific methodology to philosophical problems, as a few here are doing.


Science only deals with subjects insofar as they are material, or physical, if you like. It is only those kinds of inquiries that can be rigorously tested. Other kinds of ideas (like the synthetic generalizations I mentioned earlier) are what we (collectively) cannot imagine being otherwise. Then there things which are true as a matter of logic.

What category do you think the idea your OP consists in is based on?
Wayfarer June 24, 2023 at 02:33 #817377
Quoting Janus
What category do you think the idea your OP consists in is based on?


Philosophy, I would hope. I think the lineage of the argument can plausibly be traced back to the Phaedo.
Janus June 24, 2023 at 02:34 #817378
Reply to Wayfarer That's a non-answer, or an argument from authority, (whivh amounts to being a non-answer) which I suspect you know.

Do you think your argument and/or philosophy itself belongs to any of the categories I outlined? One of them, (variously) all of them or none of them?

And please note, what the ancients, people of other cultures, or other times generally, cannot imagine being otherwise is not necessarily the same as what we moderns cannot imagine being otherwise.
Srap Tasmaner June 24, 2023 at 02:55 #817381
Quoting Wayfarer
By giving reasons, yes?


Proceed.
Wayfarer June 24, 2023 at 05:31 #817389
Reply to Srap Tasmaner I can see you have not been persuaded by the argument thus far and probably won’t be, until you can see a reason why you should accept. At that point, you might typically say I see. So - what is it that you see? (Or in the other case, what is it you’re not seeing?) Whatever it is (or isn’t) it won’t be seen as a consequence of anything physical that has passed between us.
Alkis Piskas June 24, 2023 at 06:28 #817393
Quoting wonderer1
Thoughts are more events than things. See the following link for information about scientists detecting thought events:
https://www.eedesignit.com/oh-no-ai-now-reads-minds/

Interesting! Now, neurologists resort to AI to find answers about the process of thinking!
So they say, "If the participant is open to having their thoughts decoded, their listening to a new story or imagining telling a story allows the machine to generate corresponding text from brain activity alone."
If the decoding of thoughts is a fact, that would have shaken not only the whole scienific community but the whole world!
And also, my dreams would become true! I have been thinking of such a possibily since a very long time ago: being able to project one thoughts on a monitor! What a thrill, eh?
Unfortunetely though, this still belongs to sci-fi ...

Yet, if this some day becomes true, who can guarantee that it is the brain that creates this process and not that it just reacts --as a stimulus-response mechanism that it is-- to thoughts produced from some other source than the brain? Ha!
In fact, this is what's happening: When I'm thinking of a stressful life event willfully --not, unintentionally-- this has inpact on my body because it produces adrenaline and sensorial disorders. This of course happens in communication with the brain, which controls the pituary gland, etc. But it's not the brain that thinks of that stressful event. The brain can only receive stimuli (signals) and react to them.


Alkis Piskas June 24, 2023 at 06:56 #817395
Quoting NOS4A2
If you want to find the “process of thought” watch a person think. Human thought, like believing and reasoning, is an action performed by persons, and not by any other collection of things and processes. If you want to see a cartwheel or a backflip you do the same thing: watch a person perform these actions.

:up:
But if you train your brain hard enough, it will be able to do these things itself, automatically. You wouldn't have to do anything at all. Not even thinking about doing them! :grin:

Quoting NOS4A2
If thoughts are not persons thinking, beliefs are not persons believing, and reasons not persons reasoning, then they are nothing but words without a referent. There is no other way around it.

:up:

Metaphysician Undercover June 24, 2023 at 11:25 #817424
I think Hume provides us with a very good approach to this problem with his discussion of induction and causation. We create inductive rules through empirical observation of what has happened in the past. And from this we can, and do, produce very effective predictions concerning the future. However, we have no empirically derived principle which tells us that things in the future must necessarily be the same as they have been in the past.

So the prediction must always be expressed in terms of probability rather than the necessity which we desire of logic. This means that empirical observation along with inductive reasoning cannot provide us with an understanding of causation, and prediction in general. There is a feature of the universe which allows that empirical observation, with inductive reasoning, can provide us with very good predictions, but empirical observations with inductive reasoning cannot provide us with an understanding of this feature. That is because this mode of prediction which employs empirical observation along with inductive reasoning, to make accurate predictions, takes this feature of the universe for granted.

By taking this feature for granted, as a sort of foundational premise, the logical system built on top of this premise has no capacity to understand the meaning of that premise, it just employs it. The premise employed, or feature of the universe represented, can be portrayed as a temporal continuity. This is the same temporal continuity which is taken for granted by Newton's first law. When these "laws" which are in themselves actually derived from inductive reasoning, are taken for granted as a premise for a logic system (an axiom), the logical system cannot be turned back on itself to question, inquire, or analyze the premise in a skeptical way, because the premise must be taken for granted in order for the system to be applied. So to analyze the fundamental premise (axiom), in order to understand it, we must use principles from outside that system, therefore an independent system.

Now when we get to the very bottom of empirical observation, and the inductive reasoning which deals with it, and we want to understand these features, and how they work in our universe, we must employ principles which are not empirically derived. Therefore our logic must transcend empirical observation, or else we could not ever understand the relationship between past and future.
Mww June 24, 2023 at 12:54 #817438
Reply to Wayfarer

A most valiant effort.
Fooloso4 June 24, 2023 at 13:33 #817443
Quoting Wayfarer
Surely this does at least suggest 'a transcendent realm accessible to the wise'?


In the first paragraph, what " is thought to rule and lead us by nature" does not suggest a transcendent realm. Nor does "what is noble and divine". Contemplation is called divine but it is not and does not lead to transcendence. It is, according to Aristotle, the highest human activity. It is part of the fulfillment or realization or actualization of our nature, not a transcendence of it.

As to the use of the term 'divine': In the Phaedo Socrates calls Homer divine. In the Iliad Homer call salt divine (9.214).

A wise person must have a true conception of unproven first principles
...

Contemplation is that activity in which one's ???? intuits and delights in first principles."


Only a person who is wise can have a true conception of unproven first principles, but only a person who is wise can know whether a conception of unproven first principles is true. If we are not wise then at best we can have an opinion about first principles we assume is true, but cannot prove.

Theoria (contemplation) is related to our term 'theater', to view or see. Plato's cave is a theater. A place in which the distinction between seeing and acting is most pronounced. The prisoners are shackled to the wall and can observe but not act. What they behold is taken to be true and nothing they are able to know contradicts that belief.

The image of the cave is an image of religious practices that seek to experience the sacred and divine. The problem is there is nothing they see that shows them that what they see is not true. Contemplative activity does not provide a reality check. Rather than transcendence it is far more likely to entrap us in a cave with its hope and expectation of transcendence.





Jamal June 24, 2023 at 14:29 #817451
Quoting Wayfarer
I can see you have not been persuaded by the argument thus far and probably won’t be, until you can see a reason why you should accept. At that point, you might typically say I see. So - what is it that you see? (Or in the other case, what is it you’re not seeing?) Whatever it is (or isn’t) it won’t be seen as a consequence of anything physical that has passed between us.


If you think that the use of reason within this discussion demonstrates the soundness of the argument from reason, I think you’ve misunderstood Srap’s criticism.

He asked you to defend this:

Quoting Wayfarer
No belief is rationally inferred if it can be fully explained in terms of nonrational causes


That you will defend it with reason is irrelevant (although you haven’t actually defended it yet).

I’d add that metaphysical naturalism can be true even if beliefs cannot be fully explained in terms of nonrational causes. Nonreductive materialism is popular among philosophers. Which takes us back to @180 Proof’s early response, which I don’t think you’ve addressed.
Gnomon June 24, 2023 at 16:33 #817478
Quoting Tom Storm
Tom, your unwillingness to commit to at least a provisional position on the Random Chaos vs Rational Cosmos question is puzzling to me. — Gnomon
I think that's mostly a problem for you and may explain things. Also 'unwillingness' is not a good word, it implies an ought - I 'ought' to be able to, right? I would say 'inability' would be more appropriate. I hold tentative positions on some matters, and was just writing elsewhere above -

Yes. Our different attitudes towards opinions "may explain things". You seem motivated to avoid dogmatic positions, while I'm interested in discovering moderate "provisional positions". And yet, you do occasionally express a brief succinct opinion on some specific topics. Maybe you only avoid a priori topics that cannot be definitively proven true or false.

Perhaps you think broad general questions --- "some matters", such as Random vs Rational Reality --- are more likely to be answered imperiously, and perhaps based on debatable religious or political postures, instead of hard scientific facts. I'm keenly aware of that danger, but I'm willing to take a chance on exchanging opinions on such fraught topics, on the outside chance that I might learn something philosophically important. Such as "why some opinion exchanges are more polarized than others". :smile:

PS___Regarding the "ought" (moral obligations) aspects of expressed opinions, some might hold that scientific views (beliefs, opinions) ought to be expressed in terms of Factual Particulars, while philosophical perspectives ought to be expressed in Generalities & Possibilities --- or not expressed at all. I've noticed that some posters on this philosophy forum seem to deliberately avoid voicing general or speculative opinions (philosophy ; rationalism), and to restrict their views to particular & empirical facts (science ; naturalism)*1 *2. And they can be rather dogmatic about defending what they see as a wall of separation between Fact (science) & Fiction (philosophy). In that case, perhaps they ought not to be posting on a wishy-washy philosophy forum at all. :cool:


*1. Reply to Janus :
[i]Science only deals with subjects insofar as they are material, or physical, if you like. It is only those kinds of inquiries that can be rigorously tested. Other kinds of ideas (like the synthetic generalizations I mentioned earlier) are what we (collectively) cannot imagine being otherwise. Then there things which are true as a matter of logic.
What category do you think the idea your OP consists in is based on?[/i]
Reply to Wayfarer :
Philosophy, I would hope. I think the lineage of the argument can plausibly be traced back to the Phaedo.

*2 Philosophy and Its Contrast with Science :
We’ll start with what has historically been the most dominant view of the nature of philosophy:[1] let’s call this view ‘rationalism.’[2] After looking at this traditional perspective, we’ll review a more recent view of what philosophy is or should be: ‘naturalism
https://1000wordphilosophy.com/2018/02/13/philosophy-and-its-contrast-with-science/

Srap Tasmaner June 24, 2023 at 17:18 #817485
Reply to Jamal

Obviously I agree, but I'll add one more point, which might count as a sort of non-reductionism.

It seems to me a lot of our traditional "mental" vocabulary does not refer to exclusively internal states of human beings, but rather to mental rather than, I guess, bodily interactions with the environment and objects. We distinguish, and presumably have for a very long time, between chopping down a tree and looking at it, wondering if it's big enough for the beam we need. Both descriptions involve both the guy with the sharp implement and the tree, so just as doesn't map cleanly onto postures and movements of my body alone, in the absence of a tree, so needn't map onto something going on in my brain in the absence of a tree.

As it happens, representational theories of mind will map the necessary tree onto my internal representation of the tree, and you'll see often on this forum theories that claim my goal in either case to produce a certain state of my internal model. I think that's a very different issue from whether our everyday vocabulary around thinking, perceiving, imagining, remembering, and so on, not only presupposes objects for these activities but folds them into terms that are in some ways holistic.

Does that make any sense?
180 Proof June 24, 2023 at 19:20 #817492
Quoting Jamal
Nonreductive materialism is popular among philosophers. Which takes us back to 180 Proof’s early response, which I don’t think you’ve addressed.

@Wayfarer won't because he dogmatically cannot.
Wayfarer June 24, 2023 at 21:51 #817512
Quoting Jamal
That you will defend it with reason is irrelevant (although you haven’t actually defended it yet).


I believe I have successfully defended it in numerous places in this dialogue, and I don’t think Srap’s criticisms come to terms with the argument. Say if I do persuade you to believe any proposition whatever - not necessarily this one - where you agree that you 'see the point' of the argument - how can that be understood in any terms other than rational persuasion? What is it that you see, when you see the point? What about that kind of transaction can be described in physical terms? I think that's a very direct question, which prompted nothing more than circumlocution.

Reply to 180 Proof This is because it is my dogmatic belief that matter does not act, but is only acted upon.
Wayfarer June 24, 2023 at 22:32 #817524
Quoting Jamal
Nonreductive materialism is popular among philosophers.


And I would guess that this is because it purportedly gets them off the hook from having to defend the ridiculous implications of reductive materialism, whilst not giving ground to anything that sounds like dualism or idealism. In which context, 'supervenience' serves as a kind of convenient gap-filler.

//furthermore, the OP stipulates that the argument from reason is directed at reductive materialism. It is arguable that the reason for the development of such views as 'non-reductive materialism' was just because of such challenges as the argument from reason.//

Quoting Srap Tasmaner
our everyday vocabulary around thinking, perceiving, imagining, remembering, and so on, not only presupposes objects for these activities but folds them into terms that are in some ways holistic.


That would be the subjective unity of perception - which is another thing that is not explainable in physicalistic terms, as the linked article explains.
Janus June 24, 2023 at 22:57 #817530
Quoting Wayfarer
Say if I do persuade you to believe any proposition whatever - not necessarily this one - where you agree that you 'see the point' of the argument - how can that be understood in any terms other than rational persuasion?


The neural networks in your brain have been formed by your unique life experience and genetic constitution such that hearing the proposition affects those networks so as to produce the result that the proposition is convincing to you, whereas it might not be so to others, whose brains' neural structures have formed differently, due to different life experiences. If it was all just pure sovereign reason, then why would everyone who can think rationally not agree about everything?

Of course, I'm not claiming this explanation or at least something along those lines is anything other than the best empirical explanation we currently have. If logic doesn't by itself doesn't tell us anything, and a primary principle of rational thinking is not to believe anything without sufficient evidence, then what other possible cogent explanation is there?

Should we believe something simply because it feels right to us? I don't say we shouldn't, but I also believe in that case we must relinquish the claims that our belief is rationally-based and accept that is is a matter of faith. Matters of faith cannot be cogently argued for.

The other point of inconsistency in your position seems to be that you think rational persuasion is a matter of free will; but how could it be if rational persuasion is as strict as valid logical deduction is?
Srap Tasmaner June 24, 2023 at 23:09 #817531
Quoting Wayfarer
I don’t think Srap’s criticisms come to terms with the argument


That may very well be, but I have tried. I'm not even sure I've posted a criticism of the argument so much as I've tried and failed to understand it.

Apropos seeing the point, a gift:


Wayfarer June 24, 2023 at 23:16 #817534
Quoting Srap Tasmaner
I'm not even sure I've posted a criticism of the argument so much as I've tried and failed to understand it.


That's what it seems to me, but I appreciate the fact that you directly addressed it, rather than changing the subject or digressing.

Quoting Janus
If it was all just pure sovereign reason, then why would everyone who can think rationally not agree about everything?


That's subjectivism and relativism. It's obvious that, for example, in the American political scene, there is a huge polarization, but it's no coincidence that a large part of the cause of this is a leading political figure who quite openly tells enormous lies which large numbers of the electorate are willing to swallow. Surely a political scientist or pollster can come up with reasons why they do that, but it doesn't change the fact that they're believing lies, and that they are lies, regardless of anyone's opinion.
Janus June 24, 2023 at 23:24 #817540
Reply to Srap Tasmaner :rofl: Gave me a laugh!

Quoting Wayfarer
That's subjectivism and relativism. It's obvious that, for example, in the American political scene, there is a huge polarization, but it's no coincidence that a large part of the cause of this is a leading political figure who quite openly tells enormous lies which large numbers of the electorate are willing to swallow. Surely a political scientist or pollster can come up with reasons why they do that, but it doesn't change the fact that they're believing lies, and that they are lies, regardless of anyone's opinion.


The question was about people who are (presumably) capable of thinking strictly rationally, philosophers for example; what explains the fact that there is so much disagreement among them?
Wayfarer June 24, 2023 at 23:33 #817543
Quoting Janus
The other point of inconsistency in your position seems to be that you think rational persuasion is a matter of free will; but how could it be if rational persuasion is as strict as valid logical deduction is?


As you point out, logic itself is not necessarily descriptive of the world of experience - something can be logically true but physically impossible, and things happen which defy logic. The argument from reason is not presenting reason as a panacea or magic bullet, but as an indispensable faculty which can't be explained in physicalist terms (among other things.

Quoting Janus
what explains the fact that there is so much disagreement among them?


It's the old Indian elephant parable; that we've only ever got a grasp of some aspect of the elephant, its tail or its ear or tusk and we form conclusions on that basis. Reality, or rather, Being, is infinitely vast, and coming to an understanding exceedingly difficult.
Janus June 24, 2023 at 23:45 #817548
Quoting Wayfarer
As you point out, logic itself is not necessarily descriptive of the world of experience - something can be logically true but physically impossible, and things happen which defy logic. The argument from reason is not presenting reason as a panacea or magic bullet, but as an indispensable faculty which can't be explained in physicalist terms (among other things.


I don't think many would disagree that reason is indispensable to human life. However, even if the semantic content of logic or rational thought cannot be described in terms of fundamental physics (to expect that would be to commit a category error), it doesn't follow that there can be no physical, neurological or evolutionary explanations for the fact that humans are capable of rational thought.

Quoting Wayfarer
It's the old Indian elephant parable; that we've only ever got a grasp of some aspect of the elephant, its tail or its ear or tusk and we form conclusions on that basis. Reality, or rather, Being, is infinitely vast, and coming to an understanding exceedingly difficult.


No one is omniscient, or even close to it, and some know more than others to be sure. A neuroscientist presumably knows much more about neuroscience than you or I do, for example, and knowledge has become so specialized that it seems laypeople such as ourselves are in no position to criticize a neurological understanding of the mind and brain unless we have sufficient training in the discipline.
Wayfarer June 24, 2023 at 23:52 #817551
Quoting Janus
it doesn't follow that there can be no physical, neurological or evolutionary explanations for the fact that humans are capable of rational thought.


You're quite right. Please note the terms of reference provided in the original post. It is quite specific about the type of physicalism the argument has in its sights. The fact that naturalism has changed signficantly in the last 60 odd years in regard to physicalism (for example, with the introduction of biosemiotics) might be due to the growing realisation of the inadequacy of physicalism, although the original physicalism is still defended by the philosophers that I have mentioned in this thread (such as Daniel Dennett).

As I've said many times, I do not doubt that fact of evolution, but the way I put it is that h. sapiens has developed the capacity for reason. I still say it's a mistake to say conceive of reason as result of evolutionary biology, because it reduces reason to mere adaptation (the argument at the basis of Nagel's Evolutionary Naturalism and the Fear of Religion* and also Plantinga's evolutionary argument against naturalism.)

*From which: 'I suspect that there is a deep-seated aversion in the modern “disenchanted” Weltanschauung to any ultimate principles that are not dead--that is, devoid of any reference to the possibility of life or consciousness.'
Janus June 25, 2023 at 00:13 #817555
Quoting Wayfarer
I still say it's a mistake to say conceive of reason as result of evolutionary biology, because it reduces reason to mere adaptation.


I'm not seeing why acquiring the capacity for reason should not be thought of as a result of evolutionary biology, driven by adaptation. We know that thinking is correlated with neuronal processes, and we also know that we can formulate the principles or rules of rational thought or logic. And further to that we know, due to the obvious adaptive superiority of humans over other animals, which most plausibly would seem to be on account of language and the evolutionary complexification of rational thought it enables, that language and rational thought must have adaptive advantages. What am I missing?
Wayfarer June 25, 2023 at 00:30 #817557
Quoting Janus
I'm not seeing why acquiring the capacity for reason should not be thought of as a result of evolutionary biology, driven by adaptation.


Because there's nothing in the theory that addresses it specifically. The theory is about the factors that contribute to the survival and evolution of species. It's a biological theory, but the view that it accounts for everything about human nature and the human condition is biological reductionism (whereas I would say that we're 'underdetermined' by biological factors). Historically, because evolutionary biology stepped into the vacuum left by the abandonment of religion, then such capabilities as the faculty of reason are simply assumed to be explained by that theory. But Alfred Russel Wallace, for one, never accepted it - see his essay Darwinism Applied to Man

I should add, the expression 'neo-Darwinian materialism' which I already mentioned, describes the view that living things are material in nature, that were formed by some as-yet-to-be-understood process designated 'abiogenesis', allowing the 'Darwinian algorithm' (Dennett's phrase) to take off, giving rise to ever-more complex life-forms over hundreds of millions of years. This is the sense in which neo-Darwinianism is materialist. Whereas the kind of dualist philosophy that I'm tentatively exploring, sees the earliest life as the manifestation of intentionality, not necessarily as a consequence of theistic creation (although that might be thought of as a metaphorical account), but as an expression of a latent attribute of the Cosmos being made manifest through the process of evolution. In other words, that conscious beings manifest through the processes of evolution. This leaves space for consciousness to be understood as having a formative role, rather than as an accidental byproduct of a fortuitous process (Bertrand Russell's 'accidental collocation of atoms'.)
L'éléphant June 25, 2023 at 00:44 #817558
Quoting Wayfarer
I'm not seeing why acquiring the capacity for reason should not be thought of as a result of evolutionary biology, driven by adaptation. — Janus

Because there's nothing in the theory that addresses it specifically. The theory is about the factors that contribute to the survival and evolution of species.

No, the study of civilization, which includes evolution, is also the study about the intelligence of humans over epochs. What Janus might be referring to is the study of logic, which is a modern development. The "capacity for reason", as civilization reveals, is actually the capacity to use tools, for example.
Janus June 25, 2023 at 00:54 #817560
Quoting Wayfarer
. It's a biological theory, but the view that it accounts for everything about human nature and the human condition is biological reductionism (whereas I would say that we're 'underdetermined' by biological factors).


I'd say that with the advent and evolutionary development of culture, factors other than the merely biological certainly came into play, plausibly to the point of predominance. Don't forget, though that evolution is not merely a biological determinant, since animals, including humans have to cope with and survive in environments. Another point is that cultures themselves and their survival and flourishing are determined by geographical and biological, as well as human factors, so there is no clear line beyond which biology becomes irrelevant.

In any case saying that the capacity for language and rational thought evolved, driven initially by adaptive advantage, is not to say that everything about humans is determined by fitness for survival and reproduction.

Quoting L'éléphant
No, the study of civilization, which includes evolution, is also the study about the intelligence of humans over epochs. What Janus might be referring to is the study of logic, which is a modern development. The "capacity for reason", as civilization reveals, is actually the capacity to use tools, for example.


:up: Yes, tool use is one very significant thing that distinguishes us from most other animals, and the degree to which we have developed that distinguishes us from all other known animals. Language and logic can also be considered to be tools, but I would be loath to say that they are nothing but tools.
Paine June 25, 2023 at 01:00 #817563
Reply to Wayfarer

I have many conflicting views of what is "spiritual" But I am not down with this:

A.R. Wallace: As contrasted with this hopeless and soul-deadening belief, we, who accept the existence of a spiritual world, can look upon the universe as a grand consistent whole adapted in all its parts to the development of spiritual beings capable of indefinite life and perfectibility. To us, the whole purpose, the only raison d'être of the world--with all its complexities of physical structure, with its grand geological progress, the slow evolution of the vegetable and animal kingdoms, and the ultimate appearance of man--was the development of the human spirit in association with the human body. From the fact that the spirit of man--the man himself--is so developed, we may well believe that this is the only, or at least the best, way for its development; and we may even see in what is usually termed "evil" on the earth, one of the most efficient means of its growth. For we know that the noblest faculties of man are strengthened and perfected by struggle and effort; it is by unceasing warfare against physical evils and in the midst of difficulty and danger that energy, courage, self-reliance, and industry have become the common qualities of the northern races; it is by the battle with moral evil in all its hydra-headed forms, that the still nobler qualities of justice and mercy and humanity and self-sacrifice have been steadily increasing in the world. Beings thus trained and strengthened by their surroundings, and possessing latent faculties capable of such noble development, are surely destined for a higher and more permanent [[p. 478]] existence; and we may confidently believe with our greatest living poet--



That life is not as idle ore,
But iron dug from central gloom,
And heated hot with burning fears,
And dipt in baths of hissing tears,
And batter'd with the shocks of doom
To shape and use.
Wayfarer June 25, 2023 at 01:05 #817566
Reply to Paine Fair point. A lot of Wallace' Victorian prose is pretty hideous in today's terms, specifically all the references to 'savages', but I still think the argument that such faculties as mathematical and artistic abilities can't be accounted for in terms of the theory is sound. And also the fact that the guy credited as co-discoverer of natural selection had such divergent views on those matters to Darwin.
Paine June 25, 2023 at 01:42 #817574
Reply to Wayfarer
I am not (only) appalled by Wallace's ranking of different societies. Darwin did not fill in the cultural development dimension that Wallace does. Maybe that silence counts for something.
I like sushi June 25, 2023 at 02:30 #817585
Reply to Wayfarer I have never heard the term ‘scare quotes’. You used ‘exist’ so think of it in those terms. I was being cautious with your possible interpretation of what ‘exist’ means.

I would like you to explain what you mean by ‘true belief’ if you have the time. I have a feeling you do not wish to dive into any epistemic issues here but given that what I said makes no sense to you there must be something I failed to take into account?

My general point is that rationality is applied to experience. I felt like there was an error with mixing abstract and real.
Srap Tasmaner June 25, 2023 at 02:48 #817588
Quoting Wayfarer
mathematical and artistic abilities can't be accounted for in terms of the theory


We're the only critters we know that have math and art, and we are the way we are because of natural selection, so evidently it does account for math and art.

As someone somewhere on this forum once said, the answer to "How long would it take monkeys to compose the complete works of Shakespeare?" is about 300,000 years. That experiment has already been run.

Question: how important to the argument from reason is your unusual interpretation of human evolution?
180 Proof June 25, 2023 at 03:45 #817594
Reply to Janus :up:

Reply to Srap Tasmaner :up: :up:

Quoting Wayfarer
This is because it is my dogmatic belief that matter does not act, but is only acted upon.

If "matter does not act", then "matter" "is only acted upon" by what? Please cite an example.
Wayfarer June 25, 2023 at 05:01 #817603
Quoting 180 Proof
If "matter does not act", then "matter" "is only acted upon" by what


Say if I suggested 'mind' or 'consciousness' as a hypothetical answer - how could you go about defining that in objective terms? You can't ever cognise mind as an object - it doesn't appear to us, it appears as us.

Quoting I like sushi
I would like you to explain what you mean by ‘true belief’ if you have the time. I have a feeling you do not wish to dive into any epistemic issues here but given that what I said makes no sense to you there must be something I failed to take into account?

My general point is that rationality is applied to experience. I felt like there was an error with mixing abstract and real.


The term 'scare quotes' is used to refer to the use of quotes to indicate that a word is being used in an ironic, referential, or otherwise non-standard sense.

In respect of 'mixing abstract and real', I can see what you mean but the type of argument that it is does not appeal to empirical validation.

Quoting Srap Tasmaner
"How long would it take monkeys to compose the complete works of Shakespeare?" is about 300,000 years. That experiment has already been run.


Really? They must have started a long while ago!

Quoting Srap Tasmaner
Question: how important to the argument from reason is your unusual interpretation of human evolution?


My view is that whilst h. sapiens evolved in line with the understanding of evolutionary biology (subject as it is to frequent revision) that the faculty of reason, and other specifically human attributes and powers, can't necessarily be accounted for in Darwinian terms. With simpler life-forms, the issue is not applicable, as they are occupied wholly and solely with survival and procreation. Only when species evolved to the level of h. sapiens, did living beings who possess the kind of rational self-awareness to question the nature of reality and existence appear. With that ability comes existential dread and much else besides. I feel one of the unfortunate consequences of popular Darwinism is that it has lost sight of this. It doesn't really do anything to address the human condition on the philosophical level. Evolutionary materialism, such as Dennett/Dawkins, amply reinforce that impression by being so utterly, philosophically tone-deaf. (Not nearly so much the continental philosophers.)
I like sushi June 25, 2023 at 06:11 #817609
Reply to Wayfarer Fair enough. It is solely an abstract inquiry then I am not that bothered by abstract justification. I would also say that belief is not at all relevant for such argumentation either as what is true is true and demonstrable by abstract means.

A purely rational argument (viewed as wholly abstract) against naturalism/materialism/physicalism is waste of everyone’s time due to the obvious cross contamination.

I would still be interested to hear what ‘true belief;’ is in the context of your views here?
180 Proof June 25, 2023 at 07:41 #817616
Quoting Wayfarer
Say if I suggested 'mind' or 'consciousness' as a hypothetical answer -

By "mind or consciousness" you're claiming, in effect, that matter is only acted upon by immateral entities or processes – is that right?
Wayfarer June 25, 2023 at 08:05 #817619
Reply to 180 Proof I'm sure that whatever way we try and conceive of as 'an immaterial entity or process' will miss the mark. It requires, as one of the earlier contributors to this thread was wont to say, 'a paradigm shift'. Explaining all of the implications of that would take a lot of time. But one of the essays I've mentioned makes this point in respect to the differentiation between organisms and the inorganic domain. The context is that the author is arguing that an appeal to something other than physicalism doesn't necessarily imply an acceptance of some spooky 'elan vital'. He wishes to abide within the constraints of naturalism, whilst questioning mechanistic materialism. He writes:

Quoting From Physical Causes to Organisms of Meaning, Steve Talbott
The scientist observes meanings at play in organisms, and appeals to them in biological explanation. Anyone who construes this appeal as conjuring unacceptable vital forces needs not only to torch almost the entire biological literature, reconstructing it upon some new and as yet unknown basis; he also puts himself in an untenable position regarding the human being. For at least some of what we do, we do because we consciously think and intend it. If invoking this because of reason — this play of meaning and idea — in the explanation of human behavior is to rely on vital forces, then virtually everyone (in daily life, if not within their cocoon of theory) is a vitalist. If, on the other hand, we grant meaning to the human being without trying to make this meaning an expression of vital forces, then we can hardly voice the charge of “vitalism” when we observe meaningful activity in less conscious forms — for example, in the activity of cells and lower organisms.

So, no, we don’t need vital forces. If the organism as an expression of meaning requires us to recognize a different sort of order from that of inanimate nature, science offers no presumption against this. Our knowledge of some thought-relations in the world — for example, those of mathematized physical law — does not tell us what other thought-relations we might discover in various domains. The mathematical order, however, does tell us that there must be other principles of order. For mathematics alone doesn’t give us any things or phenomena at all; numbers are not things. Whatever the things may be to which our mathematical formulations refer, they either have a qualitative character that we can consciously apprehend in a conceptually ordered way, or they must remain unknown and outside our science. And that qualitative conceptual ordering cannot be predicted from the mathematics. Rather, the qualitative order is the fuller reality that determines whatever we abstract from it, including mathematical relationships.


There's another book that was mentioned by @Pierre-Normand a while back, Rational Causation, by Eric Marcus. I think it is compatible with this general line of reasoning. The abstract says:

We explain what people think and do by citing their reasons, but how do such explanations work, and what do they tell us about the nature of reality? Contemporary efforts to address these questions are often motivated by the worry that our ordinary conception of rationality contains a kernel of supernaturalism—a ghostly presence that meditates on sensory messages and orchestrates behavior on the basis of its ethereal calculations. In shunning this otherworldly conception, contemporary philosophers have focused on the project of “naturalizing” the mind, viewing it as a kind of machine that converts sensory input and bodily impulse into thought and action. Eric Marcus rejects this choice between physicalism and supernaturalism as false and defends a third way.

He argues that philosophers have failed to take seriously the idea that rational explanations postulate a distinctive sort of causation—rational causation. Rational explanations do not reveal the same sorts of causal connections that explanations in the natural sciences do. Rather, rational causation draws on the theoretical and practical inferential abilities of human beings. Marcus defends this position against a wide array of physicalist arguments that have captivated philosophers of mind for decades. Along the way he provides novel views on, for example, the difference between rational and nonrational animals and the distinction between states and events.


Bolds added. This is where the argument appears to converge with the argument from reason, although I haven't laid hands on the book yet.
180 Proof June 25, 2023 at 08:29 #817624
Reply to Wayfarer Let's keep things simple and clear, Wayfarer. I'm interested in your dogmatic statement about matter and have questioned you here
https://thephilosophyforum.com/discussion/comment/817594
and again in my last post. Either you can answer the question I have asked or you can't. Quoting walls of other people's texts without answering my questions comes across as telling me to "fuck off". :brow:
Wayfarer June 25, 2023 at 11:44 #817635
Reply to 180 Proof Shall I paraphrase?
180 Proof June 25, 2023 at 11:53 #817637
Quoting Wayfarer
Shall I paraphrase?

Given that I'd addressed your statement, sir, please "paraphrase" what you think, not what others think. You do think for yourself, don't you?

@Jamal
Jamal June 25, 2023 at 12:40 #817645
Quoting Srap Tasmaner
Does that make any sense?


I'm not finding much time to dedicate to TPF right now, except for posting photos of my lunch. I'll try to come back to it.
Fooloso4 June 26, 2023 at 13:15 #817978
Quoting Wayfarer
I'm sure that whatever way we try and conceive of as 'an immaterial entity or process' will miss the mark. It requires, as one of the earlier contributors to this thread was wont to say, 'a paradigm shift'.


The reason why we miss the mark is simple. We have no knowledge or experience of any immaterial entity of process. Absent evidence, reasoned argument that such may or must exist is idle speculation and leads nowhere. We start with material or physical entities and make the misguided move of imagining the entity or process minus what is essential to it as if nonessential.
180 Proof June 26, 2023 at 18:49 #818043
Quoting Fooloso4
We have no knowledge or experience of any immaterial entity of process. Absent evidence, reasoned argument that such may or must exist is idle speculation and leads nowhere.

:100: :up:
Paine June 26, 2023 at 20:36 #818081
Reply to Fooloso4

Lloyd Gerson:….the fact that in thinking, your mind is identical with the form that it thinks, means (for Aristotle and for all Platonists) that since the form 'thought' is detached from matter, 'mind' is immaterial too.


The arguments in Aristotle do not follow this line of reasoning. The "identity" with the object is not a simple correspondence of "forms". Aristotle goes to great pains in his Metaphysics to distinguish between the relatively easy task of grouping beings by kinds from understanding causes. The often repeated maxim is that "we move from what is known by us to what is known by nature." Toward that end, we can establish some principles by analogy and others through experience. Gerson consistently overlooks the importance of this distinction when discussing substance (ousia) in Aristotle's writings. The idea of intellect as a pure process is presented as something we will never be able to directly experience for ourselves:

Quoting Aristotle
And in fact there is one sort of understanding that is such by becoming all things, while there is another that is such by producing all things in the way that a sort of state, like light, does, |430a15| since in a way light too makes potential colors into active colors.363 And this [productive] understanding is separable, unaffectable, and unmixed, being in substance an activity (for the producer is always more estimable than the thing affected, and the starting-point than the matter), not sometimes understanding and at other times not. But, when separated, this alone is just what it is.365 And it alone is immortal and eternal (but we do not remember because this is unaffectable, whereas the passive understanding is capable of passing away), and without this it understands nothing.


This idea of a self-sufficient process is closely bound with a very messy material set of conditions:

Aristotle, On Memory, 1 450a25–b5:A problem might be raised as to how, when the affection is present but the thing producing it is absent, what is not present is ever remembered. For it is clear that one must understand the affection, which is produced by means of perception in the soul, and in that part of the body in which it is, as being like a sort of picture, the having of which we say is memory. For the movement that occurs stamps a sort of imprint, as it were, of the perceptible object, as people do who seal things with a signet ring. That is also why memory does not occur in those who are subject to a lot of change, because of some affection or because of their age, just as if the change and the seal were falling on running water. In others, because of wearing down, as in the old parts of buildings, and because of the hardness of what receives the affection, the imprint is not produced


Tom Storm June 26, 2023 at 21:20 #818090
Reply to Wayfarer While I don't think we can demonstrate that reason can't be arrived at through natural processes, I'd be interested to learn where this is heading.

Let's say that reason can not be explained by naturalism.

What follows from this, for you?

(I know this argument is a central platform in presuppositional Islamic and Christian apologetics - that the very possibility of intelligibility can't be explained by materialism and therefore materialism disproves itself.)

For you, I imagine this reasoning is foundational to idealism, right?

These arguments seem to get messy - if idealism is true than presumably it belongs to naturalism. The natural then being an ontology of consciousness? Discerning precisely what is meant by materialism, physicalism or naturalism can seem tricky.
Wayfarer June 26, 2023 at 21:44 #818100
Quoting Fooloso4
We have no knowledge or experience of any immaterial entity or process.


My claim is that the perceived dichotomy between material and immaterial is a consequence of Cartesian dualism. Recall that Descartes posited mind as 'res cogitans', literally a 'thinking thing'. This leads to the problem of how the thinking substance interacts with 'material substance' and the so called 'ghost in the machine' argument. It is foundational to the modern mind-matter conceptual division, based on the premisses that physical sciences must provide a mathematically precise quantitative description of an external reality extended in space and time and specified in terms of the primary qualities such as shape, size, and motion, and to laws governing the relations among them. Subjective appearances, on the other hand -- how this physical world appears to human perception -- were relegated to the mind, and the secondary qualities like color, sound, and smell were to be analyzed relationally, in terms of the power of physical things, acting on the senses, to produce those appearances in the minds of observers. It was essential to leave out or subtract subjective appearances and the human mind -- as well as human intentions and purposes -- from the physical world. This is the basis of the modern 'mind-matter problem', and what the argument from reason is aimed at. And within that paradigm, there are no immaterial entities as a matter of definition. That's why I said that whatever we try and conceive of as an immaterial entity will miss the mark , as it will invariably interpreted in those terms, which I'm sure you're doing.

But the question was actually this:

Quoting 180 Proof
This is because it is my dogmatic belief that matter does not act, but is only acted upon.
— Wayfarer
If "matter does not act", then "matter" "is only acted upon" by what?


In order to try and illustrate the alternative way of approaching it, I provided text of a couple of readings, apparently too long. So to try and paraphrase what I think the first reference is getting at: the non-material or non-physical factor involved is life itself. Living beings, even the very simplest beings, display attributes and characteristics that actually can't be accomodated in the mind-body duality that is embedded in the modern worldview. Steve Talbott's biological philosophy is that organisms are expressions of meaning. They're not mechanisms to which we have to attribute a spooky 'elan vital' to account for the uncanny abilities which life manifests. As soon as life begins to manifest, then we have the emergence of an order which can't be reduced to, or explained in terms of, only physical processes; organisms have their own reasons for acting as they do. At the very beginning there is the appearance of the 'epistemic split' of self-other, the beginning of cellular memory and genetic inheritance, and a very primitive form of subject-hood. So the subject, the mind, is not the accidental by-product of a material process, but way in which agency appears in the earliest forms of life. And, to appeal to Indian philosophy, tat tvam asi, 'thou art that'.

And in a broader sense, many of our intellectual processes rely on immaterial entities, such as numbers, ratios, laws, and so on. Humans are situated between two worlds, so to speak - the physical world, governed by the laws of physics, but also the world of ideas and reasons, 'the space of reasons' as it has been called. That is the argument of the second book, Rational Causation by Eric Marcus. I don't know if that book talks about the argument from reason as such (haven't had the chance to read it yet) but it seems to operate from similar premisses.

So by 'immaterial' I don't mean spooky ghosts in machines, or some 'ethereal realm of ideal forms'. But as soon as physicalism is questioned, that is what comes to mind, isn't it?

Quoting Paine
The arguments in Aristotle do not follow this line of reasoning. The "identity" with the object is not a simple correspondence of "forms".


Sure, you might be right. But in context, Gerson's point was this: 'when you think you see—
mentally see—a form which could not in principle be identical with a particular, including a
particular neurological element, a circuit or a state of a circuit or a synapse, and so on. This is so
because the object of thinking is universal, or the mind is operating universally. For example,
when you think ‘equals taken from equals are equal’ this is a perfectly universal truth which you
see when you think it. But this truth, since it is universal could not be identical with any
particular, any material particular located in space and time.' Which makes perfect sense to me.

Quoting Tom Storm
Let's say that reason can not be explained by naturalism.

What follows from this, for you?


The crux of this whole thread was an un-answered question:

I can see you have not been persuaded by the argument thus far and probably won’t be, until you can see a reason why you should accept. At that point, you might typically say 'I see'. So - what is it that you see? (Or in the other case, what is it you’re not seeing?) Whatever it is (or isn’t) it won’t be seen as a consequence of anything physical that has passed between us.


What do you make of that?


Tom Storm June 26, 2023 at 21:51 #818102
Quoting Wayfarer
The crux of this whole thread was an un-answered question:

I can see you have not been persuaded by the argument thus far and probably won’t be, until you can see a reason why you should accept. At that point, you might typically say 'I see'. So - what is it that you see? (Or in the other case, what is it you’re not seeing?) Whatever it is (or isn’t) it won’t be seen as a consequence of anything physical that has passed between us.

What do you make of that?


Yes, I read that earlier. I have no expertise in this subject. The best I can say is that intelligent, well informed people are 1) persuaded and 2) are not persuaded.
Gnomon June 26, 2023 at 21:56 #818103
Quoting Srap Tasmaner
As someone somewhere on this forum once said, the answer to "How long would it take monkeys to compose the complete works of Shakespeare?" is about 300,000 years. That experiment has already been run.

Interesting! Do you have a link to that experiment? How many monkeys involved (n=?)? Does it assume that the monkeys bang away randomly, or have they been taught to type purposefully --- as they do when pounding nuts with rocks? Compared to feckless philosophy, unfettered Science gets results. Oh, did the experiment begin 300,000 years ago, or did they use a Black Hole to accelerate time? :joke:

FWIW, here's what Wiki has to say on the Infinite Monkey Theorem : a thought experiment. :smile:

Infinite monkey theorem :
[i]The theorem can be generalized to state that any sequence of events which has a non-zero probability of happening will almost certainly eventually occur, given unlimited time. . . . .
Even if every proton in the observable universe (which is estimated at roughly 1080) were a monkey with a typewriter, typing from the Big Bang until the end of the universe (when protons might no longer exist), they would still need a far greater amount of time – more than three hundred and sixty thousand orders of magnitude longer – to have even a 1 in 10500 chance of success. To put it another way, for a one in a trillion chance of success, there would need to be 10360,641 observable universes made of protonic monkeys.[g] As Kittel and Kroemer put it in their textbook on thermodynamics, the field whose statistical foundations motivated the first known expositions of typing monkeys,[4] "The probability of Hamlet is therefore zero in any operational sense of an event ...", and the statement that the monkeys must eventually succeed "gives a misleading conclusion about very, very large numbers."[/i]
https://en.wikipedia.org/wiki/Infinite_monkey_theorem


A RATIONAL INTENTIONAL MONKEY times infinity
User image
Wayfarer June 26, 2023 at 22:02 #818104
Quoting Tom Storm
I have no expertise in this subject.


C'mon. What expertise is needed? Either you see a reason or you don't. What I'm asking you is that if I persuade you to accept something - not even the argument at hand, but anything - has anything physical passed between us?
Wayfarer June 26, 2023 at 22:04 #818105
Reply to Gnomon I'd really appreciate it if you deleted that inane graphic.
180 Proof June 26, 2023 at 22:06 #818106
Quoting Tom Storm
Discerning precisely what is meant by materialism, physicalism or naturalism can seem tricky.

They are only "tricky" for idealists like @Wayfarer who prefer to torch strawmen – mischaracterizing a speculative paradigm such as naturalism as an explanatory theory – which is far easier to do than to demonstrate that idealism is a less ad hoc, less incoherent, less subjective paradigm than naturalism, etc. Naturalism does not explain "consciousness", yet idealism – which rationalizes folk psychological concepts (often ad absurdum) – conspicuously explains "consciousness" even less so.

Quoting Wayfarer
And in a broader sense, many of our intellectual processes rely on immaterial entities, such as numbers, ratios, laws, and so on. Humans are situated between two worlds, so to speak - the physical world, governed by the laws of physics, but also the world of ideas and reasons, 'the space of reasons' as it has been called.

Unwarranted, question-begging, substance dualism as well as a reification / misplaced concreteness fallacy (à la platonism). Abstractions themselves do not "act upon matter" because they are not evental (or causal); rather instantiations (encoding / patterning) of abstractions (from matter) in matter act upon matter (e.g. typing on my keyboard these sentences you're reading on your screen), which refutes your dogma, sir, that "matter does not act but is only acted upon" (as if Newton's 3rd Law & conservation laws are violated, or miraculosly suspended, by "ideas"). :eyes: :roll:

Quoting Wayfarer
... has anything physical passed between us?

Of course, information (i.e. instantiated patterns).
Wayfarer June 26, 2023 at 22:41 #818112
Quoting 180 Proof
Of course, information (i.e. instantiated patterns).


Of course you will assume that information is physical, given that everything must be.

Quoting 180 Proof
(as if Newton's 3rd Law & conservation laws are violated, or miraculosly suspended, by "ideas").


The apparently inviolable constraints of physical laws have been transcended many times in the history of science. We have been able to discover hitherto unknown attributes and properties of the world through the unreasonable effectiveness of mathematical physics. But you continue to enjoy your procrustean bed, although I think it must be an awful fit.
Tom Storm June 26, 2023 at 22:47 #818114
Quoting Wayfarer
Either you see a reason or you don't. What I'm asking you is that if I persuade you to accept something - not even the argument at hand, but anything - has anything physical passed between us?


What 'I see' is not really relevant. I see words on a computer screen typed by a person (I assume) who has beliefs/thoughts. I see nothing so far that is not physical. Are thoughts physical? Can we demonstrate that thoughts do not originate from physical brains? Isn't this where the expertise comes in?

Quoting 180 Proof
They are only "tricky" for idealists like Wayfarer who prefer to torch strawmen – mischaracterizing a speculative paradigm such as naturalism as an explanatory theory – which is far easier to do than to demonstrate that idealism is a less ad hoc, less incoherent, less subjective paradigm than naturalism, etc. Naturalism does not explain "consciousness", yet idealism – which rationalizes folk psychological concepts (often ad absurdum) – conspicuously explains "consciousness" even less so.


Yes, it's hard for me not to agree with this.
Wayfarer June 26, 2023 at 23:08 #818118
Quoting Tom Storm
What 'I see' is not really relevant


It's highly relevant. When you say, 'ah, I see' - what is it you're seeing?

Quoting Tom Storm
Are thoughts physical?


That is indeed the point at issue. In the physicalist view that is the subject of the argument, everything that exists, does so as a consequence of physical causation, due to the causal closure of the physical domain. So in that view, your thoughts are likewise determined by physical causes, that can be understood in terms of neurobiology or physiology. Mind is an output or consequence of matter. That is exemplified by Daniel Dennett's theory that what we experience as consciousness is really the consequence of millions of physical processes giving rise to 'unconscious competence'.

The counter to that is that when you see causal relationships between ideas, that this is distinct from the mindless processes typically invoked by physicalism. You're seeing the connection between ideas. That is a different process to that of physical causation.

Furthermore, if I write something that perturbs or upsets you, that will have physical consequences - blood pressure, adrenal reaction, heart rate, etc. But what has affected you, is not a physical influence, like my giving you a tablet or striking you. Your metabolic condition has been affected by a perception of meaning. That is an example of 'top-down' causation that I think generally supports the argument.


Paine June 26, 2023 at 23:34 #818124
Quoting Wayfarer
Sure, you might be right. But in context, Gerson's point was this: 'when you think you see—
mentally see—a form which could not in principle be identical with a particular, including a
particular neurological element, a circuit or a state of a circuit or a synapse, and so on. This is so
because the object of thinking is universal, or the mind is operating universally. For example,
when you think ‘equals taken from equals are equal’ this is a perfectly universal truth which you
see when you think it. But this truth, since it is universal could not be identical with any
particular, any material particular located in space and time.' Which makes perfect sense to me.


If I am correct, then Gerson has misunderstood Aristotle. I recognize that you want to use Gerson to leverage an argument against reduction in the context of the scientific method. I don't have a clear understanding of those matters and am loath to put forward an exact definition in the style of the SEP.

But I have read enough text to question Gerson's assertions and look forward to challenging anyone who would champion his position as a scholar.
Tom Storm June 26, 2023 at 23:35 #818125
Quoting Wayfarer
The counter to that is that when you see causal relationships between ideas, that this is distinct from the mindless processes typically invoked by physicalism. You're seeing the connection between ideas. That is a different process to that of physical causation.


So you think this process undermines or disproves naturalism?

Quoting Wayfarer
Furthermore, if I write something that perturbs or upsets you, that will have physical consequences - blood pressure, adrenal reaction, heart rate, etc.


I need a bit more than this to take a view that naturalism isn't a plausible account.

As I said, we need real expertise to determine how thought or 'mind' comes from bodies or brains. I don't think anyone has resolved this and some subscribe to mysterianism.
Tom Storm June 26, 2023 at 23:39 #818126
Quoting Paine
But I have read enough text to question Gerson's assertions and look forward to challenging anyone who would champion his position as a scholar.


That's very interesting. What do you think his project is, then? Is he a tendentious advocate of Platonism at the expense of fidelity to Plato? His name comes up a lot amongst enthusiasts of Platonism.
Paine June 26, 2023 at 23:53 #818132
Reply to Tom Storm
Gerson is a devoted student of Plotinus. Plotinus had his own view of the limits of Aristotle in relation to what he thought Plato was saying. To some extent, I think Gerson is reverse engineering what Plotinus assumed to be the case.

I don't charge Gerson with some nefarious purpose. Some of his commentaries on Aristotle are very interesting. But I am not on board with the Ur Platonism argument.
Tom Storm June 26, 2023 at 23:58 #818133
Quoting Paine
To some extent, I think Gerson is reverse engineering what Plotinus assumed to be the case.


Got ya. Thanks.
Wayfarer June 27, 2023 at 00:09 #818134
Quoting Paine
If I am correct, then Gerson has misunderstood Aristotle.


I'm sure that Lloyd Gerson doesn't misunderstand Aristotle, it's more likely that I misunderstand Lloyd Gerson, or rather, have taken one of his arguments out of context.

I'm not that knowledgeable about the Platonic forms, but I do think that they're frequently misunderstood as a kind of immaterial template or blueprint purportedly 'located' in some 'ethereal realm'. It is under that kind of reading that they're usually dismissed. But I think there might be a plausible case that what is meant by the 'forms' is much nearer in modern terms to 'principle', 'concept' or 'universal' in the sense understood by Scholastic realism.

Quoting Edward Feser
Consider that when you think about triangularity, as you might when proving a geometrical theorem, it is necessarily perfect triangularity that you are contemplating, not some mere approximation of it. Triangularity as your intellect grasps it is entirely determinate or exact; for example, what you grasp is the notion of a closed plane figure with three perfectly straight sides, rather than that of something which may or may not have straight sides or which may or may not be closed. Of course, your mental image of a triangle might not be exact, but rather indeterminate and fuzzy. But to grasp something with the intellect is not the same as to form a mental image of it. For any mental image of a triangle is necessarily going to be of an isosceles triangle specifically, or of a scalene one, or an equilateral one; but the concept of triangularity that your intellect grasps applies to all triangles alike. Any mental image of a triangle is going to have certain features, such as a particular color, that are no part of the concept of triangularity in general. A mental image is something private and subjective, while the concept of triangularity is objective and grasped by many minds at once.


[quote=Bertrand Russell, The Problems of Philosophy]It is largely the very peculiar kind of being that belongs to universals which has led many people to suppose that they are really mental. We can think of a universal, and our thinking then exists in a perfectly ordinary sense, like any other mental act. Suppose, for example, that we are thinking of whiteness. Then in one sense it may be said that whiteness is 'in our mind'. ... In the strict sense, it is not whiteness that is in our mind, but the act of thinking of whiteness. The connected ambiguity in the word 'idea', which we noted at the same time, also causes confusion here. In one sense of this word, namely the sense in which it denotes the object of an act of thought, whiteness is an 'idea'. Hence, if the ambiguity is not guarded against, we may come to think that whiteness is an 'idea' in the other sense, i.e. an act of thought; and thus we come to think that whiteness is mental. But in so thinking, we rob it of its essential quality of universality. One man's act of thought is necessarily a different thing from another man's; one man's act of thought at one time is necessarily a different thing from the same man's act of thought at another time. Hence, if whiteness were the thought as opposed to its object, no two different men could think of it, and no one man could think of it twice. That which many different thoughts of whiteness have in common is their object, and this object is different from all of them. Thus universals are not thoughts, though when known they are the objects of thoughts. [/quote]


Quoting Paine
I look forward to challenging anyone who would champion his position as a scholar.


Here's his google scholar page. The paper I quoted that passage from is Platonism vs Naturalism.

Quoting Tom Storm
So you think this process undermines or disproves naturalism?


It doesn't disprove it, so much as being incommensurable with it. The activities of reason are grounded in intuitive insight into the relations between abstractions (which we designate 'facts' or 'propositions'). Whereas the naturalistic account seeks reasons in terms of antecedent physical causes. I don't get why this is such a hard distinction to grasp. (I've already noted that this objection may not apply to the more recent forms of naturalism such as biosemiotics, but then, they've also in the main moved away from the mechanistic materialism which the argument is against. But it remains a powerful influence.)
Tom Storm June 27, 2023 at 00:16 #818137
Quoting Wayfarer
It doesn't disprove it, so much as being incommensurable with it. The activities of reason are grounded in intuitive insight into the relations between abstractions (which we designate 'facts' or 'propositions').


Ok. But is it 'incommensurable' or seemingly so? Do you think we have enough information to make this call? Is anyone here defending mechanistic materialism? And does anyone here advocate Dennett in this space? The question seems to me to be, can we rule out naturalist explanations for reason (and what we call mental processes)?
Paine June 27, 2023 at 00:21 #818139
Reply to Wayfarer
I have read that essay several times. It is not an argument built upon assertions but a 'by means of negation':

The strategy I employed was to follow a sort of via negativa, examining the dialogues for the philosophical positions that are therein totally and consistently rejected. The ‘consistently rejected’ part is important because many would maintain that the difficulty in determining Plato’s philosophy is in part that his views changed over the course of the dialogues. So, we hear about the early, middle, and late Plato, terms of periodization that, we should never forget, are entirely fictitious.
The apotheosis of such fictional construction is the hermeneutic version of an astronomical
epicycle, the ‘transition’ dialogue, supposedly including those works which do not fit neatly into the early, middle, or late categories


My problems with his argument have nothing to do with this sort of speculation.
180 Proof June 27, 2023 at 02:08 #818160
Quoting Wayfarer
The apparently inviolable constraints of physical laws have been transcended many times in the history of science.

Cite an instance when and where Newton's 3rd Law and/or any conservation laws "have been transcended" even once. :lol:
Srap Tasmaner June 27, 2023 at 02:11 #818161
Quoting Wayfarer
when you see causal relationships between ideas, that this is distinct from the mindless processes typically invoked by physicalism. You're seeing the connection between ideas. That is a different process to that of physical causation.


It's a different framework, sure. The question is why you think the existence and utility of this framework, our everyday understanding of mentality, invalidates the framework used in neuroscience and biology at large.

Neuroscientists in the lab use that same everyday framework to talk to each other and their subjects. They'll continue to say things like "I'll be right there, just grabbing my coffee," even if they're about to sit down with a nice interviewer from PBS and tell them, and the audience at home, that "the self is a myth," or something like that.
Wayfarer June 27, 2023 at 02:56 #818169
Quoting Tom Storm
Is anyone here defending mechanistic materialism? And does anyone here advocate Dennett in this space?


No, they're mainly picking arguments with me. I select Daniel Dennett because he's unapologetically materialist so I can't be accused of attacking a straw man. But it's relevant to note that Dennett does defend the claim that humans are no different in principle to robots or computers. He's a walking, talking illustration of what I think is meant by the 'forgetfulness of Being'.

Quoting 180 Proof
Cite an instance....


The general point is that science has accomplished many things which were previously thought impossible according to then known physical laws. The understanding of what constitutes 'the physical' is constantly changing (as per Hempel's dilemma).

Quoting Srap Tasmaner
The question is why you think the existence and utility of this framework, our everyday understanding of mentality, invalidates the framework used in neuroscience and biology at large.


I don't necessarily think that at all. (There was an amazing segment (might be geo-blocked) on a current affairs program yesterday about a female virtuoso violinist who fell victim to a rare condition called dystonia which affected her muscular co-ordination, meaning she suddenly, and completely, lost the ability to play. She was treated with a form of magnetic resonance in an fMRI scanner, which ameliorated her condition and restored her ability, such that she's now planning to resume her career. In those kinds of cases and millions of others, like the ability to provide paralysis victims with the ability to walk, neuroscience verges on the miraculous. But that is not the point of the philosophical issue at hand. In practice, probably many neuroscientists hold a generally materialist or physicalist framework but whether they do is irrelevant to the problems they are dealing with. The philosophical issue is one of philosophy of mind and the nature of being. (Although there have been some neuroscientists, like John Eccles, Roger Sperry ('In calling myself a ‘mentalist’, I hold subjective mental phenomena to be primary, causally potent realities as they are experienced subjectively, different from, more than, and not reducible to their physicochemical elements') and Wilder Penfield, who have argued against brain-mind identity on the basis of their practical experience with neurosurgery subjects.)

As for biology, as there has been a sea-change in physics since the discoveries of the 1920's, so too there has been in biology, although there are still many hold-outs (more in biology than in physics, some say). But the change towards biosemiotics, epigenetics, and other developments, call into question the kind of neo-darwinian materialism that is the subject of the argument from reason.
Srap Tasmaner June 27, 2023 at 03:20 #818173
Quoting Wayfarer
I don't necessarily think that at all.


Then what are we talking about?

How do you feel about neuroscientists saying things like "the self is an illusion"? --- Before answering, note that no reduction is implied; it's not a claim that the self is "really" a bit of functioning brain tissue, but that our everyday ideas about our selves don't seem to have a correlate in the brain, just as our visual field has no real correlate in the brain and is, in a suitable sense, an illusion.
180 Proof June 27, 2023 at 03:29 #818175
Reply to Wayfarer I'll take your latest non-answer as a concession to my edifying points here
https://thephilosophyforum.com/discussion/comment/818106 You're welcome, Wayf. :smirk:
Wayfarer June 27, 2023 at 03:35 #818177
Quoting Srap Tasmaner
Then what are we talking about?


The philosophical distinction between physical causation and logical necessity, and the implications of that.

Quoting Srap Tasmaner
How do you feel about neuroscientists saying things like "the self is an illusion"? --- Before answering, note that no reduction is implied; it's not a claim that the self is "really" a bit of functioning brain tissue...


For eliminative materialism, the claim really is that reductionist - e.g. Dennett's 'competence without comprehension'.

As to the sense in which self is an illusion - as many have pointed out, illusions are artefacts of consciousness, a mistaken perception. I can't see how to avoid the necessity of there being a subject of such an illusion.

Quoting Srap Tasmaner
just as our visual field has no real correlate in the brain and is, in a suitable sense, an illusion


What do you mean, 'suitable', here. Are you referring to the neural binding problem in respect of the subjective unity of experience (e.g. here)?


Srap Tasmaner June 27, 2023 at 04:29 #818185
Quoting Wayfarer
The philosophical distinction between physical causation and logical necessity, and the implications of that.


If I put three cupcakes on a table otherwise devoid of cupcakes, I have caused an odd number of cupcakes to be on the table.

Which one is that, physical causation or logical necessity?
Wayfarer June 27, 2023 at 04:35 #818186
Quoting Srap Tasmaner
If I put three cupcakes on a table otherwise devoid of cupcakes, I have caused an odd number of cupcakes to be on the table.


The act of putting them there is physical, from which you then can draw mathematical conclusions. Dual, you see - physical in some respects, mental in others.
Tom Storm June 27, 2023 at 04:47 #818188
Quoting Wayfarer
As to the sense in which self is an illusion - as many have pointed out, illusions are artefacts of consciousness, a mistaken perception. I can't see how to avoid the necessity of there being a subject of such an illusion.


I heard David Bentley Hart making this argument some years ago. It almost deserves its own thread.

Quoting Wayfarer
But it's relevant to note that Dennett does defend the claim that humans are no different in principle to robots or computers.


'Moist robots'... great term. Whether it is plausible or not, I have to say I greatly enjoy the idea that much for what passes as the human might be illusory.

Out of interest, what do you think is the specific harm of Dennett's view (if accurately represented)? You seem to dislike it for aesthetic reasons - that it robs us of enchantment and special meaning.
Srap Tasmaner June 27, 2023 at 05:04 #818192
Quoting Wayfarer
The philosophical distinction between physical causation and logical necessity, and the implications of that.


Quoting Wayfarer
physical in some respects, mental in others


I give up.

If you ever figure out exactly what you want to say, let me know.

Peace
Wayfarer June 27, 2023 at 05:05 #818193
Quoting Tom Storm
You seem to dislike it for aesthetic reasons - that it robs us of enchantment and special meaning.


Speaking of Hart, take a look at his review of Dennett ('so preposterous as to verge on the deranged').

Quoting Srap Tasmaner
If you ever figure out exactly what you want to say, let me know.


I think the original post makes a perfectly intelligible point, and one not of my invention - I'm puzzled by the fact that it seems so incomprehensible to you. You said already a few times in this thread that it was drawing a distinction between different senses of the word 'because' - which is closer to what it's getting at than anything said by anyone else here. But you then say you can't see the point of the distinction. The fact that humans are physical and mental combined in a unity is hardly a novel philosophical idea. So I don't really understand what it is you're not understanding, although again, thanks for making the effort.
Tom Storm June 27, 2023 at 05:15 #818195
Reply to Wayfarer I've read it. Bentley is a gifted thinker and writer. Even if he can be a bit of a bitch. It's pretty much your argument being made here.







180 Proof June 27, 2023 at 05:40 #818197
Quoting Wayfarer
Of course you will assume that information is physical ...

For the *Quantum Woo Crew* ...

You're welcome, gents. :smirk:

@Gnomon

encore:

Jamal June 27, 2023 at 15:17 #818262
Quoting Srap Tasmaner
It seems to me a lot of our traditional "mental" vocabulary does not refer to exclusively internal states of human beings, but rather to mental rather than, I guess, bodily interactions with the environment and objects. We distinguish, and presumably have for a very long time, between chopping down a tree and looking at it, wondering if it's big enough for the beam we need. Both descriptions involve both the guy with the sharp implement and the tree, so just as doesn't map cleanly onto postures and movements of my body alone, in the absence of a tree, so needn't map onto something going on in my brain in the absence of a tree.


I agree. We talked about the mind long before we gave much thought to brains--that is, when we were describing certain activities rather than describing something the brain does--but even from this point of view, what is mental can still be seen as material, just not in the neuro-reductionist way.

Quoting Srap Tasmaner
As it happens, representational theories of mind will map the necessary tree onto my internal representation of the tree, and you'll see often on this forum theories that claim my goal in either case to produce a certain state of my internal model. I think that's a very different issue from whether our everyday vocabulary around thinking, perceiving, imagining, remembering, and so on, not only presupposes objects for these activities but folds them into terms that are in some ways holistic.


I'm not quite clear what you mean here. If you mean that a non-neuro-reductionist understanding of the mind, while it does presuppose mental objects, need not presuppose internal representations, then I think I probably agree.
Gnomon June 27, 2023 at 16:20 #818284
Quoting Wayfarer
?Gnomon
I'd really appreciate it if you deleted that inane graphic.

Does it remind you of someone you know? :smile:
Srap Tasmaner June 27, 2023 at 16:24 #818286
Quoting Jamal
what is mental can still be seen as material, just not in the neuro-reductionist way


I think scientists instinctively talk this way -- "When the light from this object passes through your retina and strikes these cones, blah blah blah". A word like "vision" describes an interaction between an organism and some part of its environment, not just the internal state of the organism, interesting though that is. And you can still describe the whole tableau in naturalist terms, which doesn't change "vision" being the sort of thing we think of as mental.

Just as I have a mind, and that takes in a lot of my interactions with my environment, I have a gait -- somewhat like my father's I am told -- which is not exactly a property of mine, is not evident when I am sitting, but is a consistent feature of how I ambulatorily (!) interact with my environment. There's nothing non-physical involved in how I walk, but how I walk is only available within a particular descriptive framework, and one that necessarily involves both my body and the ground I tread.

Quoting Jamal
If you mean that a non-neuro-reductionist understanding of the mind, while it does presuppose mental objects, need not presuppose internal representations, then I think I probably agree.


I don't mean anything in particular. There's the older reflex action model -- which James describes as the singular achievement of 19th century physiology -- which is triadic: input-processing-output. The way James tells it, you have to learn to consider thinking and friends as just this middle step between sensation and action, and action -- in furtherance of life -- is the point of the whole system. But then there's the newer model, in which it's the state of the middle part that's the point, reducing its level of excitation (through action), minimizing surprise (through prediction) to minimize future excitation. (Freud's death drive but with better math.)

All I was saying is that I don't really think we need to take sides here, let alone address thorny questions of representation, to recognize that our everyday mental vocabulary is not a vocabulary about our internal states, so there's no reason to expect our everyday vocabulary to map cleanly onto whatever neuroscience discovers about those internal states.

What throws people is the identification of consciousness with the mental -- better to allow that simpler organisms may have mind but not consciousness -- because consciousness appears to be exclusively internal. Mostly it isn't, of course, else we wouldn't have it; consciousness is primarily consciousness of our environment. But there are derivative phenomena like remembering and dreaming and analysing, where all the stuff to be thought about has already been accumulated. So you go down the empiricist rabbit-hole of starting out saying sensation is the ultimate source of all of our thoughts (the thread James picks up) and end up allowing that so far as the internal state is concerned, there's just whatever's given, and you've no real way to tell where it came from.

Even worse is going on to equate mind and consciousness and self-consciousness. Even worser is equating all of those with the non-physical.

Blah blah blah. We just don't have to get into all that for the straightforward recognition that our everyday mental vocabulary is not about our internal states, so a lot of the putative problems with neuroscience are not problems at all.
Gnomon June 27, 2023 at 17:37 #818298
Reply to Wayfarer
Of course you will assume that information is physical ... — Wayfarer
For the *Quantum Woo Crew* ...
Reply to 180 Proof

The Closer to Truth video asks "Is Information Fundamental?". And Seth Lloyd thinks it is. But, here's what The Information Philosopher says about that question :
"Seth Lloyd is quite correct that information ("bits") is physical ("its"). However, unlike things, which are concrete and material. Information is abstract and immaterial." *1 So, Information is "physical" in the same sense that Energy is physical & real : both are intangible causes that are detectable only in their effects*2. Abstractions are imaginary representations in minds.

Philosophically, you could say that the Atom of Energy is a Bit of Information. Otherwise, Energy has no measurable/quantifiable properties in itself, but only in its effects on Matter : Change, Causation. Therefore, Energy/Information is indeed "physical" and "real" in that it has effects on Material objects, even though it is not a material object/thing itself*4.

Energy is the Potential for change in Matter (e.g. motion). Information is the Potential for change in Minds (e.g. knowledge). Both are essential to knowable Reality, even though neither is a Material object. Instead, Matter is a tangible form of intangible Energy/Information : E=MC^2. Energy/Information is devoid of properties such as Mass & Velocity, but it is instead the Cause of such measurable properties*5. Energy is a Qualia (causation), but its effect/consequence is a Quanta (measurable difference).

180 seems to think that Energy/Information is "woo" simply because it it invisible & intangible, like a ghost. But most physicists believe that Energy is real, even though they have never seen a real particle of Potential (the ding an sich). Like a poltergeist, Energy/Information is knowable only when it causes a book to spontaneously fall off a shelf. To which we physicalists respond : "it was just Gravity", but what then, is gravity made of : graves, weights, heaviness?. :smile:


*1. Seth Lloyd : https://www.informationphilosopher.com/solutions/scientists/lloyd/

*2. Is energy real?[i]From a physics perspective, is there actually energy? If it's not a thing, what is it, and how do we know it really exists?

It’s a very good question. Speaking as a physics teacher, too few students ask it, too few instructors answer it, and too few textbooks define the word “energy” (although all textbooks use this word a lot). Everybody just assumes we all know what “energy” means, but we don’t know. Furthermore, energy is the most fundamental physical concept of all, because the universe is made of quantized fields that are themselves made of energy. So everybody needs to know what “energy” means.

The problem appears to stem from the great physicist Richard Feynman, who seems to have thought energy was undefinable. He was wrong, but his thinking was very influential because he was, after all, Feynman.

Energy is a very specific entity. It is not a “thing.” It is, instead, a property of things. Let’s start with some definitions: A collection of physical objects is called a “physical system” or simply a “system”. When we say a system “has energy,” we mean that it has the capacity to do work. So, what does it mean to ”do work?” When you do work, you exert forces in order to alter the positions or velocities of objects. That is, work is the ability to change things by exerting forces to move objects around. Of course, all this can be defined and measured quantitively (which I won’t do here). The units are joules, or calories, or BTUs, or electron-volts. Thus, when we say that a ball flying through the air has “10 joules of energy,” we mean its speed gives it the ability to do 10 joules of work on some other system. This type of energy (energy due to motion) is called “kinetic energy.”

The bottom line: Yes, energy is quite real. It is the ability*3 to do work.[/i]
https://www.quora.com/Does-mental-energy-actually-exist-as-a-real-type-of-energy-or-is-it-just-another-meaning-for-brain-chemicals

*3. Ability : the physical or mental power or skill needed to do something.
Synonym : Potential : not yet real.
Potential generally refers to a currently unrealized ability. ___Wiki

*4. Why information is energy?
Energy is the relationship between information regimes. That is, energy is manifested, at any level, between structures, processes and systems of information in all of its forms, and all entities in this universe is composed of information.
https://physics.stackexchange.com/questions/22084/how-is-information-related-to-energy-in-physics
Note --- A "relationship" is a mathematical Ratio or Proportion between related things or ideas.

*5. Physicalism typically involves a methodological commitment to the view that, whatever the final, accurate description of reality looks like, it will be set out in terms of physical entities:things with properties like mass and velocity.
https://iai.tv/articles/reality-is-not-revealed-by-quantum-mechanics-auid-2512
Note --- That "commitment" is a metaphysical belief based on a priori assumptions.


180 Proof June 27, 2023 at 18:03 #818305
Reply to Gnomon :eyes: :rofl:
Wayfarer June 28, 2023 at 00:07 #818439
Against better judgement, I want to revisit this exchange. My claim was that when a subject is persuaded by another's reasoning and realises the truth of a proposition, then nothing physical has transpired between the two parties. The objection was:

Quoting 180 Proof
Abstractions themselves do not "act upon matter" because they are not evental (or causal); rather instantiations (encoding / patterning) of abstractions (from matter) in matter act upon matter (e.g. typing on my keyboard these sentences you're reading on your screen),


Language is represented physically, and transmitted physically, by way of binary code across the Internet (or printed and sent). The question is, what is it that enables the 'encoding or patterning' of the specific words that have been selected, to which the answer is 'intelligence' - in the case of this example, clearly human intelligence, mine and the subjects'. And in this case, if I persuade another to see the truth of an argument, then that persuasion is indeed 'causal'. You can hardly argue that reasoned persuasion is not an enormous causal factor in human affairs.

So the question then becomes, is intelligence physical? Which is one of the key questions of philosophy of mind. Those arguing the case for dualism will answer in the negative. So while there are physical words on the screen, it is the nature of the faculty that composes and interprets them that is at issue. And I will not cede the case to materialist theory of mind.

(And as to whether 'abstractions are causal', that is another question altogether. But the formative role of mathematical physics in science at least points in that direction.)

A related issue, arising from my response to @Fooloso4's saying there can be no such thing as 'immaterial beings', was this:

Quoting Wayfarer
Living beings, even the very simplest beings, display attributes and characteristics that actually can't be accomodated in the mind-body duality that is embedded in the modern worldview.


So here, the argument concerns the materialist assumption that organisms are not ontologically different from non-organic matter, on the basis that they're all 'made from the same stuff' - that living things are just elements of the periodic table, arranged in a specific way. That is the basis of materialism and the point about which I referenced the Talbott article 'From Physical Causes to Organisms of Meaning'.

The point of this article is anticipated by the title - which is, to try and express it as succintly as possible, that organisms right from the outset exhibit attributes and characteristics that cannot be found in the inorganic domain. Talbott writes that much of the lexicon of biology - 'words like “stimulus”, “response”, “signal”, “adapt”, “inherit”, and “communicate”, in their biological sense, would never be applied to the strictly physical and chemical processes in a corpse or other inanimate object.'

So, what is it that organises the elements of the periodic table in such a way as to give rise to living beings? There are those that argue for those causes being physical ('the chemical paradigm') and those who argue for it being fundamentally different (code biology, biosemiotics). At this time it's an open question, but the implication is that living beings are not simply or only physical in nature. They embody intentionality and are subjects of experience - just those factors that have been excluded from the objective domain in Cartesian dualism.
180 Proof June 28, 2023 at 02:03 #818460
Quoting Wayfarer
So the question then becomes, is intelligence [adaptation] physical?

Yes.

So, what is it that organises the elements of the periodic table in such a way as to give rise to living beings?

Quantum computation (re: Seth Lloyd, Stephen Wolfram, David Deutsch).


Wayfarer June 28, 2023 at 04:05 #818476
Reply to 180 Proof Intelligence is the ability to adapt, among other things, but it is not exhausted by that description. More to the point, its attributes can't be either predicted or explained on the basis of physics.
Srap Tasmaner June 28, 2023 at 04:40 #818480
Quoting Wayfarer
More to the point, its attributes can't be either predicted or explained on the basis of physics.


So what's the deal with lesion studies, anesthesia, all the usual things people point to where changes in the brain affect a person's thinking and emotions in predictable ways?
Wayfarer June 28, 2023 at 05:05 #818483
Quoting Srap Tasmaner
So what's the deal with lesion studies, anesthesia, all the usual things people point to where changes in the brain affect a person's thinking and emotions in predictable ways?


No realistic dualism or idealism would deny that physical influences affect cognition and affect. But the argument from reason is about physicalism - that everything about the mind can be reduced to or explained in terms of physical causes. Can you see the distinction?

Earlier in the thread, I mentioned a subject whose behavioural abnormalities had been found to have been caused by a tumor. Of course there are many such cases of physical conditions or substances affecting behaviour. They are what I would designate as physical causes. But being persuaded by reason to accept the truth of a proposition is of a different order - that is an example of rational causation (hence my link to the book of that name - do take the time to read the abstract, at least.)
Janus June 28, 2023 at 05:29 #818484
Quoting Wayfarer
But the argument from reason is about physicalism - that everything about the mind can be reduced to or explained in terms of physical causes.


I don't see why a decision to (for example) go to the shop to buy milk, cannot be explained in terms of physical causes (environmental conditions and neural processes), but it doesn't follow that my reason for buying milk can be reduced to a set of physical and neural causes.
Wayfarer June 28, 2023 at 09:40 #818503
Quoting Janus
I don't see why a decision to (for example) go to the shop to buy milk, cannot be explained in terms of physical causes


The simplest reason is that it's intentional, and intentionality is lacking in physical causation.
Janus June 28, 2023 at 21:23 #818619
Quoting Wayfarer
The simplest reason is that it's intentional, and intentionality is lacking in physical causation.


I just see the two explanations as being different. Deciding to go to the shop can be explained in terms of intentionality or in terms of physical causes. It was because my reason for going to the shop is intentional that I said it cannot be reduced to a set of physical and neural causes.

My having a reason can be explained in those terms, but the reason itself, being intentional, cannot. To say that my reason could be explained physically would be a category error; we have two different categories of explanations: those given in terms of intentionality and those given in terms of physical causes.

To put it simply, the point was that making a decision can be explained physically, as can having a reason, but the reason for making the decision cannot; the reason itself is an explanation. It's all about context.
Srap Tasmaner June 28, 2023 at 22:38 #818634
Reply to Janus

One thing that strikes me as a little odd is that descriptions can be faithful without being exhaustive, and, knowing that, we expect there to be many faithful descriptions of a thing, only some of which compete directly. 7 is a prime number, a lucky number, the most common roll of a pair of dice, the average number of items a person can hold in short-term memory, etc., none of those to the exclusion of the others.

The same seems to be true, but maybe to a lesser degree, when we talk about explanations rather than descriptions. The macro-scale, observable phenomena we're talking about, things people do in the way of talking, reasoning, making decisions, all admit of multiple descriptions and explanations, depending, as you say, on context, on what we're interested in. --- It's even a standard technique in humor to switch descriptive framework in the middle of the joke, or to suggest one framework but reveal another. (Why did the chicken cross the road?)

Lewis's premise is that reasoning admits of only one description. He could have claimed that other accounts leave out what he's interested in, that they miss the reasoning in an act of reasoning and treat it like any other psychological or biological event. Instead he claims that no such description is even possible, and that nothing that could be so described and explained could be what he considers reasoning.

The question is, why would he think that? And it looks like the answer is: theology.
Tom Storm June 28, 2023 at 22:59 #818641
Quoting Srap Tasmaner
And it looks like the answer is: theology.


Christian theology...


I do see why philosopher George Lakoff describes consciousness as 'embodied brain'. It's hard to see how consciousness can create an awareness and point of view, without being part of a physical being. We understand the world in terms of what we can do with our bodies. We like to think of reason as being all head and no heart, but it originates from whole beings.


Wayfarer June 28, 2023 at 23:24 #818646
Quoting Srap Tasmaner
The question is, why would he think that? And it looks like the answer is: theology.


Of relevance, however, is the Marcus book I mentioned, Rational Causation, which explores similar territory without any reference to theology. 'We explain what people think and do by citing their reasons, but how do such explanations work, and what do they tell us about the nature of reality? Contemporary efforts to address these questions are often motivated by the worry that our ordinary conception of rationality contains a kernel of supernaturalism—a ghostly presence that meditates on sensory messages and orchestrates behavior on the basis of its ethereal calculations. In shunning this otherworldly conception, contemporary philosophers have focused on the project of “naturalizing” the mind, viewing it as a kind of machine that converts sensory input and bodily impulse into thought and action. Eric Marcus rejects this choice between physicalism and supernaturalism as false and defends a third way.

Marcus argues that philosophers have failed to take seriously the idea that rational explanations postulate a distinctive sort of causation—rational causation. Rational explanations do not reveal the same sorts of causal connections that explanations in the natural sciences do. Rather, rational causation draws on the theoretical and practical inferential abilities of human beings. Marcus defends this position against a wide array of physicalist arguments that have captivated philosophers of mind for decades.'

Likewise, Thomas Nagel's Evolutionary Naturalism and the Fear of Religion also examines 'the naturalisation of reason' from the perspective of secular, analytical philosophy rather than religion per se (although acknowledging that philosophers such as Plato and Spinoza have a 'quasi-religious' aspect.) That essay too argues that the 'sovereignty of reason' is called into question by evolutionary naturalism, which depicts reason as a form of adaptation, not of discerning truth. It also contains the often-quoted passage where Nagel writes that the 'fear of religion' is what drives a great deal of the 'scientism and reductionism of our times'.

Quoting Tom Storm
It's hard to see how consciousness can create an awareness and point of view, without being part of a physical being.


My take is that is very much characteristic of modern objective consciousness. Our world-picture is one of separate embodied subjects in a domain of objects configured by impersonal laws, whereas the world-picture of the pre-moderns was that the world is an expression of a will with which we ourselves have a relationship through faith. But, you know, 'compulsory disenchantment'.
Metaphysician Undercover June 29, 2023 at 00:35 #818672
Quoting Fooloso4
We have no knowledge or experience of any immaterial entity of process.


Speak for yourself here. Your use of "we" is out of place.

Quoting 180 Proof
Cite an instance when and where Newton's 3rd Law and/or any conservation laws "have been transcended" even once.


Newton's first law is "transcended" with every freely willed act. A body's motion changes without the application of force. The conservation of energy law is broken with every act which occurs, as some energy goes missing which cannot be accounted for. The idea that the laws of physics are never broken, or "transcended", is naivety at best.

Gnomon June 29, 2023 at 00:40 #818674
Quoting Wayfarer
(And as to whether 'abstractions are causal', that is another question altogether. But the formative role of mathematical physics in science at least points in that direction.)

As usual, we are treading in swampy terrain here, with pockets of philosophical quicksand all around. So, this post is likely to get your feet wet & squishy. Reply to 180 Proof will enjoy ROFLing and eye-rolling in bemused incredulity ; keeping his feet dry, by studiously avoiding the sodden speculations of theoretical Philosophy, in favor of the "hard" facts of empirical Science. Please pardon my excursion beyond the solid ground of objective Matter into the mucky bog of subjective Mind*1, on the leaky platform of a philosophy forum. :cool:

The vocabulary of dogmatic Materialism seems to exclude that which is behind the eyeballs*7 (ideal, abstract, private, subjective Representations/Meanings), and defines as "real" only that which lies on the objective (public) side of the lens, and labels the majority vote as Reality. Yet Kant --- echoing Plato --- noted that the observer actually knows only the internal ideal representation --- along with any personal biases --- not the external reality, or the ultimate ding an sich. But Materialists are not Kantians. So they naively believe that their abstract subjective image is the concrete objective thing; real enough for practical purposes.

On the other hand, some idealist Mathematicians (e.g. Tegmark) tend to think of their immaterial mental abstractions as somehow more real than the material embodiment of an essential logical structure of interrelationships. Tegmark's Mathworld is theoretical, while Dennett's Naturalistic world is empirical. Are you an Either/Or thinker? Do you define "Truth" as solely Real or only Ideal? For little ole me it's both : look to objective Reality for pragmatic (concrete) purposes, and to subjective Ideality for theoretical (abstract) reasons*4. Taken together, we may be able to get Closer To Truth.

Whereas Chemistry is mostly concerned with Matter (real malleable stuff), Physics is mainly focused on Energy (spooky invisible action)*3 . Hence, 21st century physicists no longer assume that reality is composed of hard material atoms. Instead, they imagine that the world is fundamentally an invisible Field of mathematical relationships (space-time nodes and meaningful links between them). In what sense is an abstract mathematical quantum field of Potential Energy Real*5? Quantum scientists find evidence that an invisible intangible nonlocal "cloud" of statistical possibilities can have tangible local actual effects*6. Is that real enough for your practical or theoretical purposes? :smile:


*1. Rejections of Idealism, often accuse Idealists of denying objective Reality, then metaphysically affirm the opposite. But that is an example of simplistic black vs white thinking, which defeats the purpose of Philosophy : to question assumptions, while avoiding dogmatism. I can't speak for Wayfarer, but the definitions in footnote *2 do not define my more complex integrated worldview, which is intended to combine the Objective (concrete) view of empirical science with the Subjective (abstract)*5 perspective of mind-probing philosophy, into a single holistic worldview. If you must label such a view, try inclusive portmanteau words like Re-dealism or I-realism, but please avoid the exclusive facile oppositions of Realism versus Idealism. :chin:


2. Within modern philosophy there are sometimes taken to be two fundamental conceptions of idealism :
[i]a. something mental (the mind, spirit, reason, will) is the ultimate foundation of all reality, or even exhaustive of reality, and
b. although the existence of something independent of the mind is conceded, everything that we can know about this mind-independent “reality” is held to be so permeated by the creative, formative, or constructive activities of the mind (of some kind or other) that all claims to knowledge must be considered, in some sense, to be a form of self-knowledge.[/i]
https://plato.stanford.edu/entries/idealism/

*3. Is energy real or a concept? :
What is energy? Energy is one of the most basic concepts in physics, but also one of the hardest to define.
https://cosmosmagazine.com/science/physics/what-is-energy/
Note --- Is "ability" a real thing? Is "work" a physical object? Is "causation" an observation or an inference?

*4. What is reality? :
“Reality is that which, when you stop believing in it, doesn’t go away.”
___Philip K. Dick
"Ideality is that which, when you stop reasoning, goes away."
___Gnomon

*5. What is the philosophy of abstract thought?
Abstract thinking is the ability to understand concepts that are real, such as freedom or vulnerability, but which are not directly tied to concrete physical objects and experiences. Abstract thinking is the ability to absorb information from our senses and make connections to the wider world.
https://www.healthline.com/health/abstract-thinking

*6. Does quantum physics disprove causality?
No, quantum physics does not disprove causality. On the contrary, our best working quantum theory to date, quantum field theory, quite properly respects causality both on the macroscopic and on the microscopic level.
https://www.quora.com/Does-quantum-physics-disprove-causality


*7 PHYSICAL OPTICS plus METAPHYSICAL INTERPRETATION (meaning)
See the little raindrop, full of images, in the head? Is it the pineal gland or the soul? How do you know?
User image
180 Proof June 29, 2023 at 01:41 #818683
Reply to Metaphysician Undercover Non sequiturs abound.
Metaphysician Undercover June 29, 2023 at 01:56 #818684
Reply to 180 Proof
Sorry, but misunderstanding on your part does not qualify as non sequitur on my part, as much as you like to think so. But go ahead, and think what you like.
180 Proof June 29, 2023 at 02:25 #818688
Srap Tasmaner June 29, 2023 at 02:53 #818690
Reply to Wayfarer

Yes, yes, we all know there is another framework. What you need to argue for is exclusivity.

I think your position is that naturalism itself makes an unjustified claim to exclusivity, and you're just rebutting that. I mean, yeah, you do that all the time, but the argument from reason claims that biology needs to get off reason's lawn. And that has to be argued for.

It's worth noting that this idea of competition has of course been institutionalized in science. Scientific theories do face competition, but only from other scientific theories. It's for this very reason that cdesign proponentists have been trying to pass off their faith as just as scientific as something that's really science.

As I understand it, you are not proposing an alternative scientific theory, and imagine your quest as challenging a foundational assumption of science.

But already science allows such challenges. There are some really obvious examples I dare not mention.

Your choice then is (1) present your view as a genuine scientific hypothesis; (2) challenge the methodology of science. Mostly theists opt for door number 2, and defend revelation as knowledge producing.

There is one last alternative, which is not to challenge science but to live alongside it, as religion continues to do, but also art, sports, geez all the other stuff people get up to, and most especially our standard ways of talking about things like the sun rising, tables being solid, people picturing things in their head -- all of that is fine, and scientists also do all that stuff when they're not doing science.
Wayfarer June 29, 2023 at 04:17 #818696
Quoting Srap Tasmaner
But already science allows such challenges. There are some really obvious examples I dare not mention.


Please do!

Quoting Srap Tasmaner
think your position is that naturalism itself makes an unjustified claim to exclusivity, and you're just rebutting that.


It’s more that I think some taken-for-granted elements of the scientific worldview amount to popular mythology, which has subtle but important consequences.
Srap Tasmaner June 29, 2023 at 04:24 #818699
Quoting Wayfarer
Please do!


It's the obvious ones, really.

I'll give you one, but only the definition: biological theory everyone thinks they understand.

I'm not hiding anything. I like to think of these as the philosophical equivalents of Godwin's law. So I am carefully not summoning those demons only because it amuses me.
Wayfarer June 29, 2023 at 04:27 #818700
Wayfarer June 29, 2023 at 05:36 #818705
Reply to Srap Tasmaner Would I be right in surmising that you find 'the argument from reason' an affront to common sense?
Srap Tasmaner June 29, 2023 at 05:57 #818707
Reply to Wayfarer No. What gave you that idea?
Wayfarer June 29, 2023 at 06:02 #818709
Reply to Srap Tasmaner OK, never mind. Mistaken impression on my part.
Wayfarer June 29, 2023 at 06:42 #818711
Quoting Gnomon
I can't speak for Wayfarer, but the definitions in footnote *2 do not define my more complex integrated worldview, which is intended to combine the Objective (concrete) view of empirical science with the Subjective (abstract)*5 perspective


The model I'm trying to flesh out posits mind or consciousness as being a latent attribute or dimension of reality, which manifests when and wherever the appropriate physical conditions exist (apparently a rare occurrence) through the processes we know as evolutionary biology. This implies that the mind is not the outcome of that process, but at the source of it - but not as a creator Deity, more like Schopenhauer's Will. It is also not to say that ‘everything is conscious’ in the pantheistic sense, or that sub-atomic particles have some primitive form of experience. I see that as an attempt to rescue materialism by the injection of mind-stuff.

Consider this quote:

[quote=Julian Huxley]Man is that part of reality in which and through which the cosmic process has become conscious and has begun to comprehend itself. His supreme task is to increase that conscious comprehension and to apply it as fully as possible to guide the course of events. In other words, his role is to discover his destiny as an agent of the evolutionary process, in order to fulfill it more adequately.[/quote]

In my view, mind is as fundamental as atomic structures although not reducible to them. But here's the key point: it is never encountered as an object - it only ever manifests as the subject of experience, not to the subject as an object of cognition. So it is not anything - no-thing or not-a-thing - but at the same time, is at the foundation of existence. (I don't think Julian Huxley saw it that way, but his brother Alduous might have.)

You can also see why such an insight is outside the purview of naturalism, although I think phenomenology understands it (for which see It is never known but it is the knower, Michel Bitbol, .pdf.)

Where reason comes into the picture is that reason and mathematics provides the logical structure of experience and cognition. Objects are given in perception but the received sensory data is structured by cognition and reason (per Kant, although I am more inclined to admit the reality of universals).

Notice also this snippet on C S Peirce 'The phrase "matter is effete mind" is attributed to Peirce and reflects his metaphysical views on the nature of reality. Peirce believed in a form of philosophical idealism, which means that he considered mind or consciousness to be fundamental and primary, with matter arising from it in some sense. He argued that matter, rather than being something independent or external to mind, is a product or result of mind.

He conceived of three categories that underpin his philosophical system: firstness, secondness, and thirdness. Firstness refers to the realm of pure possibility, potentiality, and quality. Secondness is associated with concrete individual instances, facts, and brute realities. Thirdness encompasses generality, law, and the relational aspects of things.

According to Peirce, mind or consciousness belongs to the category of firstness, characterized by a pure qualitative aspect. Matter, on the other hand, falls within the category of secondness, representing concrete, individual manifestations of existence. However, Peirce posited that matter, as we perceive it, is not entirely separate from mind but is derived from it or evolved from it.'

(Still trying to put the pieces together, may well not succeed.)
Metaphysician Undercover June 29, 2023 at 10:39 #818729
Quoting Srap Tasmaner
Mostly theists opt for door number 2, and defend revelation as knowledge producing.


Isn't it obvious to you though, that revelation must produce knowledge? If we were to restrict our definition of "knowledge" so that it only contained ideas produced by science, then how would we account for all the rest of the ideas that we would normally call "knowledge"? This would include the tools applied by science, like the axioms of mathematics.

This is the problem of epistemology, to give an accurate account of what knowledge actually is, rather than some definition, or set of criteria which we believe knowledge ought to be. If we say "knowledge is..." when this reflects what we think knowledge ought to be, rather than what knowledge actually is, this will mislead us in any attempt at ontology.

These two are very different, as demonstrated in Plato's Theaetetus. In that dialogue, they went looking to describe "knowledge", with a preconceived idea of some criteria as to what could constitute "knowledge". Then Socrates demonstrated how it was logically impossible that any description they came up with as to how knowledge actually exists, could not fulfil the preconceived criteria. In the end they realized that the preconceived idea must be misleading them in their quest to figure out what knowledge is.
Gnomon June 29, 2023 at 17:01 #818801
Quoting Wayfarer
The model I'm trying to flesh out posits mind or consciousness as being a latent attribute or dimension of reality, which manifests when and wherever the appropriate physical conditions exist (apparently a rare occurrence) through the processes we know as evolutionary biology. This implies that the mind is not the outcome of that process, but at the source of it - but not as a creator Deity, more like Schopenhauer's Will. It is also not to say that ‘everything is conscious’ in the pantheistic sense, or that sub-atomic particles have some primitive form of experience. I see that as an attempt to rescue materialism by the injection of mind-stuff.

Sounds very similar to my own personal project. Which I began a few years ago, after a quantum physicist remarked on what he saw on the quantum level of reality : "it's all information". That observation seemed to confirm John A. Wheeler's 1989 "It from Bit" conjecture. His Participatory Anthropic Universe sounds a lot like Panpsychism, plus the notion that human consciousness was somehow intended from the beginning of evolution. But being a scientist, he wouldn't be expected to make a religious doctrine of what he saw as a mere fact of Nature.

I'm not as familiar with philosophical literature as you are, so I Googled Schopenhauer's "Will and Representation (Idea)", and it looks to be generally compatible with my Enformationism worldview --- which I am also still "trying to flesh out". With no formal training in Philosophy, I began from the conjunction of two modern sciences -- Quantum & Information -- instead of from ancient philosophical & religious conjectures. However, I did find Plato's functional notion of First Cause to be a plausible way to express the un-knowable Source of the "Will" that is being expressed in gradual physical evolution. Early in the development of my thesis, I wrote an essay*1 to summarize my understanding of how intentional evolution might work, while avoiding the doctrinal prejudices of Intelligent Design. My primitive understanding has evolved since then, mainly due to feedback from this forum. Since I have no direct revelation from the First Cause, I can only guess at He/r characteristics & intentions, if any.

My personal worldview has a lot in common with ancient theories of Pantheism, but I would prefer to call it PanEnDeism*2, to avoid any dogmatic theistic implications. Also, I take issue with descriptions of primitive entities as "experiential". To me, that term seems to imply that sub-atomic particles consciously interact with their environment. Instead, I think of causal EnFormAction --- similar to Shop's "Will", but more like a goal-directed computer program --- as a primitive form of intentional Causation/Energy that took 14 billion years to evolve into Living & Thinking creatures, and most recently into Self-Conscious beings. This is not a Genesis account, but merely an educated guess. Why the wheelspinning of eons before the advent of philosophical cosmologists? I suppose it has something to do with FreeWill within an otherwise deterministic system of willful causation : matter + energy + laws.

I describe my thesis as a scientific/philosophical update of ancient Atomism/Materialism and Monism/Spiritualism, with new insights from Quantum & Information theories. Unfortunately, 180 scorns it as merely a sci-fi rehash of outdated mind/body Dualism/Spiritualism. :smile:

Note --- My guess is that Self-Consciousness "manifests" when the Cosmic Program of Evolution reaches the minimum necessary complexity for feedback loops of information.
"Feedback occurs when outputs of a system are routed back as inputs as part of a chain of cause-and-effect that forms a circuit or loop. The system can then be said to feed back into itself." ___Wikipedia


*1. Intelligent Evolution :
[i]If the physical universe is not eternal, then the various speculative “multi-verse” and “many worlds” theories cannot explain the brute fact of our temporal existence. Instead, we must devise a theory
that accounts for the finite beginning and formless end envisioned by the cosmological experts.[/i]
https://gnomon.enformationism.info/Essays/Intelligent%20Evolution%20Essay_Prego_120106.pdf

*2. PanEnDeism :
Panendeism is a non-religious ontological position that explores the interrelationship between G*D (The Cosmic Mind) and the known attributes of the universe. Combining aspects of Panentheism and Deism, Panendeism proposes an idea of G*D that both embodies the universe and is transcendent of its observable physical properties.
https://panendeism.org/faq-and-questions/
1. Note : PED is distinguished from general Deism, by its more specific notion of the G*D/Creation relationship; and from PanDeism by its understanding of G*D as preter-natural creator rather than the emergent soul of Nature. Enformationism is a Panendeistic worldview.
https://blog-glossary.enformationism.info/page16.html
180 Proof June 29, 2023 at 18:39 #818823
I'm not as familiar with philosophical literature as you are, so I Googled ...

With no formal training in Philosophy, I began from the conjunction of two modern sciences --

My primitive understanding has evolved ...

My personal worldview ... PanEnDeism ... rehash of outdated mind/body Dualism/Spiritualism.

Reply to Gnomon :eyes: :cry: :lol:
Janus June 29, 2023 at 22:58 #818881
Quoting Srap Tasmaner
Lewis's premise is that reasoning admits of only one description. He could have claimed that other accounts leave out what he's interested in, that they miss the reasoning in an act of reasoning and treat it like any other psychological or biological event. Instead he claims that no such description is even possible, and that nothing that could be so described and explained could be what he considers reasoning.

The question is, why would he think that? And it looks like the answer is: theology.


Is this David Lewis you are speaking about? I'm not very familiar with his work. Is he a theist? I think you are right: that it is only theology which would allow that things are just as we perceive them to be, because God thinks them into existence and then gives us perceptions which accord with his thoughts. Everything lives and has its being in God, according to classical theism.

I see that as a possibility, but not a very plausible one.
Srap Tasmaner June 29, 2023 at 23:12 #818885
Quoting Janus
Is this David Lewis you are speaking about?


C. S. Lewis
Janus June 29, 2023 at 23:32 #818887
Reply to Srap Tasmaner Ah, of course, not so familiar with him either, other than reading Mere Christianity about 40 years ago. As I remember it his arguments didn't impress me much.
Wayfarer June 29, 2023 at 23:47 #818891
Quoting Gnomon
Sounds very similar to my own personal project


The year before my first child was born - he's now 34 - I saw my first-ever laser printer, which had been bought by the place I worked (for about the price of a car). I composed a one-pager on ideas related to the Tao of Physics. When printed, it looked amazingly slick. I was so impressed with myself that I tried to start an actual publication dedicated to such ideas. Of course, it bombed - I had no experience in publishing or marketing and distribution. Tilting at windmills. Later on when blogs became a thing, I too started a blog, but it got no readership. I tried writing a couple of articles on Medium, but there are literally hundreds of thousands of people publishing there. The problem with philosophy as a field is that there are many very clever people with their own agendas. Cutting through is exceedingly difficult. That's why I respect David Chalmers and Jules Evans. Nowadays I'm more circumspect.

I try to stay within the bounds of philosophy, although I do draw on Buddhist and Hindu sources that are outside the Western canon. To my mind, a great deal of what is designated philosophy in modern culture is diametrically opposed to what philosophy originally was (as per Pierre Hadot) - there is a tension between philosophical wisdom and liberalism.


Quoting Metaphysician Undercover
Isn't it obvious to you though, that revelation must produce knowledge?


Far from being ‘obvious’, to most contributors here it would be highly objectionable. In fact you could almost say that anything designated 'revealed truth' will be discounted at the outset of any discussion. Deserves a separate thread.
Tom Storm June 30, 2023 at 00:09 #818895
Quoting Wayfarer
In fact you could almost say that anything designated 'revealed truth' will be discounted at the outset of any discussion. Deserves a separate thread.


Do it - I have an interest in this one. I would like to understand more about the nature of revealed wisdom. I had some interest back in the days when I read about Gnosticism and the notion of revealed wisdom through Gnosis. I spent quite some time talking about this (years ago) with one of Carl Jung's friends, who was a friend of my parents and a key expert on the Jung Codex.
Wayfarer June 30, 2023 at 00:17 #818896
Reply to Tom Storm Wow, you have some interesting friends. I'll see if I can get started on it.

------

I was going to add this snippet I was reading in a New Yorker obituary of Jerry Fodor:

“Neo-Darwinism is taken as axiomatic,” he wrote in “What Darwin Got Wrong,” co-written with Massimo Piattelli-Palmarini, a cognitive scientist, and published in 2010. “It goes literally unquestioned. A view that looks to contradict it, either directly or by implication, is ipso facto rejected, however plausible it may otherwise seem.”


Which reminded me of this exchange earlier in the thread:

Quoting Srap Tasmaner
mathematical and artistic abilities can't be accounted for in terms of the theory (of natural selection, according to Alfred Russel Wallace)

— Wayfarer

We're the only critters we know that have math and art, and we are the way we are because of natural selection, so evidently it does account for math and art.



Srap Tasmaner June 30, 2023 at 00:42 #818898
Reply to Wayfarer

You can always find a guy, but Fodor's writing on evolution found few defenders. Make of that what you will.
Wayfarer June 30, 2023 at 00:55 #818900
Reply to Srap Tasmaner I have noticed that. But I think his basic criticism, that neo-darwinism has become a 'theory of everything', is solid.
180 Proof June 30, 2023 at 01:18 #818903
Reply to Janus :up:

Quoting Wayfarer
... anything designated 'revealed truth' will be discounted ...

Any "truth" that lacks a truth-maker or corroborating public evidence is reasonably discountable (Hume, Kant, Clifford, Popper, Sagan), except, at best, as a fiction.
Srap Tasmaner June 30, 2023 at 01:30 #818907
Reply to Wayfarer I may be misremembering, but I think he claimed that evolution by natural selection is blatantly circular, which is clearly horseshit, and not a criticism working biologists even considered taking to heart.

But, hey, you go ahead and add him to your list of voices crying in the wilderness.
Wayfarer June 30, 2023 at 01:33 #818909
Reply to Srap Tasmaner But then, biologists may be poor judges of philosophical argument.
180 Proof June 30, 2023 at 01:36 #818910
Reply to Wayfarer What does some "philosophical argument" have to do with a well-tested biological theory? :mask:

Wayfarer June 30, 2023 at 01:39 #818913
Reply to 180 Proof Indeed, one of the main points at issue. Something to do with philosophical shortcomings of naturalism.
180 Proof June 30, 2023 at 01:46 #818915
Reply to Wayfarer Naturalism is a conceptual paradigm, biology is a natural science, NeoDarwinian Evolution is a scientific model. Try not to confuse, or conflate, them, Wayfarer. 'Objections to naturalism' are irrelevant to biology, chemistry, physics, etc.
Janus June 30, 2023 at 03:15 #818927
Quoting Wayfarer
But then, biologists may be poor judges of philosophical argument.


As may mystics and theists.

Quoting 180 Proof
Any "truth" that lacks a truth-maker or corroborating public evidence is reasonably discountable (Hume, Kant, Clifford, Popper, Sagan) except, at best, as a fiction.


:up: I really cannot see any reason why this should be denied. It seems to me that those who reject it are those who just don't want to accept it. These interminable arguments that are not really arguments at all ! :roll:
Wayfarer June 30, 2023 at 03:57 #818931
Quoting 180 Proof
NeoDarwinian Evolution is a scientific model….


…which also serves as an ideological attitude, as amply illustrated in many exchanges here.
Gnomon June 30, 2023 at 17:03 #819036
Quoting Srap Tasmaner
Yes, yes, we all know there is another framework. What you need to argue for is exclusivity. . . .
As I understand it, you are not proposing an alternative scientific theory, and imagine your quest as challenging a foundational assumption of science. . . .
Your choice then is (1) present your view as a genuine scientific hypothesis; (2) challenge the methodology of science. Mostly theists opt for door number 2, and defend revelation as knowledge producing. . . . .
There is one last alternative, which is not to challenge science but to live alongside it,

Personally, I don't read Reply to Wayfarer's modest proposals as "challenging science" or arguing for "exclusivity" of philosophical reasoning versus scientific reasoning. Like me, he seems to be content with the pragmatic scientific "revelations" of the material world. But, at the same time, he is keenly aware that the human mind is still a black box*1 for those who seek a material explanation for Mental phenomena, such as Reasoning. That's why he is not proposing "an alternative scientific theory", or "challenging a foundational assumption", but instead, exploring some ancient & modern philosophical theories --- perhaps parallel to the materialistic presumptions, rather than diametrically opposed. Black vs White oppositions are typical of politics, but when philosophy gets into politics, what you get is Sophistry.

Your insistence on a "genuine scientific hypothesis" may reveal an implicit attitude of exclusivity : "Philosophy has nothing important to say about the 'hard problem', so only a scientific hypothesis can be taken seriously". It's true that philosophical theorizing is unlikely to reveal the physical "seat of consciousness". Yet a quick overview of current scientific hypotheses reveals that the imaginary "seat" seems to be all over the place, mostly in the head*2. Each team points to a different "grid" or region of the brain. But, are these localized conjectures any more authoritative than the generalized speculations of philosophers? For example, Chalmers is asking general "why" queries (relationships), instead of specific "what" questions (neurons)*3. FYI : David Chalmers is a professor of philosophy and neural science at New York University,

This is a philosophy forum. So why would you require an amateur philosopher to provide a "genuine scientific hypothesis", when the professional scientists, after years of research, are still arguing among themselves? Why should we force Philosophy to "challenge" Science, when they are so successful in working side-by-side*4? For example, Einstein was not an empirical scientist, but a mathematical seeker after a priori or necessary truths of nature. He postulated hypotheses based primarily on imagination*5, and then waited for the empiricists to provide the hard evidence to support what he already knew to be true : teamwork. Perhaps Wayfarer is already opting for your "last alternative". :smile:

PS___I interpret Wayfarer's "revelations" to be those of Imagination, rather than of divine Inspiration. He has already explained that he is not a theist, as you seem to imply.


*1. What does the mind is a black box mean? :
To behaviorists, the mind is a “black box.” In science and engineering, the term black box refers to any complex device for which we know the inputs and outputs, but not the inner workings.
https://www.td.org/insights/why-the-brain-is-still-a-black-box-and-what-to-do-about-it

*2. Seat of Consciousness :

"The brainstem is the seat of human consciousness"
https://medium.com/@philipodegard/the-seat-of-human-consciousness-6dbce3bfa6de

" At least two regions of the brain decide what we perceive"
https://neurosciencenews.com/neuroscience-consciousness-brain-regions-1362/

"It found that consciousness may emerge from a grid-like interconnection of neurons at the back of the head. . . . The reigning theory is just a first win. The opposing team—which thinks consciousness stems from the executive frontal parts of the brain—is ready to fight back with a new test design."
https://singularityhub.com/2023/06/27/where-does-consciousness-originate-two-leading-theories-go-head-to-head/

*3. Physics vs Experience :
The hard problem of consciousness (Chalmers 1995) is the problem of explaining the relationship between physical phenomena, such as brain processes, and experience (i.e., phenomenal consciousness, or mental states/events with phenomenal qualities or qualia).
http://www.scholarpedia.org/article/Hard_problem_of_consciousness

*4. Empirical vs Theoretical Science :
Science is about empirical knowledge; philosophy is often about that but is also about a priori knowledge (if it exists). Science is about contingent facts or truths; philosophy is often about that but is also about necessary truths (if they exist).
https://1000wordphilosophy.com/2018/02/13/philosophy-and-its-contrast-with-science/

*5. PHILOSOPHY IS APPLIED IMAGINATION
To imagine is to represent without aiming at things as they actually, presently, and subjectively are. One can use imagination to represent possibilities other than the actual, to represent times other than the present, and to represent perspectives other than one’s own. Unlike perceiving and believing, imagining something does not require one to consider that something to be the case. Unlike desiring or anticipating, imagining something does not require one to wish or expect that something to be the case.
https://plato.stanford.edu/entries/imagination/

User image
Janus June 30, 2023 at 23:18 #819086
Quoting Wayfarer
…which also serves as an ideological attitude, as amply illustrated in many exchanges here.


When it is presented as a philosophical position it is not so much an ideology as an inference to the best explanation, which is that humans evolved from a common ancestor with other primates to become what they are today. I don't think anyone with half a brain would deny that social and cultural factors also played a huge part in that human evolution, and that language and all it enables sets us apart, for better or worse, from all other animals. To say that the whole thing was somehow planned, a claim for which there can be no evidence, would amount to espousing an ideology.

Reply to Gnomon I don't think Einstein was thinking about imagination as a faculty standing free from science, but rather in its service.
Wayfarer June 30, 2023 at 23:24 #819088
Reply to Gnomon Thank you.
Wayfarer June 30, 2023 at 23:39 #819094
Quoting Janus
To say that the whole thing was somehow planned, a claim for which there can be no evidence, would amount to espousing an ideology.


Indeed. Conversely, what philosophical point do you think is being made by this oft-cited trope?

Quoting Srap Tasmaner
As someone somewhere on this forum once said, the answer to "How long would it take monkeys to compose the complete works of Shakespeare?" is about 300,000 years. That experiment has already been run.


Janus June 30, 2023 at 23:47 #819096
Reply to Wayfarer The philosophical point seems to be that we evolved from ancestral animals along with all the other contemporary species, and that what we are able to do courtesy of language is not something which sets us apart in the sense of being altogether different kinds of beings. Looked at from the perspective of ecology language is one enormous adaptive advantage in one sense, and an enormous disadvantage in another; in that we have been able to continue to overuse resources to our own detriment, whereas other species who do that get knocked back in a timely way.
Wayfarer July 01, 2023 at 00:28 #819103
Reply to Janus I took the point to be the claim that life originates as a chance event. The analogy of monkeys typing represents the random combination of elements that just happened to form themselves into organisms.
Janus July 01, 2023 at 00:43 #819104
Reply to Wayfarer Considering the number of posdible combinations of English words, not to mention the almost infintely greater number of possible combinations of the letters, punctuations and spaces, the idea that one of Shakespeare's plays could have been produced by random typing is absurd.

On the other hand the elements combine in only very limited, lawlike ways, and there are only four 'letters' in the genetic code. so it seems a poor analogy.
180 Proof July 01, 2023 at 00:53 #819106
Quoting Janus
I don't think Einstein was thinking about imagination as a faculty standing free from science, but rather in its service.

:up:

Quoting Wayfarer
I took the point to be the claim that life originates as a chance event.

"Biological evolution" models the development of life just as "Big Bang cosmology" models the development of the universe – neither model explains the "origin" of life or the universe, respectively. However, as reasons to the best explanation, both models (usually) eliminate intelligent reliance on non/super-natural "origin stories".

Janus July 01, 2023 at 01:20 #819110
Reply to 180 Proof Right, the chain of explanation is always potentially without end. The child can always endlessly ask 'why?'.
Srap Tasmaner July 01, 2023 at 02:06 #819114
Quoting Wayfarer
Indeed. Conversely, what philosophical point do you think is being made by this oft-cited trope?

As someone somewhere on this forum once said, the answer to "How long would it take monkeys to compose the complete works of Shakespeare?" is about 300,000 years. That experiment has already been run. — Srap Tasmaner


Probably not a great number for me to have chosen, since that's the emergence of homo sapiens. Something in the millions for hominids or for simians would have been a better choice.

Quoting Janus
Looked at from the perspective of ecology language is one enormous adaptive advantage in one sense


I wasn't making any claim about language, or about the adaptive value of language. The point stands if you ask "How long would it take mammals to produce the work of Shakespeare?" and move the starting-point back even more -- but it's not as picturesque as the monkeys.

Quoting Wayfarer
I took the point to be the claim that life originates as a chance event.


Truly bizarre. I am speechless.

Quoting Wayfarer
The analogy of monkeys typing represents the random combination of elements that just happened to form themselves into organisms.


Now, see, if you had thought about it for a minute, you might have realized that I was making exactly the opposite point. Evolution gets results in the timeframes that it does by not being random. It threw up mammals, then simians, then hominids, then finally something like us. Took millions of years to keep ratcheting up the complexity that would so dramatically increase our cognitive capacity that we might have among us one -- and even then only one, among all the humans who have ever lived -- with the mind of Shakespeare.

I don't have to claim that the ability to write blank verse revenge tragedies is an adaptive advantage. I'm not insane.

The point I have been making is only that the creature that produced Lear shares, what is it, 99.5% of his DNA with chimpanzees, and more than a little with plenty of other terrestrial life forms. He is a product of the same process that produced every living thing we know of.

Now you want to say that his body is, but his mind -- no, no, that's, I don't know, magic, or whatever it is you think makes humans dramatically different from everything else living. And yet it's perfectly obvious that evolution endowed other creatures with mind as well. We are not so unique as all that. And there's more and more evidence that our minds have little flaws that betray their evolutionary origins, just as our bodies do, just as all animals do. Evolution loves a workable kludge.
Srap Tasmaner July 01, 2023 at 02:14 #819117
Quoting Gnomon
Personally, I don't read ?Wayfarer's modest proposals as "challenging science" or arguing for "exclusivity" of philosophical reasoning versus scientific reasoning.


Did you read the OP?
Janus July 01, 2023 at 02:23 #819120
Quoting Srap Tasmaner
I wasn't making any claim about language, or about the adaptive value of language. The point stands if you ask "How long would it take mammals to produce the work of Shakespeare?" and move the starting-point back even more -- but it's not as picturesque as the monkeys.


I took you to be saying that a monkey descendant has produced the work of Shakespeare. and thus that the experiment has already been run. Obviously the work could not have been produced without language, so I took the role of language in the experiment as implcitly given.
Srap Tasmaner July 01, 2023 at 02:30 #819124
Quoting Janus
I took you to be saying that a monkey descendant has produced the work of Shakespeare. and thus that the experiment has already been run. Obviously the work could not have been produced without language, so I took the role of language in the experiment as implcitly given.


Sure, sure. I just don't have to commit to anything about the origin of language, I don't think.

It was an ape that wrote Lear. Obviously it was an ape that could write. So he was a member of a species that it is capable of language use, however that happened.
Srap Tasmaner July 01, 2023 at 02:32 #819125
Reply to Janus

It's just the old "Make an apple pie from scratch" joke.
Janus July 01, 2023 at 02:32 #819126
Reply to Srap Tasmaner Quoting Srap Tasmaner
Sure, sure. I just don't have to commit to anything about the origin of language, I don't think.

It was an ape that wrote Lear. Obviously it was an ape that could write. So he was a member of a species that it is capable of language use, however that happened.


:up:
Srap Tasmaner July 01, 2023 at 02:41 #819129
Reply to Janus

You know, if you had seen little Will Shakespeare as an infant, as a toddler, you wouldn't have thought him capable of writing Lear, if only because until he did it, nobody knew it was a thing that could be done. And, of course, because his feet were muddy and he didn't know very many words yet.

But that child does grow into Shakespeare, a wonder of human history.

Well so it is with his species. To see those little furry things skulking about, burrowing underground or climbing trees to avoid being eaten my those freakin' reptiles, you couldn't guess their descendants would include Will Shakespeare, or that they would one day transform this planet's ecosystem or build machines that could take them into space. But we don't have to guess because we know it did happen.

(This isn't a rebuttal of anything you said, just some further thoughts in response.)
Wayfarer July 01, 2023 at 05:54 #819152
Quoting Srap Tasmaner
Evolution gets results in the timeframes that it does by not being random.

I took it as a reference to the million monkeys trope https://en.m.wikipedia.org/wiki/Infinite_monkey_theorem, often invoked as an account of how life could have started as a consequence of chance.
Wayfarer July 01, 2023 at 05:57 #819153
Quoting Srap Tasmaner
The point I have been making is only that the creature that produced Lear shares, what is it, 99.5% of his DNA with chimpanzees, and more than a little with plenty of other terrestrial life forms. He is a product of the same process that produced every living thing we know of.


Plainly, but the difference makes a difference. H Sapiens has passed an evolutionary threshold with the faculty of reason and language being key to that. That brings with it capabilities which I don’t believe are reducible to biology.
Wayfarer July 01, 2023 at 06:08 #819155
Quoting Srap Tasmaner
t threw up mammals, then simians, then hominids, then finally something like us.


I think you will find that any idea of there being progress in this sense is rejected by mainstream biology on account of it being orthogenetic which is defined as the theory that evolutionary variations follow a particular direction and are not sporadic and fortuitous. As it happens the process has turned out h. sapiens, but there's no reason given in the theory as to why that particular outcome. And indeed the fact that there is no reason in that sense is central to the whole argument.
Srap Tasmaner July 01, 2023 at 06:22 #819157
Quoting Wayfarer
I took it as a reference to


Good lord, yes, of course it is.

Quoting Wayfarer
often invoked as an account of how life could have started as a consequence of chance


Okay, maybe, sure, someone might've said that, but the use of the image for various purposes is long established, it wasn't invented just for talking about abiogenesis or for talking about evolution, though yes it's a favorite crackpot argument of creationists, which is why people like me refer directly to it in rebutting them. Do you really not understand how this works?

Quoting Wayfarer
evolutionary threshold


Define. Better yet, define it non-circularly so that Fodor's ghost doesn't haunt you.

Quoting Wayfarer
capabilities which I don’t believe are reducible to biology


Neat. Did we acquire these capabilities biologically?

By the way, what does it mean for a capability to be reducible to biology?

Quoting Wayfarer
t threw up mammals, then simians, then hominids, then finally something like us. — Srap Tasmaner

I think you will find that any idea of there being progress in this sense is rejected by mainstream biology


You may notice that the connectives there are then's -- not and then even better's. I could have said what you think I said, but, as it happens, I didn't.

But you read it that way, which means you assumed I was talking about progress, which means you ought to be chiding yourself not me. Go easy on yourself though.

Another way to read what I actually wrote was from the general to the specific, just taxonomy spread out chronologically, something speciation tends to do.
Wayfarer July 01, 2023 at 06:44 #819162
Quoting Srap Tasmaner
Another way to read what I actually wrote was from the general to the specific, just taxonomy spread out chronologically, something speciation tends to do.


Your use of the word 'finally' clearly suggests goal-directedness.

I have said numerous times in this thread that h. sapiens has plainly evolved in the sense described by the theory of evoluition. But that with the development of language and reason, we transcend purely biological determination in a way that other animals do not. Only humans can consider questions such as whether there are domains of being beyond the sensory world, for example, not to mention more quotidian abilities, such a mathematics, science, and so on.
Tom Storm July 01, 2023 at 14:40 #819227
Quoting Wayfarer
Only humans can consider questions such as whether there are domains of being beyond the sensory world, for example, not to mention more quotidian abilities, such a mathematics, science, and so on.


Perhaps and yet I envy animals who are self-sufficient and need no cars or porn or bad movies by Disney; who have no reason and no governments and no jails and no persecutions or prejudices nor layered psychological cruelties or stupid dead end jobs. I can't help feel that it is animals who often live the superior life, precisely because they don't need to speculate, think or fester and can live in the moment, taking no more than they need, as they need it. :razz:
Srap Tasmaner July 01, 2023 at 14:58 #819234
Quoting Wayfarer
Your use of the word 'finally' clearly suggests goal-directedness.


'Finally' clearly suggests 'last', the last item in my sequence, since we have not seen further speciation since modern humans emerged.

This little inquisition to determine whether I have sinned against Science is pointless anyway. I was not telling the story of evolution but of the story of Shakespeare, beginning from the first mammals, so I know where my story ends (or 'finishes') and the plot highlights those events that lead there.

Quoting Wayfarer
But that with the development of language and reason, we transcend purely biological determination in a way that other animals do not.


A quick search yielded a couple of nice popular articles on the problem-solving abilities of crows: one at Ars Technica on a specific experiment and one from the BBC that ranges somewhat wider. Both articles struck me, as a layman, as pretty balanced. Both seemed to assume, more or less as I do, that our intelligence is on some kind of continuum with other animals. "Continuum" is not a great word there, though, because it may not be a matter of having more or having less of one thing, general intelligence, but of having more or having fewer cognitive skills, particular abilities. (I think the BBC article mentioned that chimps seem to have better short-term memory than we do, which I didn't know.)

I don't know how to fit your talk of "transcendence" with the way scientists and science journalists talk about intelligence in animals, including us. Language, for example, allows displacement, the ability to communicate about objects not in our present surroundings; you could describe that as "transcending" the limit of referring only to what other animals can or do perceive. What's not clear is in what sense displacement would transcend "purely biological determination," since it's not clear what that is, and if we have transcended it what other sort of determination is at play in what we actually do in our humble, biological way.
Srap Tasmaner July 01, 2023 at 14:59 #819237
Reply to Tom Storm

Whitman:
I think I could turn and live with animals, they are so placid and self-contain'd,
I stand and look at them long and long.

They do not sweat and whine about their condition,
They do not lie awake in the dark and weep for their sins,
They do not make me sick discussing their duty to God,
Not one is dissatisfied, not one is demented with the mania of owning things,
Not one kneels to another, nor to his kind that lived thousands of years ago,
Not one is respectable or unhappy over the whole earth.
Gnomon July 01, 2023 at 16:05 #819256
Quoting Janus
?Gnomon
I don't think Einstein was thinking about imagination as a faculty standing free from science, but rather in its service.

Of course! I posted the quote only because Wayfarer's "revelations" were being implicitly compared to divine revelations, in the service of religion instead of science. I just wanted to remind forum posters that informed imagination is not a no-no on a philosophy forum.

Both philosophical and scientific theories are imaginary conjectures (speculations), not empirical observations. As Einstein noted, imagination points the way to future knowledge. And, as the OP implied : our current knowledge of the human Mind --- as contrasted with the Brain --- is quite sketchy, and based mostly on guessing. Moreover, the provenance and role of Reason (rational imagination) is suspect in some quarters, perhaps due to its being subject to the whims of Emotion. So, I think Way was being accused of being driven by passionate Emotion, instead of dispassionate Reason. I beg to differ. :smile:

David Hume on Reason :
"Reason is, and ought only to be the slave of the passions, and can never pretend to any other office than to serve and obey them."
https://sites.pitt.edu/~mthompso/readings/hume.influencing.pdf
Gnomon July 01, 2023 at 16:32 #819259
Quoting Srap Tasmaner
Personally, I don't read ?Wayfarer's modest proposals as "challenging science" or arguing for "exclusivity" of philosophical reasoning versus scientific reasoning. — Gnomon
Did you read the OP?


Yes. But perhaps I read it with different preconceptions.

I understood him to be questioning the metaphysical assumptions of doctrinal Materialism, not the empirical methods of practical Science. As pragmatist/naturalist John Dewey noted, modern Materialism pretends to be Monistic, but as an explanation for the emergence of mental phenomena from a material substrate, logically it must assume Dualistic origins : similar to Aristotle's theory of Hylomorphism. :smile:

PS__The "foundational assumption" he was "challenging" is that of philosophical Materialism, not of pragmatic Science. So, his "alternative theory" was philosophical, not scientific.


THE METAPHYSICAL ASSUMPTIONS OF MATERIALISM :
In his first published article, "The Meta-physical Assumptions of Materialism" (written in 1881), Dewey found the doctrine "which declares that matter and its forces adequately account for all phenomena -- those of the material world, commonly so called, and those of life, mind, and society" -- to be lacking both in clarity and logical consistency. Of the several destructive conclusions which he discovered to be implicit in monistic materialism one was to be of particular importance for his later naturalism: if the materialist begins with the assumption that mind and the molar forms of matter are constructed ultimately from molecular blocks of matter, he must end "with the conclusion that the ultimate form of matter has dualistic 'mind' and 'matter' properties . . . . If a materialist were to say that this double-sided substance is what he means by matter, we could only reply that he is playing with words--that it is just as much mind as it is matter.
http://home.uchicago.edu/~rjr6/articles/Dewey.pdf

John Dewey :
John Dewey was a leading proponent of the American school of thought known as pragmatism, a view that rejected the dualistic epistemology and metaphysics of modern philosophy in favor of a naturalistic approach that viewed knowledge as arising from an active adaptation of the human organism to its environment.
https://iep.utm.edu/john-dewey/

Hylomorphism, (from Greek hyl?, “matter”; morph?, “form”), in philosophy, metaphysical view according to which every natural body consists of two intrinsic principles, one potential, namely, primary matter, and one actual, namely, substantial form.
https://www.britannica.com/topic/hylomorphism

Reply to Wayfarer
Srap Tasmaner July 01, 2023 at 17:59 #819288
Reply to Gnomon

It's a class and category thing. The first premise claims that rational and biological are classes, and a given phenomenon can be in one or the other but never both. The response (beginning with Anscombe) has most often been that rational and biological are categories, and there's no reason at all something can't be both. (Calling these both 'dualisms' obscures the distinction.)

Here's a bit from the conclusion of the article you linked:

In the early years of his career he was an absolute idealist and in no way could be considered a materialist. The second stage of his thought on this question, reaching definitive proportions with Experience and Nature, revealed him to be a neutralist: the ultimate reality is neither physical nor mental, but such that it permits the ascription of those properties through inquiry. The sense in which he might be considered a materialist at this stage is in his disavowal of mind as an independent entity shaping the destinies of matter. In the final period of his thought Dewey still affirmed the ultimately neutral character of natural events, but saw their transactional phases so inextricably linked in the situational complex that the hope was provided that with the advance of scientific inquiry someday the necessary and sufficient conditions for mental behavior might be given in terms of its physical matrix. Thus, if the hypothesis that the proper manipulation of the physical properties of the human organism can assure control of its mental properties is materialistic, then in his last years Dewey was indeed a materialist.


I've only begun reading Dewey, so I don't know him remotely as well as I should. His early essay on the importance of Darwin and how evolution ought to reshape philosophy I thought extraordinary. (Might make more sense for y'all to reach for James, who, though a genuine working scientist, always left more than a little room in his philosophy for spirituality and religion.)

I'm not sure what's to be gained from lining up on two sides to say "There's one kind of thing!" or "There's two!!" More interesting is what you can do with such a claim. Naturalism is pretty straightforward as a working assumption, rather than a dogma; you know how to proceed, what sorts of things to look for, how to design experiments, how to craft a research program. I'm not clear what the other side offers except a defense of people's common pre-scientific beliefs. How are we to investigate the transcendence of biology? Is there a way to do that scientifically? If so, bring it on.

This is why I have tried to force y'all to be more specific. If you say, here's something evolution can't do, what do you mean by that? Are you in the trenches of biology, offering an alternative theory? Evidently not. Are you challenging science's approach to knowledge production? No one will say so. If you're saying that here's something that by definition evolution can't do, then you're playing semantic games and the rest of us can ignore you.
Gnomon July 01, 2023 at 22:00 #819333
Quoting Srap Tasmaner
It's a class and category thing. The first premise claims that rational and biological are classes, and a given phenomenon can be in one or the other but never both. The response (beginning with Anscombe) has most often been that rational and biological are categories, and there's no reason at all something can't be both. (Calling these both 'dualisms' obscures the distinction.)

I agree that philosophers, for the sake of argument, often make such compartmentalized distinctions, regarding controversial topics. From a general philosophical perspective, "Reason" and "Biology" and "Psychology" are in separate classes (type or kind or categories of being) with completely different physical and semantic characteristics. Yet, from the standpoint of a monistic Materialistic belief system, they are merely convenient categories for discussion, but ultimately all features & phenomena of the world are presumed to be subject to the rationally-inferred laws (regularities) of Physics.

Then, there is a monistic Meta-Physical*1 perspective, in which Matter is limited by the restrictions of physical laws, but Mental phenomena are free from such strictures. Which is why the human mind is able to believe and to tell lies. In that case, Reason is only subject to the Laws of Logic, which are essentially the same as Mathematics. :nerd:

*1. Meta-Physical : anything not perceivable via the physical senses, but conceivable by the mental faculties --- intangible, abstract


Quoting Srap Tasmaner
This is why I have tried to force y'all to be more specific. If you say, here's something evolution can't do, what do you mean by that? Are you in the trenches of biology, offering an alternative theory? Evidently not. Are you challenging science's approach to knowledge production? No one will say so. If you're saying that here's something that by definition evolution can't do, then you're playing semantic games and the rest of us can ignore you.

Specificity is a necessity for philosophical discussions, because each of us comes to the argument with a personal belief system (set of prejudices). That's why a primary rule of dialog is : first, define your terms. And that's why Wayfarer specified that his terminology is not limited to definitions classified under the heading of Materialism*2. He may not have been as specific about which alternative dictionary (or philosophical tradition) he draws his meanings from. But, I'm sure he will give you that information, if you ask him.

Regarding the evolution of Reason, I can't speak for Way. Our worldviews seem to be similar, but his philosophical background is quite different from mine. And he might disagree with some of my unconventional ideas & terminology. For myself though, I will specify that, for all practical purposes, I am a materialist, "living in a material world" (pace Madonna). And for scientific purposes, I assume that the modern synthesis of Evolutionary theory is generally correct. Anything that currently exists in the world, is something that Evolution could do, and did. And that includes things/ideas that are not made of massive matter, but of meaningful information.

So, for my personal philosophical purposes, I have developed my own "alternative" theory, which does not exclude Mental phenomena from consideration*4. My private personal theory of evolution includes insights from Quantum & Information sciences, that were not known to Darwin. And it specifically attempts to explain how the immaterial class of Mind could evolve over eons of time from the initial conditions & laws & causes in place at the beginning of space-time. This alternative theory is intended to help explain how Evolution did somehow produce immaterial Minds, only after 14 billion years of physical/material interactions , not accidentally, but guided by the inherent Laws of Nature. :smile:

PS___Regarding "semantic games", when posters on a philosophy forum do not share, or attempt to understand, the worldviews (belief system & its assumptions) of their fellows, a dialog soon devolves into a "semantic game". So, lets make an effort to see the topic from someone else's perspective.


*2. Wayfarer from OP :
In order to clearly frame the argument from reason, it is necessary to understand what it is opposed to. This is usually said to be ‘naturalism’, but I will instead propose that its target is better named physicalism or materialism.
https://thephilosophyforum.com/discussion/14418/the-argument-from-reason/p1
Note --- He didn't say he was opposed to the Nature or the scientific method, only to certain belief systems that claim the absolute authority of Scientism.

*3. The Mind-Evolution Problem :
The Difficulty of Fitting Consciousness in an Evolutionary Framework
https://www.frontiersin.org/articles/10.3389/fpsyg.2018.01537/full

*4. Evolution of Mind :
But then the conscious mind constitutes a special dilemma, since this modern picture was produced precisely by excluding all mental properties from physical nature.
https://www.thenewatlantis.com/publications/the-illusionist

Reply to Wayfarer
Tom Storm July 01, 2023 at 22:58 #819339
Whitman:I think I could turn and live with animals, they are so placid and self-contain'd,
I stand and look at them long and long.

They do not sweat and whine about their condition,
They do not lie awake in the dark and weep for their sins,
They do not make me sick discussing their duty to God,
Not one is dissatisfied, not one is demented with the mania of owning things,
Not one kneels to another, nor to his kind that lived thousands of years ago,
Not one is respectable or unhappy over the whole earth.


:up: He's on the money here. Thanks for the verse.
Wayfarer July 01, 2023 at 23:17 #819342
Quoting Tom Storm
I can't help feel that it is animals who often live the superior life...


Perhaps that innocence is what was lost in the mythology of the Fall.

Quoting Srap Tasmaner
Both seemed to assume, more or less as I do, that our intelligence is on some kind of continuum with other animals. "Continuum" is not a great word there, though, because it may not be a matter of having more or having less of one thing, general intelligence, but of having more or having fewer cognitive skills, particular abilities.


The problem-solving abilities of crows and such are often cited in this context. Parrots are also clearly intelligent with problem-solving skills. I suppose it's seen as the basis for a continuum, a proto-version of what our intelligence turns out to be, so conforming with the idea of 'gradual development'.

But I see a radical break - an ontological distinction, in philosophical terms - at the point where humans become fully self-aware, language-using and rational creatures.

I think the cultural dynamics behind this, is that for modern culture, 'nature' is now the nearest thing we have to 'the sacred'. Hence the (laudable) reverence for environment, first nations peoples, and so on. Conversely Biblical religions are said to have encouraged the subjugation of nature. So the assertion of a radical difference between humans and animals is, I think, seen as a relic of Judeo-Christian mythology.

There's a passage in Max Horkheimer's book, The Eclipse of Reason, about this dynamic:

[quote=Eclipse of Reason, pp. 123-127]In traditional theology and metaphysics, the natural was largely conceived as the evil, and the spiritual or supernatural as the good. In popular Darwinism, the good is the well-adapted, and the value of that to which the organism adapts itself is unquestioned or is measured only in terms of further adaptation. However, being well adapted to one’s surroundings is tantamount to being capable of coping successfully with them, of mastering the forces that beset one. Thus the theoretical denial of the spirit’s antagonism to nature – even as implied in the doctrine of interrelation between the various forms of organic life, including man – frequently amounts in practice to subscribing to the principle of man’s continuous and thoroughgoing domination of nature. Regarding reason as a natural organ does not divest it of the trend to domination or invest it with greater potentialities for reconciliation. On the contrary, the abdication of the spirit in popular Darwinism entails the rejection of any elements of the mind that transcend the function of adaptation and consequently are not instruments of self-preservation. Reason disavows its own primacy and professes to be a mere servant of natural selection. On the surface, this new empirical reason seems more humble toward nature than the reason of the metaphysical tradition. Actually, however, it is arrogant, practical mind riding roughshod over the ‘useless spiritual,’ and dismissing any view of nature in which the latter is taken to be more than a stimulus to human activity. The effects of this view are not confined to modern philosophy.[/quote]

(Important to note that Horkheimer is referring to 'popular Darwinism' which he distinguishes from what Darwin himself wrote.)

Quoting Srap Tasmaner
Language, for example, allows displacement, the ability to communicate about objects not in our present surroundings; you could describe that as "transcending" the limit of referring only to what other animals can or do perceive.


That's certainly part of it. There's a book I've never got around to (one of thousand) by Chomsky and a collaborator, Why Only Us? In that book Chomsky and Berwick argue that human language is a distinct and innate cognitive capacity that sets us apart from other animals. They propose that language is not a result of gradual evolution, as commonly believed, but rather a sudden and unique emergence in human history. They emphasize that the development of language cannot be explained by natural selection acting on incremental changes, as is the case with other biological traits. (Chomsky however always intends to stay within the confines of naturalism, whereas I myself don't suffer from that inhibition.)

In any case, in traditional philosophy and religion, I believe the reverence accorded to reason as a kind of 'divine instrument' is at least symbolically meaningful. After all, here we are, the day before yesterday we were chasing wildebeest around the savanah, now we're able to weigh and measure the Universe. Something which no crow will ever do.

Quoting Srap Tasmaner
If you're saying that here's something that by definition evolution can't do, then you're playing semantic games and the rest of us can ignore you.


Again - it's not evolutionary theory that is at issue, but darwinian materialism, represented in popular culture by Dawkins and Dennett, but implicit in a great deal of naturalism. Not all though. There are non-materialist evolutionary biologists, many of those mentioned by Apokrisis, for instance Robert Rosen:

[quote=Life Itself]For centuries, it was believed that the only scientific approach to the question "What is life?" must proceed from the Cartesian metaphor (organism as machine). Classical approaches in science, which also borrow heavily from Newtonian mechanics, are based on a process called "reductionism." The thinking was that we can better learn about an intricate, complicated system (like an organism) if we take it apart, study the components, and then reconstruct the system-thereby gaining an understanding of the whole.

However, Rosen argues that reductionism does not work in biology and ignores the complexity of organisms. Life Itself, a landmark work, represents the scientific and intellectual journey that led Rosen to question reductionism and develop new scientific approaches to understanding the nature of life. Ultimately, Rosen proposes an answer to the original question about the causal basis of life in organisms. He asserts that renouncing the mechanistic and reductionistic paradigm does not mean abandoning science. Instead, Rosen offers an alternate paradigm for science that takes into account the relational impacts of organization in natural systems and is based on organized matter rather than on particulate matter alone.[/quote]

Quoting Srap Tasmaner
Are you in the trenches of biology, offering an alternative theory?


I’m not ‘in the trenches’, but there's a lot of dissent from the neo-darwinian orthodoxy.

[quote=The Third Way of Evolution; https://www.thethirdwayofevolution.com/] The vast majority of people believe that there are only two alternative ways to explain the origins of biological diversity. One way is Creationism that depends upon intervention by a divine Creator. That is clearly unscientific because it brings an arbitrary supernatural force into the evolution process. The commonly accepted alternative is Neo-Darwinism, which is clearly naturalistic science but ignores much contemporary molecular evidence and invokes a set of unsupported assumptions about the accidental nature of hereditary variation. Neo-Darwinism ignores important rapid evolutionary processes such as symbiogenesis, horizontal DNA transfer, action of mobile DNA and epigenetic modifications. Moreover, some Neo-Darwinists have elevated Natural Selection into a unique creative force that solves all the difficult evolutionary problems without a real empirical basis. Many scientists today see the need for a deeper and more complete exploration of all aspects of the evolutionary process.[/quote]

Steve Talbott whom I mentioned previously is represented on this site, along with many others, none of them ID representatives (which is the inevitable suggestion for anyone who questions the consensus.)
Janus July 01, 2023 at 23:27 #819344
Quoting Srap Tasmaner
But that child does grow into Shakespeare, a wonder of human history.

Well so it is with his species. To see those little furry things skulking about, burrowing underground or climbing trees to avoid being eaten my those freakin' reptiles, you couldn't guess their descendants would include Will Shakespeare, or that they would one day transform this planet's ecosystem or build machines that could take them into space. But we don't have to guess because we know it did happen.


Yes, it is remarkable what language (and the opposable thumb?) has enabled those little naked apes to do. Of course, fossil fuels have been a fortuitous blessing or curse, without which we arguably would never have breached the atmosphere or even have gotten very far off the ground.

And I agree; none of this would have been predictable. We like to think we have a certain destiny, but that is radically mistaken, human hubris.

Reply to Srap Tasmaner Nice! I love Whitman!

Quoting Gnomon
Of course! I posted the quote only because Wayfarer's "revelations" were being implicitly compared to divine revelations, in the service of religion instead of science.


I don't know what others had in mind, but I was responding just to your posting of the Einstein quote. There are those who think the metaphysicalist imagination should be unfettered by science, by physicalism, and I don't think Einstein was one of them. That is all my response was concerned with.
Wayfarer July 01, 2023 at 23:31 #819347
Quoting Janus
We like to think we have a certain destiny, but that is radically mistaken, human hubris.


also a convenient way to dodge the implications of our existential situation.
Janus July 01, 2023 at 23:39 #819351
Reply to Wayfarer I have no idea what you are referring to. The implications of our existential situation are a matter of interpretation as I see it. I see science as playing a huge role in any rationally informed understanding of our existential situation.
Srap Tasmaner July 02, 2023 at 03:02 #819396
Quoting Wayfarer
But I see a radical break - an ontological distinction, in philosophical terms - at the point where humans become fully self-aware, language-using and rational creatures.


Yes. I've read one or two of your posts. Is there a radical break when creatures started living on land? When they took to the skies? When they started using tools? No? When they developed communication? When they developed the ability to navigate across thousands of miles by sensing the earth's magnetic field? No? When they developed social structures? No?

None of the things characteristic of any other species count as transcending biology. Why on earth would something unique to us? Why do we alone transcend biology?

And I ask that, still not knowing what it means, or how it is supposed to have happened. Was it a biological process by which we transcended biology? Is there any way to know whether dolphins have too? Or maybe octopuses?
Wayfarer July 02, 2023 at 03:23 #819399
Quoting Srap Tasmaner
Why do we alone transcend biology?


Aristotle differentiated man as ‘the rational animal’. Rationality is certainly one aspect - it enables h sapiens to see well beyond the stimulus-response perception of other creatures. There are other faculties as well, artistic, spiritual, philosophical, scientific - I think these differentiate h. Sapiens from other creatures. You don’t agree?
Wayfarer July 02, 2023 at 03:58 #819408
Another relevant snippet from Horkheimer:

We might say that the history of reason or enlightenment from its beginnings in Greece down to the present has led to a state of affairs in which even the word 'reason' is suspected of connoting some mythological entity.
:lol: :lol: :lol:
Srap Tasmaner July 02, 2023 at 04:20 #819412
Reply to Wayfarer

That we do things other creatures don't? Of course we do. And other creatures do things we don't. Where does the transcending biology come in? Is being a living thing not extraordinary enough?

Now and then, I look around the yard, trees and grass, deer, birds, squirrels, and I think there is really only one life form on earth -- it's DNA and cells packed with protein machines everywhere you look, the rest is just details, specialization. We share about half our genes with those trees, and more than half with the squirrels and deer. Is that not extraordinary enough? That cedar tree is my kin. That moth too. How we all got this way is an interesting story, but I don't see the transcending of life anywhere in it.

If we are capable of extraordinary things other plants and animals aren't, science and art but also weapons of mass destruction and chattel slavery, it's because life is capable of those things, and we just happen to be the specific form of life realizing those possibilities. We are apes that wear clothes. We have Mozart, but we also kill each other for made-up reasons. The "rational animal," sure we are.
Wayfarer July 02, 2023 at 04:33 #819417
Quoting Srap Tasmaner
Where does the transcending biology come in? Is being a living thing not extraordinary enough?


I feel there's a distinction here that you're not seeing. But then, that's been the case all through this thread. Thanks all the same for your responses.
Srap Tasmaner July 02, 2023 at 04:35 #819418
Quoting Wayfarer
I feel there's a distinction here that you're not seeing.


Oh I'm pretty sure I know what you think you see, I just think it's not there.
Srap Tasmaner July 02, 2023 at 04:44 #819421
Reply to Wayfarer

Did you know that humpback whales -- I think I'm remembering this right -- fuck with orcas?

It's a curious thing. When orcas are trying to kill seals and such, sometimes a couple whales will intervene. They will even scoop up a seal by rolling over and diving under it so that they surface with the seal on their belly, where the orcas can't get to it. There's even a case of whales remaining in the area to protect the dead body of a seal the orcas killed, just to keep them from actually eating it.

At first the theory was that they look enough like whale calves that it's kind of a mistake. But it turns out they will also interrupt the killing of sea turtles, of just about anything. They go out of their way to fuck with orcas.

The current theory is that it's more or less revenge. Orcas do attack whale calves, and so the whales have a clear sense of who the enemy is, and they side against them, for no other reason, with nothing to gain from it. Two whales swim in and call other whales from miles away, and they'll all swim in a ring around the seals or whatever keeping the orcas at bay, and they'll do this for hours, were still doing it at dusk when the researchers observing them headed back to shore.
Wayfarer July 02, 2023 at 05:10 #819428
Quoting Srap Tasmaner
Did you know that humpback whales -- I think I'm remembering this right -- fuck with orcas?


Gee, I must have missed that. I'd better go back and read the Argument from Reason again.
Srap Tasmaner July 02, 2023 at 05:11 #819429
Reply to Wayfarer You do that.
wonderer1 July 02, 2023 at 06:03 #819436
Quoting Srap Tasmaner
We share about half our genes with those trees, and more than half with the squirrels and deer. Is that not extraordinary enough?


:up:
Gnomon July 02, 2023 at 16:24 #819527
Quoting Janus
I don't know what others had in mind, but I was responding just to your posting of the Einstein quote. There are those who think the metaphysicalist imagination should be unfettered by science, by physicalism, and I don't think Einstein was one of them. That is all my response was concerned with.

Yes. But Reply to Wayfarer is not one of "those" preferring "unfettered" imagination. The negative reactions to Wayfarer's OP seem to be falsely accusing him of making unsubstantiated scientific (physical) assertions, while ignoring his explicit framing of the topic in terms of philosophical (metaphysical) concepts. He was not arguing against Evolution or Biology, but against the axiomatic (unprovable) metaphysical beliefs of Materialism*1. :smile:

PS___I quoted pragmatist Thomas Dewey above : " if the materialist begins with the assumption that mind and the molar forms of matter are constructed ultimately from molecular blocks of matter, he must end "with the conclusion that the ultimate form of matter has dualistic 'mind' and 'matter' properties . . . . If a materialist were to say that this double-sided substance is what he means by matter, we could only reply that he is playing with words--that it is just as much mind as it is matter." The problem is that doctrinaire Materialists seem to omit Mind (the observer) from their metaphysical assumptions. However, Aristotle spoke of just such a mind/body dualism in his concept of Hylomorphism : matter + mind (nous ; form) = natural bodies. So the Monism of Materialism is missing an essential ingredient to explain the emergence of abstract thought (i.e. Reason) from physical evolution. :nerd:


*1. Materialism as a belief system :

Materialism asserts that everything is or can be explained in relation to matter.
https://philosophynow.org/issues/42/What_is_Materialism

Materialism is a form of philosophical monism which holds that matter is the fundamental substance in nature, and that all things, including mental states and consciousness, are results of material interactions of material things.
https://en.wikipedia.org/wiki/Materialism

Materialism holds that the only thing that can be truly proven to exist is matter. Thus, according to Materialism, all things are composed of material and all phenomena are the result of material interactions, with no accounting of spirit or consciousness.
https://www.philosophybasics.com/branch_materialism.html
javra July 02, 2023 at 17:42 #819553
Reply to Srap Tasmaner

Here in reference to the OP’s basic contention:

Do you uphold that the basic principles/laws of thought which Aristotle brought to light are emergent properties of ever-reducible matter that, thereby, can then change with changes in the ordering of their underlying constituents of matter?

This would be in line with materialism: that laws of thought are the mutable constructs of purely material constituents. But it would also logically entail that all principles of logic/reasoning are, when ontologically addressed, a relativistic free for all—this relativity existing in relation to the order of underlying material constituents from which these principles of thought emerge—a relativism that, again, is thereby devoid of any impartial, existentially fixed standards (in the form of principles or laws) by which all variants of logic/reasoning manifest. This then being a conclusion that I take to be at the very least an unnerving suggestion; one could express it as inevitably leading into a nihilism or reasoning—i.e., in respect to logic as an authoritative means of discerning what is real or true and what is not.

The alternative doesn’t then necessitate that biology, genotypical evolution, and a concurrent phenotypical evolution of behavior (with the generality of “behavior” including that of (reason-adherent) cognition) are in any way bogus. (I, for one, uphold that lesser animal can and do utilize reasoning to various lesser degrees). Instead, it would only necessitate that the materialist paradigm (including its just aforementioned entailment of relativity in relation to laws of thought) is mistaken as a metaphysical outlook.
Srap Tasmaner July 02, 2023 at 18:39 #819566
Quoting javra
logic/reasoning


Short answer is that I wouldn't write these with a slash between them. Logic is a system of relations among propositions; reasoning is something people do, and they can do it well ("logically") or poorly ("illogically").

@apokrisis would have me say that even logic is just habitual, patterns of inference that have proved their worth, but he's got a whole metaphysics that makes that the natural move, and I'm not there yet.

So I don't think insisting that reasoning is something living creatures do requires me to reduce logic itself to biology.
javra July 02, 2023 at 18:46 #819568
Quoting Srap Tasmaner
Short answer is that I wouldn't write these with a slash between them. Logic is a system of relations among propositions; reasoning is something people do, and they can do it well ("logically") or poorly ("illogically").


I've addressed both of them, with a slash for concision of expression, in that the occurrence of both are dependent on basic laws of thought. (edit: this irrespective of how either term might be interpreted, given both terms' multiple meanings)

So the dilemma I've presented remains: namely, either the soundness of materialism's position which inevitably results in a nihilism of both reasoning and logic (that, I'll add, can then be applied to materialism itself) or, else, the necessity that materialistic metaphysics is, in some way, erroneous.
javra July 02, 2023 at 19:05 #819570
Quoting Srap Tasmaner
apokrisis would have me say that even logic is just habitual, patterns of inference that have proved their worth, but he's got a whole metaphysics that makes that the natural move, and I'm not there yet.


BTW, while I can't speak for apokrisis's metaphysical outlook, Peirce would have it that laws of nature evolve as global habits exactly via globally fixed laws, else expressed, principles - hence, via metaphysically fixed global principles that supersede natural laws and which are not themselves emergent from the latter: Peirce has a trifold system to this effect and - something that apo so far has disallowed in my conversations with him - the principle of Agapism as ultimate cosmic goal. Such that neither of these are emergent habits of the cosmos but, instead, are the immutable global principles via which the habits of effete mind (i.e., of physicality and its natural laws) evolve.
Srap Tasmaner July 02, 2023 at 19:46 #819582
Quoting javra
all principles of logic/reasoning are, when ontologically addressed, a relativistic free for all—this relativity existing in relation to the order of underlying material constituents from which these principles of thought emerge—a relativism that, again, is thereby devoid of any impartial, existentially fixed standards (in the form of principles or laws) by which all variants of logic/reasoning manifest


It's a very good question, and I thank you for it.

As I read your response more closely (which I shouldn't be doing since I'm at work!), it seems the question cashes out like this: would logic be the same, and thus the rules of valid inference, even if nature were very different?

That's a very difficult hypothetical, but I am inclined to say no. I think we think the way we do, and find success thinking the way we do, because nature is the way it is. We do think of logic as being above natural law, as being prior to it, but in a universe that behaved very differently than this one, if there could even be creatures like us to speculate, insisting upon the logic that works in this universe would look foolish, and nothing like the high road to truth.

That's to say, what counts as logic for us presumes a universe in which that version of logic is reasonable, is successful, does tend to lead to truth.

So I'll put my chips on what seems to me a naturalist and pragmatist view, and find some way to fight off the threat of nihilism.
javra July 02, 2023 at 19:53 #819586
Quoting Srap Tasmaner
So I'll put my chips on what seems to me a naturalist and pragmatist view, and find some way to fight off the threat of nihilism.


:grin: :up: Fair enough.

And since you're at work, I won't push too much in this direction. All the same, even if what you say might be true in regard to alternative worlds, it would still remain a reality that we - here, in the world we inhabit - could only fathom any such alternative world only if it were to abide by the law of identity, and then other laws of thought that could be argued derivatives of this one.

At any rate, thanks for the compliment! (Re: it being a good question :razz: )
Srap Tasmaner July 02, 2023 at 20:06 #819589
Quoting javra
we - here, in the world we inhabit - could only fathom any such alternative world only if it were to abide by the law of identity, and then other laws of thought that could be argued derivatives of this one


Oh yeah, that's a mess. Hmmm.

If you have further thoughts, do post, and I'll try to give better responses later.
javra July 02, 2023 at 21:04 #819597
Quoting Srap Tasmaner
If you have further thoughts, do post, and I'll try to give better responses later.


Well, OK. Maybe this is nothing worthy of debate. All the same:

As a fallibilist, I'm as confident as one can be that no one can ever infallibly settle this (or any other) issue - namely, that of it being possible to have a world devoid of ontically fixed laws of thought (or, at the very least, some ontically fixed law(s) to the like). Laws that therefore don't emerge out of constituent stuff, whatever this "stuff" might be - but instead "just are" and thus govern what is.

So - in keeping with the pragmatism that I myself uphold - the previously mentioned issue regarding laws of thought can then only be satisfactorily resolved via (not infallible proof, but) optimal explanatory power. Then:

If no inconsistencies are found in the dilemma I've provided, the claim that materialism (at least as its currently understood by most, and as it was addressed in my first post today) is a sound metaphysics can only end up being a self-refuting logical, or rational, proposal - for it results in a nihilism in relation to logic and reason (and/or rationality) which then negates its own logical, or rational, validity as a metaphysics. In short, materialism does not present a self-consistent explanation for the way things are - and, due to this, lacks cogent explanatory power.

Pragmatically, the following holds: optimal explanatory power evidences the, always fallible, and always to be improved, truth of theories.

By this I then conclude on pragmatic grounds the following: materialism must then be an erroneous stance.

Hence, in sum: ontic reality could only validly be non-materialist (else here expressed, non-physicalist, this again as physicalism is typically understood).

------

Note that I'm not one to equate either pragmatism or, more importantly, naturalism to materialism. So I maintain that both the former can be viably held in a non-materialist (else, non-physicalist) world view.

But, as a maybe important caveat to this given for the sake of frankness: Yes, the latter specified world view can also - not validate, but - logically allow for the possibility of the supernal (a view that is contradicted by naturalism). This possibility, however, would clearly remain outside the purview of the empirical sciences ... which as study can only address that which is objectively manifest and so equally apparent to all (this in principle if not also in practice). So, just as it currently is, the possibility of the supernal would remain an empirically unverifiable belief at best (or worst?) consisting of unverifiable personal experiences ... or, otherwise, an impossible to infallibly validate disbelief, this in cases where naturalism is being maintained.

That said, personally, I couldn't give a hoot as to whether or not anything supernal (e.g., angels, deities, forest fairies, etc.) might ontically be, same with the possible occurrence of extraterrestrial aliens. What I deem most important is unbiased reasoning and, maybe more importantly than this, ethical conduct. Well, just saying.
Wayfarer July 02, 2023 at 22:35 #819609
Quoting Gnomon
Wayfarer's OP seem to be falsely accusing him of making unsubstantiated scientific (physical) assertions, while ignoring his explicit framing of the topic in terms of philosophical (metaphysical) concepts. He was not arguing against Evolution or Biology, but against the axiomatic (unprovable) metaphysical beliefs of Materialism


:100:

Quoting Srap Tasmaner
I think we think the way we do, and find success thinking the way we do, because nature is the way it is. We do think of logic as being above natural law, as being prior to it, but in a universe that behaved very differently than this one, if there could even be creatures like us to speculate, insisting upon the logic that works in this universe would look foolish, and nothing like the high road to truth.


Notice this rhetorical sleight-of-hand which re-frames necessary truths as contingent. This is often deployed by way of speculations about the ‘multiverse’. It relativises the issue by suggesting that logic is 'for us', again, something of our own manufacture. Even what were customarily considered necessary truths are only conventions, after all. To be so bold as to suggest the origins of 'natural law' in Neoplatonism would be 'metaphysical speculation' but to casually introduce the possibility of 'other universes' barely causes a ripple. Even though it is:

Quoting javra
a relativism that, again, is thereby devoid of any impartial, existentially fixed standards (in the form of principles or laws) by which all variants of logic/reasoning manifest.


Quoting javra
Peirce has a trifold system to this effect and - something that apo so far has disallowed in my conversations with him - the principle of Agapism as ultimate cosmic goal.


I’ve noticed that Apokrisis tends to acknowledge only those aspects of Peirce’s philosophy which are pragmatically useful for modelling semiotic relationships whilst often disavowing his broader idealism. As Thomas Nagel put it, 'Even without God, the idea of a natural sympathy between the deepest truths of nature and the deepest layers of the human mind, which can be exploited to allow gradual development of a truer and truer conception of reality, makes us more at home in the universe than is secularly comfortable'. I think that discomfort is often on display in these kinds of discussions.



javra July 02, 2023 at 22:57 #819615
Quoting Wayfarer
I’ve noticed that Apokrisis tends to acknowledge only those aspects of Peirce’s philosophy which are pragmatically useful for modelling semiotic relationships whilst often disavowing his broader idealism.


Yea, I very much noticed that too. :smile:

Quoting Wayfarer
Thomas Nagel put it, 'Even without God, the idea of a natural sympathy between the deepest truths of nature and the deepest layers of the human mind, which can be exploited to allow gradual development of a truer and truer conception of reality, makes us more at home in the universe than is secularly comfortable'.


I find this quote quite beautiful. :up:

Janus July 02, 2023 at 23:52 #819627
Reply to Srap Tasmaner Considering the history of whaling, it's a wonder they don't also fuck with humans.
Srap Tasmaner July 03, 2023 at 00:04 #819631
Quoting Wayfarer
Notice this rhetorical sleight-of-hand which re-frames necessary truths as contingent.


That implies intent to deceive or mislead, which I assure you was not present.

Look, I'm between the devil and the deep blue sea here. I've been on almost exactly the other side of this argument, right here on this forum, many times. I have defended philosophy against the encroachment of psychology. I had a long argument with @Metaphysician Undercover over the necessity of object permanence to counting!

I am not *comfortable* allowing logic itself to be something like a fact of our universe -- maybe it is something more like a necessity for any universe, or at least for any intelligible universe.

The reasons for my decision here are several: I have never found an account of the status of logic or mathematics I like, never, and it comes up over and over again; I'm not sure we have much to show for defending our turf against psychology, which seems to have been making more than a little progress without our help; something in @javra's phrasing really crystallized the choice for me, a heaven of eternal logic versus naturalism; and finally I've been reading William James, whose approach to pragmatism really does feel informed by his work in physiology and psychology. James was famously open to the supernatural, to religion and spirituality, even to the paranormal, so his pragmatism is not a matter of dogmatic anti-supernaturalism, but his starting point is always life.

So I think maybe I'm ready to give up the idea of necessary truths. But maybe I'm not, we'll see. Quine waffled on this very issue for decades, with a set of commitments and inclinations similar to mine. It's hard.
Janus July 03, 2023 at 00:06 #819633
Quoting Wayfarer
Notice this rhetorical sleight-of-hand which re-frames necessary truths as contingent. This is often deployed by way of speculations about the ‘multiverse’. It relativises the issue by suggesting that logic is 'for us', again, something of our own manufacture.


How do you know that some truths are necessary? How do you know that logic is not "something of our own manufacture"?

Quoting Wayfarer
I’ve noticed that Apokrisis tends to acknowledge only those aspects of Peirce’s philosophy which are pragmatically useful for modelling semiotic relationships whilst often disavowing his broader idealism. As Thomas Nagel put it, 'Even without God, the idea of a natural sympathy between the deepest truths of nature and the deepest layers of the human mind, which can be exploited to allow gradual development of a truer and truer conception of reality, makes us more at home in the universe than is secularly comfortable'. I think that discomfort is often on display in these kinds of discussions.


Is that an argument from authority? Are you suggesting that if Peirce was right about semiotics, that he must be right about idealism and/or God? Why would the human mind, being a part of nature not be in accordance with the nature it, according to you, constructs, or according to others is partially affected by, and partially constructs? And why should that be "uncomfortable" for a secular thinker? It's a kind of ad hominem move to attempt to dismiss secular conceptions of nature by explaining them away as being "blind spots" due to being unable to cope with "transcendence" or some such "discomforting'" idea. What you don't seem to get is that not everyone is convinced by the arguments you are, even though they understand those arguments perfectly well enough.

And what are you and Nagel actually even arguing for, if neither of yout identify as theists?
Wayfarer July 03, 2023 at 00:10 #819635
Quoting Srap Tasmaner
That implies intent to deceive or mislead, which I assure you was not present.


Not accusing you of that at all. Overall your criticism has been relevant...well, up until the excursion into marine biology. It's the zeitgeist, the spirit of the times. A related topic I often explore is the vexed question of mathematical platonism - whether numbers are discovered or invented. The connection between the two questions ought to be pretty clear. The point being that the heavy-hitters in philosophy of maths all decry any form of platonism, on the grounds that it verges on a spooky ability to grasp non-physical truths.

Quoting Janus
How do you know that some truths are necessary? How do you know that logic is not "something of our own manufacture"?


Quoting Janus
The child can always endlessly ask 'why?'


javra July 03, 2023 at 00:45 #819645
Quoting Srap Tasmaner
something in javra's phrasing really crystallized the choice for me, a heaven of eternal logic versus naturalism


Ha! But then on what grounds would - needless to add, insentient - global constraints on what is and can be (the Heraclitean logos, so to speak, since it would occur in the so called heavens just as much as in the bowels of the earth and in everything in-between) then be validly defined as unnatural? Else expressed, as "non-naturalistic"?

This "heaven of eternal logic versus naturalism" (if I interpret the expression right) so far seems to me a false choice, since to me the two are not mutually exclusive by necessity. Such Heraclitean logos of sorts, if in fact existent, then being part and parcel of nature - the very same by which naturalism is defined.

If I'm missing out on something - other than the implicit status quo stance of materialism/physicalism - I'd like to be made aware of it.
Metaphysician Undercover July 03, 2023 at 00:49 #819647
Quoting Wayfarer
The point being that the heavy-hitters in philosophy of maths all decry any form of platonism, on the grounds that it verges on a spooky ability to grasp non-physical truths.


Perhaps, but those same "heavy-hitters in the philosophy of maths" accept without questioning, the axioms (basic set theory for example), which represent numbers as immutable objects. The philosophy of math is full of hypocrisy.
Srap Tasmaner July 03, 2023 at 02:05 #819662
Quoting Janus
Considering the history of whaling, it's a wonder they don't also fuck with humans.


Remember Crocodile Dundee and the kangaroo shooting back at the hunters? Love that.

Quoting javra
global constraints on what is and can be


I'm not sure we can reach quite that far. There may be a halfway point, a sort of anthropic principle -- if that's the right word for this kind of selection bias -- that I gestured at above, what sort of universe could be intelligible to creatures somewhat like us.

We're pretty far afield here, but I want to mention another way of approaching this, instead of pondering the status of possible constraints on the physics of a possible universe.

We have good reason to believe infants acquire the concept of object permanence before the concept of object identity. Think about that for a moment. That means it is possible for a creature to live in a world in which, so far as they can tell, ducks sometimes turn into trucks, but they never simply disappear. What's going on there? At some point -- I'm not sure how old -- they would no longer accept the possibility of such a transformation, but it appears there is a time when they do. Can they reason yet? Hard to say, but they express surprise when there's no object where they expect one, so the predictive machinery is certainly running already, it just doesn't need object identity to get going.

In effect, small infants live in a different world from us, with different or perhaps only fewer laws of thought. They transition to ours, mostly. Are they discovering more about how our universe works, about how any universe must? Maybe. Are they making richer and more rewarding predictions about their environment? Certainly. But they did live in that other world first, as we all did.

Is that world devoid of reason because the law of identity is absent? Maybe. Must we say so? What would be the point of saying so? There is a class of predictions the infant does not make that we do, and they are the poorer for it, presumably. We can say that, from our position, having been successfully relying on the law of identity; we know, that is, what they're missing out on.

But do we know what we are missing out on? Is it impossible that there are other laws of thought of whose operation we remain ignorant? That to some alien race we might appear like infants unable to conceptualize the simplest facts about our universe?

Still working at your last few posts, @javra. Might help me make sense of them if you compared your use of the terms "materialism" and "naturalism". (I've never been very comfortable arguing the merits of isms, hence my reliance on whales and infants and play-writing hominids.)
javra July 03, 2023 at 02:50 #819666
Quoting Srap Tasmaner
Can they reason yet? Hard to say, but they express surprise when there's no object where they expect one, so the predictive machinery is certainly running already, it just doesn't need object identity to get going.


Object identity is not an identical property to that of what the law of identity stipulates. I'd like to rephrase the latter (for better clarity) in this way: every X that we can be in any way aware of - be it an abstract given, a concrete given, a percept, or anything else - can only be X, this at the same time and in the same respect. This whether or not one is consciously aware of this property of thought (quite arguably as most, if not all, philosophers prior to Aristotle were consciously ignorant of this property - to give just one example). As far as we can fathom, lesser animals, though not consciously aware of this property of thought as a concept, can only think via its occurrence (e.g., unless one considers them to be automatons, a predator thinks, aka reasons - forethought included - in terms of how to best capture prey that is evading and holds the potential to injure the predator, either singularly or by working cooperatively as a team - obviously this without use of language; apes however give the best known examples, including of holding "eureka moments" that occur after strict contemplation in respect to a problem. I know its arguable to philosophers, but I'll uphold this.)

So understood, the law of identity's operation (its praxis, not its theory) is required for an understanding of what an object's identity is, but does not necessitate the latter. The latter is a complex concept and, thus, abstraction regarding what is. So, for those lifeforms that don't recognize the object identity of relevant objects instinctively (unthinkingly, this via genetic inheritance), it only makes sense that the aptitude is learned.

All this, I know, can be debated. But is by no means contradictory to either reason or empirical evidence. Back to the quote above: the "predictive machinery" which is surprised at there not being an object where an object is expected could not function in the absence of the law of identity as praxis. For then there would not be any "X" to be expected in the awareness of the infant.

What's relevant to a law of thought's occurrence is not our conceptual grasp of it as such but that it ontically occurs. It is only in this manner that laws of thought can be discovered - rather then invented - by us.

Quoting Srap Tasmaner
Might help me make sense of them if you compared your use of the terms "materialism" and "naturalism". (I've never been very comfortable arguing the merits of isms, hence my reliance on whales and infants and play-writing hominids.)


In brief, to my mind, materialism specifies that everything that is or can be is fully reducible to, and emergent from, insentient material constituents. Naturalism, on the other hand, specifies that all which does and can occur is that which is natural - thereby nature at large - this in contrast to that which is deemed to not be natural (again, for example, angels, deities, forest fairies, etc.). Materialism mandates naturalism of a certain kind, but naturalism is not limited to the possibility of materialism.



Janus July 03, 2023 at 03:00 #819669
Quoting Wayfarer
How do you know that some truths are necessary? How do you know that logic is not "something of our own manufacture"?
— Janus

The child can always endlessly ask 'why?'
— Janus


Sure, but you are yet to even begin an argument for necessary truths as far as I can tell. In any case if you did present an argument, I would address it on its own terms, not try to evade the issue by asking further 'why' questions. (Note: the only necessary truths as far as I can tell are tautologies, things true by definition).

And, as usual, you have no answers for the questions I asked...way to engage!
Janus July 03, 2023 at 03:18 #819673
Quoting Srap Tasmaner
Remember Crocodile Dundee and the kangaroo shooting back at the hunters? Love that.


:up: Sure do...me too!
Srap Tasmaner July 03, 2023 at 03:26 #819675
Quoting javra
What's relevant to a law of thought's occurrence is not our conceptual grasp of it as such but that it ontically occurs. It is only in this manner that laws of thought can be discovered - rather then invented - by us.


I thought that might be what you're saying. That makes such a law a fact about the universe (if I understand "ontic occurrence" as you intended). There are two questions that naturally arise:

(1) What is the real difference between such a law and other natural laws, such as the laws of thermodynamics?

(2) How can we tell whether such a law happens to hold in our universe, or whether it must hold? What would make it necessary, and how could we know?

(Okay the second one's two questions. My bad.)

Quoting javra
Naturalism, on the other hand, specifies that all which does and can occur is that which is natural - thereby nature at large - this in contrast to that which is deemed to not be natural (again, for example, angels, deities, forest fairies, etc.).


Huh. For discussion I'll go with it -- especially since all I mean by "naturalism" is, roughly, "amenable to scientific investigation," and that's not much of a definition either. My ersatz definition is essentially an exclusion of magic, behavior that is inherently unlawlike and thus incomprehensible to science. Your version of naturalism countenances immaterial entities so long as -- what exactly? They are not traditionally identified as supernatural?
apokrisis July 03, 2023 at 03:41 #819677
Quoting Srap Tasmaner
I am not *comfortable* allowing logic itself to be something like a fact of our universe -- maybe it is something more like a necessity for any universe, or at least for any intelligible universe.


It ought to help to strip "logic" down to its ultimate simplicities. We do grant it too much psychological status, even though we don't then want to eliminate it as the thread of "cosmic intelligibility" that runs through life and mind too.

The metaphysics of the systems view goes back to Anaximander's apeiron and the general Greek enthusiasm for understanding nature as a dialectic, a unity of opposites.

What it boils down to is the logical principle that whatever doesn't self-contradict is free to be the case. Peirce encoded this in his dichotomy of tychism~synechism – freedoms and the habits of constraint that must emerge once everything starts happening and discovers how that mass of interacting results in its own restricting limits.

If you have to argue against any principle, this would seem the hardest to refute.

Something popping out of nothing doesn't compute. There is no logic in that. But somethingness being whatever is left after a great clash of clashing contrarieties does compute. It is an undeniable conclusion. Everythingness is its own filter as all that is simply symmetric will eliminate itself, leaving whatever is uncancellably asymmetric as a possibility.

If you step one metre left and one metre right, nothing has effectively changed. But if you step one metre left and it is over the edge of a cliff, now you have an event that can't so easily be cancelled out.

This is exactly the logic of the path integral that extracts a concrete world from the probabilities of particle fields. The calculation takes all possible particle events and then discovers the degree to which all the many options cancel each other out.

The vast weight of the possibilities are symmetric – virtual particle pairs that create and annihilate without leaving any trace. But this self-winnowing eventually leaves some "collapsed" actual particle event making its mark on the world.

So both in our earliest metaphysics and our best current physical models, the same deep logical trick is at work. Something emerges out of a self-cancelling sum over everything. We have a foundational truth that we can rely on. Or at least pragmatism tells us no other principle has better survived the test.

I should add that dichotomies encode asymmetries, or hierarchical order.

If everything simply self-cancelled, then that would indeed leave nothing. So what can survive all the cancellation is the dichotomous order that gives reality two complementary directions in which to be forever moving apart from itself.

Again, that is the metaphysics of the quantum view. The Universe is eternally cooling and spreading – spreading because it cools, and cooling because it spreads.

In any instant, from the level of individual quantum vacuum fluctuations, the world has grown both larger and cooler. This in itself is enough to promote some of the self-cancelling virtual pairs to long-term reality.

At the event horizon of a black hole or the edge of the visible universe, you have even the briefest-lived self-annihilating pairs being separated for just long enough to find themselves existing in different lightcones or world lines.

Back during the hot dense fury of the Big Bang, inflation itself was separating virtual fluctuations with enough vigour to keep even a lot of very short-lived particles going. There was a lot of crud to spill into a rapidly cooled void and reheat it with the matter we are familiar with as the lucky survivors.

So the objection to maths and logic is largely to do with the way these fields have wandered off as their own research subjects, remote from the concerns of natural philosophy - the tradition that connects ancient metaphysics to modern physics.

Maths is hell of an arbitrary exercise in the freedoms from physical reality that it grants itself. The logic choppers likewise have strayed from the constraints of pragmatics.

But what we mean by an intelligible cosmos is in fact so simple in terms of its logic and maths that this isn't a great problem. The Darwinian principle of cosmic self-selection tells us somethingness is the product of a symmetry-breaking so rigorous that it left behind only uncancellable asymmetry. The dichotomy that results in the hierarchy.

In the end, the cooling~expanding Universe will end in its Heat Death. All the crud will get broken down into the faintest rustle of a quantum vacuum and exported across cosmic horizons. It will still be something of course, but as near to absolute nothingness as we can intelligibly conjecture.






Srap Tasmaner July 03, 2023 at 04:08 #819687
Quoting apokrisis
What it boils down to is the logical principle that whatever doesn't self-contradict is free to be the case.


If I'm following this, one point is that logic (at least in these sorts of discussions) is often conceived primarily as a constraint, contradiction is forbidden, non-identity is forbidden, and so on, but that's clearly not the whole story because you need some generative principle as well. (Hence tychism?)

But much of what you write is about how constraints themselves are generated, rather than simply being given, and this is where symmetry breaking comes in, yes?

It would certainly be more satisfying to have a story in which a single process gives rise to the constraints on its continued operation. Without such a story, you in effect imagine the universe to exist within a bigger universe in which there are already certain rules in place -- the rules of universe creation, these laws of thought -- and you simply decline to explain that one. You would face a similar problem if anything simpler and more general than your story were conceivable -- but you knew that going in and have aimed at maximal simplicity and generality.

Do I have any of that right?
javra July 03, 2023 at 04:40 #819700
Quoting Srap Tasmaner
That makes such a law a fact about the universe (if I understand "ontic occurrence" as you intended).


Yes, its an accurate interpretation of my view.

Quoting Srap Tasmaner
(1) What is the real difference between such a law and other natural laws, such as the laws of thermodynamics?

(2) How can we tell whether such a law happens to hold in our universe, or whether it must hold? What would make it necessary, and how could we know?


As apokrisis illustrates, these questions will be contingent on one's - hopefully self-consistent and explanatory power endowed - metaphysical worldview. (I'm myself still working on expressing the worldview I have in mind. It's a very long ways from being finalized, but it's what the website in my profile points to - this to be transparent and not seem to be posturing. Rather than trying to engage in detailed explanations of my, sometimes unorthodox, metaphysical perspectives in a forum format, I'll just point to that website and leave it at that.)

As to (1), I follow Peirce's view and hold that the former is existentially fixed while the later is in a perpetual process of (very gradual) development - such that the latter depends on the former, but while I take both to be fully natural.

As to (2), as I've previously expressed, the optimal explanatory power that will result from such a worldview would in this case evidence (always fallibly) that the law must and does hold in [s]our[/s] the universe (or in the cosmos, if one prefers).

Whatever the correct, reality-conformant answer to these questions might in fact be, however, in relation to this thread, the pivotal issue remains: how can materialism and physicalism uphold their own rational validity when their rational validity is (for reasons so far discussed) undermined by the very metaphysical stance they maintain?

Quoting Srap Tasmaner
Your version of naturalism countenances immaterial entities so long as -- what exactly? They are not traditionally identified as supernatural?


Can you better explain what you mean by "immaterial entities" in this context? Would these include things like numbers, natural laws, and teloi (e.g., intentions)? Or do you by the term intend things like angels, deities, and forest fairies? As to the latter, they would again not be consistent with what I take naturalism to be - this due to not being explicable in terms of the physical and what the physical essentially entails. However, as to the former, tmk they are not amenable to scientific investigation - by which I understand the empirical sciences' use of the scientific method - but are nevertheless aspects of reality writ large upon which all at least modern scientific investigation depends. And, personally, I do take the former set to consist of natural, though immaterial, givens - hence to quality as natural aspects of the nature which naturalism demarcates.

apokrisis July 03, 2023 at 04:45 #819702
Quoting Srap Tasmaner
(Hence tychism?)


Or better yet – if Peirce had completed his logic of vagueness – he was defining tychism more clearly as that to which the PNC doesn't apply (and generality or synechism as that to which the LEM doesn't apply, so framing the metaphysics more clearly in terms of the familiar laws of thought).

But yes, free generation. Just like a quantum vacuum.

Quoting Srap Tasmaner
But much of what you write is about how constraints themselves are generated, rather than simply being given, and this is where symmetry breaking comes in, yes?


Constraints develop or evolve. They are the habits that survive – in that they are not disturbed by mere passing fluctuations any longer.

So at the level of pure tychism, fluctuations are symmetry breaking in equilibrium with symmetry-restoration. Virtual particles are defined by their creation~annihilation operators that don't give them long enough to get established as real particles.

But once there is a spacetime context that is larger in than the Planck scale, brief time, then now there is a filter on this hot and cancelling action. Some kinds of symmetry breaking start to stick. In particular, you get the electroweak phase transition where local fermion gauge breaking gets entangle with global goldstone boson symmetry breaking – the Higgs field that gives particles an effective mass and traps them in longer-lived states.

Whole species of anti-particles get wiped out leaving the particles nothing to annihilate with any longer. The CP-symmetry breaking story.

So all this is just routine particle physics. You can get global constraints kicking in as fast as the spacetime metric grows and so allows interactions on that larger collective scale. A Darwinian filter sorts everything out down to protons, electrons and neutrinos.


Srap Tasmaner July 03, 2023 at 04:54 #819704
Quoting javra
Can you better explain what you mean by "immaterial entities" in this context?


Just a placeholder for "not reducible to matter", since that's the other thing. I think I've got it now with the distinction between numbers and angels, although I do wonder why the problem with angels is that they're not physical and the problem with numbers is nothing at all.

Quoting javra
how can materialism and physicalism uphold their own rational validity when their rational validity is (for reasons so far discussed) undermined by the very metaphysical stance they maintain?


And naturalism gets around this, on your view, by countenancing laws of thought as "natural, though immaterial, givens"; that is, you get to rely on logical inference and the materialist does not. Is that your position? I mean, that seems like cheating, like the Russell line about "the advantages of the method of positing."

Whatever I end up thinking naturalism amounts to, I don't think that'll be it.
javra July 03, 2023 at 05:19 #819711
Quoting Srap Tasmaner
although I do wonder why the problem with angels is that they're not physical and the problem with numbers is nothing at all.


Yea, one way to tersely address this is that the cogency of scientific knowledge is in many ways contingent on the occurrence of numbers but couldn't care less about the occurrence or non-occurrence of angels. Numbers - more accurately, quantity - is something the occurrence of a physical reality essentially entails (otherwise one would have a quantity-devoid, partless, etc. reality - which is not what the physical presents itself to be). Angels, on the other hand, are not essentially entailed by the physical reality we all share.

Quoting Srap Tasmaner
And naturalism gets around this, on your view, by countenancing laws of thought as "natural, though immaterial, givens," that is, you get to rely on logical inference and the materialist does not. Is that your position?


No, it is not. My position - which I think I've repeated one too many times already in this thread - is that the logical inferences of materialists when it comes to their metaphysics result in the conclusion that all logical inferences are relative - such that one might as well declare that "to each their own equally valid logic and reasoning". In contrast, maybe there is some impartial, nonfabricated set of rules, laws, or principles that we all must adhere to when it comes to logic and reasoning if we are to pragmatically survive - immaterial thought these might be - such that 2 + 2 does not equal 5 no matter how much one tries to make it happen.

apokrisis July 03, 2023 at 05:20 #819712
Quoting Srap Tasmaner
It would certainly be more satisfying to have a story in which a single process gives rise to the constraints on its continued operation. Without such a story, you in effect imagine the universe to exist within a bigger universe in which there are already certain rules in place -- the rules of universe creation, these laws of thought -- and you simply decline to explain that one. You would face a similar problem if anything simpler and more general than your story were conceivable -- but you knew that going in and have aimed at maximal simplicity and generality.


Oh, on this, we start with the need to explain at least one world - ours.

And to the degree that a tale of immanence and self-creation is achieved, that does serve to rule out a more pluralistic metaphysics.

For example, some use the quantum collapse issue to argue for an infinite multiverse. But a Darwinian self filtering mechanism - like quantum decoherence - can then close the story in effective fashion. You can prune away the modal argument that all possibilities exist. Only the possible universe exists as it is the one making all the other universes impossible.

There are still problems of course. To include vagueness in a concrete fashion is a delicate operation if vagueness is suppose to be an ultimate lack of the concrete.

So I might talk about the Apeiron, quantum vacuum, Ungrund, vagueness, or other concrete attempts to frame this notion of unlimited and formless potential. A sea of pure fluctuation … that pre-exists … any existence.

But again, we start knowing that there is indeed a something with intelligible structure. There is a world. We can start in the middle of things, as Peirce urged, and work out to the edges.



Srap Tasmaner July 03, 2023 at 05:37 #819714
Quoting apokrisis
We can start in the middle of things


Hey that part I understand and, for what it's worth, agree.

Quoting javra
the logical inferences of materialists when it comes to their metaphysics result in the conclusion that all logical inferences are relative - such that one might as well declare that "to each their own equally valid logic and reasoning".


Wait, really? I thought the relativism at stake was to the ordering of nature, but you meant relative to the individual reasoner as a bit of ordered matter? Is materialism really committed to that sort of simplistic perspectivism? Why cannot materialism call on the laws known to hold in this material universe, logical and natural, and leave it at that? This universe is logical, and so logical inference is appropriate here -- no eternity needed. --- Or is the materialist unable, in your view, to recognize that, say, the law of non-contradiction holds in our world?

I'm missing something. Apologize if I'm just misreading you.
javra July 03, 2023 at 06:14 #819719
Reply to Srap Tasmaner

I’m currently tired, so my bad if I’m reading you wrong. You reply as though I’m pushing you into buying something and you’re not yet prepared to buy it – but I have nothing to sell. I’ve simply presented my fallible perspective. I’ve by now expressed my views repeatedly, and at this point find the prospect of doing so once more to likely not be beneficial for anyone. So I’ll now ask you in turn for your own perspective:

Do you find that the basic laws of thought are fixed for everyone today, yesterday, and tomorrow?

If so, on what coherent materialism-based grounds do you so conclude?

If not, on what coherent grounds do you find that reasoning and logic can serve as means for discerning what is real?
Srap Tasmaner July 03, 2023 at 15:08 #819766
Quoting javra
You reply as though I’m pushing you into buying something and you’re not yet prepared to buy it


Sorry if it came across that way. I just don't understand, that's all.

Quoting javra
So I’ll now ask you in turn for your own perspective


Yikes. My views are in flux, even more than usual at the moment.

Quoting javra
Do you find that the basic laws of thought are fixed for everyone today, yesterday, and tomorrow?


As far as I can tell, this means the world has a logical structure, so any belief that violates the usual canons of logic cannot be true.

I'm going to lean "no". I think the picture presented here is of a static world modeled after the medium-sized dry goods we are used to interacting with. But the universe wasn't always like this, which means insofar as we see order around us it is not eternal but temporary -- it had to emerge and it will go away.

What's more, our intuitions about things at our scale don't translate well to the much larger or the much smaller. Space itself bends -- what the hell? And I don't have a clue what's going on at quantum scales, but the stuff @Andrew M has tried to explain to me does not match how my keys and my breakfast cereal and my books behave.

So I'm inclined to think these laws of thought -- phrase I really detest -- are an approximation, in almost exactly the way that Newtonian mechanics is an approximation. It's only an approximation and even that only applies because of where and when we live.

That's not to deny its utility at all, but its utility is the point, hence my leaning harder all the time toward pragmatism.

Quoting apokrisis
The twisty journey that all must take from lumpen realism, to the body shock of idealism, to the eventual resolution of enactivism and pragmatism.


That's how I read Hume, and it would be true of me except I've never found idealism appealing so there are things about the predictive modeling view that wouldn't bother an idealist but freak me out a bit, as I'm getting it all at once.

Quoting javra
If not, on what coherent grounds do you find that reasoning and logic can serve as means for discerning what is real?


As above, because that's all they're for, at least as predictive approximations.

You seem to accept the thrust of the argument from reason, that if materialism is true then there really isn't anything you'd call logic. I think there's no real argument presented at all, but there is something like a conflict of definitions. This discussion has at least forced me to consider which side of the fence I want to be on, and if I have to give up the eternal truths of logic to stick with natural science, then so be it.

I've taken the opportunity to choose, but I don't think it's actually forced by the argument. Denying that the universe comes with a built-in logical structure we discover does not require us to deny that systems we have ourselves constructed can have the logical structure we give them. To say otherwise looks like a simple genetic fallacy to me. The logic I "find" in the world is an approximation I make; the logic in a mathematical proof or a computer program or a game of chess is fact, because we made it so. Is there some other sense in which the logic we imbue these artifacts with is eternal and unchanging? If so, it's something different from what we've been talking about.
javra July 03, 2023 at 20:38 #819816
Quoting Srap Tasmaner
Is there some other sense in which the logic we imbue these artifacts with is eternal and unchanging? If so, it's something different from what we've been talking about.


Yes to the first; no to the second.

That said, thanks for the informative reply. What you say of logic makes sense, but then, it also unintentionally equivocates what is at issue – this at least in terms of the dilemma I’ve repeatedly presented.

It hence equivocats between logic proper and that which it necessarily depends upon: the basic laws of thought, of which I exemplified the most fundamental of these: the law of identity.

Logic – here presuming we’re addressing formal logic and not informal logic – is largely axiomatically founded. With most of these axioms being creations which we for whatever reason (a best inference, a shot in the dark, and so forth) thereby fabricate and then build logical systems upon. It is because of this that different systems of (formal) logic can be obtained (consistent, paraconsistent, etc.). In comparison, informal logic is simply the process of solving problems in linear, step-by-step manners of thought – irrespective of how this might come about – and so is for the most part equivalent to rationality and reasoning.

Either type, however, would not be able to be engaged in in any manner whatsoever if not for our utterly fundamental manner of being aware – this in respect to anything, including of empirical givens (e.g., the sky’s color), of introspective givens (e.g., a felt emotion), of proprioceptive givens (a felt pain), and so forth. Fully innate to all aspects of our awareness is something that is so utterly basic, so utterly necessary, that quite a few might be at a loss for why it should be presented as a law that we follow to begin with. Namely: Anything (e.g., an entity, a process, a quality, a property, etc.; else, a set of any of the aforementioned or mixture of these; and so forth) one is aware of at time t can only be at time t the very same given one is aware of – this in comparison to being both itself and something other at the same time and in the same respect. E.g., when one (consciously) strictly focuses on X at time t one does not – cannot - then strictly focus (consciously) on Y at time t (needless to add, this in the same respect).

This is (of if one prefers, appears to be) a property of all awareness – hence, derivatively, of all possible conscious thought (one cannot think without being aware of one’s thought(s)). It is not something that some person decided one day to concoct as an axiom thinking that, upon its being so axiomatized, it might then be put to use as foundation for further reasoning by him/herself and others. It is not something that can be invented by some awareness that, at some initial time of its creation, lacked the very ability addressed.

It is then termed a law – not because some individual or cohort decided to make it a law for others to follow – but because it is deemed to be an immutable governing factor of all possible awareness (and, hence, of all possible thought).

Becoming, if you’d like, meta-aware of this innate aspect of all awareness is a discovery of what is and has always been – but an awareness does not need to be endowed with this meta-awareness in order to be aware and, thus, to be governed by this, so traditionally termed, law of identity.

I know of those who doubt the reality of the LNC. I have yet to learn of any philosopher or non-philosopher who in any way doubts the law of identity as just specified.

-----

Hopefully I’ve addressed the law of identity more clearly this time around than I did when I was replying to the issue of infants’ lack of recognition of object identity (which, again, is a complex concept which we likely learned as infants via trial and error and which we adults entertain habitually in mostly unthinking manners – but which could not be obtained in the absence of that property of awareness just specified, i.e. in the absence of the law of identity).

If so, then the question should be addressed not in terms of logic but, again, in terms of the basic laws of thought upon which all forms of logic are contingent. To further simplify things, I'll here strictly address the law of identity (as it was just described):

Can – or, better asked, does – this utterly innate law of identity change over time?

If so, on what grounds can we consider any aspect of our current thought process to arrive at any existential truth – such as that of materialism (for example)?

If not, how can materialism account for this utterly immutable governing principle of all awareness?
apokrisis July 03, 2023 at 22:12 #819825
Quoting Srap Tasmaner
The logic I "find" in the world is an approximation I make;


I think it helps to see we are pursuing a model of causality when we talk about the logic, structure, rationality or intelligibility of nature. Otherwise you are getting too hung up on the human mathematical practice and it’s semiotic peculiarities. You get caught out by the fact that semiosis is itself a symbolic and mechanical way of regulating the material world, and the material world then has its own organic logic - that of a self-organising dissipative structure.

So we have to be able to step back and see the whole of a modelling relation in which the model and the world have their own causalities, their own logics. And what would then universalise them as aspects of the one “logical” cosmos is the further step of understanding how the mechanical and the organic are the two poles of a meta-dichotomy. They can mutually exist as “other” to each other.

So the usual monistic and foundationalist instinct is to reduce our models of cosmic logic or cosmic cause to a single central truth. But semiosis and systems science explain why this quest proves so hard. It is because causality/logic is irreducibly complex in the triadic way that systems thinking describes.

The approach to the goal of a unity of opposites has to begin with the grant that anything might be the case. The “ground” of being is a pure material potential/pure logical vagueness. An everythingness of unstructured fluctuation.

We can know this is the most reasonable starting point or initial conditions as we can wind back from the developed state of the Universe today. We can see that the Universe is the most fundamental expression of an organic causality in that it is the grand self-organising dissipative structure that constrains all else due to the laws of thermodynamics and symmetry breaking.

And we can also see that one of the possibilities that exists in this general dissipative state of order is life and mind - the evolution of an informational causality, a localised symbolic and mechanical regulation that cuts across the Universe’s global dissipative project.

Modelling can control entropy flows for its own purposes as it has codes to create memories that stand outside the world of ordinary self-organising dynamics. It can manufacture networks of switches - like enzymes, neurons, utterances, logic gates - that switch material flows off and on with dichotomous certainty.

So we can say nature just self organises under the logic/causality of dissipative structure. But that itself is a statement we are then making about “the real world” from the cosy mechanistic confines of our modelling relation with that world.

And therein lies the problem. Or at least the knot of complexity which makes people’s heads explode with its self-referential Goedelian incompleteness. We can only seem to map the territory. We realise we are applying a mechanical perspective to what it itself now claimed as being the “other” of the organic. This doesn’t compute. The regress appears infinite. We are spat back out of our metaphysics into dualistic confusion.

Yet this need not be the case. If we can understand the world vs model relation in terms of an actual natural dichotomy - one that unites the organic and mechanical as complementary and reciprocal aspects of being - then we can formulate a triadic vantage point that sees both sides to the one more complex whole.

That is why I champion biosemiosis. It goes beyond pragmatism to speak to the scientific “how” of how negentropic organisms exist by mechanically ratcheting the entropic gradients of a dissipating cosmos.

In a nutshell, the material world has physical dimension. It costs time, space and energy to cause things to happen. And the way to then stand outside this world so as to model and regulate it is to thus lack the constraints of being dimensioned. To systematically reduce that dimensional involvement between model and world until it becomes the zero cost presence of an information bit or Euclidean 0D point.

The sly trick of life and mind, with their memories and informational order, is that they don’t actually have to dematerialise themselves to be literally dimensionless ghosts or whatever. As processes, they just have to be able to encode the world using a symbolic machinery that involves a small and constant entropic cost per unit of information captured and stored.

The brain burns nearly a quarter of our calories. But we can afford that because we use our brains to feed our mouths. And while feeding our mouths has been top of mind for most of history, the brain is a flexible enough bit of hardware that it could also learn to run verbal and mathematical systems of semiosis on top of its evolved genetic and neural ones. Homo sapiens could ascend the ladder of world models to ever greater dimensionless abstraction and so aspire to regulate the material dimensional world from some ever less materially constrained - and hence more purely informational - place.

So again, our big metaphysical question here is what is the fundamental model of the causality/logic of the Universe? And this quest is already irreducibly complex because we are mechanical systems modelling an organic world in the mechanical way that looks to contradict everything about that world.

This seems an irresolvable problem. But it is also the larger semiotic inevitability. Organic order - the evolution of a Universe structure by the logic of dissipation - had to create the possibility of its own mechanical regulation to the degree it became crisply dimensional and thus gave causal reality to whatever could then conceal its dimensionality - ie: the further realm of 0D symbols. Or at least semiotic systems which could afford information storage as a very small and constant cost.

A word is a puff of air. But we can make it stand for anything in terms of some unit of socially meaningful information. An enzyme is a chain of amino acids, but the body can toss one into the biochemical fray and regulate the direction of any metabolic reaction. A neuron is an abrupt act of depolarisation, but it can stack up levels and levels of Bayesian predictive routines. A number can be encoded in the 1s and 0s of a logic gate, but the resulting mathematical structure encoded in a pattern of electrically-powered switching can function as a useful model in any way a human cares to imagine.

The brute materiality of the Cosmos is the ground of its own antithesis in terms of bringing to life - being the cause - of the mechanical symbol processing that transcends the Comos’s strictly dimensional existence.

Yet look closely enough - understand semiosis as physical process with a small but constant cost per unit - and you can reunite the model with its world. We see that life and mind are part of the Cosmic fabric as all they really do is aid its dissipation by applying Bayesian modelling to the creation of paths that locally accelerate the global diktat of the Second Law.

We speak of causality when we talk of material structure. We speak of logic when we talk about informational structure. And both those things are different, yet also fundamentally connected. In formal terms, we would get closest to the metaphysical truth by being able frame them as two halves of the one larger dichotomy. And to do that, we have to dig away at the third thing of the way the two halves indeed connect. The story of the switches and ratchets which are the Janus interface bridging the epistemic machines with their ontologised environments.

Or if we switch from this synchronic perspective to the “other” of the developmental/evolutionary diachronic perspective, then we have to dig away at the third thing of the way the two halves could co-arise from some shared initial state or a vagueness … which is neither dimensional, nor dimensionless, but now “other” to dimensionality in general. In other words, the Apeiron - the everythingness that is neither yet a something, nor yet distinguishable from a nothing, but “exists” as a logical ground for any resulting structured material being.

This is certainly a twisty saga. But it is the systems metaphysics Anaximander first articulated, Peirce formalised, and which science is cashing out in twin pronged fashion as it comes to model the Universe as a self-organising dissipative structure and life/mind as itself the machinery of models that can further ratchet this generalised cosmic flow.

Wayfarer July 03, 2023 at 22:49 #819828
Quoting Srap Tasmaner
In effect, small infants live in a different world from us, with different or perhaps only fewer laws of thought. They transition to ours, mostly.


And generally learn to speak as a part of that. Something which is also unique to h. sapiens.

Quoting javra
Object identity is not an identical property to that of what the law of identity stipulates.


The law of identity itself originates with Greek philosophy with Parmenides assertion that "what is, is," and "what is not, is not", suggesting that something is what it is and cannot be what it is not. Later, in the Phaedo, there is the 'argument from equals' where Socrates argues that, in order to ascertain that two objects have the same length, we must already have the 'idea of equals' in order to make the judgement as to sameness and difference.

My belief is that our every rational act is suffused with such judgements of sameness and difference, is/is not, equals/unequal. And because it structures our cognition, these are also inherent in reality as experienced by us.

Quoting apokrisis
So again, our big metaphysical question is what is the fundamental model of the causality/logic of the Universe?


The lesson of philosophy since Kant is that we can't see the Universe from some point outside our own cognitive apparatus - that the world and the subject are inextricably intertwined. The world and the self have distinctive existences within that matrix but the world remains an experience-of-the-world. Within that matrix logic and mathematics are formal structures which are replicable and inter-subjectively verifiable, but I say it's a mistake to think of them as being 'in the mind'. They're neither 'in here' nor 'out there' but structures within the experience-of-the-world. Reason provides the means to abstract, predict, and generalise, which is why Greek philosophy held reason in such esteem as it provided insight into the [s]causal[/s] formal domain, whereas sensory consciousness only reveals the phenomenal domain.
javra July 03, 2023 at 23:20 #819835
Quoting Wayfarer
My belief is that our every rational act is suffused with such judgements of sameness and difference, is/is not, equals/unequal. And because it structures our cognition, these are also inherent in reality as experienced by us.


I'm in agreement. Could try to splurge on the idea a little, but am thinking this would only muddle matters.

However, in relation to what has been so far discussed by me, on one hand there is the issue of whether the law of identity is immutable for all awareness and, thus, for all consequent thought. Then, on the other hand, there is the issue of whether physical reality in fact does conform to this same principle: that what is X at time t cannot in the same respect be non-X at time t - not just epistemically but also onticaly.

Since we're all quite familiar with introductory notions of quantum mechanics, this issue, for instance, can then apply to the wave-particle duality of quantum particles: Is a particle a particle at time t when its so measured to be? Is this so called "measurement" only a best inference - rather than an immediate percept of what is - that can thereby be mistaken (due to mistaken reasoning) in the identity of what is being measured - such that what's measured is neither strictly particle nor wave, but something different? Or is it, in physical reality, both a strict particle and a strict wave simultaneously and in the same respect - contradictory though this is? But the latter can then signify that a chair can ontically be a swimming pool at the same time and in the same way - and so so much for reasoning.

For those of us who don't know how to "shut up and just do the math", how one address the law of identity as an existential given - one that might be applicable to physical reality at large as well as all our awareness - will directly impact one's possible choices in perspective regarding this issue of particle-wave duality just mentioned.

I don't have a ready answer for this issue up my sleeve - just my convictions. And this subject is probably a distraction for the thread's primary theme. All the same, it's interesting stuff to me - ontologically speaking.

------

Edit: BTW, thanks for the head's up as to the historic background to the law of identity. I misspoke in that previous post you quoted from: the principle was known of prior to Aristotle.
Janus July 04, 2023 at 00:41 #819845
Quoting javra
Numbers - more accurately, quantity - is something the occurrence of a physical reality essentially entails (otherwise one would have a quantity-devoid, partless, etc. reality - which is not what the physical presents itself to be).


"[i] NUMBER IS DIFFERENT FROM QUANTITY
This difference is basic for any sort of theorizing behavioral science, for any sort of imagining of what
goes on between organisms or inside organisms as part of their processes of thought.
Numbers are the product of counting. Quantities are the product of measurement. This means that
numbers can conceivably be accurate because there is a discontinuity between each integer and the next.
Between two and three, there is a jump. In the case of quantity, there is no such jump; and because jump is missing in the world of quantity, it is impossible for any quantity to be exact. You can have exactly three tomatoes. You can never have exactly three gallons of water. Always quantity is approximate.
Even when number and quantity are clearly discriminated, there is another concept that must be
recognized and distinguished from both number and quantity. For this other concept, there is, I think, no
English word, so we have to be content with remembering that there is a subset of patterns whose
members are commonly called "numbers." Not all numbers are the products of counting. Indeed, it is the
smaller, and therefore commoner, numbers that are often not counted but recognized as patterns at a
single glance. Card-players do not stop to count the pips in the eight of spades and can even recognize the characteristic patterning of pips up to "ten."
In other words, number is of the world of pattern, gestalt, and digital computation; quantity is of the
world of analogic and probabilistic computation.[/i]"

Gregory Bateson Mind and Nature- A necessary Unity
javra July 04, 2023 at 00:56 #819849
Quoting Janus
Between two and three, there is a jump. In the case of quantity, there is no such jump; and because jump is missing in the world of quantity, it is impossible for any quantity to be exact. You can have exactly three tomatoes. You can never have exactly three gallons of water.


Alright, so you're saying (via your quote) that tomatoes are not quantifiable?

As to the "exactness" of what a gallon consists of, this applies to all measurements in general, doesn't it? As in "exactly 1 yard or meter" doesn't quite exist in every day physical reality - other than good enough approximations - but only occurs as an abstracted concept ... much the same as a (perfect) circle doesn't occur in physical reality. Take the standard meter by which all measuring devices (say, a tape measure) are measured and built, place this very standard on any flat surface: which molecules pertain to the meter and which don't? And then there's smaller constituents of the physical. Its exactness doesn't occur in physicality.

Till shown otherwise, I'm calling BS on the quote's contents. One can't have numbers (such that they mean anything) in the absence of quantity, I still say.
Janus July 04, 2023 at 01:32 #819862
Quoting javra
Alright, so you're saying (via your quote) that tomatoes are not quantifiable?


Sure, language is ambiguous, and we can say that there is a certain quantity of tomatoes or a certain number of tomatoes. But the quantity of tomatoes, although it can be given as a number, could also be given as a weight or a volume. Of course, these weights or volumes are also expressed in numbers, but Bateson's point is that number is discrete, whereas quantity, insofar as a distinction can be made between it and number, is not.

So, we can have exactly three tomatoes, but we cannot have exactly three kilograms or cubic centimeters of tomatoes.
apokrisis July 04, 2023 at 01:52 #819869
Quoting Wayfarer
The lesson of philosophy since Kant is that we can't see the Universe from some point outside our own cognitive apparatus - that the world and the subject are inextricably intertwined.


So where are you standing when you see that embodied modelling relation?

Pragmatism/semiosis is about going the next step of discovering the extremes of a disembodied view by starting with the extremes of the embodied view.

This is why Peirce began with phenomenology. He tried to excavate the Firstness of qualia of “mere flashes of feeling and sensation” as they are before being complexified by language and logic.

Then he flipped that to show how that then looks from the other perspective of maximally disembodied material reality. Hence his “objective idealism”. Which might as well be called his “subjective realism”. The two are just reciprocal in being organised by dialectical inquiry.

So one thing can always be described or measured in terms of its dichotomous other. Instead of the equals sign - which would describe their dynamical balance, you just need to have an inversion sign (a reciprocal equation) that defines the same dynamical balance in terms of its complementary limits.

Gray = gray. But black = 1/white. And vice versa.

The same game allows us start where we first find ourselves - usually in a grey fog of vagueness or uncertainty - and figure out that there are these psychological limits in terms of self and world, first and third person views, embodied and disembodied cognition.

Kant started a ball rolling. But he was not the last word.

Quoting Wayfarer
They're neither 'in here' nor 'out there' but structures within the experience-of-the-world.


So everything is “just models”. Except then there is the further thing of not all models being equally rigorous.

You suggest a dichotomy of formal-phenomenal. That is the right start in terms of logical rigour as it posits qualities that mutually quantify.

But I think you can see how it bakes in category errors. It looks to mix up epistemological and ontological qualities. Or if you are really wanting to talk about Platonic ideas, then you are straying into the pitfalls of Cartesian substance dualism.

I see some sense to the cut you want to make. But I don’t think it is a sufficiently clean cut.

If you hit the dichotomy between its eyes, you would then also have a metric by which formal thought and bare consciousness could be measured. The qualities would be quantified by their status as the polar extremes or a reciprocally causal relation.

In my view, you are talking about mindfulness at the level of neurobiology vs sociocultural construction. But I argued that these are all hierarchical levels of semiosis done with increasingly formal abstraction and material range.

Types of “consciousness” - or the Bayesian modelling relation - are being stacked up with broadening scope and greater organismic being.

Semiotics sorts out the foundational basics of what is going on causally. Now we are talking about the specifics of grades of semiosis and how they are indeed developing along a natural arc towards the limits of symbolic abstraction.

Forget naked feels and atomistic qualia. They aren’t evidence of anything definite or rigorous, despite what phenomenologist would like to claim. Or at least, rigorous phenomenology discovers holistic states of sensory, motor and affective integration when it zeroes in on “what it is like to experience redness”. Qualia are reductionist constructs. Holism discovers gestalts when it indulges in drilling down to psychological Firstness.

So yep, there is always the dichotomy, the dialectic, to make our ideas clear enough to be measured. And neurobiology has its system of measurement in the way we respond bodily to a “stimulus”. Social construction had its own measurements ranging from the rather informal constraints of “does this conform to traditional convention” to fully formalised acts like reading the numbers off a dial.

There are levels of semiosis that express a sort of dichotomy in developing along the path from concretely embodied to abstractly disembodied. But genes are not “other” to numbers. Biosemiosis is about how information breaks entropic symmetries.

Organisms exist by giving nature a direction. And broadly this is always metabolic. Even philosophers have to eat. But then we have gone on to create metabolisms at the level of agrarian societies and industrialised economies.

Homo sapiens invented articulate speech and kicked the collective metabolism up a gear. The Ancient Greeks took the Pythagorean turn of seeing maths could be closed by proofs and so opened the door to the giddiness of Platonic idealism. But what then actually made maths and logic matter in human lives? It wasn’t the discovery of a new realm of being. It was just the practicalities of turning a shit-load of fossil fuel into population growth that wasn’t just exponential but hyperbolic. The rate of increase was increasing up until about 1990.

Pragmatism says sure, maths and logic seem to speak to maximal disembodiment. Silicon Valley believes it to be the future reality. The Metaverse and the Singularity. But those wet dreams of computer science forget that humans are still just doing metabolism. The only thing really changing is the scale.

This is the anthropocene. The population is topping out at 10 billion hungry mouths and ecology says we way overshot the actual carrying capacity. Our economic models that got us here were all wrong as they did not factor in the other side of the equation - the environmental capital that is being converted into social capital. Etc.

So again, a clear view of the reality is critical. Maths may be unreasonably effective in describing nature - from the much simplified reductionist point of view. But the holistic use of maths in modelling is a whole lot tougher. We need robust systems modelling to rejig our metabolic systems to continue any further as civilised beings who can also live within ecological limits.

All this is to say that slicing across the complexity with the right dichotomies has become mission critical. Ways of thought appropriate to agrarian societies are in the past. Ways of thought constructing the world as it is today are about to hit the wall.

That is why I promote the combo of dissipative structure and semiotic regulation. Or what some call ecological economics. A practical philosophy of how to be an organism with all these stacked up levels of semiosis and cranked up future expectations.

Platonism is fun with ideas. My point is that in some sense the formal does transcend the materiality of being. But human systems of thought are then closely tied in practice to achieving basic needs. And for the majority, a bucket of KFC is the ultimate Platonic good. (Well, it has pragmatically become the quickest way to get protesting inmates down off the roofs around these parts.)

So judging by what society does rather than what philosophers might call “the truth”, the realm of civilised concern remains the metabolic reality of giving the entropy of chemical reactions a usefully constructive direction. And reductionist maths is unreasonably effective when pointed towards that end.

javra July 04, 2023 at 02:17 #819876
Quoting Janus
So, we can have exactly three tomatoes, but we cannot have exactly three kilograms or cubic centimeters of tomatoes.


I of course accept this, but so far fail to see its significance.

Rather than focusing on the absolute exactitude of things in the physical world (which I would grant does not exist, all of it being in flux and such - tomatoes very much included), I'd instead focus on the discreetness of physical givens as discerned by awareness. Something which, as an indefinite amount of something, we commonly term quantity in the English language. Which we then use numbers to more precisely quantify in definite manners.

I don't know the background of the guy you've quoted. Is the guy trying to conceive of what reality is like, or would be like, in the complete absence of all awareness in the cosmos? As one avenue of enquiry into this: under the a materialist's reductionist microscope where everything material (i.e., everything) is reducible to the quantum vacuum, in the absence of awareness discreteness would be hard to specify, if at all present. Sure. But then in a world devoid of all awareness so too would numbers not be present.

The exactitude of numbers has everything to do with awareness's aptitudes - especially here addressing that of humans - the very same awareness which discerns discreteness, and hence quantity, in the physical world (to not mention in is own thoughts, in its own emotions, in it own perceptions, etc.).

Take that meter I previously addressed: its composed of ever moving quantum parts; it, as a physical meter stick, has no absolute exactitude. And yet it is not simply an absolutely precise, abstract, free floating number; it instead is a discrete amount of something, a quantity - which we address in definite terms via use of numbers. Hence, "there is one meter stick there".

At any rate, in reference to what seems to be your disagreement with my stance that you've previously quoted:

Either via the idealism of Platonic Realism or the materialism of today's mainstream views, how can one have numbers in the complete absence of discrete amounts of givens - i.e., of quantities? (if nothing else, there would yet be a quantity of numbers by the shear presence of the number(s) addressed)

Again, in reference to what I initially said, the physical reality we know of entails the presence of quantity and, due to this, of number, (the later at least for us linguistic animals). (As an apropos, ravens and other animals are known to be able to count, obviously this without the use of language, via which numbers are specified).

Srap Tasmaner July 04, 2023 at 02:31 #819878
Quoting javra
I don't know the background of the guy you've quoted.


Gregory Bateson
Janus July 04, 2023 at 02:37 #819879
Quoting javra
I of course accept this, but so far fail to see its significance.


The significance, as I see it, is that thinking in terms of number is, along with identity, an artificially exact mode of thought. Also, I think it is good practice to draw conceptual distinctions wherever we find that we can; it helps to understand how we think.

Quoting javra
I'd instead focus on the discreetness of physical givens as discerned by awareness. Something which, as an indefinite amount of something, we commonly term quantity in the English language. Which we then use numbers to more precisely quantify in definite manners.


That something stands out perceptually is not that it is actually separate from its environment; it is just that we can distinguish it. That said observations of animals show that they often distinguish the same things as we do; notably items of food.

Our distinguishing things depends on our nature, and on the nature of what is distinguished. When we distinguish a single tomato, there is not an "indefinite quantity of something" in the sense that you were using the term 'quantity', that is as number: on the contrary there is an exact number of tomatoes; in this case one. We might even say that there is an exact mass and volume of tomato there, although we cannot say precisely what that is.

Quoting javra
I don't know the background of the guy you've quoted. Is the guy trying to conceive of what reality is like, or would be like, in the complete absence of all awareness in the cosmos?


No, Bateson was one of the seminal "information" thinkers; well worth a read.

Quoting javra
The exactitude of numbers has everything to do with awareness's aptitudes - especially here addressing that of humans -


Apropos of this, another quote from the same book:

[i]"Some birds can somehow distinguish number up to seven. But whether this is done by counting or by
pattern recognition is not known. The experiment that came closest to testing this difference between the
two methods was performed by Otto Koehler with a jackdaw. The bird was trained to the following
routine: A number of small cups with lids are set out. In these cups, small pieces of meat are placed.
Some cups have one piece of meat, some have two or three, and some cups have none. Separate from the cups, there is a plate on which there is a number of pieces of meat greater that the total number of pieces in the cups. The jackdaw learns to open each cup. Taking off the lid, and then eats any pieces of meat that are in the cup. Finally, when he has eaten all the meat in the cups, he may go to the plate and there eat the same number of pieces of meat that he got form the cups. The bird is punished if he eats more meat from the plate than was in the cups. This routine he is able to learn.

Now, the question is: is the jackdaw counting the pieces of meat, or is he using some alternative method
of identifying the number of pieces? The experiment has been carefully designed to push the bird toward
counting. His actions are interrupted by his having to lift the lids, and the sequence has been further
confused by having some cups contain more than one piece of meat and some contain none. By these
devices, the experimenter has tried to make it impossible for the jackdaw to create some sort of pattern or rhythm by which to recognize the number of pieces of meat. The bird is thus forced, so far as the
experimenter could force the matter, to count the pieces of meat.

It is still conceivable, of course, that the taking of the meat from the cups becomes some sort of rhythmic
dance and that this rhythm is in some way repeated when the bird takes the meat from the plate. The
matter is still conceivably in doubt, but on the whole, the experiment is rather convincing in favor of the
hypothesis that the jackdaw is counting the pieces of meat rather than recognizing a pattern either of
pieces or of his own actions.

It is interesting to look at the biological world in terms of this question: Should the various instances in
which number is exhibited by regarded as instances of gestalt, of counted number, or of mere quantity?
There is a rather conspicuous difference between, for example, the statement "This single rose has five
petals, and it has five sepals, and indeed its symmetry is of a pentad pattern" and the statement "This rose has one hundred and twelve stamens, and that other has ninety-seven, and this has only sixty-four." The process which controls the number of stamens is surely different from the process that controls the number of petals or sepals. And, interestingly, in the double rose, what seems to have happened is that some of the stamens have been converted into petals, so that the process for determining how many petals to make has now become, not the normal process delimiting petals to a pattern of five, but more like the process determining the quantity of stamens. We may say that petals are normally "five" in the single rose but that stamens are "many" where "many" is a quantity that will vary from one rose to another.

With this difference in mind, we can look at the biological world and ask what is the largest number that
the processes of growth can handle as a fixed pattern, beyond which the matter is handled as quantity. So far as I know, the "numbers" two, three, four, and five are the common ones in symmetry of plants and animals, particularly in radial symmetry."[/i]

javra July 04, 2023 at 03:13 #819886
Quoting Janus
That something stands out perceptually is not that it is actually separate from its environment; it is just that we can distinguish it.


I'm not getting this. Edit: A predator's perceived prey that stands out perceptually isn't separate from the prey's environment?

Quoting Janus
When we distinguish a single tomato, there is not an "indefinite quantity of something" in the sense that you were using the term 'quantity', that is as number: on the contrary there is an exact number of tomatoes; in this case one.


"A quantity" is an unspecified amount. "A number" is a specified quantity. Q: "What is the quantity of tomatoes you've purchased from the store?" A: "One." Conceptually, quantities consist of numbers - whether or not the latter are specified. To go back to Bateson's initial quote, what would a numberless measurement of length, for example, be?

As to Bateson's latest quote, interesting as it is to read, it only speculates without evidencing what is speculated.

Just found this on line:

https://www.quantamagazine.org/animals-can-count-and-use-zero-how-far-does-their-number-sense-go-20210809/

it starts:

An understanding of numbers is often viewed as a distinctly human faculty — a hallmark of our intelligence that, along with language, sets us apart from all other animals.

But that couldn’t be further from the truth. Honeybees count landmarks when navigating toward sources of nectar. Lionesses tally the number of roars they hear from an intruding pride before deciding whether to attack or retreat. Some ants keep track of their steps; some spiders keep track of how many prey are caught in their web. One species of frog bases its entire mating ritual on number: If a male calls out — a whining pew followed by a brief pulsing note called a chuck — his rival responds by placing two chucks at the end of his own call. The first frog then responds with three, the other with four, and so on up to around six, when they run out of breath.

Practically every animal that scientists have studied — insects and cephalopods, amphibians and reptiles, birds and mammals — can distinguish between different numbers of objects in a set or sounds in a sequence. They don’t just have a sense of “greater than” or “less than,” but an approximate sense of quantity: that two is distinct from three, that 15 is distinct from 20. This mental representation of set size, called numerosity, seems to be “a general ability,” and an ancient one, said Giorgio Vallortigara, a neuroscientist at the University of Trento in Italy.


And it gets better as you read the article.

But wait, what if all this is not counting but "pattern or rhythm recognition"? I'll skip on this debate.

Instead, you could reply to what I initially asked.

Quoting javra
Either via the idealism of Platonic Realism or the materialism of today's mainstream views, how can one have numbers in the complete absence of discrete amounts of givens - i.e., of quantities? (if nothing else, there would yet be a quantity of numbers by the shear presence of the number(s) addressed)


Janus July 04, 2023 at 03:23 #819887


Quoting javra
I'm not getting this. Edit: A predator's perceived prey that stands out perceptually isn't separate from the prey's environment?


I meant not separate in any ontological sense. The prey and the predator are parts of the environment that only stand out in the sense of being noticed.

Quoting javra
As to Bateson's latest quote, interesting as it is to read, it only speculates without evidencing what is speculated.


Speculation or conjecture, more or less informed, is all we have; what more do you expect?

Quoting javra
But wait, what if all this is not counting but "pattern or rhythm recognition"? I'll skip on this debate.


As Bateson says pattern recognition is possible with small numbers of things; beyond that we would need to count them to know how many there are.

Quoting javra
Conceptually, quantities consist of numbers - whether or not the latter are specified. To go back to Bateson's initial quote, what would a numberless measurement of length, for example, be?


Apart from counting, quamtities do not strictly consist in numbers, or at least not in number alone. A certain quanity of tomatoes will weigh a certain number of milligrams, a different number of grams, kilograms, ounces pounds, stones and so on. Same with volumes. So it is numbers plus units of measurement.

There can be no numberless measurement, but a length is a length, regardless of whether it is measured.

Quoting javra
Instead, you could reply to what I initially asked.

Either via the idealism of Platonic Realism or the materialism of today's mainstream views, how can one have numbers in the complete absence of discrete amounts of givens - i.e., of quantities? (if nothing else, there would yet be a quantity of numbers by the shear presence of the number(s) addressed)


I'm not sure what you are saying or asking there. I'll attempt an answer if you care to clarify.
javra July 04, 2023 at 03:24 #819888
Quoting Janus
I'm not sure what you are saying or asking there. I'll attempt an answer if you care to clarify.


That's OK. Thanks
Janus July 04, 2023 at 03:30 #819890
Reply to javra OK, suit yourself.
Count Timothy von Icarus July 04, 2023 at 16:30 #819947
Reply to Wayfarer

I meant to respond to this when the thread first came out because I am working on a different sort of argument from reason. I do not think this argument works with common, highly nominalist versions of reductive physicalism.

In most versions of physicalism, which tend to embrace the computational theory of mind (still seemingly the most popular theory in cognitive science), a belief is just an encoding of the state of the external environment. This encoding exists within a system that can be defined as an agent. Agents need not be conscious, they simply need goals and a set of possible behaviors to decide from when attempting to actualize those goals. Decisions on how to act given some goal x and some set of beliefs y can be described in computational terms. This "set of beliefs," is represented as a database of atomic propositions, a "knowledge base," and the behavior selection process can be described well enough through backwards chaining searches on the knowledge base.

Under this view, a belief is true just in case the representation of the enviornment of which the agent is a part (the world) corresponds to the actual enviornment. It's that simple. The claim that "no belief is rationally inferred if it can be fully explained in terms of nonrational causes," is simply the result of a misinterpretation of what "rational," should be taken to mean. The world is rational because it obeys a set of rules that govern how it progresses from state to state which can be fully described mathematically, perhaps even fully described computationally. Future states are deducible from current ones. All beliefs are thus the product of a rationally describable set of steps, essentially a "program running on a quantum computer," as it is often put. States in a program are all logical consequences of prior states, so "rational." Assuming "rational," to mean "the result of an agents beliefs," as Lewis does is arguably begging the question.


The encodings inside the physical system making up the agent don't have to fully describe the world as it is to be true. Indeed, organisms cannot encode all the data they are exposed to without succumbing to entropy (Terrance Deacon's "Towards a Science of Biosemiotics," has a good explanation of this). This means that a true belief is just a representation of the world that is in some ways isomorphic to actual states of affairs in the world. A belief can be consistent with many states of affairs and is true just in case it corresponds to the actual state of affairs.

Thinking through computable toy universe examples, there doesn't seem to be any reason why a toy universe can't contain a subsystem that instantiates the logical computations that (allegedly) result in the creation of agents (and conscious agents at that). Parts of the universe simply interact such that the agent subsystem comes to represent a compressed, partial description of the universe within itself. These descriptions are the knowledge base, which is what it uses to compute ways of achieving its goals (goals which generally include maintaining homeostasis and reproduction, with these goals being explained by reference to natural selection).

Beliefs then are just other names for physical subsystems within a physical agent, e.g. patterns of neuronal activity. Such beliefs are created due to physical causal mechanisms. Belief, the verb, is just the description of what the enumeration of these physical belief subsystems "feels like" to a conscious agent.

Knowledge then is justified true beliefs. Beliefs are justified if the methods employed by the agent to vet their beliefs have proven themselves to be successful in the past (inductive support) and if any deduction used in vetting/creating these beliefs is sound. Since AI can already build proofs, I don't think there should be too much argument that causal processes can be used to cross check soundness.

This view works regardless of how consciousness arises, or even if it is eliminated, because agents are not defined in terms of possessing first person perspective, but rather through having goals. First person experience is thought to be totally described, somehow, by physical casual processes anyhow, so adding it won't change anything.

Given this view, I don't see how this argument works at all. Lewis seems to conflate the proposition that "the universe and causal forces are [I]meaningless[/I]," as in, "devoid of moral or ethical value and describing nothing outside themselves," with agent's beliefs necessarily also being "meaningless," as in "the beliefs must not actually be in reference to anything else." The first sense of the term "meaningless" is not the same as the second. A toy universe with just a bunch of floating balls and a Pac Man that tries to eat them can be meaningless in the first sense while the Pac Man could have meaningful representations of the locations of the balls encoded within itself provided there is some medium for interaction (e.g. light waves bouncing around to hit the Pac Man's eyes).

Basically, a lack of external reference does not imply a lack of internal reference.
180 Proof July 04, 2023 at 17:26 #819951
Reply to Gnomon FYI: One of the pioneers of digital philosophy (re: pancomputationalism/digital physics) died a couple of weeks ago, Edward Fredkin. If you are not familiar with him, here's a wiki article with a summary of view on the fundamental nature of information ...

https://en.m.wikipedia.org/wiki/Digital_physics#Pancomputationalism

I became aware of Fredkin through references to him in the writings / interviews of
David Deutsch,
Seth Lloyd,
Max Tegmark,
Stephen Wolfram,
Richard Feynman,
John Wheeler,
Frank Tipler,
Eric Drexler,
Douglas Hofstadter,
Nick Bostrom et al.
In the history of Western philosophy, speculations as divergent as Peirce's semiosis-tychism (pragmaticist), Leibniz's monadology (rationalist) and Democritus-Epicurus' atomism (materialist) are the closest analogues to digital philosophy I've yet found.

I'm not convinced (it does not seem to me to follow), however, 'that if physical events-regularities are computable (which they are), then physical reality must be a "computer" executing a nonphysical program (and, in your case, Gnomon, that's written by a "nonphysical programmer")' – at best, this hasty generalization is too unparsimonious and the pseudo-speculative equivalent of (neo-Aristotlean / neo-Thomistic / neo-Hegelian) "intelligent design". :eyes:

@Wayfarer @universeness @apokrisis @Janus
Count Timothy von Icarus July 04, 2023 at 17:52 #819959
Reply to Metaphysician Undercover
I feel like Platonism is so heavily ingrained in mathematics that even those trying to run from it can find themselves simply lapsing into it from another direction. Part of this has to do with specialization in academia IMO. If, being a scholar focused on mathematical foundations, you're not generally going to be able to do a lot of work or teaching on other fields, the fields where a Platonist would say numbers are instantiated, then your field, by definition, places you in a silo where your experience of mathematics is necessarily "floating free of the world."

Rather than a set of immutable numbers, which seems less defensible today, we can have a set of possible, contextually immutable axioms, which define a vast, perhaps infinite space of systems. The truths in the systems are mutable, because there are different systems, but then there is a sort of fall back, second-order Platonism where the existence of the systems themselves, and relations between them, are immutable.


Reply to javra

To go back to Bateson's initial quote, what would a numberless measurement of length, for example, be?


Couldn't this be accomplished by simply referencing objects' extension in relation to one another? Indeed, this is how our measurement systems tend to work. We take an arbitrary phenomenon and use it as a base and describe other phenomena in terms of their relation to the base. I wouldn't agree that a ratio is essentially a number either, as a ratio is necessarily a comparison between things, be they discrete entities or parts of a whole.

But more to the point on animals having some ability to conceive of numbers, I'm not sure if that demonstrates too much in either direction. Human nature seems to produce a strong tendency to want to think of things in terms of discrete objects. We have some good reason to think this tendency is the result of evolution, since it causes a great deal of difficulty in trying to conceptualize how the world appears to actually work at very large or very small scales. That is, the discrete object view appears to work only at the scales relevant to evolution. It also makes it hard for us to conceive of continua, hence the endless appeal of the Eleatic Paradoxes. However, mathematics also shows us that this conception of numbers is much shakier than was originally thought. I feel like there is support for the supposition that the illusion of discreteness is just a useful survival trick as much as for the idea that innate numeracy denotes the existence of numbers "out there, sans mind."

how can one have numbers in the complete absence of discrete amounts of givens - i.e., of quantities?


Imagine a continuum, for example a line, of finite length. Our line has an uncountably infinite number of points but also a finite length. Take some section of the line, arbitrarily, and compare how many lengths of the section fit within the whole. There are sections of the line that exist such that the line can be broken into n segments of equal length, where n is a natural number. No initial discreteness required, right? All that is required is that the points of the line differ from each other in some way; then we can define this difference in reference to a given segment's length relative to the whole to produce numbers for a coordinate system.

I've always found the reverse argument more interesting, the claim that numbers are essential for reality, or at least our understanding of it.

We ought to have ontological commitment to all and only the entities that are indispensable to our best scientific theories.
Mathematical entities are indispensable to our best scientific theories.
Therefore, we ought to have ontological commitment to mathematical entities.


That's a brief summary of the Quinte-Putnam Indispensability Argument. In response to this, I know some folks actually have made some headway in describing areas of physics without reference to numbers, although it isn't exactly pragmatic to do so. Anyhow, if some hitherto unformulated version of logicalism is true, and numbers are reducible to logos, it seems to me like this argument is moot (and that the concept of logos spermatikos ends up beating out divine nous as a better explanation of "how things are," IMHO.)
Count Timothy von Icarus July 04, 2023 at 18:09 #819963
Reply to 180 Proof

Quite a few pancomputationalists also seem to embrace ontic structural realism, that the universe is the mathematical structure describing it, so I'm not even sure if it makes sense to talk about a physical computer and a non-physical program, as the distinction seems to collapse.

I think the whole concept of pancomputation suffers from the fact that computation itself is poorly defined. The most common explanations make reference to "what Turing Machines do," because that's the easiest way to describe computation, but then Turing Machines are themselves an attempt to define what human beings do when carrying out instructions to compute things. But then human consciousness is also explained in terms of computation, making the whole explanation somewhat circular.
180 Proof July 04, 2023 at 18:24 #819967
Reply to Count Timothy von Icarus Read Seth Lloyd, David Deutsch and Stephen Wolfram – 'computation' has been operationally defined quite rigorously for decades.
wonderer1 July 04, 2023 at 19:50 #819996
Quoting Count Timothy von Icarus
The most common explanations make reference to "what Turing Machines do," because that's the easiest way to describe computation, but then Turing Machines are themselves an attempt to define what human beings do when carrying out instructions to compute things. But then human consciousness is also explained in terms of computation, making the whole explanation somewhat circular.


Do you have a citation supporting "Turing Machines are themselves an attempt to define what human beings do"?

I've been under the impression that Turing conceived of the TM as a sort of minimalist CPU, and I wonder whether Turing himself would have thought of TMs as more than a conceptual tool for use in thinking about what human beings do, rather than an attempt at defining what human being's do.

Anyway, I think computationalism has a large element of looking for the keys under the street lamp because that's where the light is. Computation is a human invention that is as it is, because it is relatively easy for us to think in such terms.

Connectionism is much closer to where it's at when considering the way human thought really works. Perhaps it is harder for most people to think in connectionist terms though.
javra July 04, 2023 at 20:09 #820010
Reply to Count Timothy von Icarus Thanks for engaging with what I've previously asked!

Quoting Count Timothy von Icarus
To go back to Bateson's initial quote, what would a numberless measurement of length, for example, be? - javra

Couldn't this be accomplished by simply referencing objects' extension in relation to one another?


Yes, you are quite right. (Realized this after posting, but deemed that editing it would be a bit much ... in any case, my bad here.) I would however better reword the concern I have in this way:

-- Can one have any measurement that is devoid of any discrete givens which we - either thinkingly or unthinkingly - enumerate (i.e., determine the amount of) via numbers?

If not, then it currently seems to me that measurement necessitates number in some way or another.

Notice that in the very quoted sentence number (a definite amount; i.e., a definite quantity) is necessarily specified in order for cogent semantics to obtain (what I've boldfaced). Likewise, while a ratio might not be itself interpreted as a number (debatable) it will yet, I so far find, necessarily consist of a relation between numbers - at the very least between quantities (the plurality of which is itself a quantity) which we hold the potential to enumerate. Else, in measurements that strictly concern relations, such as greater than or lesser than, there will always be an at the very least implicitly addressed number of givens to which the relations applies. I'm for example weak on pure theoretical mathematics, but I so far can't find any exception to this.

Quoting Count Timothy von Icarus
I feel like there is support for the supposition that the illusion of discreteness is just a useful survival trick as much as for the idea that innate numeracy denotes the existence of numbers "out there, sans mind."


As to whether lesser animals can count, as philosophy it's right up there with whether lesser animals are in fact conscious - to which might as well be appended the issue of other minds. In short, I'm convinced that they do, but, as with those who'd disagree, can't provide conclusive philosophical evidence of it - at least not in a forum format. So, I won't debate the issue.

Still, the pivotal issue I was addressing is that, as I currently find it, discreteness is contingent on the occurrence of awareness - such that if awareness then discreteness (and as an important meta-example: the occurrence of one awareness or more will each be a discrete given). And, furthermore, that numbers are only then contingent on the occurrence of discreteness. This irrespective of one's metaphysical interpretation(s) regarding the consequent significance in respect to the cosmos we inhabit. (e.g., a materialists' view that an awareness-devoid cosmos is possible or, else, an idealist's view that such is impossible - as two among other metaphysical perspectives)

So I'm here in full agreement that "numbers 'out there, sans minds' [by which I here understand, tersely stated, "a plurality of discrete awareness"]'" can only be a fallacy.

Quoting Count Timothy von Icarus
how can one have numbers in the complete absence of discrete amounts of givens - i.e., of quantities? - javra

Imagine a continuum, for example a line, of finite length. Our line has an uncountably infinite number of points but also a finite length.


Right here, in the very semantics of what a line is, is the occurrence of quantity in the form of "points" - such that this quantity minimally consists of more than one point. In addressing "a" continuum one is likewise specifying a quantity - not two or three continuums but one. So the occurrence of quantity is a requite aspect of any continuum - be it real or strictly conceptual.

Quoting Count Timothy von Icarus
ake some section of the line, arbitrarily, and compare how many lengths of the section fit within the whole. There are sections of the line that exist such that the line can be broken into n segments of equal length, where n is a natural number. No initial discreteness required, right? All that is required is that the points of the line differ from each other in some way;


Maybe the "discreteness" here addressed by you has a specialized mathematical meaning? But in the ordinary sense I've addressed it it specifies something being separate, distinct, individual. Hence, in the sense I intend the very sections of the line that are compared are thereby discrete (to our awareness of them as such - otherwise no comparison could be made).

BTW, one could then address point-free topology as another example to be provided - but, here too, tmk there will be discerned some form of separateness somewhere (e.g., sets), such that discreteness (and hence quantity) yet obtains.

So, I again find that the (maybe I should specify, cosmic) occurrence of quantity is requisite for the occurrence of numbers (be the latter's occurrence also cosmic or, else, strictly located in individual minds as some would have it).

Quoting Count Timothy von Icarus
I've always found the reverse argument more interesting, the claim that numbers are essential for reality, or at least our understanding of it.


This in fact isn't too different from my own personal metaphysical views. Only that I maintain quantity - as in "discrete givens" - to be essential to existence rather than to reality (with "existence" here roughly understood as all which "stands out" in any way) and, thereby, to physical reality (which exists); this, thereby, concurrently necessitating the ontic occurence of numbers in the cosmos (however sentience might represent them symbolically; e.g. as "IV", as "4", or as "four"). But I don't want to digress into my own metaphysical views concerning this.

All the same, of main interest here is the issue of how numbers could be had in the complete absence of quantity.

Quoting Count Timothy von Icarus
Anyhow, if some hitherto unformulated version of logicalism is true, and numbers are reducible to logos, it seems to me like this argument is moot (and that the concept of logos spermatikos ends up beating out divine nous as a better explanation of "how things are," IMHO.)


:grin: I could go with that. (but then these touchy terms hold different connotations to different people, for instance, that of "spermatikos" say by compassion to the terms "in-fluence" or even that of "inspiration (aka, to breath in or, more archaically, roughly, to be breathed into psychically)") But yes, a Heraclitean-like, cosmic logos of the type addressed stands in direct logical contradiction to an omnipotent and omniscient creator deity whose "words" make up the world.
apokrisis July 04, 2023 at 20:48 #820027
Quoting 180 Proof
I'm not convinced


Yeah, digital physics fails at the gate for me even as an epistemology, let alone an ontolotgy.

As Wiki notes:
Extant models of digital physics are incompatible with the existence of several continuous characters of physical symmetries,[7] e.g., rotational symmetry, translational symmetry, Lorentz symmetry, and the Lie group gauge invariance of Yang–Mills theories, all central to current physical theory.


Informational atomism has to be able to handle the dichotomy of the discrete-continuous. It must make a world that exists as a dynamical balance between these metaphysical limits. Every bit must be both locally separated and yet globally connected. The openness of local degrees of freedom must be closed by global constraints. Etc.

So even constructing a map of the territory is an issue according to this no go argument - https://arxiv.org/pdf/1109.1963.pdf

And when it comes to taking literally the claim that “reality is a computer program”, you have to scratch your head at how it can in any sense run without material hardware or a handy power socket.

Digital physics was a weird one. The Planck triad of constants does surely tells us something deep about the fundamental grain of the Universe. But that would be that nature is triadically structured (is irreducibly complex) in systems fashion and arises out of the dichotomisation or symmetry breaking that can oppose spacetime extent to spacetime content.

https://thephilosophyforum.com/discussion/comment/586530

It is relations rather than computations that are going to be metaphysically fundamental.


180 Proof July 04, 2023 at 21:47 #820065
Quoting apokrisis
And when it comes to taking literally the claim that “reality is a computer program”, you have to scratch your head at how it can in any sense run without material hardware or a handy power socket.

:up:
Gnomon July 04, 2023 at 22:15 #820077
[quote="Wayfarer;819609"] Reply to 180 Proof sarcastically & superciliously ridiculed your & my spooky immaterial opinions in this thread about general Reason instead of particular Things. Specifically, he poo-poos my information-based posts postulating something like a data-processing-universe theory.

From his reply : "I'm not convinced (it does not seem to me to follow), however, 'that if physical events-regularities are computable (which they are), then physical reality must be a "computer" executing a nonphysical program (and, in your case, Gnomon, that's written by a "nonphysical programmer")' – at best, this hasty generalization is too unparsimonious and the pseudo-speculative equivalent of (neo-Aristotlean / neo-Thomistic / neo-Hegelian) "intelligent design". Apparently he connects my personal theory with those of several famous mathematicians & physicists. So, it looks like I'm in good company. FWIW, since I avoid engaging in put-down polemics, here's my response indirectly addressed to 180 :

My only knowledge of Ed Fredkin came in Robert Wright's 1988 book : Three Scientists And Their Gods, Looking For Meaning in an Age of Information. Wright said, "He posits not only laws but a law enforcement agency : a computer. Somewhere out there, he believes, is a machinelike thing that actually keeps our our individual bits of space abiding by the rule of the universal cellular automaton, With this belief Fredkin crosses the line between physics and metaphysics".

Wright later asks, "where is this computer that Fredkin keeps talking about". Fellow physicist and Information theorist, Rolf Landauer noted the flaw in Fredkin's theory, infinite regress : "It's turtles all the way down". Wright goes on to ask, "if matter is made of information, what is the information made of?" Apparently Fredkin's god-like cosmic computer is just an ontological metaphor, and not subject to ultimate empirical evidence. Likewise, my own hypothetical god-like Enformer is just an immaterial metaphor, and not amenable to materialistic challenges. It simply provides a way to think about the kind of Ultimate Origin questions that Plato & Aristotle answered with the metaphor of a First Cause. Sure the container of the metaphor is a material brain, but what is the substance of an abstract metaphor?

Would you (180) also accuse Fredkin, a certified genius, of "hasty generalization" and "unparsimonious and the pseudo-speculative equivalent of (neo-Aristotlean / neo-Thomistic / neo-Hegelian) intelligent design"? Is his "law enforcement agent" a god-of-the-gaps posit to cover our ignorance of ultimate answers? Is his "computer" a self-programmed natural intelligence, or an artificial intelligence created by an even more intelligent Programmer? Is human intelligence merely an accidental pattern of a hypothetical "universal cellular automaton"? Perhaps, it just doesn't matter, literally. :smile:

PS__180's Materialistic worldview seems to force him to assume that the information & ideas encoded into a computer program are made of some kind of exotic matter. He doesn't understand that the matter is a container for information, not the ideas per se.

Wayfarer July 04, 2023 at 22:45 #820091
Quoting Count Timothy von Icarus
In most versions of physicalism, which tend to embrace the computational theory of mind (still seemingly the most popular theory in cognitive science), a belief is just an encoding of the state of the external environment.


All very well, but this seems to me to be overlooking or taking for granted a great deal of what is required by such an encoding. I can see how it is perfectly applicable when it come to animals responding to stimuli, but reason is able to do a great deal more than that. It generalises and abstracts, from the particular to the universal. Sure, if you treat life as a kind of model or a game, this approach makes sense - 'Sim Life'. But is it philosophy?

Quoting Count Timothy von Icarus
This view works regardless of how consciousness arises, or even if it is eliminated, because agents are not defined in terms of possessing first person perspective, but rather through having goals.


That's what makes it reductionist. You can set aside the first person perspective, and with it, the reality of existence, by treating it as a model, or a board game, as if you were surveying the whole panorama from outside it - when you're actually not.

Quoting Count Timothy von Icarus
Lewis seems to conflate the proposition that "the universe and causal forces are meaningless," as in, "devoid of moral or ethical value and describing nothing outside themselves," with agent's beliefs necessarily also being "meaningless," as in "the beliefs must not actually be in reference to anything else."


I think his argument is situated with reference to the classical atheistic materialism of the culture of his day. Have you ever encountered Bertrand Russell's A Free Man's Worship? Published around the turn of the 20th century, it was an anthemic essay dedicated to the purportedly brave and clear-eyed scientific vision of a Universe driven wholly by the forces of physics and man as 'accidental collocation of atoms'. Jacques Monod's Chance and Necessity was another landmark work in similar vein. That vision is also the subject of enormous commentary in 20th century literature, art, drama and philosophy. But I think that hard-edged mentality is already on the wane, 21st century science is more engaged with questions of meaning, acknowledgements of the human aspect of all the sciences and a renewed sense of mystery. The hard edged kind of atheistic materialism that his argument was aimed at will always have its advocates, but there's an abundance of alternatives today (I am enjoying Adam Frank's articles, he's a physics scholar and popular science commentator, see his Science Claims a God's Eye View and his current 'mission statement'.)

'In Aristotle, nous is the specific faculty that enables human beings to reason. For Aristotle, this was distinct from the processing of sensory perception, including the use of imagination and memory, which other animals can do. For Aristotle, nous is what enables the human mind to set definitions in a consistent and communicable way, and provides the innate potential for different persons to understand those universal categories in the same logical ways (and note that Kant adopted Aristotle's categories almost unchanged)..... In this type of philosophy, later considerably elaborated by neoplatonism, it came to be argued that the human understanding (nous) somehow stems from this cosmic nous, which is however not just a recipient of order, but also a creator of it.'

[quote=Immanuel Kant]Two things fill the mind with ever new and increasing admiration and awe, the more often and steadily we reflect upon them: the starry heavens above me and the moral law within me. I do not seek or conjecture either of them as if they were veiled obscurities or extravagances beyond the horizon of my vision; I see them before me and connect them immediately with the consciousness of my existence.[/quote]

180 Proof July 04, 2023 at 23:26 #820100
Quoting Gnomon
Would you (180) also accuse Fredkin ... of "hasty generalization" and "unparsimonious and the pseudo-speculative equivalent of (neo-Aristotlean / neo-Thomistic / neo-Hegelian) intelligent design"?

Yes and no.

Yes, Fredkin's "computer universe" proposal/conjecture is, in fact, an unparsimonious hasty generalization. Read Seth Lloyd, David Deutsch & Stephen Wolfram.

No, I only "accuse" you, Gnomon, of pseudo-speculations, etc: "enformationism" = "intelligent design" = "pan-en-deism" = "first cause/unmoved mover" (i.e. WOO-of-the-gaps). :smirk:

Is his "law enforcement agent" a god-of-the-gaps posit to cover our ignorance of ultimate answers?

I'll drink to that. :up:

Is his "computer" a self-programmed natural intelligence, or an artificial intelligence created by an even more intelligent Programmer?

This "computer" metaphor amounts to an infinite regress – it's "enformers" all the way down. :lol:

Is human intelligence merely an accidental pattern of a hypothetical "universal cellular automaton"?

Define "human intelligence". :sparkle:

Count Timothy von Icarus July 04, 2023 at 23:41 #820107
Reply to 180 Proof

Sure, computation has been operationally defined since Turing's "On Computable Numbers With an Application to the Entscheidungsproblem," and Church's introduction of Lambda Calculus, and their findings re: the Church-Turing Thesis was later extended conceptually to physical systems more generally with the discovery that relatively simple cellular automata could simulate a Universal Turing Machine. The computable numbers, as Turing says, "may be described briefly as the real numbers whose expressions as a decimal are calculable by finite means."

However, an operational definition is not what is wanted when discussing the ontological underpinnings of the universe in the same way that explaining matter as "the amount the spring on a spring balance stretches when something is placed upon it," fails to adequately explain matter in all its aspects.

In any event, even the operational definition is widely acknowledged not to be fully rigorous, as the term "algorithm," on which the Church-Turing thesis hangs itself lacks a rigorous definition.

The problem gets even more dicey when one starts talking about the physical instantiation of computation. There is considerable debate and myriad different camps vis-á-vis the question of how to define computation in this respect. Central to this debate is the problem that, if pancomputationalism is true, and every physical system is a computer, then the core thesis of computational theory of mind, that the brain is a computer, becomes trivial (this is why pancomputationalists have jumped on IIT, although to my mind I don't get how IIT doesn't imply panpsychism, which Tegmark at least seems prepared to bite the bullet on).

On the other hand, permissive mapping accounts of physical computation allow virtually every system to be "computing," every possible computable function (Putnam and Searle make this argument) making the concept of computation itself trivial. I for one can certainly see how these issues can motivate people to cast in with semantic explanations of computation, even if I'm not ready to join them.

There is certainly no widely accepted definition of what makes a physical system a computer, and in this aspect the current operational definition itself is sorely lacking. I am not aware of many attempts at a theoretical definition, maybe because Liebnitz seems to have done as good of a job as possible off the bat? Although I also think the shadow of Platonism in mathematics makes it difficult to work on defining an abstract process that necessarily take "steps" (time) and which is itself defined by recursion (e.g. Godel's early work on computation).

Reply to wonderer1

Sure, check out his original paper "On Computable Numbers With an Application to the Entscheidungsproblem." The term "computer" in Turing's day referred to a person who computed figures for their job. He makes specific references to people in crafting the idea of a TM. For example, the requirement that the machine's memory be finite is justified by "the fact that the human memory is necessarily limited." Turing is specifically idealizing what a human being does when "computing" figures with a pen and paper. For more detail you can also check out: https://plato.stanford.edu/entries/church-turing/#MeanCompCompTuriThes

I agree with the keys and streetlight metaphor but I also think pancomputationalism does get at an essential element of how the world works. I am just not convinced that Turing's definition is appropriate for what we wish to describe, in part because continua do appear to exist in physics, although a fully discrete universe certainly hasn't been ruled out.

It's worth noting that Turing's claim is not that "anything that follows law-like behaviors or instructions must be computable." His claim is merely that a Universal Turing Machine can compute all functions that any Turing Machine can compute and this statement is paired with an argument for why the UTM is a good definition for effective computation, while this definition is bolstered by providing an example of an uncomputable number. This does not imply that there cannot be machines that can do things UTMs cannot do.

The reason I mention the above is that, even if it were satisfactorily confirmed that some elements of physics are surely uncomputable via a UTM, I don't not think this would be a death blow to pancomputationalism. Rather, we might be able to adopt some sort of new formalism to describe such uncomputable law-like behavior, and we'd likely give it some sort of new name like "super computation," although hopefully it'd be something more clever than that.
wonderer1 July 04, 2023 at 23:54 #820111
Count Timothy von Icarus July 05, 2023 at 00:13 #820112
Reply to Wayfarer

That's what makes it reductionist. You can set aside the first person perspective, and with it, the reality of existence, by treating it as a model, or a board game, as if you were surveying the whole panorama from outside it - when you're actually not.


Exactly. However, the problem of whether or not consciousness can actually be fit into such a physicalist model, which is something such models need to be able to explain if they are to be satisfactory to most people, seems like a separate problem from the one Lewis is pointing out. If we assume that physical systems, as described per physicalism, can indeed produce first person experience, then Lewis' argument doesn't seem to work.

I don't think abstraction is a particularly hard problem for the physicalist either. Tropes and universals can be described in mathematical, computable terms.

So my point would be that argument just doesn't seem to add much. Obviously it is true that physicalism is deeply broken if it is unable to ever explain the most obvious fact of existence, first person experience. No extra argument is really needed if you can prove that an ontology has a giant "the world we experience," hole in it. However, since no system can currently "explain everything," and since plenty of previously mysterious phenomena have successfully been explained in physical terms, I don't think this is a KO of physicalism either. It certainly doesn't work the way arguments that superveniance, as presented in popular forms of physicalism, is incoherent work. If accepted, these do seem to "KO" at least popular varieties of physicalism. That seems to be the type of argument Lewis is going for, but I don't think it works.

Have you ever encountered Bertrand Russell's A Free Man's Worship?


No, I generally tend to steer clear of primary sources for Russell, at least on that sort of thing, because I find him to be one of the most uncharitable, self-assured philosophers out there and it rubs me the wrong way. I'm familiar with the vision from Stace's "Man Against the Darkness," though. I find it sort of funny in a way, because for the Stoics and many early Christians the fact that the world did move in such a law-like way was itself evidence of the divine Logos, not an argument against the divine.
Wayfarer July 05, 2023 at 00:34 #820117
Quoting Count Timothy von Icarus
If we assume that physical systems, as described per physicalism, can indeed produce first person experience...


But we cannot so assume.

Quoting Count Timothy von Icarus
Tropes and universals can be described in mathematical, computable terms.


By rational agents - human beings - augmented with intentionally-designed artefacts - computers and calculators. Were those rational abilities absent, there would be no apprehension of tropes or universals. I know it's already been suggested that crows can count, but try explaining the concept of prime to them.

Quoting Count Timothy von Icarus
I find it sort of funny in a way, because for the Stoics and many early Christians the fact that the world did move in such a law-like way was itself evidence of the divine Logos, not an argument against the divine.


That is a subject in history of ideas (associated with Arthur Lovejoy's book The Great Chain of Being: A Study of the History of an Idea, published 1936.) One of the major historical themes explored is 'man's changing vision of the cosmos' - from the demiurgos of Plato, to the biblical creation myths, to the mechanical universe of the early modern period, where God became a ghost in his own machine. The concept of natural laws was originally identified with ideas emanating from the One, then as divine commandments, and then as physical laws absent any intentionality or design - which is the view expressed by Russell in that essay. It is that kind of materialism which is challenged by the argument from reason.
javra July 05, 2023 at 01:20 #820125
Quoting Wayfarer
I know it's already been suggested that crows can count, but try explaining the concept of prime to them.


To try clarify what was suggested, by me at least: If crows can count, then crows (as with apes and other lesser animals) obviously cannot count the way humans do: via use of language and its conceptual constructs. But then crows can't understand a rock as object the same way we do either - e.g., that it consists of minerals, subatomic particles, etc. Still, crows can hold awareness of some rudimentary properties of rocks well enough to intentionally make use of some rocks as tools - not like us, they can't build a skyrocketing pyramid from them, but they can drop them on things in attempts to crack these things open.

In parallel to their, by comparison, minuscule understanding of physical objects in the world, crows might well be able to apprehend rudimentary aspects of definite quantities existing in the world which we humans linguistically refer to by the term "numbers".

This teeny-weeny aptitude of apprehension will quite obviously differ by great magnitudes in comparison to an average human's. But this does not mandate that there is an absolute on/off switch in-between.

That our human psyches are leaps and bounds more awareness-endowed than those of all lesser animals does not of itself then indicate that there was no evolutionary cline in awareness and the intelligence that accompanies it.

As another parallel, that one can't explain calculus to a five-year-old doesn't then imply that the five-year-old has no awareness of specific quantity whatsoever.

Not claiming to know that lesser animals do or don't count. But if they do, this does not then make their aptitudes of understanding on par to our own. Its why we term them "lesser" animals - due to us being the most evolved (here strictly meaning, developed) animal we know of by far.


Janus July 05, 2023 at 01:36 #820127
Quoting 180 Proof
I'm not convinced (it does not seem to me to follow), however, 'that if physical events-regularities are computable (which they are), then physical reality must be a "computer" executing a nonphysical program (and, in your case, Gnomon, that's written by a "nonphysical programmer")' – at best, this hasty generalization is too unparsimonious and the pseudo-speculative equivalent of (neo-Aristotlean / neo-Thomistic / neo-Hegelian) "intelligent design". :eyes:


:100: I totally agree with you here.
Metaphysician Undercover July 05, 2023 at 01:55 #820131
Quoting Count Timothy von Icarus
Rather than a set of immutable numbers, which seems less defensible today, we can have a set of possible, contextually immutable axioms, which define a vast, perhaps infinite space of systems. The truths in the systems are mutable, because there are different systems, but then there is a sort of fall back, second-order Platonism where the existence of the systems themselves, and relations between them, are immutable.


I don't think that this works. The reason why different systems are needed is because incompatibilities arise between one and another. Incompatibility makes it impossible to have immutable axioms which would be applicable to all systems.

The problem is right at the foundation, the nature of a unit, one. We can assume an ideal unit, "one", but then the things which we allow ourselves to do with "one" through stated axioms, must be consistent with what we can actually do with one object, in practise. In practise though, we find that different types of things, or objects, allow us to do different types of activities with them. This is very evident with division. Each type of thing has specific ways which it can be divided. In mathematics, the common principle is to allow that the unit "one" can be divided in any possible way. This does not properly represent the restrictions on division which exist in reality.
Janus July 05, 2023 at 02:09 #820134
Quoting Metaphysician Undercover
I don't think that this works. The reason why different systems are needed is because incompatibilities arise between one and another. Incompatibility makes it impossible to have immutable axioms which would be applicable to all systems.


:up:
Count Timothy von Icarus July 05, 2023 at 16:21 #820270
Reply to Metaphysician Undercover


Incompatibility makes it impossible to have immutable axioms which would be applicable to all systems.


Absolutely. That's why the pivot is to just think in terms of all the possible coherent systems. There isn't one set of immutable axioms but rather a landscape of systems as your new fixed objects. At least that's how I've seen the conception developed in some cases.

But, as I understand it, while numbers tend to get grounded in quite abstruse work within set theory that there is less general confidence in, they can also be grounded using category theory. Barry Mazur has some relatively approachable stuff on this, although I certainly don't get all of it.

Timelessness remains either way, mathematics is eternal, not involved in becoming— in most takes at least. This, I think, may be a problem. Mazur had an article on time in mathematics but it didn't go that deep. But I recently discovered Gisin's work on intuitionist mathematics in physics, and that is quite interesting and sort of bound up with the philosophy of time. The Nature article seems stuck behind a paywall, but there is this Quanta article and one on arXiv.

https://arxiv.org/abs/2011.02348

https://www.quantamagazine.org/does-time-really-flow-new-clues-come-from-a-century-old-approach-to-math-20200407/
Gnomon July 05, 2023 at 16:51 #820283
Reply to 180 Proof Quoting Gnomon
From his reply : "I'm not convinced (it does not seem to me to follow), however, 'that if physical events-regularities are computable (which they are), then physical reality must be a "computer" executing a nonphysical program (and, in your case, Gnomon, that's written by a "nonphysical programmer")' – at best, this hasty generalization is too unparsimonious and the pseudo-speculative equivalent of (neo-Aristotlean / neo-Thomistic / neo-Hegelian) "intelligent design".


Reply to Wayfarer Again, I'll reply to you, because dialoging with 180 is like talking to a snarky wall. He says that, in his superior wisdom, he is convinced that physical events are indeed computable --- as Wolfram, Tegmark, etc claim --- but he is "not convinced . . . that physical reality must be a computer executing a nonphysical program". His incredulity toward anything "nonphysical" is built-in to the dogma of Materialism, and non-physical Intelligence is taboo. But many mathematicians, such as Wolfram & Tegmark are platonist in their view of the ontological status of mathematics/logic*1. That's mainly because math & logic are about systematic interrelationships (meanings), not isolated things (atoms in void). Likewise, a computer program is not a material object, but an imagined sequence/system of events, mathematically encoded into a complex Algorithm (a conceptual procedure, not a physical thing).

So, a real computer --- perhaps including the universe --- is a physical/material machine designed by a Designer to process some mathematical software encoded by a Programmer, with some eventual end in mind. The input-to-output procedure is typically aimed at future knowledge of something not otherwise knowable --- otherwise, why bother to do the math? If that statement is true of the PC running non-physical software on your desktop, why is it a "hasty generalization" to conclude that the evolving universe also requires some kind to mind to explain the origin of both the hardware computer and the software code? Apparently, for 180, there is no such thing as software : it's all hardware, all the way down*3.

180's Materialistic worldview seems to be based on pragmatic scientific Reduction, instead of theoretical philosophical Generalization*2. But reduction to what? The ancient Atom of Materialism has proved elusive & uncertain to the quantum scientists, who now mostly think of Reality as a Mathematical Field of some kind. Which is why Tegmark wrote his book : Our Mathematical Universe, My Quest for the Ultimate Nature of Reality. Yet, for him, the universe itself seems to be the ultimate Atom of reality. So he doesn't bother to make the next logical step, from computer (logical) universe to a universal (rational)] Programmer. Ironically, due to the inherent randomness of cosmic evolution, the fate of the universe is unpredictable. Unlike the instant creation of Genesis, even the Programmer of gradual evolution could not know the final outcome, except by running the Program to its conclusion*4. :smile:


*1. Is math non physical? :
Math is not physical (composed of matter/energy), though all physical things seem to conform to it.
https://www.askphilosophers.org/question/24527

*2. Philosophical Generalization :
A generalization is a form of abstraction whereby common properties of specific instances are formulated as general concepts or claims. Generalizations posit the existence of a domain or set of elements, as well as one or more common characteristics shared by those elements (thus creating a conceptual model).
https://en.wikipedia.org/wiki/Generalization

*3. Software :
Software of a computer is actually a collection of data or some type of instructions that dictates the computer how to perform some work. From this definition it is quite evident that software is a completely massless collection of instructions. So, it can't be the physical part of a computer.
https://www.quora.com/Are-all-the-physical-parts-of-a-computer-called-software
Note --- The computer is a material hardstuff (hyle), but the software is immaterial mindstuff (morph, form)

*4. " Ultimately, it would be fair to say the Universe itself is chaotic and therefore unpredictable."
https://www.sciencefocus.com/science/chaos-theory-why-the-universe-is-a-massive-unpredictable-mess/
NotAristotle July 05, 2023 at 17:38 #820296
Reply to Wayfarer I am not sure I am getting the argument. In particular, I am not sure I understand Premise 1. Why would the explanation of a belief in terms of non-rational causes prohibit the belief from being rationally inferred? Is the concern overdetermination of the belief?
Wayfarer July 05, 2023 at 22:13 #820366
Quoting NotAristotle
Is the concern overdetermination of the belief?


Something like that. Put it this way: if a belief is a consequence of a brain condition, then it is not held on the basis of logical necessity. It is arguing that if our ability to reason logically is merely a result of physical processes, such as the firing of neurons, then there are no grounds to trust that our logical conclusions are valid. Our logical reasoning would be explained in terms of physical processes, undermining the sovereignty of reason.
Wayfarer July 05, 2023 at 22:56 #820371
Quoting Gnomon
Again, I'll reply to you, because dialoging with 180 is like talking to a snarky wall.


Totally hear you on that. But your use of the metaphors of information and information processing introduce many difficulties from a philosophical point of view. My own approach is more oriented around 'history of ideas' and understanding how ideas influence cultural dynamics and entrenched attitudes, leavened somewhat with my engagement with Buddhist praxis. I try and situate what I write against that context. I am not much in favour of 20th century Anglo-American philosophy which overall is oriented around scientific naturalism and armed to the teeth against anything suggesting idealism (although there is a healthy idealist strain in current culture also.)

Quoting Gnomon
(The) Materialistic worldview seems to be based on pragmatic scientific Reduction


I'll take a step back. How modern physicalism, naturalism or materialism evolved is, I think, not very difficult to discern. The watershed was René Descartes, and the confluence of his work with Newtonian physics and Galileo's cosmology. This sets up the modern worldview (bearing in mind we're now situated in a post-modern world, but I'll leave that aside for now.) The 'universal science' at the heart of this method, based on precise analysis of measurable attributes using Cartesian algebraic geometery and calculus, is the basis of the success of the modern scientific method. The famous Cartesian description of the mind as 'res cogitans', literally, 'a thinking thing', however, has had calamitous consequences, as it seemed very difficult to establish what, exactly, it means. Meanwhile, the concentration on the purely measurable and quantitative aspects of the universe in the discovery of modern scientific method provided many astounding breakthroughs. Within that context, scientific materialism is the consequence of attempting to apply the very successful methods deployed by science to the problems of philosophy (in the absence of any real insight into what those problems are.) That's it in a nutshell, as far as I'm concerned. (I think in all likeliood, phenomenology and existentialism is far nearer the mark than anglo-american philosophy, but I'm not well-schooled in that either.)

As regards mathematical platonism - I had a minor epiphany about that. It was simply this: that whilst every material object is composed of parts and has a beginning and an end in time, this does not apply to numbers and other mathematical objects (although later I realised that only prime numbers are strictly indivisible.) At the time of this realisation, I thought 'aha! This is why the ancients held mathematics in such high esteem: they're nearer the "unconditioned origin of being"'. And the fact that the intellect is able to grasp these ideas is evidence for a kind of dualism, although not of the Cartesian kind. (I've attempted to follow that thread through the labyrinth, but the subject matter is arcane and difficult, and demands a much greater knowledge of all the classical texts than I will ever have.)

One of my all-time favourite Buddhist texts was subtitled 'Seeking truth in a time of chaos'. Don't loose sight of the fact that modernity - actually, post-modernity - is chaotic. There's a lot of turmoil, vastly incompatible opinions and worldviews all jostling one another for prominence. Learn to live with it, but I recommend not trying to tame the waters. It's beyond any of us to to that.



Tom Storm July 05, 2023 at 22:59 #820374
Quoting Wayfarer
One of my all-time favourite Buddhist texts was subtitled 'Seeking truth in a time of chaos'. Don't loose sight of the fact that modernity - actually, post-modernity - is chaotic. There's a lot of turmoil, vastly incompatible opinions and worldviews all jostling one another for prominence. Learn to live with it, but I recommend not trying to tame the waters. It's beyond any of us to to that.


I hear you.

Gnomon July 05, 2023 at 23:20 #820381
Quoting Wayfarer
Tropes and universals can be described in mathematical, computable terms. — Count Timothy von Icarus
By rational agents - human beings - augmented with intentionally-designed artefacts - computers and calculators. Were those rational abilities absent, there would be no apprehension of tropes or universals. I know it's already been suggested that crows can count, but try explaining the concept of prime to them.

Again, we have here an instance of looking at one side (the apparent side) of the whole world. Since scientists have concluded, from available evidence, that big-brained homo sapiens emerged on a minor planet on the margin of an ordinary spiral galaxy, only after 14 billion earth-years of gradual development. If so, did "tropes & universals" exist in the natural world for all those eons of evolution, or are they a result of "artificial" reasoning? What about "mathematics"?*1 Is that a natural thing, or an unnatural product of human reasoning? If the universe was "computing" the inputs & outputs of Nature since the system was suddenly turned-on in a Big Bang of matter/energy interaction, who/what encoded the program of evolution? Was it a sapient counting crow? (just kidding)

A materialistic worldview intentionally avoids dealing with the elephant in the world : the human mind, the rational observer --- those annoying unnatural pests who lit-up the world with artificial light. Perhaps, as you once noted, that evasion may be due to exaggerated "fear of organized religion", or of abstract reason. Reply to 180 Proof in a post above, responded to my question : "Is human intelligence merely an accidental pattern of a hypothetical "universal cellular automaton"?", with : "Define 'human intelligence' ". Of course, he was not really interested in my opinion on the subject ; just looking for another opportunity to demonstrate the superiority of his mountaintop-Illuminati intellect, unburdened by the superstitions of such spooky concepts as immaterial (functional) Minds & Intelligences --- presumably including gods & ghosts & angels. Judging by superficial appearances, a doctrinaire Materialist might not see any meaningful difference between a counting crow and a computing scientist.

The implicit assumption of monistic Materialism is that anything the human mind imagines --- that can't be counted --- does not matter. And that countless category includes the distinction between Brains & Minds, or Seeing & Knowing. If he placed a coin on a table and asked me if it was heads or tails, he would ridicule my holistic BothAnd answer, because you can't see the downside of the coin ; so it doesn't count, even though you know that it is there, by reasoning from prior experience. Sure, reasoning can lead to erroneous conclusions, but so can discounting what is not directly apparent to the 5 senses. Likewise, discounting the value of human reasoning, just because it a natural outcome of mundane evolution --- instead of a divine miracle --- can lead to a one-sided worldview. A belief system that ignores ideas, reasons, and other abstractions as immaterial*2. :smile:

PS___Yes, " universals can be described in mathematical, computable terms" because, like any other man-made language, computer code can represent abstractions with symbols*3.

*1. What is Mathematics? :
the abstract science of number, quantity, and space.
__Oxford

*2. What are Abstractions? :
the quality of dealing with ideas rather than events
___Oxford

*3. What is a Symbol? :
something visible that by association or convention represents something else that is invisible.
https://www.vocabulary.com/dictionary/symbol

Gnomon July 06, 2023 at 00:01 #820399
Quoting Wayfarer
But your use of the metaphors of information and information processing introduce many difficulties from a philosophical point of view. My own approach is more oriented around 'history of ideas' and understanding how ideas influence cultural dynamics and entrenched attitudes, leavened somewhat with my engagement with Buddhist praxis

I am aware that you and I are coming from completely different backgrounds : mine in the sciences, yours in history & literature. But, surprisingly, we have come to similar conclusions about some of the most controversial topics discussed on this forum. Hence, though wearing different uniforms, we are forced to stand back-to-back, fending-off the forces of encircling orthodox Scientism.

For example : "The argument from reason challenges the proposition that everything that exists, and in particular thought and reason, can be explained solely in terms of natural or physical processes". Personally, I don't interpret the existence of abstract Thought & logical Reason as evidence of a "supernatural" act of intervention, in the traditional sense of many world religions. Instead, I attribute the ubiquitous role of mathematical/material Information in the world to a mysterious preter-natural source, similar to the abstract principles that Plato & Aristotle called First Cause or Logos or Prime Mover. Except for that hypothetical Ontological beginning, everything else in the world is a natural result of evolutionary programming. No superstitions necessary, it's just coding.

Since I have no divine revelation or Buddhist insights, I have no basis for making more specific conjectures about the postulated metaphorical Enformer or Programmer : it's just a theory, like the Big Bang. Hence, it does not prescribe any unique shoulds & oughts & thou shalts. From the perspective of doctrinaire Naturalists though, that pre-natural + natural postulation leaves me suspended between mythical or superstitious religions, and empirical pragmatic sciences. My worldview is completely natural & mundane, up to the point where physical Nature began in an astronomically-unlikely bang in the dark. Beyond that, anything I, or anyone else, might say is a shot in the dark. :smile:


180 Proof July 06, 2023 at 00:36 #820406
Quoting Gnomon
?180 Proof in a post above, responded to my question : "Is human intelligence merely an accidental pattern of a hypothetical "universal cellular automaton"?", with : "Define 'human intelligence' ". Of course, he was not really interested in my opinion on the subject ...

And, of course, once again, you project by impugning my motives for requesting clarification in order to deflect from the conspicuous fact that you have no idea, Gnomon, what the hell you're gibber-jabbering about. :yawn:
Wayfarer July 06, 2023 at 11:52 #820494
Quoting Gnomon
we are forced to stand back-to-back, fending-off the forces of encircling orthodox Scientism.


User image

Gnomon July 06, 2023 at 22:44 #820603
"The argument from reason challenges the proposition that everything that exists, and in particular thought and reason, can be explained solely in terms of natural or physical processes".___OP

Quoting Wayfarer
My own approach is more oriented around 'history of ideas' and understanding how ideas influence cultural dynamics and entrenched attitudes, leavened somewhat with my engagement with Buddhist praxis

My knowledge of the 'history of ideas', and of Hindu/Buddhist philosophy --- not to mention "praxis" --- may be superficial compared to yours. And my knowledge of Western Philosophy --- especially of the modern era --- is superficial to that of Reply to 180 Proof. So, I don't pretend to compete in those arenas. But my "expertise" --- relatively speaking --- is in the 21st century sciences of Quantum Physics (QP) and Information Theory (IT).

The pioneers of QP, baffled by the variance of their experiments from classical/mechanical Newtonian Physics, turned to Hindu & Buddhist vocabularies --- not the religions --- to express the Holistic & Immaterial (mental) aspects of sub-atomic reality. John A. Wheeler even went so far as to combine QP and IT in his famous "It from Bit" postulation. Then, he went on to propose the Participatory Anthropic Principle*1, which implied that the observer's mind could have real effects upon the material world --- at least on the sub-atomic scale. He eventually toyed with the idea of a Weak Anthropic Principle, and even a Strong Anthropic Principle*2.

Obviously, such spooky-woo notions (per 180) are anathema to a believer in hard Materialism, in which the Mind is a minor side-effect of Brain functions, with no magical powers over the material world. Being a practicing scientist, not a philosopher, Wheeler didn't expand his Information/Matter/Mind theory to its ultimate conclusion. So others took-up the gauntlet for him. And their Information-centric (not matter-centric) reasoning led to the conclusion that the human mind must be a descendant of a Prime Mind of some kind : a Logos or Enformer/Programmer.

Both the woke Buddha and itty-bitty Wheeler, being pragmatic practitioners in their own fields, astutely avoided making the philosophical inference of an ultimate Mind, who planted the enforming seed that eventually sprouted in homo sapiens (rational ape). Yet, since I am an amateur philosopher, with no peer review to revile my unorthodox ideas for conformance to established orthodoxy*3, I feel free to take the "it from bit" seed to its logical ontological conclusion.

However, since I have no interest in founding a popular religion, I'm content to use the ancient vocabulary of Plato (Logos) and a neologism of my own devising (EnFormAction) to express my personal opinions on an opinion-sharing forum. As for Plato, my Logos/Enformer is no Satan or Savior, just Order-Organization-Reason in a growing-maturing-complexifying material world --- where the only Nirvana is a refuge of your own imagination. :smile:



*1. "A participatory anthropic principle (PAP) was proposed by the American physicist John Archibald Wheeler. He suggested that if one takes the Copenhagen interpretation of quantum mechanics seriously, one may conclude that, because no phenomenon can be said to exist until it is observed,"
https://www.britannica.com/science/participatory-anthropic-principle
Note : This strange statement by a physicist, echoes Berkeley's Idealist assertion : "Berkeley's immaterialism argues that "esse est percipi (aut percipere)", which in English is to be is to be perceived (or to perceive). ___Wiki

*2. "THE STRONG ANTHROPIC PRINCIPLE The ultimate form of anthropic reasoning is to assert that the coincidences we have remarked on are more than that: that the universe must be such as to admit the production of intelligent life at some time. This idea is known as the strong anthropic principle."
https://ned.ipac.caltech.edu/level5/Peacock/Peacock3_5.html
Note -- Weak Principle = Intelligence from Coincidence. Strong Principle = Intelligence from Intelligence.

*3. On this forum, 180 proof has appointed himself as the un-official inquisitor of Scientism/Naturalism/Materialism orthodoxy. And what vexes his know-it-all-neurons most is the mutual ideas : a> that there is some immaterial force in the world (energy/information) and b> that it originated in some preter-natural Intelligence. He need not worry though. The Enformationism philosophy, with a single adherent, is just a personal opinion (like that of Gallileo), based on eye-opening observations of the squishy (non-mechanical) foundations of the material world.
No, 180, I'm not equating my little QP/IT notion with Gallileo's church-threatening revelations. Perhaps Information-centric physicists, Wheeler & Paul Davies, though, will find a similar place in the future history of science & philosophy. All I can say, as an amateur with no credentials, is that the all-purpose role of Information in the world makes sense of many philosophical & scientific enigmas for me : such as the emergence of intentional Reasoning in a world of dumb Matter & dynamic Energy.

*4. "By the time of Socrates, Plato, and Aristotle, logos was the term established to describe the faculty of human reason and the knowledge men had of the known world and of other humans. Plato allowed his characters to engage in the conceit of describing logos as a living being in some of his dialogues."
https://www.newworldencyclopedia.org/entry/Logos


User image
Metaphysician Undercover July 08, 2023 at 01:11 #820900
Quoting Count Timothy von Icarus
But, as I understand it, while numbers tend to get grounded in quite abstruse work within set theory that there is less general confidence in, they can also be grounded using category theory. Barry Mazur has some relatively approachable stuff on this, although I certainly don't get all of it.

Timelessness remains either way, mathematics is eternal, not involved in becoming— in most takes at least. This, I think, may be a problem. Mazur had an article on time in mathematics but it didn't go that deep. But I recently discovered Gisin's work on intuitionist mathematics in physics, and that is quite interesting and sort of bound up with the philosophy of time. The Nature article seems stuck behind a paywall, but there is this Quanta article and one on arXiv.

https://arxiv.org/abs/2011.02348

https://www.quantamagazine.org/does-time-really-flow-new-clues-come-from-a-century-old-approach-to-math-20200407/


I haven't had a chance to read the referred material yet, but I think that this is an important subject, very much related to the op. As I see it the crux of the matter is the nature of "order". The first principle of mathematics is order, yet pure mathematicians do not want to be constrained by any natural order. So they propose a fundamental orderless condition which would allow infinite freedom for creations of order. This is the set which is not ordered.

The problem with this "not ordered" condition is that time is a type of order, and this puts the "not ordered condition as outside of, or prior to time. Now we can ask in what sense is this condition "prior" because it really cannot be prior in a temporal sense. And we might say that it is "logically prior". But this ought to be questioned. So the question might be, does it make sense logically, to speak of something which is "not ordered". If this idea, the existence of something without any order, is itself illogical, then the "priority" implied by placing this condition as prior to temporal order, cannot be said to be a logical priority, because it would appear more like an illogical type of priority.
Leontiskos July 15, 2023 at 01:52 #822618
Quoting Paine
My problems with his argument have nothing to do with this sort of speculation.


May I ask for your precise critique of his argument as it relates to this thread?

In a well-known argument Gerson claims that the immateriality of the intellect disproves materialism. In some places Gerson associates this argument with Aristotle, and it is this association that you seem to want to reject. But what exactly is it that you are rejecting? Which of Gerson's key claims, as presented in this thread, are non-Aristotelian?

I read all of your posts in this thread and this one seems to be most pertinent:

Quoting Paine
The "identity" with the object is not a simple correspondence of "forms".


For example, I would want to say, "What matters is whether the identity implies the requisite immateriality, not whether it is a simple correspondence of 'forms'."

If Gerson truly got this wrong then I should be interested to know how he got it wrong.
Paine July 15, 2023 at 02:37 #822624
Yours is a fair challenge. I will try to gather a proper response as I can. In the meantime, I can ask about something in your statement:

Quoting Leontiskos
For example, I would want to say, "What matters is whether the identity implies the requisite immateriality, not whether it is a simple correspondence of 'forms'."


Aristotle puts a lot of emphasis on the priority of the being one encounters. The generality of being a kind of thing is a pale shadow of the actual being. If that is the case, how 'forms' work in hylomorphic beings is different in the various "Platonic" models.

I figure that should be discussed before getting into Gerson's interpretation.
Leontiskos July 15, 2023 at 02:45 #822625
Quoting Paine
Yours is a fair challenge. I will try to gather a proper response as I can.


Okay, thanks.

Quoting Paine
Aristotle puts a lot of emphasis on the priority of the being one encounters. The generality of being a kind of thing is a pale shadow of the actual being. If that is the case, how 'forms' work in hylomorphic beings is different in the various "Platonic" models.


I don't at all doubt that this is the case. In fact my assumption is that the critique would involve the claim that Gerson is projecting some variety of non-Aristotelian Platonism onto Aristotle. Of course Gerson also advances the somewhat controversial thesis that Aristotle is best seen as a Platonist, but although this is related I'm not sure we need to get into it right now.
Paine July 15, 2023 at 02:53 #822626
Reply to Leontiskos
I suggest reading enough Plotinus to see his objections to Aristotle. Gerson does not simply take those remarks as the only way to read Aristotle. But it does change the perspective of what Platonism is about.

I don't claim to understand all the moving parts.

Edit to add: Gerson has been discussed numerous times here. I made an argument against one of his positions here.

For a more exhaustive discussion of the differences between Plato and the 'Neo-Platonists' there is Fooloso4's OP on Phaedo to consider. From that, you can see that people here have been disagreeing for years about it.

I realize that I am not up for rekindling those debates right now. It is summertime and the living is easy.
Wayfarer July 17, 2023 at 10:33 #823086
Quoting Leontiskos
In a well-known argument Gerson claims that the immateriality of the intellect disproves materialism.


Just noticed your post now.

I have read some of Lloyd Gerson's work, but I find his corpus pretty unapproachable, as it is directed almost solely at his academic peer group, or so it seems to me. He's written a series of books, including one contentiously called Aristotle and Other Platonists, but they're dense with footnotes and polemical skirmishes with competing interpretations. It's a shame his work is not more approachable, because I think his central thesis - that Platonism basically articulates the central concerns of philosophy proper, and that it can't be reconciled with today's naturalism - is both important and neglected. It would be great if there could be a compendium of his writing edited for a more general audience, although I suppose it would still be only a small general audience. (Rather a good lecture on his Possibility of Philosophy here.)

I've long been interested in various aspects of scholastic and platonic realism, i.e. the view that universals and abstract objects are real. There's precious little interest in and support for such ideas here, or anywhere, really. But I'm of the view that it was the decline of scholastic realism and the ascendancy of nominalism which were key factors in the rise of philosophical and scientific materialism and the much-touted 'decline of the West'. But it's a hard thesis to support, and besides, as I say, has very little interest, it's diametrically at odds with the mainstream approach to philosophy.

Some of the sources I frequently cite in support include Bertrand Russell's chapter on The World of Universals, the transcript of a lecture by Jacques Maritain The Cultural Impact of Empiricism, a book section about Augustine on Intelligible Objects, a book called The Theological Origins of Modernity by Michael Allen Gillespie. And this excerpt from a book on Thomistic philosophy which re-states, I think, the same argument Gerson refers to in respect of the immateriality of nous.

Tom Storm July 17, 2023 at 11:01 #823089
Quoting Wayfarer
I frequently cite in support include Bertrand Russell's chapter on The World of Universals, the transcript of a lecture by Jacques Maritain The Cultural Impact of Empiricism, a book section about Augustine on Intelligible Objects, a book called The Theological Origins of Modernity by Michael Allen Gillespie. And this excerpt from a book on Thomistic philosophy which re-states, I think, the same argument Gerson refers to in respect of the immateriality of nous.



A digression via some questions. Plato seems to regard nous as the highest form of understanding - the ability to contemplate the ultimate nature of reality via the Forms. Do you consider this to be approximately the same as enlightenment? Or something not quite as elevated? And is the idea that reasoning or intellectual intuition can help us to access a higher realm of knowledge - i.e., the reality behind the world of appearances? I'm interested in how this access is theorised to work. A rational process. I understand we can get there through anamnesis 'remembering' as we become reawakened through dialogue and learning.

What does having direct access to the Forms do for the perceiver?
Wayfarer July 17, 2023 at 11:21 #823090
Reply to Tom Storm These are all very deep questions. I’m hardly equipped to make a comparison between Platonic philosophy and Asiatic teachings of enlightenment (although one of my books, The Shape of Ancient Thought, Thomas McEvilly, does go into that in detail.)

Bertrand Russell says in his chapter on Pythagoras that the numerological and rationalist tendency in Pythagoras and in the later Greek tradition is one of the things that differentiates it from Asiatic mysticism. Seems to me that the Greek approach was far more likely to give rise to later science than the Indian and Chinese traditions. But it’s also true that since the Renaissance, the West has kept those elements of the tradition which were useful for science and technology while abandoning the ethical and moral precepts of Aristotelian thought (cf Alisdair MacIntyre). That might be because of the absorption of those principles into theology so that they became rejected along with religion.

I firmly believe that Greek philosophy held to the necessity of ‘the philosophical ascent’, but that with the loss of the qualitative dimension, ‘’the great chain of being’, then the idea becomes unintelligible. (Hence the tension between tradition and naturalism.) There is no axis along which the idea of ‘higher’ makes any sense. Everything is said to have arisen from self-organising matter, ideas only exist in the minds of h. Sapiens, chiefly to serve instrumental purposes. The modern world is very much at odds with the philosophical vision of the Greeks in that sense.
Tom Storm July 17, 2023 at 11:30 #823092
Quoting Wayfarer
Bertrand Russell says in his chapter on Pythagoras that the numerological and rationalist tendency in Pythagoras and in the later Greek tradition is one of the things that differentiates it from Asiatic mysticism.


Yes, this is where I was heading. Reason as pathway to higher awareness - very different. It does strike me that the notion of a reawakening of the wisdom we held before brith 'anamnesis' has within it some of the characteristics of enlightenment traditions.


Quoting Wayfarer
Seems to me that the Greek approach was far more likely to give rise to later science than the Indian and Chinese traditions.


Yes, this is also what I was wondering. Thanks.

Quoting Wayfarer
There is no axis along which the idea of ‘higher’ makes any sense.


We retain some of the ghostly afterlife of this through various notions of wisdom, I guess, but it's faint.

Metaphysician Undercover July 17, 2023 at 11:40 #823093
Quoting Wayfarer
It's a shame his work is not more approachable, because I think his central thesis - that Platonism basically articulates the central concerns of philosophy proper, and that it can't be reconciled with today's naturalism - is both important and neglected.


I believe that understanding the various forms of post-Platonism (rather than Neo-Platonism) is very significant to any study of metaphysics. Plato exposed many ontological and metaphysical problems inherent within the conventions of his day. He pointed in numerous different directions as to possible resolutions. The different ways that various philosophers have taken up his challenges is very indicative of the problems which philosophy encounters in addressing the nature of reality.

Quoting Wayfarer
But I'm of the view that it was the decline of scholastic realism and the ascendancy of nominalism which were key factors in the rise of philosophical and scientific materialism and the much-touted 'decline of the West'.


The decline was predicted by Plato, in "Republic", Bk. 8, 546. There is a number which relates the circumference of circles to the fertility of living creatures. That number is also related to the powers of 3,4, and 5 (Pythagorean theorem?) in some obscure way. Knowledge of the "perfect number" is required for divine birth. And the rulers of the state, lacking this knowledge will inevitably provide for the births of human beings who are not good natured and fortunate. Because of this, even the best state, as proposed, will decay and face dissolution.
Fooloso4 July 17, 2023 at 15:44 #823125
Quoting Tom Storm
A digression via some questions. Plato seems to regard nous as the highest form of understanding - the ability to contemplate the ultimate nature of reality via the Forms.


There is no better source of why this is not true than the works of Plato. Several of the dialogues can be cited, but Timaeus, in which Socrates remains mostly silent, presents a clear picture of the inadequacy of the Forms. In this dialogue, much or which is a monologue, Socrates expresses the desire to see the city he creates in the Republic at war. He wants to see the city in action. The story of the city in the Republic is incomplete. It is a city created by intellect (nous) without necessity (ananke), that is, a city without chance and contingency. A city that could never be.

For a more detailed discussion: Shaken to the Chora
Tom Storm July 17, 2023 at 19:55 #823169
Reply to Fooloso4 Not really for this thread, but I understand Plato's notion of the Forms evolved throughout his writings and that he was sometimes 'self-critical' - there are explorations of the problems of participation (Phaedo) and the issue of infinite regress, 'the Third Man Argument' (Parmenides). But does Plato stop thinking of the Forms as a source of truth and ultimate reality?
Fooloso4 July 17, 2023 at 21:57 #823183
Quoting Tom Storm
But does Plato stop thinking of the Forms as a source of truth and ultimate reality?


I don't know if he ever thought of them as a source of truth. Although Parmenides is generally considered a late dialogue, it is contextually an early dialogue based on the chronology of the dramatic settings of the dialogues. Socrates is a young man. What is the significance of this? Placing the dialogue at an early stage of Socrates journey suggests that he was from early on aware of the problems raised in the dialogues regarding Forms.

In Plato's Second Letter he says that his is a Socrates made young and beautiful.

In the Seventh Letter he says:

There is no treatise (suggramma) by me on these subjects, nor will there ever be. (341c)


In the Apology Socrates denies is having knowledge of anything "????? ??? ????", very much or great and good or beautiful. (21d)

In the Phaedo Socrates calls his hypothesis of the Forms "safe and ignorant". In addition to the Forms, he later recognizes the necessity of admitting physical causes such as fire and fever (105c)

In the Republic Socrates calls the Forms "stepping-stones and springboards" (511b). They are intended to free us from what has been hypothesized. But when asked he is circumspect but clear in stating that he does not actually have knowledge of the Forms:

"You will no longer be able to follow, my dear Glaucon," I said, "although there wouldn't be any lack of eagerness on my part. But you would no longer be seeing an image of what we are saying, but rather the truth itself, at least as it looks to me. Whether it is really so or not can no longer be properly insisted on. But that there is some such thing to see must be insisted on. Isn't it so?" (533a)


The truth as it looks to him may not be the truth, and he is not insisting that it is. But he insists that there is “some such thing to see”. What he shows us is a likeness of what the beings must be, that is, an image. The Forms are, ironically, images.

All of this is consistent with the many "likely stories (ton eikota mython)" in Timaeus. We are human beings, capable of telling likely stories, but incapable of discerning the truth of such things. Timaeus proposes it is best to accept likely stories and not search for what is beyond the limits of our understanding.

Socrates approves and urges him to perform the song (nomos). Nomos means not only song but law and custom or convention. In the absence of truth there is nomos. But not just any song, it is one that is regarded as best to accept because it is told with an eye to what is best. One that harmonizes being and becoming.



Tom Storm July 17, 2023 at 22:03 #823184
Reply to Fooloso4 Interesting. Thank you.

Quoting Fooloso4
We are human beings, capable of telling likely stories, but incapable of discerning the truth of such things. Timaeus proposes it is best to accept likely stories and not search for what is beyond the limits of our understanding.


That's generally been my game plan. I think I'll try to find a thread on the Forms to see what's been said here.
Paine July 17, 2023 at 22:12 #823190
Quoting Leontiskos
Which of Gerson's key claims, as presented in this thread, are non-Aristotelian?

Okay, I will give it a try.

The problem with Gerson is that he does not distinguish between the different roles Matter (? ???) plays amongst the 'Ur-Platonists' he assembles to oppose the team of 'Materialists' he objects to.

Plotinus says:

Ennead, II, 5, 5, translated by Katz:What conception then shall we for of matter? In what sense does matter exist? Its existence consists in potentiality. It is in the sense that it is potential. It exists in as much as it is a substrate of existence. "Existence" with regard to it signifies possibility of existence. The being of matter is only what it is to be. Matter is potential not just some particular thing, but all things. Being nothing by itself and being what it is, matter is nothing actually. If it were something actually, it would no longer be matter, that is , it would not be matter in the absolute sense of the term, but only in the sense in which bronze is matter.


In developing his ideas of actual being in relation to potential being, Aristotle says this:

Physics, 192a, translated by H.G. Apostle:Other thinkers, too, have perceived this nature (the belief in generation, destruction, and change in general) but not adequately. For, in the first place, they agree that there is unqualified generation from nonbeing, thus granting the statement of Parmenides as being right, secondly, it appears to them that if this nature is numerically one, then it must be also one potentially, and this makes the greatest difference.

Now we maintain that matter is distinct from privation and that one of these, matter, is nonbeing with respect to an attribute but privation is nonbeing in itself, and also that matter is in some way near to substance but privation is in no way such.

These thinkers, on the other hand, maintain that the Great and the Small are alike nonbeing, whether these two are taken together as one or each is taken separately. And so they posit their triad in a manner which is entirely distinct from ours. Thus, they have gone so far as to perceive the need of some underlying nature, but they posit this as being one, for even if someone [Plato] posits the Dyad, calling it the Great and Small, he nevertheless does the same since he overlooks the other [nature].

Now in things which are being generated, one of these [two natures] is an underlying joint cause with a form, being like a mother, so to speak, but the other part of the contrariety might often be imagined, by one who would belittle it, as not existing at all. For, as there exists an object which is divine and good and something to strive after, we maintain that one of the principles is contrary to it, but that the other [principle], in virtue of its nature, by nature strives after and desires that object. According to the doctrine of these thinkers, on the other hand, what results is that the contrary desires its own destruction. Yet neither would the form strive after itself, because it does not lack it, nor does it strive after the contrary, for contraries are destructive of each other. Now this [principle] is matter, and it is like the female which desires the male and the ugly which desires the beautiful, but it is not by itself that the ugly or the female does this, since these are only attributes.


For myself, the many points Plotinus and Aristotle may agree upon are not as interesting as where they clearly do not.
Wayfarer July 17, 2023 at 22:15 #823191
Quoting Fooloso4
Timaeus proposes it is best to accept likely stories and not search for what is beyond the limits of our understanding.


A theme also found in Kant.

Quoting Tom Storm
I think I'll try to find a thread on the Forms to see what's been said here.


Fooloso4's reading of Plato generally deprecates the widespread view that the knowledge of the forms corresponds to insight into a higher realm of truth. Plato's dialogues are open to a variety of interpretations by their very nature, and I don't think I agree with his interpretation. But to disagree would require re-visiting and re-reading many a dusty tome, so I think I'll regard his as one among other possible interpretations.

Tom Storm July 17, 2023 at 22:32 #823194
Reply to Wayfarer Understand. This is a tendentious subject. I just like having a sense of the scope of enquiry. :wink:
Paine July 17, 2023 at 22:33 #823195
Quoting Wayfarer
But to disagree would require re-visiting and re-reading many a dusty tome, so I think I'll regard his as one among other interpretation.


Agreeing or disagreeing with interpretations aside, are you saying that pursuing authorial intent in the writings is a foolish enterprise because supporting or deconstructing a particular set of opinions is just another opinion?
Wayfarer July 17, 2023 at 23:51 #823200
Reply to Paine Of course not. But there is plenty of scope for different interpretations.
Paine July 18, 2023 at 00:18 #823207
Reply to Wayfarer
There are many different interpretations. As one who has gotten dusty from the tomes, I am not sure how to read you balancing your interest in the works as works against a more general response to the ideas.

That is not an argument against what I think you might think but a sense of dislocation. I cannot address what you have collected and you have put yourself outside of what I can gather.
Wayfarer July 18, 2023 at 00:56 #823209
Reply to Paine Had I been schooled in the Classics I would have a much better knowledge of the texts. Regrettably it was not part of my education, a lack that I have only come to regret much later in life, and one which I don’t think I will ever really overcome. All I have are a few straws to grasp at, grounded on the scattered readings I have done. And also the intuition that many contemporary readings of Plato downplay or redact out those elements which are incompatible with the type of naturalism which prevails in today’s academia. That is in line with what I believe is the forgetting or occlusion of the qualitative dimension of existence which begins to become apparent in Hume - the loss of the ‘vertical axis’.

One of those straws is the belief that the parable of the cave does indeed present an allegory for a kind of intellectual illumination or an insight into a higher domain of being, and that those who have ascended to it see something which others do not, as I think the allegory plainly states. (I’m of the view that this is what is represented by the later term ‘metanoia’ which is not found in the Platonic dialogues but which means in this context an intellectual conversion or the breakthrough into a new way of seeing the world.) I suppose one secondary source I could refer to for support is this SEP entry on ’divine illumination’ in Greek philosophy.

My revisionist interpretation of the basic issue revolves around the question of the reality of universals, numbers, and other such intellectual objects (such as logical laws etc). I have the idea that number (for example) does not exist in the sense that phenomenal objects exist, but it is nonetheless real. Hence this is an important ontological category that this applies to, that is constitutive of rational thought, but which is not phenomenally existent. I think this understanding is kind of implicit in Platonic epistemology with the distinction between different levels of knowledge in the Analogy of the Divided Line. Platonic realism developed into Aristotelian realism, which was maintained up until scholastic realism, after which it was overthrown by nominalism and later by scientific empiricism. (This is the subject of the book I mentioned, The Theological Origins of Modernity.)

About the only contemporary representative of those kinds of views is Edward Feser (and also maybe Gerson who we already discussed.) I’m trying to fill in the very many blanks in this account, but to date I haven’t encountered anything which would cause me to think it’s entirely mistaken.



wonderer1 July 18, 2023 at 01:36 #823212
Reply to Gnomon

According to wikiquote that statement that you are propagating, as being from Heisenberg, is misattributed.
Fooloso4 July 18, 2023 at 13:46 #823258
Quoting Wayfarer
Fooloso4's reading of Plato generally deprecates the widespread view that the knowledge of the forms corresponds to insight into a higher realm of truth.


Perhaps the warning not to kill the messenger is apt. How well the widespread view holds up in light of the passages I sighted is up to the reader to decide.

Quoting Wayfarer
But to disagree would require re-visiting and re-reading many a dusty tome, so I think I'll regard his as one among other possible interpretations.


I can understand this, I feel that way about some philosophers, but for me Plato is not dusty tomes.
In my opinion, one must learn how to read Plato. Given the topic of this thread I will only say that the dialogues to not present the argument from reason. Certainly they contain reasoned argument, but if they

Quoting Wayfarer
arrive at a true understanding


it is an understanding of ourselves and our limits. It is not an understanding of a disembodied rational being. The dialogues are imitations or images of actual human beings known to Plato's first readers. Particular human beings with their various and particular ambitions, desires, and limits. In short, the true turning of the dialogues is not to an imagined realm of unchanging truths, but to the development of self-knowledge.









Gnomon July 18, 2023 at 16:14 #823269
Quoting wonderer1
According to wikiquote that statement that you are propagating, as being from Heisenberg, is misattributed.

OK. I'll delete the quote.

But I wasn't trying to "propagate" anything. I had never heard that quote before. And it doesn't even indicate the point I was trying to make : The quantum pioneers who used concepts from Eastern philosophy, were not trying to "propagate" the religions associated with the Holistic concepts.

Apparently Heisenberg was at least a nominal Christian, not a Hindu or Buddhist. On this forum Holistic ideas are often dismissed as "woo". But, Holism (e.g. entanglement) is a primary distinguishing factor of Quantum physics compared to Classical Newtonian physics. Yet, Newton himself was at least a nominal Christian, who dabbled in Alchemy. Which would be dismissed today as "woo". :smile:
180 Proof July 18, 2023 at 19:36 #823290
Quoting Fooloso4
There is no better source of why this is not true than the works of Plato. Several of the dialogues can be cited, but Timaeus, in which Socrates remains mostly silent, presents a clear picture of the inadequacy of the Forms. In this dialogue, much or which is a monologue, Socrates expresses the desire to see the city he creates in the Republic at war. He wants to see the city in action. The story of the city in the Republic is incomplete. It is a city created by intellect (nous) without necessity (ananke), that is, a city without chance and contingency. A city that could never be.

For a more detailed discussion: https://thephilosophyforum.com/discussion/12008/shaken-to-the-chora/p1

:fire: I very much appreciate this insight. Thanks!

Reply to Fooloso4 :up:
Count Timothy von Icarus July 18, 2023 at 20:24 #823298
Reply to wonderer1

Connectionism is much closer to where it's at when considering the way human thought really works. Perhaps it is harder for most people to think in connectionist terms though.


Just happened to come back to this. I imagine it's because we have defined what computers can do and we also know that relatively simple cellular automata can do anything a computer can. I think the move to thinking of the mind as (or the result of) a computer is another example of "looking for the keys under the streetlight."

To date, it is unclear that cellular automata, neural networks, or the like can do anything that Universal Turing Machines cannot. But this seems like a replay of the old Meno Paradox. We can't check if these entities can do something novel and possibly related to consciousness until we know what that "thing," is so we can go look for it. .
Leontiskos July 18, 2023 at 21:41 #823302
Quoting Paine
Gerson has been discussed numerous times here...


Thanks for the references. I just reread Phaedo last week so I will be curious to have a look at the thread.

Quoting Paine
I realize that I am not up for rekindling those debates right now. It is summertime and the living is easy.


Fair enough. :smile:

Quoting Paine
The problem with Gerson is that he does not distinguish between the different roles Matter (? ???) plays amongst the 'Ur-Platonists' he assembles to oppose the team of 'Materialists' he objects to...


Okay, that is an interesting difference between Aristotle and Plotinus. I am much more familiar with Aristotle than Plotinus. I found a source which corroborates what you say, and may also relate to what you say about Aristotle and Plotinus' disagreement on the soul in the other thread:

Quoting Sui Han, Review
The anti-Aristotelian conclusions [in Ennead II.5] are two. While sensible reality, according to Aristotle, involves continuity of change based on the actualisation of proximate matter, Plotinus breaks this continuity by defining matter only as prime matter which can never be actualised. While Aristotle mentions in De Anima II.5 a certain potentiality in the soul, Plotinus argues that it is rather active power than passive potentiality.


Quoting Paine
For myself, the many points Plotinus and Aristotle may agree upon are not as interesting as where they clearly do not.


The article that has already been referenced in this thread is Gerson's "Platonism Versus Naturalism." There he gives this definition of anti-materialism:

Quoting Lloyd P. Gerson, Platonism Versus Materialism | cf. From Plato to Platonism, 11
Anti-materialism is the view that it is false that the only things that exist are bodies and their properties. Thus, to admit that the surface of a body is obviously not a body is not thereby to deny materialism. The antimaterialist maintains that there are entities that exist that are not bodies and that exist independently of bodies. Thus, for the antimaterialist, the question "Is the soul a body or a property of a body?" is not a question with an obvious answer since it is possible that the answer is no. The further question of how an immaterial soul might be related to a body belongs to the substance of the positive response to [Ur-Platonism], or to one or another version of Platonism.


Are you then of the opinion that either Aristotle or Plotinus are not "antimaterialists"? That Gerson has not categorized them correctly?
Fooloso4 July 18, 2023 at 22:21 #823305
Quoting Leontiskos
I just reread Phaedo last week so I will be curious to have a look at the thread.


A couple of years ago I started a thread on Phaedo. When I completed it I compiled my separate posts so that they can be read through without having to read the whole thread. I would be interested in your comments.

1. https://thephilosophyforum.com/discussion/10914/platos-phaedo/p1


2. https://thephilosophyforum.com/discussion/comment/534860


3. https://thephilosophyforum.com/discussion/comment/535343


4. https://thephilosophyforum.com/discussion/comment/535924


5. https://thephilosophyforum.com/discussion/comment/536573


6. https://thephilosophyforum.com/discussion/comment/537114


7. https://thephilosophyforum.com/discussion/comment/537698


8. https://thephilosophyforum.com/discussion/comment/538481


9. https://thephilosophyforum.com/discussion/comment/539501


10. https://thephilosophyforum.com/discussion/comment/540733



Fooloso4 July 18, 2023 at 22:31 #823306
Quoting 180 Proof
I very much appreciate this insight.


And I very much appreciate your appreciation.
Paine July 18, 2023 at 23:04 #823307
Reply to Leontiskos
I appreciate your efforts to compare the texts.

I think Sui Han's points are Important and will look into his writings.

Lloyd P. Gerson, From Plato to Platonism, 11:Thus, for the antimaterialist, the question "Is the soul a body or a property of a body?"


In the portion I quoted, Plotinus separates 'embodiment' from matter. What is at issue is how to understand properties.
Wayfarer July 18, 2023 at 23:22 #823308
From the OP (based on the C S Lewis form of the argument):

Quoting Wayfarer
The argument from reason challenges the proposition that everything that exists, and in particular thought and reason, can be explained solely in terms of natural or physical processes. It is, therefore, an argument against materialist philosophy of mind.


And quoted above, from Gerson's paper

Quoting Lloyd P. Gerson, Platonism Versus Materialism | cf. From Plato to Platonism, 11
The antimaterialist maintains that there are entities that exist that are not bodies and that exist independently of bodies


The convergence of the two quotations ought to be clear - which is hardly surprising, since Lewis, as a Christian intellectual, no doubt defends a broadly Platonist point of view, considering the incorporation of many elements of Platonism into Christianity.

As we've now re-introduced Gerson, I'll provide a bit more detail from the essay quoted above:

[quote=Lloyd Gerson, Platonism vs Naturalism](Gerson defends the) thesis that most of the history of philosophy, especially since the 17th century can be characterized as failed attempts by various Platonists to seek some rapprochement with naturalism and, mostly in the latter half of the 20th century and also now, similarly failed attempts by naturalists to incorporate into their worldviews some element or another of Platonism. I would like to show that what I am calling the elements of Platonism...are interconnected such that it is not possible to embrace one or another of these without embracing them all. In other words, Platonism (or philosophy) and naturalism are contradictory positions. [/quote]

The elements of ‘Ur-platonism’, according to Gerson’s hypothesis are: anti-materialism, anti-mechanism, anti-nominalism, anti-relativism, and anti-scepticism, summarised below

Anti-materialism rejects the notion that only bodies and their properties exist. It allows for entities beyond bodies, like souls. Anti-mechanism states that materialist explanations are inadequate and proposes non-bodily explanations for material phenomena. Anti-nominalism denies that only individuals exist and allows for sameness in difference (i.e. the role assigned to forms or universals). Anti-relativism opposes the view that truth and goodness are subjective and emphasizes their objective or transcendental determinations. Anti-scepticism asserts that knowledge is possible, countering scepticism's doubt about attainable knowledge. (Refer to reference above for detailed content).

Quoting Paine
What is at issue is how to understand properties.


It is in this respect where I proposed the revisionist understanding of the nature of intelligibles (such as forms, numbers, the soul, and so forth.) This is not of my devising, as it is elaborated in an historical source, namely the writings of theological philosopher Scotus Eriugena (as I'll explain). The gist of this argument is that 'the soul' does not exist in the sense that the terms 'entity' and 'exist' are understood within the framework of naturalism. As an illustrative example, numbers and other intelligibles, such as 'the concept of prime', likewise do not exist in the sense that chairs, tables, and the objects of natural science exist. Their existence is purely intelligible, i.e. only perceptible to a rational mind. They are nevertheless real, in that they have the same value for all who can grasp them. As Bertrand Russell says in The World of Universals, 'universals are not thoughts, though when known they are the objects of thoughts.' The rational soul (psyche) is what can perceive these ideas but itself is not a cognizable entity. So to make of the soul an entity, as the body is an entity, is an (understandable) error of reification (literally, 'to make a thing out of').

This is from the SEP entry on Scotus Eriugena. (I have taken the liberty of replacing 'to be' with 'to exist' to draw out the point I'm trying to articulate):

Quoting John Scotus Eriugena, Dermot Moran, SEP
Eriugena lists “five ways of interpreting” the manner in which things may be said to exist or not to exist. According to the first mode, things accessible to the senses and the intellect are said to exist, whereas anything which, “through the excellence of its nature”, transcends our faculties are said not to exist. According to this classification, God, because of his transcendence is said not to exist. He is “nothingness through excellence” (nihil per excellentiam). (On this, also see Whalon, God does not Exist).

The second mode of existence and non-existence is seen in the “orders and differences of created natures” whereby, if one level of nature is said to exist, those orders above or below it, are said not to exist:

For an affirmation concerning the lower (order) is a negation concerning the higher, and so too a negation concerning the lower (order) is an affirmation concerning the higher. (Periphyseon, I.444a)

...This mode illustrates Eriugena’s original way of dissolving the traditional Neoplatonic hierarchy of being into a dialectic of affirmation and negation: to assert one level is to deny the others. In other words, a particular level may be affirmed to be real by those on a lower or on the same level, but the one above it is thought not to be real in the same way.


This is obviously a rather recondite set of distinctions, but to me it is the only way to make sense of the reality of intelligibles, such as universals, number, and the like, because it restores a dimension of reality or being that has been 'flattened out' in the transition to modernity with its exclusive concentration on material and efficient causation. Consequently, there is no conceptual space for the idea that there are different levels or domains of reality - to us, things either exist or do not exist, they do not exist 'in different ways'.
Leontiskos July 18, 2023 at 23:27 #823309
Reply to Paine
Thanks for the clarification. That's what I figured--I just wanted to make sure. To be fair, Aristotle's metaphysical account of proximate matter always struck me as a little wobbly, so Plotinus' critique makes some sense.

Reply to Fooloso4
Yes, Paine pointed me to the same thread. I am a layman, but I do look forward to reading it. :smile:
wonderer1 July 19, 2023 at 00:07 #823311
Quoting Count Timothy von Icarus
To date, it is unclear that cellular automata, neural networks, or the like can do anything that Universal Turing Machines cannot.


I didn't mean to suggest that I think neural nets can do things that a Turing machine couldn't do in principle. Remember I was talking about "the way human thought really works".

We don't have thinking based on Turing machines. We have thinking based on neural networks, and understanding the nature of the more analogish sort of information processing that occurs in neural networks is conducive to improving one's understanding of oneself.

For example Peter Tse's book The Neural Basis of Free Will: Criterial Causation discusses aspects of understanding free will, in light of scientific understanding of the way we think. It's not the sort of free will that many people want to believe in, but there is a lot of pragmatic value in understanding it.



Tom Storm July 19, 2023 at 00:07 #823310
Reply to Wayfarer Thank you that's a very succinct and informative summary of the position you've been articulating.
Leontiskos July 19, 2023 at 01:28 #823319
Quoting Wayfarer
It's a shame his work is not more approachable, because I think his central thesis - that Platonism basically articulates the central concerns of philosophy proper, and that it can't be reconciled with today's naturalism - is both important and neglected.


That's true, but at the same time Gerson is opening up the can of worms within his own discipline and therefore providing a stepping stone for someone to do the work of translating it into the world of modern philosophy. I am fairly certain that this will happen.

Quoting Wayfarer
I've long been interested in various aspects of scholastic and platonic realism, i.e. the view that universals and abstract objects are real. There's precious little interest in and support for such ideas here, or anywhere, really. But I'm of the view that it was the decline of scholastic realism and the ascendancy of nominalism which were key factors in the rise of philosophical and scientific materialism and the much-touted 'decline of the West'. But it's a hard thesis to support, and besides, as I say, has very little interest, it's diametrically at odds with the mainstream approach to philosophy.


In the Catholic world it is just the opposite! That thesis is so prevalent that it is thought to be trite. That line says that the nominalism that was conceived with Duns Scotus and came to maturity with William of Ockham is the crucial error that fueled the loss of realism and set the stage for the modern period. I think there's a lot of truth to it, although there is nuance to be had.

Quoting Wayfarer
Some of the sources I frequently cite in support...


Thank you for all the sources! I am especially interested to read the essay by Maritain.

Let me find some time and get back to you with more sources regarding this thesis, including at least one popular adaptation of Gerson.

Quoting Wayfarer
The current form of the argument from reason was popularised by C S Lewis in 1947, subsequently revised and reformulated after criticism from G.E.M. Anscombe.


Regarding the OP, I have always found Lewis' argument to be sound. I would be curious to read Anscombe's criticisms beyond the footnote he gives. She is a very competent philosopher.
Janus July 19, 2023 at 01:33 #823321
Quoting Wayfarer
Consequently, there is no conceptual space for the idea that there are different levels or domains of reality - to us, things either exist or do not exist, they do not exist 'in different ways'.


This is patently false: objects exist for us in a different way than sensations, thoughts or emotions. Inanimate objects exist for us in a different way than animals. Humans exist for us in a different way than animals. Concepts and numbers exist for us in a different way than concrete objects, and so on.
Wayfarer July 19, 2023 at 02:52 #823333
Quoting Janus
Concepts and numbers exist for us in a different way than concrete objects, and so on.


:lol: There is huge controversy over their reality and whether number is invented or discovered and so on. Empiricist philosophers like yourself generally reject the notion that they have any reality apart from as the product of the mind (read 'brain'.)

Quoting What is Math?
Platonism, as mathematician Brian Davies has put it, “has more in common with mystical religions than it does with modern science.” The fear is that if mathematicians give Plato an inch, he’ll take a mile. If the truth of mathematical statements can be confirmed just by thinking about them, then why not ethical problems, or even religious questions? Why bother with empiricism at all?

Massimo Pigliucci, a philosopher at the City University of New York, was initially attracted to Platonism—but has since come to see it as problematic. If something doesn’t have a physical existence, he asks, then what kind of existence could it possibly have? “If one ‘goes Platonic’ with math,” writes Pigliucci, empiricism “goes out the window.” (If the proof of the Pythagorean theorem exists outside of space and time, why not the “golden rule,” or even the divinity of Jesus Christ?)


Why not indeed? (My bolds, and also my point.)

Quoting Leontiskos
In the Catholic world it is just the opposite! That thesis is so prevalent that it is thought to be trite.


Which I suspect is the reason for its unpopularity outside that world. Incidentally you'll find a breakdown of Anscombe's criticism of Lewis' argument in Victor Reppert's essay (Reppert authored a book on the argument.)

Quoting Leontiskos
That line says that the nominalism that was conceived with Duns Scotus and came to maturity with William of Ockham is the crucial error that fueled the loss of realism and set the stage for the modern period. I think there's a lot of truth to it, although there is nuance to be had.


The role of Duns Scotus and the eclipse of scholastic realism is also central to John Milbank's 'Radical Orthodoxy' as I understand it (see this blog post).



Janus July 19, 2023 at 03:02 #823334
Quoting Wayfarer
There is huge controversy over their reality and whether number is invented or discovered and so on. Empiricist philosophers like yourself generally reject the notion that they have any reality apart from as the product of the mind (read 'brain'.)


Firstly, I am not an Empiricist philosopher and secondly, I am not making any claim about any "ultimate" explanation for the existence of anything. Numbers exist for us, trees exist for us, emotions exist for us, sensations exist for us, the world is full of many things which exist, and they all exist for us in different ways, a fact which refutes your claim that different things do not exist in different ways.
Ø implies everything July 19, 2023 at 03:02 #823335
Turing machines are material. If they can "reason" without some immaterial force, why can't we?

I'm not saying our reasoning is identical (see Penrose), but what is it about our reasoning that is different in a way that matters in this context? Intention? Well, a Turing machine also seemingly intends to arrive at a conclusion too. There are material forces in it that pushes out a calculation, and maybe that too is true for us?

Yet, we feel that intention. And that's really the actual challenge of materialism; The Hard Problem of Consciousness; why/how do we experience things in a completely material universe?
Wayfarer July 19, 2023 at 03:11 #823336
Quoting Janus
I am not an Empiricist philosopher…


Don’t make me go back and copy the hundred thousand times you’ve claimed that we all learn abstract concepts through experience.

Quoting Ø implies everything
If they can "reason"


Having to use scare quotes on that context vitiates whatever comes next.
Leontiskos July 19, 2023 at 03:33 #823339
Quoting Wayfarer
The role of Duns Scotus and the eclipse of scholastic realism is also central to John Milbank's 'Radical Orthodoxy' as I understand it...


Yes, Milbank takes the thesis into new frontiers. My sense is that almost all Medievalists accept the thesis with qualifications, and modern philosophers don't know the medieval period well enough to consider the thesis. When Milbank brings this up in those circles he is often met with blank stares rather than incredulity. Rowan Williams is another outlier who supports the thesis. And most any Thomist philosopher will support it to one degree or another, because the shift away from Aquinas' form of realism in the late middle ages is undeniable.

Quoting Wayfarer
Incidentally you'll find a breakdown of Anscombe's criticism of Lewis' argument in Victor Reppert's essay (Reppert authored a book on the argument.)


Great, thank you!
Ø implies everything July 19, 2023 at 04:09 #823341
Quoting Wayfarer
Having to use scare quotes on that context vitiates whatever comes next.


No, because what I wrote next only agrees with the quotation marks. Turing machines might not reason the way we do; thus, whatever their equivalent is, I call it "reason".
Janus July 19, 2023 at 07:24 #823354
Quoting Wayfarer
Don’t make me go back and copy the hundred thousand times you’ve claimed that we all learn abstract concepts through experience.


We do learn abstract concepts via experience; how else would we learn them, if we are not born with them? Anyway, that is a red herring: why don't you try to deal with the fact that your argument that things all exist for us in the same way is refuted, and actually attempt to engage in some discussion, rather than trying to deflect via dismissal by labelling?
Wayfarer July 19, 2023 at 08:07 #823358
Quoting Janus
your argument that things all exist for us in the same way


That's about the opposite of what I stated.
Janus July 19, 2023 at 08:13 #823360
Quoting Wayfarer
That's about the opposite of what I stated.


Right, what I should have said is "your argument that there is no conceptual space for the idea that things exist for us in different ways", that we moderns generally think that different things exist in the same way. That is a strawman of the dominant view I would say.
180 Proof July 19, 2023 at 10:09 #823370
Reply to Janus :up:

Quoting Wayfarer
In other words, Platonism [s](or philosophy)[/s] and naturalism are [s]contradictory[/s] positions.
— Lloyd Gerson, Platonism vs Naturalism

What's referred to here as "naturalism" I think is more cogently conceived of as Pre-Platonism (e.g. Milesian, Ionian & Eleatic cosmologies) from which subsequent "Platonism" is abstracted (and then, IMHO, reified (fallaciously) into transcendent "forms" "categories" "essences" "emanations" "universals" "patterns" etc).

Anyway, Wayf, reviews of Gerson's book are intriguing so I'll pick it up (unless @Fooloso4's arguments / objections (here or elsewhere) persuade me not to bother).
Metaphysician Undercover July 19, 2023 at 12:15 #823380
Quoting Wayfarer
One of those straws is the belief that the parable of the cave does indeed present an allegory for a kind of intellectual illumination or an insight into a higher domain of being, and that those who have ascended to it see something which others do not, as I think the allegory plainly states. (I’m of the view that this is what is represented by the later term ‘metanoia’ which is not found in the Platonic dialogues but which means in this context an intellectual conversion or the breakthrough into a new way of seeing the world.) I suppose one secondary source I could refer to for support is this SEP entry on ’divine illumination’ in Greek philosophy.


The breakthrough referred to here, intellectual conversion, or illumination, is to apprehend the priority of conceptions and ideas. Ideas are first in the mind of the individual, as what are present to the mind. This is Kant's starting point in the Critique of Pure Reason, the priority of what's in the mind, phenomena. But for Plato the priority is not just a logical priority, but also a temporal priority as well. He sees ideas as causal through the reality of "the good". This way of looking at things is outlined in The Symposium, when the student of love grasps the beauty of human artefacts and institutions, and learns of a Beauty which transcends the beauty of any particular artefact.

So the cave allegory expresses this causal relationship of temporal priority. What the people in the cave see, are shadows on the wall, and the shadows are representative of artificial material objects. Behind the scenes, what few people properly relate to, and understand, is that human ideas, along with ambition, desire, intention or good, are the cause of these artificial products. Human intention and ambition is represented as the fire, and we are directed to apprehend the material products as simple representations of the human ideas, the shadows which come into being through the means of the fire.

That is the first stage of the philosopher's illumination, and the important point is that the ideas, along with the intention or good (the fire), are temporally prior to the material products as the cause of them. This temporal priority is what validates Plato's claim of a higher degree of reality to the intelligible realm (what's inside the mind), as the artificial material products are simply a copy or reflection of what's inside the mind. Aristotle adopts this position, and assigns actuality to form.

The second stage of the illumination is when the philosopher exits the cave. Then the philosopher sees the entirety of the natural world under this conceptual structure of temporal priority. All material objects are seen as reflections of the Form which produces them. The human good (the fire) is replaced by the natural good (the sun) and the philosopher sees all natural material objects illuminated by the sun, as reflections of the Forms which produce them. The Forms are temporally prior to the material objects as the cause of their existence, and this priority is also a logical priority as expressed in Plato's Timaeus and Aristotle's Metaphysics as the cosmological argument.

The logical necessity of the priority of the Forms is the result of assigning actuality to form, and this is validated by the concept of final cause, the causal force of human ideas, and the existence of artificial objects. You'll notice that Plotinus' proposition of "the One" as prior to all, fails in logical necessity because "the One" is assigned the character of unlimited potential. So "the One" as Plotinus' first principle, lacks in the required actuality to be causal. Therefore it lacks the logical force required as a first principle.
Mww July 19, 2023 at 13:22 #823389
I understand you guys are talking about Plato, but I wanted to inject this in the interest of conformity with the textual record.

Quoting Metaphysician Undercover
This is Kant's starting point (….) the priority of what's in the mind, phenomena.


Priority of what’s in the mind…..yes; that the priority of what’s in the mind is phenomena……no.

At best, with respect to phenomena, it can only be said that the priority in the mind is the antecedent conceptual conditions by which they are possible, which is the deduction of the pure conceptions, better known as the categories.

Ideas, remaining with Kant, have priority in the mind regarding that which is not as yet, or may never be, phenomena.

Unless I misunderstood, in which case…..never mind.



Fooloso4 July 19, 2023 at 14:47 #823393
Quoting Wayfarer
the parable of the cave


The irony here is that although with the image of the cave Plato is warning against the persuasive power of images he does so using images. And this is often taken to be not an image but the truth itself.

The cave story is, as Socrates says when telling it:

... an image of our nature in its education and want of education ... (Republic 514a)


The escape from the cave is an escape from the bonds of our education, an escape from the images of the truth. Replacing an image with another image, one of a transcendent realm of Forms, is not to escape the cave, but to remain bound within it. The image makers, the educators, that is, the poets, are not replaced by all knowing truth telling philosophers, but by the image maker Socrates.

In the Apology Socrates denies having knowledge of "anything very much or great and good or beautiful" (21d). And yet in the Republic he tells this story of transcendent knowledge, a knowledge he does not possess. In the Phaedrus he says he has an 'erotic art' (257a). And in the Symposium he claims to know nothing except things about eros (177d). It is this knowledge of eros or desire that informs his story of transcendence. The philosopher desires, but does not possess, transcendent knowledge.

The education of the philosopher living in the city, which is to say, the cave, is an education is how to educate those he must educate if there is to be any possibility of justice for the philosopher. Socrates does so by imitating the theologians, those who claim to have knowledge of divine things. But in doing so he replaces the willful capriciousness of the gods with the Forms.
Janus July 19, 2023 at 21:59 #823478
Reply to Fooloso4 Yes, it has always seemed to me that the 'cave' metaphor in Plato is better read as the contrast between unthinking acceptance of the shadows on the wall as being the Real, and the philosophical attitude of questioning the reality of those shadows, and not as a contrast between ignorance and enlightenment; a matter of coming to know, not the truth, but that the truth you thought you knew is not the truth; Socrates' wisdom of knowing that you do not know.
Wayfarer July 19, 2023 at 22:37 #823480
Quoting Fooloso4
The escape from the cave is an escape from the bonds of our education, an escape from the images of the truth. Replacing an image with another image, one of a transcendent realm of Forms, is not to escape the cave, but to remain bound within it.


Your interpretation is at odds with the text, though, and every interpretation of the meaning of the Allegory of the Cave that I've read. In the allegory 'prisoners' represent those ignorant of the forms:

[quote=Republic VII;https://www.platonicfoundation.org/republic/republic-book-7/]For in the first place, do you think such people [i.e. the prisoners in the cave] would ever have seen anything of themselves, or one another, apart from the shadows cast by the fire onto the cave wall in front of them?[/quote]

So the education in question, is the education necessary to overcome their attachment to the illusory domain and to perceive the real (i.e. be closer to 'what is'):

“Now,” I said, “consider what liberation from their bonds, and cure of their ignorance, would be like for them, if it happened naturally in the following way. Suppose one of them were released, and suddenly compelled to stand up, crane his neck, walk, and look up towards the light. Would he not be pained by all this, and on account of the brightness be unable to see the objects whose shadows he previously beheld? And if someone were to tell him that he beheld foolishness before, but now he sees more truly, since he is much closer to ‘what is’, and is turned towards things which partake of more being, what do you think he would say? Moreover, if they showed him each of the passing objects and forced him to answer the question ‘what is this?’, do you not think he would be perplexed, and would believe that what he saw before was truer than what he is now being shown?”


What do you think it means to 'partake of more being'?

Quoting Fooloso4
The philosopher desires, but does not possess, transcendent knowledge.


Then what to make of all this? First there is the passage where the philosopher (I presume it's a philosopher) returns to the Cave, but his eyes are now unaccustomed to the gloom:

Now, suppose that he had to compete once more with those perpetual prisoners in recognising these shadows, while his eyesight was still poor, before his eyes had adjusted. Since it would take some time to become accustomed to the dark, would he not become a figure of fun? Would they not say that he went up, but came back down with his eyes ruined, and that it is not worth even trying to go upwards? And if they could somehow get their hands on and kill a person who was trying to free people and lead them upwards, would they not do just that?

“Definitely,” he said.

“Then, dear Glaucon,” I said, “you should connect this image, in its entirety, with what we were saying before.[2]Compare the realm revealed by sight to the prison house, and the firelight within it to the power of our sun. And if you suggest that the upward journey, and seeing the objects of the upper world, is the ascent of the soul to the realm known by reason, you will not be misreading my intention, since that is what you wanted to hear. God knows whether it happens to be true, but in any case this is how it all seems to me. When it comes to knowledge, the form of the good is seen last, and is seen only through effort. Once seen, it is reckoned to be the actual cause of all that is beautiful and right in everything, bringing to birth light, and the lord of light, in the visible realm, and providing truth and reason in the realm known by reason, where it is lord. Anyone who is to act intelligently, either in private or in public, must have had sight of this.”

“I also hold the same views that you hold,” he said, “after my own fashion, anyway.”

“Come on then,” I said, “and agree with me about something else. Do not be surprised that those who have attained these heights have no desire for involvement in human affairs. Their souls, rather, are constantly hastening to commune with the upper realm. For I presume that is what is likely to happen, if this really does accord with the image we described earlier.”


Bolds added. I think 'the realm known by reason' makes the meaning perfectly clear, and that as a whole that the allegory shows that, on the whole, prisoners of the cave (the hoi polloi) do not inhabit that realm, but require a 'painful education' in order to reach it.
Wayfarer July 19, 2023 at 22:40 #823482
Quoting Metaphysician Undercover
Ideas are first in the mind of the individual, as what are present to the mind.


'universals are not thoughts, though when known they are the objects of thoughts.' ~ Bertrand Russell

Paine July 19, 2023 at 23:54 #823485
Reply to Wayfarer
I meant my remark about properties to apply specifically to Plotinus assigning a different role to 'matter', as potential, from what Aristotle did. For Plotinus, the hylomorphism that makes each creature different from another, is said to be an illusion, a trick of the light. Once a body takes a determinate shape, it is no longer 'material' as an expression of potential. With 'matter' no longer having a portion of being, the shared life of being a soul like all the other forms of life, where our need for nutrition or use of perception and movement is the same as plants and other animals, is overruled by an idea of individual souls.

ibid. III, 1, 8:We shall have to introduce among the number of beings another principle, the soul. The soul is a principle of no little importance. She is the force that binds all things together. Unlike the other things she is not born of some seed but is a primary cause. When she is outside the body, she remains absolute mistress of herself, free and independent even of the cause which administers the world. As soon as she has descended into a body, she is no longer fully independent, for she then forms part of an order with other things. She yields in part to the influence of the accidental circumstances into which she fell, but also dominates and directs them according to her wishes. This power of domination depends on the degree of her excellence. When she yields to temperaments of the body, she is necessarily subjected to desire or anger, is discouraged in poverty, is proud in prosperity, and is tyrannical in the exercise of power. But when she resists all these evil tendencies and her nature is a good one, she changes her surroundings more than she is changed by them. Then she alters some things, while she tolerates others without herself falling into vice.


While I admire this passage for bringing forth the importance of being an individual human, it does not reflect the serious consideration by Plato and Aristotle to recognize the indeterminate events and accidents that studying a natural world require of us.
Wayfarer July 20, 2023 at 00:02 #823486
Reply to Paine Thank you :pray:

ibid. III, 1, 8:Unlike the other things she is not born of some seed but is a primary cause. When she is outside the body, she remains absolute mistress of herself, free and independent even of the cause which administers the world.


You can definitely see echoes of this in Christian theology, in which each soul is created by God (i.e. 'not born of some seed').
Paine July 20, 2023 at 01:06 #823490
Reply to Wayfarer
But Plotinus is not introducing a personal God to witness the activities. We get with the program because we understand our situation, or we do not.


Fooloso4 July 20, 2023 at 01:11 #823491
Quoting Wayfarer
Your interpretation is at odds with the text, though, and every interpretation of the meaning of the Allegory of the Cave that I've read.


My interpretation is in line with both the text and well known and well regarded interpretations of it.

Quoting Wayfarer
So the education in question, is the education necessary to overcome their attachment to the illusory domain and to perceive the real (i.e. be closer to 'what is'):


Quoting from the text:

... an image of our nature in its education and want of education ... (Republic 514a)


The nature of the education of the cave dwellers, that is, our education, is that:

... such men would hold that the truth is nothing other than the shadows of artificial things. (515c)


What is wanting is an education in the truth.

“Then, dear Glaucon,” ...


Note that in the middle of this passage you quote Socrates says of the story:

A god doubtless knows if it happens to be true.


A god would know if it happens to be true, but Socrates does not. And we do not.

Quoting Wayfarer
Once seen, it is reckoned to be the actual cause of all that is beautiful and right in everything ...


But this is not something that Socrates has seen and not something that we have seen. For us it too is an image, a story about something we have no experience of.

Anyone who is to act intelligently, either in private or in public, must have had sight of this.


And so, based on our education and want of education we do not have the knowledge to act intelligently. Such knowledge cannot be given to us by this or any other story.

“I also hold the same views that you hold,” he said, “after my own fashion, anyway.”


Glaucon makes the same mistake that you do. He holds a view about something he has not seen. He takes an image to be the truth.

Quoting Wayfarer
I think 'the realm known by reason'


If we follow the divided line this in not the realm known by reason. The realm is not known by reason (dianoia) but by nous. It is the realm of what is seen by the mind, not something known by reason. Not something that can be taught. Not something we have seen and not something we know to be true.

Added: I don't know which translation you are citing but the Bloom translation does not say "the realm known by reason".








Janus July 20, 2023 at 02:05 #823497
Quoting Wayfarer
Your interpretation is at odds with the text, though, and every interpretation of the meaning of the Allegory of the Cave that I've read. In the allegory 'prisoners' represent those ignorant of the forms:

For in the first place, do you think such people [i.e. the prisoners in the cave] would ever have seen anything of themselves, or one another, apart from the shadows cast by the fire onto the cave wall in front of them?


That quoted passage says nothing about the "forms".
Wayfarer July 20, 2023 at 02:25 #823501
The translation is this https://www.platonicfoundation.org/republic/republic-book-7/

Quoting Fooloso4
Quoting from the text:

... an image of our nature in its education and want of education ... (Republic 514a)

The nature of the education of the cave dwellers, that is, our education, is that:

... such men would hold that the truth is nothing other than the shadows of artificial things. (515c)

What is wanting is an education in the truth.


Quoting selectively from the text. Those 'dwelling in the cave' only know the appearances (shadows on the wall), and the 'education in the truth' is described in the following:

“Now,” I said, “consider what liberation from their bonds, and cure of their ignorance, would be like for them, if it happened naturally in the following way. Suppose one of them were released, and suddenly compelled to stand up, crane his neck, walk, and look up towards the light. Would he not be pained by all this, and on account of the brightness be unable to see the objects whose shadows he previously beheld? And if someone were to tell him that he beheld foolishness before, but now he sees more truly, since he is much closer to ‘what is’, and is turned towards things which partake of more being, what do you think he would say?"


This is in reference to those who have 'ascended from the cave', and seen the true light of the good, compared here with the Sun

Again, what is the meaning of 'things which partake of more being'? What does it mean to be 'much closer to what is?'

Quoting Fooloso4
A god would know if it happens to be true, but Socrates does not.


But he says immediately afterward:

When it comes to knowledge, the form of the good is seen last, and is seen only through effort. Once seen, it is reckoned to be the actual cause of all that is beautiful and right in everything, bringing to birth light, and the lord of light, in the visible realm, and providing truth and reason in the realm known by reason, where it is lord. Anyone who is to act intelligently, either in private or in public, must have had sight of this.”


There is no suggestion that this is something he himself hasn't seen.

Quoting Janus
That quoted passage says nothing about the "forms".


'When it comes to knowledge, the form of the good is seen last, and is seen only through effort.'

Quoting Paine
But Plotinus is not introducing a personal God to witness the activities.


Of course not, but there are echoes of his doctrines in Christianity, due to the considerable influence of platonism on later Christian theology (for better or worse). I read recently that it's possible that Plotinus and Origen (one of the Church Fathers) were both disciples of the same teacher.





Janus July 20, 2023 at 02:39 #823504
Quoting Wayfarer
Your interpretation is at odds with the text, though, and every interpretation of the meaning of the Allegory of the Cave that I've read. In the allegory 'prisoners' represent those ignorant of the forms:


Quoting Wayfarer
That quoted passage says nothing about the "forms".
— Janus

'When it comes to knowledge, the form of the good is seen last, and is seen only through effort.'


I was referring to the passage you presented as a rebuttal of this

Quoting Fooloso4
The escape from the cave is an escape from the bonds of our education, an escape from the images of the truth. Replacing an image with another image, one of a transcendent realm of Forms, is not to escape the cave, but to remain bound within it.


which I quoted.



Paine July 20, 2023 at 07:34 #823530
Quoting Wayfarer
Of course not, but there are echoes of his doctrines in Christianity, due to the considerable influence of platonism on later Christian theology (for better or worse).


Yes, and Augustine said Plotinus was a better Platonist than Plato was in the City of God. But that sense of what is a natural good was still separated from the grace that only God could bestow upon a believer.
Wayfarer July 20, 2023 at 08:45 #823536
Reply to Paine Perhaps. But I don't know if the 'form of the Good' could be described in terms we would now call naturalistic although I agree there's nothing corresponding to 'divine grace' in Plato's dialogues.
Metaphysician Undercover July 20, 2023 at 11:27 #823546
Quoting Mww
At best, with respect to phenomena, it can only be said that the priority in the mind is the antecedent conceptual conditions by which they are possible, which is the deduction of the pure conceptions, better known as the categories.


I do not agree with your interpretation of Kant here. The categories are produced by judgement, and I think that Kant does not properly characterize judgement. This is where the issues of his system are evident, and Kant runs into problems. I think you and I have discussed the nature of judgement before.

The conditions for sensibility, phenomena, are the pure intuitions of space and time. And "intuition" is not well defined by Kant. It is not even implied that intuition is necessarily within the mind. But these pure intuitions are necessarily prior to phenomena. But the categories are created, or discovered as a means for judging phenomena. They are not necessarily prior to phenomena, as Kant described them as conforming to the appearance of phenomena. Nor are the categories properly called "intuitions" because they are already judgements of some sort, and judgement is posterior to intuition. But then he seems to want to assert that such tools of judging phenomena are prior to phenomena, though he formulates his categories as conforming to phenomena.

Quoting Wayfarer
'universals are not thoughts, though when known they are the objects of thoughts.' ~ Bertrand Russell


As an "object" of thought, we can ask where that object exists. Thinking occurs within the mind, and we can conclude that the object of thinking, the goal or end, is within the mind as well, as directing the thought. Other objects of thought must exist within as well.

The problem with Plato's analogy, comparing the good with the sun, and the intelligible object with the visible object, is that the sun and visible object are external to the mind. This may create the impression that the good and the intelligible object are external to the mind as well. I believe the proper interpretation of the analogy is to compare the internal "realm" of thinking, mind, and intellect, with the external "realm" of sensation. This places the good, the intellect, and the intelligible object as internal to the mind.

However, since there is ambiguity as to how the intelligible object, and the intelligible realm are to be understood, there is also ambiguity as to the proper location of "the good". Accordingly, Aristotle distinguished between the apparent good, and the real good. This distinction is commonly used, and misused in Christianity. It is often proposed that the real good is the external good, as supported by God, and the internal good is the apparent good. However, God cannot support the real good, as demonstrated by the Euthyphro problem. And it becomes evident from the problems of the immoral human being knowing what is good (as external good), yet acting in a contrary way (guided by the internal good), that the real good must be the internal good. This is "the good" which motivates the actions of a human being, therefore it is the real good, and the external good, the one supposedly supported by God is the apparent good, as other external objects are only appearances of objects as well.

Quoting Wayfarer
What does it mean to be 'much closer to what is?'


I think that is best interpreted as temporal priority, "what is", is the present, therefore what is meant is closer to the present. The shadows are the effect, therefore in the past. If you come to apprehend the causal role of the internal, then a vast realm of "inner space" with its own mode of relations, distinct from external relations, will be revealed to you. The temporal order, which is supported in some degree with science by the concept of spatial expansion, is from the inside outward. The future, with all its related features lies within the internal realm which the human mind partakes of in a very limited degree. The internal (future) manifests at the present in an outward process and this is what gives the internal intelligible objects causal capacity.
Mww July 20, 2023 at 11:58 #823547
Quoting Metaphysician Undercover
I do not agree with your interpretation of Kant here.


Be that as it may…..I mean, you pretty much disagree with everybody…..it is clear that priority in the mind, as such, cannot be phenomena.
Paine July 20, 2023 at 14:25 #823565
Reply to Wayfarer
The point I wish to make is that the tension between the natural order and the truth of religion that occupied the Scholastic philosophers did not exist for Plotinus.

This disconnect is a separate one from the issue Gerson opines upon. The difference between Plotinus and Aristotle regarding matter undercuts Gerson's attempt to group their views as sharing a common view of the order of nature. Much of the Ennead's arguments are oppositions to Aristotle, sometimes expressed specifically as such but more often by citing as incorrect descriptions that resemble Aristotle's positions.

While Plotinus has positions that do not agree with Plato, he does not discuss those as differences. To the best of my knowledge, Plotinus always knows what Plato really meant.
Fooloso4 July 20, 2023 at 15:13 #823568
Quoting Wayfarer
Those 'dwelling in the cave' only know the appearances


Those dwelling in the cave are just like us:

... you should compare our nature, in respect of education and lack of education, to a condition such as the following.


When Glaucon says how strange this image is Socrates replies:
They are just like us ...[/quote]

Quoting Wayfarer
and the 'education in the truth' is described in the following


This does not describe our education, the education of the dwellers:

[quote ]“Now,” I said, “consider what liberation from their bonds, and cure of their ignorance, would be like for them, if it happened naturally in the following way. Suppose one of them were released, and suddenly compelled to stand up, crane his neck, walk, and look up towards the light. [/quote]

How can the bonds that keep the prisoners from turning around release "naturally"?

A bit further on:

“And,” I said, “if someone were to drag him forcibly from there ...


If this someone forces a prisoner out of the prison then it does not occur naturally.

Quoting Wayfarer
There is no suggestion that this is something he himself hasn't seen.


In that case he would not say:

God knows whether it happens to be true, but in any case this is how it all seems to me.


It would not be how it seems to him, he would know that it is true. He would have divine knowledge rather than the human knowledge he professes. He would have the cure for his ignorance.








Leontiskos July 20, 2023 at 18:04 #823602
Quoting Paine
The point I wish to make is that the tension between the natural order and the truth of religion that occupied the Scholastic philosophers did not exist for Plotinus.


Good point. It's interesting, though, that in Eastern Christianity the nature/grace and reason/faith distinctions are not as stark as they are in the West. There does not seem to have been as much of a focus on Pelagianism in the East, and Augustine does not occupy such a central role.

This was a very interesting passage from Plotinus. Thanks for sharing.
Metaphysician Undercover July 21, 2023 at 01:44 #823653
Quoting Mww
Be that as it may…..I mean, you pretty much disagree with everybody…..it is clear that priority in the mind, as such, cannot be phenomena.


Sure, I might not be very agreeable, but if you read Stanford's article on a priori justification, you'll see that there are many problems with Kant\s system. So it's not just me.

The problem I have with the idea of a priori judgements, is that if the justification comes from within the mind, this just produces an infinite regress, as each a priori judgement would require an a priori justification, which would be an a priori judgement requiring a further a priori justification, and there would be no substance upon which all these justifications would be supported, just an implied infinite regress.

What Kant does do though, is grounds, or substantiate the a priori in intuition. However, this removes "intuition" from the mind, making it prior to the mind, as the basis for the a priori judgements within the mind. But intuition is also the necessary condition for phenomena. Therefore he provides no real principles which would place a priori judgements as prior to phenomena in the mind. In reality, he just uses "intuition" in an ambiguous, obscure way, to hide the problems with his proposed system.
Mww July 21, 2023 at 11:50 #823707
Paine July 23, 2023 at 18:46 #824118
Reply to Leontiskos
My pleasure.

Is there a philosopher (or more than one) from the Orthodox side you see as a counterpoint to the western Scholastics?

Perhaps another way to ask that is, was there a parallel equivalent of the Renaissance on the other side of the Schism?
Leontiskos July 23, 2023 at 19:57 #824133
Quoting Paine
Is there a philosopher (or more than one) from the Orthodox side you see as a counterpoint to the western Scholastics?


There is an interesting book by Martin Laird that grew out of his dissertation. It is called Gregory of Nyssa and the Grasp of Faith. I think it strikes a counterpoint to Augustine, and to my knowledge it is characteristic of the Cappadocian Fathers as a whole (Basil, Gregory of Nyssa, and Gregory of Nazianzus).

Quoting Paine
Perhaps another way to ask that is, was there a parallel equivalent of the Renaissance on the other side of the Schism?


There very much was. The Palamite or Hesychast controversy took place the 14th century and began with the disagreement between Gregory Palamas and Barlaam of Calabria. Barlaam was one of many renaissance thinkers in the East who were hearkening back to Greek philosophy and learning, and encouraging a more syncretistic approach.

A common Eastern narrative would say that the West accepted the renaissance of Greek and Aristotelian thinking in the medieval period whereas the East rejected it, and this deepened the schism. I think that is simplistic, but there is some truth to it.
Paine July 23, 2023 at 21:42 #824157
Reply to Leontiskos
Much to consider in your comments. I will try to get up to speed.
180 Proof July 24, 2023 at 02:36 #824201
Quoting Fooloso4
The irony here is that although with the image of the cave Plato is warning against the persuasive power of images he does so using images. And this is often taken to be not an image but the truth itself.

:fire:

Reply to Janus :up: